GI Board Exam

Download as pdf or txt
Download as pdf or txt
You are on page 1of 281

SPIEGEL • KARSAN

Hepatology is an expanding field—it’s hard to keep up. The liver has been sitting there the whole

Acing the Hepatology Questions on the GI Board Exam


time, but the knowledge surrounding this perplexing organ is exploding; this makes it hard to
prepare for board and recertification exams, where 1 in every 4 questions pertains to hepatology.
Until now, there has been no single, slim, but high-yield volume that summarizes the hepatology
you need to know for the boards. This book has it nailed. The authors have collected every pearl

T H E U LT I M AT E C R U N C H  T I M E R E S O U R C E
of wisdom, high-yield factlet, liver “board buzzword,” hepatic micrograph, and classic liver im-
aging study they could muster, all while keeping the book a manageable size—because who has
time for more than manageable when you’re busy?
The answer to your study questions (and study time!) can be found inside Acing the Hepatology
Acing the Hepatology Questions on the

GI Board Exam
Questions on the GI Board Exam: The Ultimate Crunch-Time Resource.
Traditional textbooks usually feature long and detailed discussions that are not directly related to
Board and recertification exams. On the flip side, many Board review manuals provide lists and
bullet points lacking sufficient background and context. Inside Acing the Hepatology Questions
on the GI Board Exam, Drs. Brennan M. R. Spiegel and Hetal A. Karsan present time-tested and
high-yield information in a rational, useful, and contextually appealing format.
Why You Will Need to Read Acing the Hepatology Questions on the GI Board Exam:
T H E U LT I M AT E C R U N C H  T I M E R E S O U R C E
• Focuses exclusively on hepatology review – an area that comprises 25% of the Board exam
• Carefully vetted board-style vignettes with color images
• Comprehensive yet succinct answers using a high-yield format
• Emphasis on key clinical pearls and “board buzzwords”
• Answers to classic board “threshold values” questions that you need to know but
always seem to forget:
If an echinococcal liver cyst exceeds XX cm, then the risk of rupture is clinically
significant and surgery is warranted
• Rapid fire crunch-time exam with 135 classic one-liners such as:
Spider web collaterals + caudate lobe hypertrophy = Diagnosis
Stepwise fever + temperature-pulse dissociation + rose spots + hepatitis = Diagnosis
With its focus on pearl after pearl, emphasis on images, and attention to high-yield “tough stuff”
vignettes you don’t know the answers to (yet), Acing the Hepatology Questions on the GI Board
Exam is truly the ultimate crunch-time resource for acing the often vexing liver section of the
examination, taking recertifying examinations, looking good on clerkship rounds, or for just
challenging yourself with interesting and entertaining vignettes.

BRENNAN M. R. SPIEGEL HE TAL A. KARSAN


SLACK
I N C O R P O R A T E D

slackbooks.com

MEDICAL/Gastroenterology
SLACK Incorporated
®
BRENNAN M. R. SPIEGEL, MD, MSHS, FACG
Chief of Education and Training, UCLA
Division of Digestive Diseases
Program Director, UCLA
Gastroenterology Fellowship Training Program
Director, UCLA
Center for Outcomes Research and Education (CORE)
Associate Professor of Medicine,
David Geffen School of Medicine at UCLA
Los Angeles, California

HETAL A. KARSAN, MD, FACG, FACP


Clinical Assistant Professor of Medicine,
Emory University
Atlanta Gastroenterology Associates
Atlanta, Georgia
www.slackbooks.com
ISBN: 978-1-55642-953-8
Copyright © 2012 by SLACK Incorporated

Brennan M.R. Spiegel, MD, MSHS, FACG has no financial or proprietary interest in the materials presented
herein.
Hetal A. Karsan, MD, FACG, FACP has no financial or proprietary interest in the materials presented herein.
All rights reserved. No part of this book may be reproduced, stored in a retrieval system or transmitted in any
form or by any means, electronic, mechanical, photocopying, recording or otherwise, without written permission
from the publisher, except for brief quotations embodied in critical articles and reviews.

The procedures and practices described in this publication should be implemented in a manner consistent with
the professional standards set for the circumstances that apply in each specific situation. Every effort has been
made to confirm the accuracy of the information presented and to correctly relate generally accepted practices.
The authors, editors, and publisher cannot accept responsibility for errors or exclusions or for the outcome of the
material presented herein. There is no expressed or implied warranty of this book or information imparted by it.
Care has been taken to ensure that drug selection and dosages are in accordance with currently accepted/recom-
mended practice. Off-label uses of drugs may be discussed. Due to continuing research, changes in government
policy and regulations, and various effects of drug reactions and interactions, it is recommended that the reader
carefully review all materials and literature provided for each drug, especially those that are new or not fre-
quently used. Some drugs or devices in this publication have clearance for use in a restricted research setting by
the Food and Drug and Administration or FDA. Each professional should determine the FDA status of any drug
or device prior to use in their practice. Any review or mention of specific companies or products is not intended
as an endorsement by the author or publisher.
SLACK Incorporated uses a review process to evaluate submitted material. Prior to publication, educators or
clinicians provide important feedback on the content that we publish. We welcome feedback on this work.
Published by: SLACK Incorporated
6900 Grove Road
Thorofare, NJ 08086 USA
Telephone: 856-848-1000
Fax: 856-848-6091
www.slackbooks.com
Contact SLACK Incorporated for more information about other books in this field or about the availability of
our books from distributors outside the United States.
Library of Congress Cataloging-in-Publication Data
Spiegel, Brennan M.R., 1972-
Acing the hepatology questions on the GI board exam : the ultimate crunch-time resource / Brennan Spiegel
and Hetal A. Karsan.
p. ; cm.
Includes index.
ISBN 978-1-55642-953-8 (pbk.)
1. Liver--Diseases--Examinations, questions, etc. 2. Hepatology--Examinations, questions, etc. I. Karsan, Hetal
A., 1971- II. Title.
[DNLM: 1. Liver Diseases--Examination Questions. WI 18.2]
RC845.S65 2011
616.3’620076--dc23
2011025868
For permission to reprint material in another publication, contact SLACK Incorporated. Authorization to photo-
copy items for internal, personal, or academic use is granted by SLACK Incorporated provided that the appropri-
ate fee is paid directly to Copyright Clearance Center. Prior to photocopying items, please contact the Copyright
Clearance Center at 222 Rosewood Drive, Danvers, MA 01923 USA; phone: 978-750-8400; website: www.copy-
right.com; email: info@copyright.com

Printed in the United States of America.

Last digit is print number: 10 9 8 7 6 5 4 3 2 1


DEDICATION
To my parents, who always encouraged me to “do the best you can, because
nobody can ask for anything more, and you won’t be satisfied with anything
less.” I’ve always remembered that.

—Brennan M. R. Spiegel, MD, MSHS, FACG

To my wife, Lina, who remains the most selfless person on this planet, and my
children, Sonia and Rajan, who inspire me daily with their boundless enthusi-
asm. Iamque opus exegi. Om Shanti Shanti Shanti.

—Hetal A. Karsan, MD, FACG, FACP


CONTENTS

Dedication.................................................................................................................v
Acknowledgments ...................................................................................................ix
About the Authors ...................................................................................................xi
Preface ................................................................................................................... xiii

Chapter 1 Liver Disease on the GI Board Exam ............................................... 1


Chapter 2 “Tough Stuff” Vignettes ................................................................... 7
Chapter 3 60 Hepatology Board Review “Clinical Threshold Values” ............. 215
Chapter 4 “Crunch-Time” Self-Test—Time to Get Your Game On ...................221

Appendix A: Answers to “Crunch-Time” Self-Test…………………………………....237


Appendix B: “Crunch-Time” Self-Test Scoring Guide………………………………...243

Bibliography .........................................................................................................245
Index.....................................................................................................................251
ACKNOWLEDGMENTS
The authors wish to thank the following individuals who provided images for
this book:
z Stanley Dea, MD (Olive View–UCLA Medical Center)
z Francisco Durazo, MD (UCLA Medical Center)
z Alton B. Farris, MD (Emory University)
z Steven Hanish, MD (Emory University)
z Barbara Kadell, MD (UCLA Medical Center)
z Bobby Kalb, MD (Emory University)
z Dennis Jensen, MD (UCLA Medical Center)
z Charles Lassman, MD (UCLA Medical Center)
ABOUT THE AUTHORS
Brennan M. R. Spiegel, MD, MSHS, FACG is an Associate Professor of Medicine
in the Division of Digestive Diseases, UCLA School of Medicine, and in the
Division of Gastroenterology, VA Greater Los Angeles Health Care System. He is
the section chief for Health Services Research at the UCLA Division of Digestive
Diseases, and Chief of Education and Training in the UCLA GI Fellowship Training
Program, which is amongst the largest GI Training Programs in the country.
Dr. Spiegel attended Tufts University where he majored in Philosophy and
Community Health, and obtained his medical degree from New York Medical
College. He received training in internal medicine at Cedars-Sinai Medical
Center in Los Angeles, completed a fellowship in Gastroenterology at UCLA, and
completed advanced studies in Health Services Research in the UCLA School
of Public Health, where he received a master’s degree in Health Services. He is
board certified in Internal Medicine and Gastroenterology. He currently teaches
in the Schools of Medicine and Public Health at UCLA.
Dr. Spiegel’s research interests have focused on acid-peptic disorders, chronic
liver disease, gastrointestinal hemorrhage, and functional bowel disorders such
as irritable bowel syndrome and dyspepsia. He has performed research across
a range of health services methodologies, including health-related quality of life
measurement, survey design and administration, systematic review, meta-analy-
sis, multivariable regression analysis, survival analysis, expert panel research,
quality improvement, cost-effectiveness analysis, budget impact modeling, and
use of clinical informatics to support decision making. He is a peer reviewer
for numerous medical journals, and is on the editorial boards for the American
Journal of Gastroenterology and Clinical Gastroenterology and Hepatology. He
has contributed to the publication of more than 90 peer-reviewed papers, as well
as numerous abstracts, book chapters, and monographs.

Hetal A. Karsan, MD, FACG, FACP is a Clinical Assistant Professor of Medicine


in the Division of Digestive Diseases at Emory University School of Medicine in
Atlanta. He is a gastroenterologist and hepatologist at Atlanta Gastroenterology
Associates, one of the largest gastroenterology groups in the country, at which he
is a partner. He is also a practicing clinician at Emory University.
Dr. Karsan attended Indiana University in Bloomington, Indiana, where he
received his Bachelor of Science in Biology. While an undergraduate, he taught a
course on Evolution and Diversity to fellow college students and won awards for
undergraduate biomedical research from the Howard Hughes Medical Institute.
He obtained his medical degree from Indiana University School of Medicine and
went on to train in Internal Medicine at Boston University, where he completed
his medical internship and residency. Dr. Karsan served as acting Chief Medical
Resident while at Boston University. He completed fellowship at UCLA Medical
Center, where he trained in gastroenterology, advanced interventional endoscopy,
and transplant hepatology. While at UCLA, he won several awards and was nomi-
nated for the Teaching Fellow of the Year by the Department of Internal Medicine.
He also sought formal training in clinical outcomes research at UCLA.
Dr. Karsan is active in numerous professional organizations and was elected
as Fellow by both the American College of Gastroenterology and the American
College of Physicians. In addition to contributions to various peer-reviewed
manuscripts, abstracts, and book chapters, he also serves as a peer reviewer for
medical journals and is on an editorial board. Dr. Karsan is board certified in
internal medicine, gastroenterology, and transplant hepatology. He enjoys endos-
copy and actively participates in clinical outcomes research. In his spare time,
he enjoys playing sports, traveling, and spending time with his family.
PREFACE
You’re holding the second book in the now-growing Acing the GI Board Exam
series of review manuals. The first book, entitled Acing the GI Board Exam:
The Ultimate Crunch-Time Resource, proved to be a well-received and unique
approach to studying for the GI Board examination. Buoyed by positive reviews
and seemingly satisfied readers of the first book, we opted to expand the series
to the field of hepatology, a topic that constitutes 25% of the American Board
of Internal Medicine (ABIM) GI Board examination. The purpose of this book is
to complement the original effort by drilling deeper into liver knowledge for the
GI exam, but not drilling so far that the result is diminishing returns. We have
packed this volume with time-tested pearls that will help not only for the Board
exam, but also for everyday clinical practice.
The first Acing book included a wide variety of liver questions. Here we provide
new questions to round out the liver content likely to show up on a Board exam.
We have minimized content overlap between the books to ensure this volume is
unique from its predecessor. Between the two books, we now cover almost every
major topic in liver disease, with a focus on the “tough stuff” liver vignettes you
may not know the answer to (yet).
We’ve made every effort to write a modern, up-to-date liver textbook. The field
of hepatology is constantly evolving; even hepatologists find it difficult to keep
up with management guidelines for viral hepatitis, for example. We have tried to
predict what might be on a Board exam 2 to 3 years from now, but have mainly
kept to the traditional liver content that shows up in Board review. That said,
we’ve not shied away from newer information. For example, if you’re not yet aware
of IL-28B polymorphism genotyping and its impact on hepatitis C management,
then you should go read about it. We have included the use of the IL-28B poly-
morphism genotype test in this book, even though it’s a relatively new discovery.
In essence, we’ve attempted to reach a balance between novel information and
time-tested knowledge, with an emphasis on the latter. We think we’ve hit the
right balance, but you will be the ultimate judge.
Although the first Acing book was a single-author effort, the current book is a
collaborative effort. Joining the project is Hetal Karsan, who brings experience
not only as an academic hepatologist originally trained at UCLA and now prac-
ticing at Emory University, but also as a community-based general gastroenter-
ologist and hepatologist with years of practical experience in a large GI practice
group—Atlanta Gastroenterology Associates. Together, we’ve striven to ensure
that this book maintains an appropriate balance between the academic theory
and everyday clinical reality you need to know for success on the Board exam
and clinical practice.
In the remainder of this Preface, we explain how the Acing approach works, and
how we will try to get you where you need to be without going too far. We’ve main-
tained much of the same wording that was in the preface to the original Acing
book, since the content is just as relevant to hepatology as it is to general gastro-
enterology. Then, in Chapter 1, entitled “Liver Disease on the GI Board Exam,” we
discuss issues specific to hepatology and describe how the current book aims to
focus on liver knowledge of relevance to the general gastroenterologist.
xiv Preface

At this point in your career you know a lot. It’s been a hard-earned battle, but
after years of reading books, sitting through lectures, and working with patients,
you now have a pretty good sense of what is important and what is, well, less
important. You are also busy, and your time is limited. So now that you have to
study for Boards or prepare for a clerkship, your goal is to learn the information
you don’t know, not review the content you already do know.
Yet, for some reason, we all continue to practice an inefficient approach to
studying for Board exams. This usually consists of comprehensively reviewing
the entirety of a topic area without thinking about (a) whether we are adding
incremental information to our pre-existing storehouse of knowledge, and (b)
whether we are learning things that are actually on the examination. Presumably
you have already done the painstaking work of learning the basics of your trade.
Now you have to get down to business and ace a test. Those are two very differ-
ent activities.
Yet the inefficient approach to Board review is perennially fostered by tradi-
tional “Board review” textbooks in which content areas are laid out in chapter-
by-chapter (and verse) detail, fully laden with facts both high and low yield—both
relevant and irrelevant to actual examinations. There’s a time and place for the
chapter and verse approach to learning your trade, but Board review crunch time
probably is not it.
Board review books often present information that is not on the Boards (nor
ever will be on the Boards) with information that is merely of personal interest to
the chapter authors. That is, many Board review books suffer from the affliction
of academia running roughshod over practical information. Such information is
usually prefaced by the standard forerunners, like “Recent data indicate that…”,
or “Our group recently discovered that…”, or “Although there is still a lack of
consensus that…”, and so forth. This kind of information is interesting and
important for so many reasons, but is not relevant for Board review. When you
are in crunch time, you should not have to read about pet theories, areas of utter
controversy (and thus ineligible for Board exams), or brand new or incompletely
tested data that are too immature for Board exams. You need to know about
time-tested pearls that appear year after year—not cutting-edge hypotheses,
novel speculations, new epidemiological oddities, or anything else not yet ready
for prime time. Board exams are all about prime time.
The Acing books are different. They aim for the sweet spot between what you
already know and what you do not already know (or have forgotten)—but that
may be on the Boards. They try to avoid the lower extreme of information you
have known since birth and the upper extreme of academic ruminations that are
great for journal club or staying on the cutting edge, but that sit on the cutting
room floor in Board exam editorial offices. You may find that you do know some
of the content in this book, and if so, that’s great. That means you’re almost
ready for the exam. But you will also find that you do not know (or have forgotten)
much of what is in this book. And that’s the point—you should be reading what
you do not know, not reviewing content you already know well.
The information in this book has been culled from years of clinical practice
and teaching Board review to our gastroenterology fellows at UCLA. We have
come to realize that our fellows, who are among the “best of the best,” know a
lot about their specialty but are not necessarily ready for Boards. That’s because
we purposefully do not teach for Boards during everyday training—we instead
teach the skills and knowledge that support rational and evidence-based deci-
sion making in clinical practice. Unfortunately, Board exams do not always tap
directly into those skill sets. Great clinicians can do poorly on Board exams.
And great test-takers can be suboptimal clinicians. We all recognize that it’s
Preface xv

primarily important to be a great clinician and secondarily important to be a


great test-taker.
But with that caveat, it’s still important to ace the Boards. Acing the Boards
means that you ace not only the stuff you know, but also the “tough stuff” you
may not yet know. This tough stuff tends to recur year after year.
This book consists of a series of “high-yield” vignettes on topics that are peren-
nial Board review favorites—generally on the more difficult side—and full of
pearls that may come in handy at Board time. All of these are originals; none
are from an actual Board exam, naturally. But all have been endorsed, through
an ongoing process of content development with our UCLA fellows and faculty, as
being generally reflective of the content that might appear on GI Board exams. It
goes without saying that we have no idea what will be on your particular Board
exam—and even if we did, we sure as heck won’t give you the answers in a book!
Instead, we can make the more general statement that the content covered in
this book is probably in the ballpark of things you should know to help you on
the exam.
Here are some highlights of this book:
z Focus on clinical vignettes. We see actual patients in clinical practice.
And, to the Board’s credit, most Board questions are clinical vignettes. This
book presents questions in the form of clinical vignettes, not sterile, fact-
laden blocks of text.
z Relatively short. Most Board review books are better suited for arm curls
than for rapidly and effectively teaching their content. Said another way,
they are not “bathroom reading.” Instead, most Board review books are read
at a desk with a highlighter in hand. Unlike traditional didactic volumes,
this book is big-time bathroom reading. You know, you sit down, open it up,
and take in high-yield “tough stuff” in a hopefully entertaining format in
short order. This is not a definitive text for comprehensive Board review, but
a one-stop shop for high-impact content presented in a novel and interactive
way. This book can be used in concert with longer volumes if you are looking
for more extensive topic coverage.
z Focus on stuff you don’t know. The goal of this book is for you to learn
something new on every page, not to rehash what you already know. This
book is relatively short, but it’s dense with material you may not know yet.
And that’s the point—to learn stuff you don’t know yet, not keep reading and
re-reading stuff you’ve known forever.
z Emphasis on pearl after pearl after pearl. Students, residents, fellows,
and even attendings love clinical pearls. And so do the Boards. After every
vignette in this book, there is a pearl explicitly stated at the bottom intro-
duced by the phrase, “Here’s the Point!”
z Random order of vignettes. The Boards present questions in random order,
not in nice, neat chapters. This book is meant to emulate the Board experi-
ence by providing vignettes in random order. It’s a way to introduce cogni-
tive dissonance into your learning by constantly switching directions. After
all, patients appear in random order, so why not Board review material? If
there’s a specific topic you want to review, then you can look in the index
to find the relevant pages. But again, keep in mind that this is not meant
to be a treatise on any single topic, but instead a rapid-fire review of high-
yield content.
xvi Preface

z No multiple-choice questions. Multiple-choice questions are boring. They


often test process of elimination more than knowledge and aptitude. When
we teach Board review, we present a vignette, and then ask, “So what next?”
Or, “What treatment will you give?” It’s more entertaining and it’s more
realistic. A patient who comes into the office doesn’t have a multiple-choice
grid floating over his or her head in a hologram. So we find open-ended
questions to be more engaging and interesting, even if the Boards empha-
size multiple choice. We have no doubt that if you can get these questions
right without multiple choice, you will most definitely get them right with
multiple choice.
z Emphasis on “clinical thresholds.” Many Board questions require that
the test-taker has memorized some numerical threshold value. Like: “If an
echinococcal cyst exceeds XX cm, then the risk of rupture is clinically sig-
nificant and surgery is warranted.” Or, “If the PMN count in ascites exceeds
XX in cirrhosis, then the patient has spontaneous bacterial peritonitis and
must be treated.” And so forth. Throughout this book, we call these values
clinical thresholds. They are emphasized through the vignettes and are
separately cataloged toward the end of the book (pp. 215). The catalog is
a one-stop shop for all the little numerical facts that everyone forgets but
everyone needs to know. We often refer to this list ourselves because it’s so
easy to forget some of these critical threshold values.
z Comprehensive yet parsimonious explanations. Some books provide
multiple-choice questions and give only the letter answer. The ABIM GI
questions are a case in point. Other books provide short explanations. Still
others provide full explanations but with information that isn’t relevant. In
this book we’ve striven to provide comprehensive answers that are also suc-
cinct, emphasizing the key clinical pearls. In other words, we’ve attempted
to give you enough information to understand how to answer the questions
correctly without overwhelming you with additional details. Board review
isn’t about ruminating forever about personal areas of interest—it’s about
cutting to the chase and keeping information on target.
z Avoidance of mind-numbing prose. Too many review books are boring
as heck. They take away our will to live. This book is purposefully written
in a manner that acknowledges that studying for Boards can be painful.
We’ve tried to include interesting vignettes, provide answers that draw from
real-life clinical experience, and avoid unnecessary jargon and excessive
academic descriptions. We also love to use witty humor to keep your atten-
tion!
z Emphasis on images. Clinical medicine is a visual art, and GI and hepatol-
ogy, in particular, are visual subspecialties. The Boards acknowledge this
by including lots of questions with images. Many of the vignettes in this
book are accompanied by carefully selected images designed to “bring the
content to life” and aid in understanding the key points of the case.
This book was greatly enhanced by the feedback and input from current and
former GI fellows at UCLA. We remain especially grateful to Benjamin Weinberg,
who helped develop the title of the original Acing the GI Board Exam book, of
which this is the second in the series. We would also like to thank Barbara Kadell
Preface xvii

(Department of Radiology) and Charlie Lassman (Department of Pathology)


at UCLA for providing key images to complement the text of this book, along
with Alton B. Farris (Department of Pathology) and Bobby Kalb (Department
of Radiology) at Emory University for additional images. These images greatly
enhance the visual appeal and pedagogy of the text, and for that we are greatly
appreciative.
1
LIVER DISEASE ON THE
GI BOARD EXAM
General Observations
As mentioned in the Preface, we have no idea what will show up on your Board
exam. Nor do we have any knowledge about specific vignettes that have appeared
on recent exams. We do, however, know which general content areas seem to
be popular for Board review and which seem to be relatively de-emphasized. Of
course, this may or may not correspond with what shows up on your exam. But
with that caveat, here are some observations about general liver topics that tend
to show up routinely.

Perennial Board Review “Favorites”


z Pregnancy. The Boards seem to love pregnancy. We pointed this out in the
original Acing book and provided a range of vignettes in which pregnancy
affects intestinal and hepatic function. But when it comes to preparing for
the liver part of the exam, learning about pregnant women is absolutely
necessary. So we have peppered this book throughout with the classics.
These include acute fatty liver of pregnancy, HELLP syndrome, viral hepa-
titides in pregnancy, hyperemesis gravidarum, intrahepatic cholestasis of
pregnancy, gallstones in pregnancy, route of delivery, and vertical transmis-
sion of viral hepatitis. When it comes to the Board exam, be sure to know
this stuff.
z Cirrhosis. You just know cirrhosis will be all over the exam. But cirrho-
sis is a broad area, so it’s helpful to focus on particularly high-yield top-
ics within cirrhosis. In particular, be sure to know how cirrhosis affects
the kidney. Common Board review topics include refractory ascites with
an elevated creatinine, type I versus type II hepatorenal syndrome, mixed
cryoglobulinemia with glomerulonephritis in hepatitis C liver disease, and
medication-induced renal failure in cirrhosis. We have included several
Spiegel BMR, Karsan HA.
Acing the Hepatology Questions on the GI Board
1 Exam: The Ultimate Crunch-Time Resource (pp 1-6)
© 2012 SLACK Incorporated
2 Chapter 1

vignettes about these topics (see the “cirrhosis/renal throwdown” in


Vignettes 70-75). Other commonly tested areas in cirrhosis include hepatic
encephalopathy, hepatopulmonary syndrome, specifics of ascites manage-
ment, portal hypertension, management of esophageal varices, and evalua-
tion and treatment of spontaneous bacterial peritonitis.
z Drug-Induced Liver Injury (DILI). With so many drugs, herbals, and other
substances that can affect the liver, it’s virtually a lock to see some DILI
questions on the Board exam. Know what drugs cause microvesicular ver-
sus macrovesicular steatosis (covered in the first Acing book), and be able
to recognize pathognomonic patterns of individual drugs. Classic Board
review topics include the phospholipid-laden lysosomal lamellar bodies
seen with amiodarone hepatotoxicity, antibiotic-induced cholestasis from
trimethoprim-sulfamethoxazole, erythromycin, or amoxicillin-clavulanate,
and autoimmune-type DILI from diclofenac, minocycline, nitrofurantoin,
phenytoin, and propylthiouracil. Classic herbals that can affect the liver
include kava kava, ephedra, mistletoe, and herbal teas. And don’t forget
about acetaminophen. Finally, remember that various illicit drugs can
cause acute liver injury. We have addressed all of these plus many more
in the “DILI Delight” section of the book (see Vignettes 14 to 22) and other
vignettes.
z Metabolic Liver Diseases. It’s almost a guarantee that you will be asked
about Wilson disease, hereditary hemochromatosis (HHC), and α-1 anti-
trypsin (AT) deficiency. Just know these. Be sure to know classic “factlets”
such as treat HHC before hepatitis C (HCV) in HCV-infected patients; low
alkaline phosphatase (ALP) and hemolytic anemia are seen in Wilson dis-
ease; and PAS-positive globules accumulated in hepatocytes are seen in α-1
AT deficiency. We focus on these and many other facts you need to know
in the metabolic liver disease vignettes in this book. For example, you will
find the top 10 facts of Wilson disease in Vignette 36. We have also included
characteristic micrographs of these classics. Images are important, and
this book does not skimp on them.
z Vascular Diseases Affecting the Liver. Be sure to review Budd-Chiari
syndrome, veno-occlusive disease (also called sinusoidal obstruction syn-
drome, or SOS), and nutmeg liver, seen in right heart failure. Also, with
the variety of patients who present with hepatic bruits on physical exam,
it’s important to know the differential diagnosis for hepatic bruits, which
includes acute alcoholic hepatitis, arteriovenous fistula, hepatic artery
aneurysm, and so forth. We’ve got these covered.
z Nutritional Deficiencies. The American Board of Internal Medicine (ABIM)
states that nutritional deficiencies compose a small part of the “general”
category, which makes up 7% of the exam. Thus, you might expect only 1 or
2 questions about nutritional deficiencies. So it’s not necessarily high-yield
to learn all the nutritional deficiencies from the standpoint of total ques-
tions, but it is high-yield from the standpoint of the likelihood that these
deficiencies will show up on the exam, even if in small numbers. Perennial
Board review favorites include zinc deficiency, biotin deficiency, and seleni-
um deficiency (cardiomyopathy). If you’re in crunch time, it’s almost a sure
bet that spending 20 minutes on the deficiencies will yield at least a couple
Liver Disease on the GI Board Exam 3

of correct answers. We have included a section called “Vitamin Power!”


which reviews major vitamin deficiencies with a focus on fat-soluble vita-
min deficiencies commonly seen in cholestatic liver diseases (see Vignettes
52-55).
z Dermatological Manifestations of Liver Diseases. This is a favorite
Board review topic because it reminds all of us that we are internists first
and subspecialists second. Moreover, it’s easy to test this material because
the content is set up for picture images. It’s a high-yield activity to review all
the major liver-dermatology links, including migratory necrolytic erythema
in glucagonoma, porphyria cutanea tarda in HCV, and skin rashes from
interferon and ribavirin therapy.
z HCV Infection. This is one example where the prevalence of the disease in
the community appears to be matched by its prevalence on the examina-
tion. As with hepatitis B virus (HBV) treatment, HCV therapy remains a
rapidly changing area, so you will generally be held accountable for older,
well-supported facts, not recent innovations or cutting-edge data from the
newest combination therapies (with some exceptions, noted in the book). Be
sure that you can interpret responses to combination therapy with inter-
feron and ribavirin, including knowing the definitions for rapid virologic
response (RVR), early virologic response (EVR), sustained virologic response
(SVR), slow responder, nonresponder, and null responder. We’ve got these
definitions covered (see Vignette 64). We have also added a vignette covering
prime-time information regarding the IL-28B genotype and how it predicts
response to interferon and ribavirin therapy in genotype 1 HCV. We think
this is important to know for the exam, even if it’s relatively new.
z HBV Infection. Because you probably memorized the HBV serotype pat-
terns back in medical school, we have not included a vignette on basic sero-
logic patterns. But to succeed on the GI Board exam, you will need to know
more nuanced details about HBV. Be acutely aware of how to manage HBV
in pregnancy, what the differences are between various HBV genotypes,
and how to manage acute HBV liver failure. Know the relationship between
HBV and polyarteritis nodosa and between HBV and hepatocellular cancer.
Learn about flares following anti-tumor necrosis factor (TNF) therapy or
steroid administration. The treatment algorithms for HBV seem to vary all
the time, and the existing guidelines are themselves inconsistent, so you
should not be asked too much about treatment algorithms. But you should
at least understand the impact of HBeAg status and how that might affect
duration of treatment.
z Nonviral Liver Infections. When studying for the Board exam, it’s easy
to be lulled into the siren song of viral hepatitis. While you should study
viral hepatitis carefully because it’s very likely to show up on your exam,
it’s also important to focus on a number of other liver infections. Some
classics include salmonella hepatitis (think temperature-pulse differential,
rose spots, and a stepwise fever along with hepatitis), fascioliasis (a nasty
disease, with an awesome case described in this book), echinococcal liver
cysts (think sheepherder with “eggshell calcifications” on a liver cyst),
schistosomiasis, clonorchiasis, and Entamoeba histolytica abscess (think
“anchovy paste”). There are Board buzzwords for all of these conditions, and
4 Chapter 1

it will probably be worth it for you to link the buzzwords with the diagnoses.
We will cover these topics throughout the book. But in the meantime, try to
name the nonviral infectious diagnosis for each of the following buzzwords:
Katayama fever, transverse myelitis, anaphylaxis after cystic rupture,
undercooked fish, cyst filled with hydatid sand, tortuous tracks in liver on
CT, contaminated freshwater plants, portal vein granulomas with fibrosis. If
some of these have you stumped, stay tuned—all of these are covered in the
vignettes, and we have pulled them together in a table of Board buzzwords
with their associated conditions (see Table 44-1). That table is gold!

Generally De-Emphasized Topics


In contrast to the topics just discussed, other areas of hepatology have histori-
cally been de-emphasized in Board review. Of course, that does not mean that
next year’s exam won’t be filled to the brim with these topics. And furthermore,
“historically de-emphasized” does not mean absent from the exam altogether—
just de-emphasized compared to other topics. Here are some topics that we think
will probably get relatively short shrift on the exam (but who knows...?).

z Liver Transplantation. Hepatology is an exploding field, and the GI Board


exam heavily emphasizes this field. In fact, the ABIM states that 1 in 4
Board questions pertains to hepatology. So, if you are not a big “liver fan,”
now is the time to become one because you have to know liver. But, there is
a fine line between the hepatology that a general GI should know and the
specialized knowledge that a full-fledged hepatologist should know. This
is not a test for budding transplant hepatologists. There is a move afoot to
better define the hepatology curriculum, and there is now a separate ABIM
examination for transplant hepatology. Liver transplantation is a very spe-
cialized area that is generally reserved for “official” transplant hepatologists.
It’s not expected that community gastroenterologists know how to manage
the intricacies of post-transplant patients. Pre-transplant issues, such as
staging with the Model for End-Stage Liver Disease (MELD), are relevant to
practicing GIs and are also on the Board examination. However, periopera-
tive and postoperative transplant topics have not been heavily emphasized
on the general GI examination. If you’re in crunch time, it’s probably low
yield to start learning those topics, particularly if you have not already
been exposed to post liver transplant patients in much detail. But know the
basics: What is MELD? How is MELD used? What are the indications and
contraindications to liver transplant? What are the criteria for hepatocel-
lular carcinoma (HCC) and transplant eligibility? What is the most common
de novo malignancy in the long term after liver transplantation (no, not
lymphoma...)? and so forth. If you can answer these basic questions, then
you should be well on your way to getting the basic transplant-related ques-
tions correct on the Boards. We covered these topics in the first Acing book
and have added some additional vignettes in this book to round out these
areas.
z Nonbiliary Radiology. You need to know basic cholangiograms, but the
Board examiners should hopefully recognize that you are a gastroenter-
ologist—not a radiologist. Even though GI and hepatology rely heavily on
Liver Disease on the GI Board Exam 5

abdominal imaging in everyday clinical practice, it is expected that you will


be working with radiologists when interpreting images. So, on the Board
exam you should rarely, if ever, be given a radiograph without supporting
information. There should be enough information in the vignette for you to
piece together what is happening in the image, even if you cannot actually
interpret the image. It’s still useful to be able to recognize basic radiographic
patterns (eg, ascites, subcapsular hepatic adenoma, spoked-wheel pattern
of focal nodular hyperplasia [FNH], the difference between FNH and fibrola-
mellar hepatocellular carcinoma, and hemangiomas), but at this point you
pretty much know what you know when it comes to liver imaging. It will be
generally low yield to spend Board review crunch time carefully studying
different CT, MR, or ultrasound images. If you are going to study images,
we recommend spending time reviewing cholangiograms. For nonbiliary
images, you should be able to rely on your general knowledge about imaging
and couple that with your specific knowledge about liver diseases, since—as
noted earlier—the accompanying vignettes should have sufficient data to
allow correct answers even if you can’t precisely interpret the provided
image. With that said, we have included a wide variety of classic images in
this book for several liver disorders (reprinted with permission from Barbara
Kadell from UCLA, possibly the best GI radiologist to ever walk the Earth.
Seriously, she is unmatched... once, she diagnosed breast cancer just by
looking at the cecum of a KUB, and was dead right!). If you can interpret the
images included in this book, then you should be well on your way to rec-
ognizing most of the major liver radiographs that might show up on a Board
exam. We worked carefully with Dr. Kadell to select some of the highest-yield
images we could think of. You will see them scattered throughout the text.
z Pathology Interpretation. Pathology interpretation is a fundamental skill
for practicing gastroenterologists and hepatologists. Every biopsy we take
needs pathologic interpretation, and the results almost always have impor-
tant clinical implications. But at the same time, we are gastroenterologists
and hepatologists, not pathologists. Similar to the radiology discussion, it’s
generally the case that Board questions tied to pathology results will provide
sufficient information to answer the question without having to precisely
interpret the histopathology. Undoubtedly, you need to know some basic
patterns of histopathology: interface hepatitis with plasma cells in autoim-
mune hepatitis, “onion skinning” in primary sclerosing cholangitis (and
other chronic cholestatic conditions), “florid duct lesions” in primary biliary
cirrhosis, Mallory bodies, hepatocyte ballooning and portal inflammation
in nonalcoholic fatty liver disease, among many others. But in each of these
cases, the exam should provide concurrent clinical information, and you
should be able to piece together the answer without being an expert in histo-
pathologic interpretation. In situations where you are called upon to directly
interpret a pathology slide, chances are the abnormality will be right
under your nose and “classic”—not some strange variation. Leave it to the
pathologists to be experts in interpreting fine degrees of separation between
conditions. You should know the basics and be able to tie those basics to
clinical information. We expect the Board will focus on the pathognomonic
classics when it comes to requiring histopathologic interpretation. You will
find many classic liver micrographs featured in this book (reprinted with
6 Chapter 1

permission from Dr. Charles Lassman from UCLA and Dr. Alton B. Farris
from Emory University—both outstanding teachers and top liver histopa-
thology experts)—we think we’ve got almost all of the classics covered.
2

“TOUGH STUFF” VIGNETTES

In the pages that follow are 95 “tough stuff” vignettes. As described in the
Preface, these have been culled from years of teaching Board review and prac-
ticing gastroenterology and hepatology and have been iteratively reviewed and
vetted with our fellows at UCLA. As you go through these vignettes, keep the
following points in mind:
z These are generally difficult. That is by design. You may nonetheless know
the answers to many of the questions in these vignettes—a sign that you
are well prepared for the exam. But if you cannot get them all right, that is
fine, too. That’s the whole point of this book—to ensure that you’re gaining
incremental information, not just reviewing content you already know. Keep
in mind, however, that for every tough question that’s in this book, there
will be a bunch of “gimmies” on the exam. The entire Board exam won’t be
full of “tough stuff” questions. So don’t get demoralized if you can’t answer
all of the questions in this book correctly. Rest assured that you already
know most of the “gimmies” just by virtue of paying attention and learning
during your clinical experiences.
z These are in completely random order—there is no explicit rhyme or reason.
See the Preface for our rationale for this setup.
z The vignettes appear on one page, followed by one or more open-ended
questions. The answers are provided on the next page. Before you turn the
page, take a moment to really think about the answers. Even if you’re not
sure of an answer, at least take a moment to think about the potential dif-
ferential diagnosis, or other information you might need to better answer
the question. This form of active learning is more useful than merely flip-
ping the page and reading the answer. Seriously… don’t just flip the page
until you’ve given the vignette at least a nanosecond of thought. The answer
will be more meaningful if you’ve first struggled a bit to think through the
vignette.
Spiegel BMR, Karsan HA.
Acing the Hepatology Questions on the GI Board
Exam: The Ultimate Crunch-Time Resource (pp 7-214)
7 © 2012 SLACK Incorporated
8 Chapter 2

z After each answer there is a short section entitled “Why Might This Be
Tested?” The purpose of this discussion is to emphasize why it’s important
to know the content of the vignette, vis-à-vis the Board exam in particular.
It puts you in the mind of the Board examiners to better understand their
potential reasoning, which might help you better remember the vignette.
z At the bottom of each answer page there is a box entitled “Here’s the Point!”
This summarizes the key issue or issues that appear on the page. If you are
really in crunch time, at the very least make sure you know the “Here’s the
Point!” bottom line for each vignette. The “Crunch-Time Self-Test” on page
221 catalogs all of these factlets (and more) into a 135-question test that
quizzes you on the key points from each vignette.
z Some of the answer pages also have a “Clinical Threshold Alert,” followed by
the presentation of an explicit clinical threshold (see the Preface for details).
Sixty of these clinical thresholds are cataloged on page 215 for your conve-
nience during crunch time.
“Tough Stuff” Vignettes 9

Vignette 1: Severe “Transaminemia”


A 50-year-old man presents to the emergency room with a chief complaint of
malaise. The following laboratory values are noted:

AST = 1200
ALT = 2100
LDH = 300

` Without knowing anything else, what is the most likely diagnosis?


` Why?
10 Chapter 2

Vignette 1: Answer
This is acute viral hepatitis. You might wonder how 3 lab values and a single
symptom would clinch this diagnosis. If so, it’s worth taking some time to break
down this short yet demonstrative vignette. It turns out that there’s enough infor-
mation here to narrow the diagnosis to acute viral hepatitis with a high degree
of reliability. You certainly can’t pinpoint the type of virus with this information
alone, but you can at least posit an underlying acute viral infection.
First off, it’s important to note that the serum aminotransferases, AST and
ALT, are extremely high—both above 1000 U/L. This degree of “transaminemia”
(not, by the way, “transaminitis,” which literally means “inflammation of the
transaminases” and makes no sense despite the term’s deep penetration into
common parlance) can occur only from a short list of conditions. When the AST
and ALT are in the 1000+ range, think about the following potential causes:
(1) acute viral hepatitis, (2) acute drug or toxin injury, and (3) shock liver. On
occasion, an acute flare of autoimmune hepatitis or an acute bile duct obstruc-
tion (ie, common duct stone) can lead to aminotransferase levels over 1000, but
that would be relatively uncommon. A wide range of other conditions can also
cause hepatitis, but not with this degree of marked transaminemia.
A classic trick question is to describe an alcoholic who presents to the emer-
gency department with an AST of, say, 2000 and an ALT of 1000, and then
ask you to make the diagnosis. The usual first temptation is to diagnose acute
alcoholic hepatitis and call it a day. Indeed, every med student knows that acute
alcohol poisoning can present biochemically with an AST:ALT ratio of 2:1 (more
on alcohol injury in Vignette 26), but not with enzymes in the 1000+ range. In
fact, it’s highly unusual for the AST to exceed 400 to 500 in the setting of pure
alcoholic liver disease without some other hepatotoxin on board. So in an alco-
holic patient who presents with a 2:1 ratio of AST to ALT but with liver enzymes
in the 1000+ range, think about something else—like underlying acetaminophen
toxicity in addition to acute alcoholic hepatitis.
Now, with a little more information, you could start to distinguish between
underlying etiologies for severe transaminemia. For example, if the patient were
in the intensive care unit and recently suffered a bout of pronounced hypoten-
sion, then shock liver would lead the differential diagnosis. If the patient had
just eaten shellfish and turned yellow, then acute hepatitis A might be high on
the list. If the patient had recently started a new herbal remedy recommended by
a homeopathic healer, then toxin-mediated injury might be the diagnosis. If the
patient had just consumed half a bottle of, say, Vicodin or Percocet, then acet-
aminophen injury would be highly suspected. If an obese, 40-year-old woman
presented with acute biliary colic, then a common bile duct stone would be con-
sidered. And so forth. But you do not have any of that information here; this is
a bare-boned question.
So, that should lead you to evaluate the LDH level. Recall that LDH has several
forms and arises from not only the liver but also the heart, reticuloendothelial
system, lungs, kidney, and striated muscles. So, it’s all over the place and not
very sensitive for liver disease, in particular. And it comes in 5 isoforms that,
for the most part, are not routinely fractionated in everyday clinical practice.
But the bottom line is that LDH is often included in metabolic panels and the
information it provides is, on occasion, quite helpful. The LDH is elevated in this
“Tough Stuff” Vignettes 11

case, although it’s not inordinately high. How can that help you narrow the diag-
nosis? The key is to evaluate the ratio of ALT to LDH. Serum LDH is especially
high in the setting of hypoxemic liver injury (“shock liver”) and in both toxin- and
drug-induced liver injury. In these situations, both the aminotransferases and
the LDH are high, with the LDH disproportionately high compared to the already
high ALT. The ALT:LDH ratio is therefore low. In acute viral hepatitis, in contrast,
LDH levels are not as high, so the ALT:LDH ratio tends to be much higher. In
the original validation study of this concept by Cassidy and Reynolds (see the
Bibliography in the back of this book), the mean ALT:LDH ratio for acute viral
hepatitis was 4.65. In contrast, the ratios for hypoxemic liver injury and acet-
aminophen injury were 0.87 and 1.46, respectively. A cutoff of 1.5 distinguished
acute viral hepatitis from hypoxemic, toxin-induced, and drug-induced hepatitis
with a sensitivity of 94% and a specificity of 84%.
In the current vignette, the ALT:LDH ratio is 7.0. The high ratio argues strong-
ly against hypoxemic, toxin-induced, and drug-induced liver injury, all of which
tend to disproportionately elevate the LDH. That leaves acute viral hepatitis as
the most likely explanation for the biochemical pattern. Fatigue is also support-
ive of viral hepatitis but is by no means specific. Although acute autoimmune
hepatitis and bile duct obstruction can also drive the aminotransferases into the
1000+ range, acute viral hepatitis would be much more likely. When all of these
factors are considered, it’s reasonable to conclude that acute viral hepatitis is the
best explanation for the marked transaminemia.
While we’re on the topic of aminotransferases, it’s worth reviewing a few other
useful factlets about their use in diagnosing and managing liver disease. First,
the degree of transaminemia is not predictive of the level of hepatic necrosis.
That is, although aminotransferase levels in the 1000+ range can be intimidat-
ing, they do not necessarily portend a bad prognosis in and of themselves. This
highlights the fact that ALT and AST are liver enzymes but are not liver function
tests, or LFTs. For some reason ALT and AST are typically referred to as “LFTs”
when, in fact, they do not measure liver function at all. They indirectly measure
hepatocyte damage, but they do not provide insight into the functional deficit
engendered by that damage.
This leads to the second point, which is to distinguish the aminotransferases
from true LFTs, including albumin, INR, and total bilirubin. These lab values
provide direct evidence of liver function and true prognostic information; they
can predict, quite literally, whether a patient with advanced liver disease will live
or die without a transplant. The aminotransferases simply cannot provide this
information and are therefore not LFTs at all.
Third, it’s notable that while the absolute aminotransferase levels do not pro-
vide prognostic information, their rate of decline may. In severe cases of acute
liver injury, the ALT and AST values may rise quickly and subsequently fall even
faster. This suggests a rapid burnout of the liver from overwhelming necrosis.
When coupled with an abrupt rise in bilirubin and INR, a rapid drop in amino-
transferase levels portends a bad prognosis. But that’s enough factlets for now.
One final point in passing—the term aminotransaminases is now favored over
the term transaminases, since it more accurately describes the function of the
enzymes. But that will not be on a Board exam. And, quite frankly, we tend to
switch back and forth when we use the terms.
12 Chapter 2

Why Might This Be Tested? For the general gastroenterology exam, you will
not be expected to know everything about hepatology, which is a burgeoning
field. But you should certainly be able to interpret basic patterns in liver tests.
Marked transaminemia is perfect for a Board question because the differential
diagnosis is quite narrow, so the choices are limited. A Board question will likely
give more information than what we have provided here, but knowing the rela-
tionship between ALT and LDH could help you answer a question more efficiently
and reliably without getting too waylaid by the inevitable red herrings test writers
like to pepper into vignettes. That was a mixed metaphor, but who cares? We’re
doctors, not English professors.
Clinical Threshold Alert: If the ALT:LDH ratio exceeds 1.5 in the setting of
severe transaminemia, think acute viral hepatitis. If the ratio is lower, think
about toxin-induced, drug-induced, or hypoxemia-induced liver injury.

Here’s the Point!

When the aminotransferases enter the 1000+ range, think:


z Acute viral hepatitis
z Shock liver
z Drug- or toxin-induced liver injury

Here’s the Point!

The ALT and AST are not LFTs. The “real” LFTs include:
z Total bilirubin
z Albumin
z INR
“Tough Stuff” Vignettes 13

Vignette 2: Bloated With Bleeding Gums


A 46-year-old woman with morbid obesity had been successfully losing weight
while on a monitored dietary regimen over the past year. However, she continued
to experience abdominal bloating and increased abdominal girth despite the
overall weight loss. Her primary care provider noted that all of her blood tests
were normal on routine annual physical examination last month, but ordered
an abdominal MRI to further evaluate the progressive abdominal bloating
(Figure 2-1).

Figure 2-1. Abdominal MRI for the patient in Vignette 2.


(Reprinted with permission of Barbara Kadell, MD, UCLA
Medical Center.)

The patient subsequently presented to the emergency department with com-


plaints of bleeding gums for the past week associated with a further increase in
abdominal girth and pain, along with progressive jaundice. There was no history
of trauma. Laboratory tests revealed the following: total bilirubin = 4.9, ALT = 51,
AST = 62, INR = 2.2, hemoglobin = 7.2, platelets = 40,000, fibrinogen = 43 mg/dL
(low), fibrin degradation products = 60 μg/mL (high), aPTT = 72 seconds (high).
The patient received transfusions of various blood products and underwent emer-
gent surgery, which resulted in resection of a large liver tumor (Figure 2-2).

Figure 2-2. Large liver tumor resected from the patient in


Vignette 2 (wow!). (Reprinted with permission of Steven
Hanish, MD, Emory University.)

` What is the diagnosis?


14 Chapter 2

Vignette 2: Answer
This is Kasabach-Merritt syndrome (KMS) associated with a giant (ie, humon-
gous) cavernous hemangioma. KMS is a consumptive coagulopathy that can lead
to disseminated intravascular coagulopathy (DIC). The syndrome was originally
described in 1940 in a young child with a rapidly growing cutaneous heman-
gioma. KMS has also been recognized as a rare complication of giant hepatic
hemangiomas. In this case, resection of the giant hemangioma cured the KMS-
associated clotting and fibrinolysis.
Cavernous hemangioma is the most common benign tumor of the liver and
has been found in nearly 10% of autopsies—so you may have one right now with-
out knowing it. Hemangiomas are more common in women than men. They are
usually noted incidentally and rarely require treatment. However, some of these
can enlarge, and they are given the designation “giant” when they grow to more
than 5 cm in diameter. These giant hemangiomas can lead to symptoms, includ-
ing chronic pain from stretching Glisson’s capsule, massive hepatomegaly, early
satiety from gastric compression, jaundice from biliary compression, vascular
compression, DIC, and even rupture! Indeed, a spontaneous rupture is a rare
occurrence in giant hemangiomas, but it can have a disastrous outcome with a
nearly 40% mortality rate. Treatment options for symptomatic giant hemangio-
mas include resection, transarterial embolization (enucleation), and even liver
transplantation.
On gross examination, giant hemangiomas have a sponge-like, purplish
appearance. Just look at that thing—it’s an amazing lesion. Furthermore, hem-
angiomas may have regions of calcification, scarring, and intraluminal throm-
bosis. On microscopic examination, there are networks of blood-filled vascular
spaces separated by thin, fibrous stroma (Figure 2-3).

Figure 2-3. Micrograph of hepatic hemangioma demon-


strating networks of blood-filled vascular spaces separated
by fibrous stroma. (Reprinted with permission of Charles
Lassman, MD, UCLA Medical Center.)

Because there is a significant risk of bleeding with biopsy of this type of


lesion (obvious just by looking at the micrograph), the diagnosis of hemangioma
is usually made on the basis of imaging alone without biopsy. On ultrasonog-
raphy, these tumors appear as hyperechoic lesions with smooth margins. On
“Tough Stuff” Vignettes 15

contrast-enhanced CT or MRI, hemangiomas are well-circumscribed lesions with


a characteristic progressive centripetal (from the periphery to the center) fill-in.
If the diagnosis is still unclear after these imaging studies are conducted, then
a technetium-labeled red blood cell nuclear scan can be employed as a noninva-
sive test to help clinch the diagnosis. With this scan, there should be increased
uptake of the isotope within the lesion during the venous phase with retention
on delayed images. Following diagnosis of a giant hemangioma, it’s reasonable
to perform follow-up imaging—especially if the lesion is subcapsular—since this
location confers a higher risk of bleeding upon tumor enlargement. Otherwise,
stable and asymptomatic lesions do not necessarily need to be followed.
Why Might This Be Tested? Since hemangiomas are the most common
hepatic tumor, you should know their characteristics. Furthermore, you need
to keep anatomic considerations in mind when determining a surveillance and
treatment plan—good fodder for a Board exam. Incidental liver lesions are one of
the most common reasons for consultation requests (either formal or curbside)
that gastroenterologists will encounter, so you need to know about all the com-
mon liver tumors that arise. We cover most of these within this volume and the
previous Acing book.

Here’s the Point!

DIC + Giant hemangioma = Kasabach-Merritt syndrome


16 Chapter 2

Vignette 3: Isolated Fundic Varices


A 62-year-old man with chronic alcoholic pancreatitis presents with 4 hours of
large-volume hematemesis that began abruptly after drinking several beers. The
patient does not have a history of cirrhosis. After stabilization in the emergency
department, the patient undergoes urgent upper endoscopy, which reveals large
varices in the fundus of the stomach (Figure 3-1). No esophageal or gastroesoph-
ageal junctional varices are noted.

Figure 3-1. Fundic varices. (Reprinted with


permission of Dennis Jensen, MD, UCLA
Medical Center.)

` What might explain these endoscopic findings?


` How should this be treated?
“Tough Stuff” Vignettes 17

Vignette 3: Answer
These are isolated fundic varices resulting from an underlying splenic vein
thrombosis from chronic pancreatitis. This vignette provides an excuse to review
the anatomy of the portal circulation, which you need to understand in order
to know why and how isolated fundic varices can form. Figure 3-2 provides a
simplified depiction of the portal circulation—refer to this figure as you read the
text that follows.

Figure 3-2. Portal circulation and related anatomy.

Recall that the portal circulation flows into and out of the liver via the portal
vein and hepatic vein, respectively. The hepatic vein joins the inferior vena cava
to return blood to the heart for recirculation. On the other side of the liver, the
portal vein is formed by the confluence of the superior mesenteric vein and the
splenic vein. The esophageal vein branches off the portal vein and heads toward
a venous plexus surrounding the gastroesophageal junction. In the setting of
cirrhosis, where there is intrinsic liver disease marked by portal hypertension,
blood cannot push its way through the portal circulation, causing hepatofugal
flow (ie, flow away from the liver). Backflow typically occurs when the hepatic
venous pressure gradient (HVPG) exceeds 12 mm Hg, at which point the back-
flow starts working its way up the esophageal vein and back down the portal
vein. This, in turn, causes the plexus of vessels around the esophagus to swell,
leading to the formation of esophageal and gastric junctional varices.
18 Chapter 2

Isolated fundic varices (ie, without junctional or esophageal varices) do not


typically occur in the setting of cirrhosis unless some other process is also pres-
ent. They may occur along with esophageal or junctional varices, but they do
not occur in cirrhosis without other sites concurrently involved. To explain how
isolated fundic varices could form, we need to posit an obstruction that is proxi-
mal to the takeoff of the esophageal vein, causing sinistral, or left-sided, portal
hypertension. This is where splenic vein thrombosis comes in. The splenic vein,
which joins with the superior mesenteric vein to form the portal vein (see Figure
3-2), runs right along the posterior length of the pancreas. The splenic vein can
serve as the bellwether for pancreatic trouble. Splenic vein thrombosis is most
common in the setting of either pancreatic cancer or chronic pancreatitis. In
both instances, there can be compression and/or stasis of the splenic vein, which
leads to secondary thrombosis. When the splenic vein is thrombosed, the blood
backs up into the spleen, which in turn becomes swollen and enlarged. This fills
the short gastric veins, which connect the spleen with the stomach. This filling,
in turn, may lead to isolated fundic varices in the stomach. Since the vascular
obstruction is well proximal to the takeoff of the esophageal vein, there is no
reason to expect esophageal varices. The history of chronic pancreatitis in this
patient raises splenic vein thrombosis as the most likely etiology.
Treatment of gastric varices can be difficult. Unlike esophageal or junctional
varices, gastric varices are not easily amenable to endoscopic therapy. Band liga-
tion and sclerotherapy may temporize active bleeding, but they don’t seem to hold
for long. Cyanoacrylate glue has proven effective but is currently unavailable in
the United States and should not be an answer choice on an ABIM Board exam.
Intravariceal injection of thrombin has also been reported to be effective, but it
too is not standard of care and therefore unlikely to appear on a Board exam.
Octreotide can help drop the portal pressure in the acute setting, but it’s not
a long-term fix for this problem. It would make more sense to fix the underly-
ing problem rather than troubleshoot the varices directly, which are essentially
vascular epiphenomena of a more fundamental disorder—in this case, the pre-
sumed splenic vein thrombosis from chronic pancreatitis. So that leads us to
the ultimate treatment, which is splenectomy. Splenectomy is curative for most
patients with isolated fundic varices due to splenic vein thrombosis. If, however,
the splenic vein thrombosis is coupled with diffuse thrombosis in the portal and
mesenteric veins, then splenectomy would not fix the whole problem.
In cases of severe and recalcitrant bleeding from underlying complex throm-
bosis involving several vessels, it may be necessary to perform surgery. There
are two basic surgical approaches—shunt and nonshunt operations. Surgical
shunts come in many types, but the ultimate goal is to decompress the portaca-
val circulation and relieve the backflow. The splenorenal shunt can work in some
cases. If the thrombosis is particularly complex, then nonshunt operations may
be necessary. A classic nonshunt operation is the Sugiura procedure, in which
the surgeon devascularizes the entire gastroesophageal junction in an effort to
take down all the varices. This is rarely performed anymore and is clearly an
option of last resort due to the high mortality rate with the surgery itself. Perhaps
even more extreme is complete esophageal transection, although this is really
directed toward esophageal, not gastric fundic, varices (but worth mentioning
for the sake of completeness). Surgical management of bleeding varices remains
unusual and mostly limited to centers with considerable experience with these
high-risk procedures.
“Tough Stuff” Vignettes 19

Why Might This Be Tested? Presenting a case of isolated fundic varices is


a good way to test your knowledge of vascular anatomy. And since knowing the
anatomy has obvious clinical indications, this is a case where remembering your
Netter atlas from med school can really come in handy.
Clinical Threshold Alert: If the HVPG exceeds 12 mm Hg, then varices begin
to form.

Here’s the Point!

Isolated fundic varices in the setting of pancreatitis = Splenic vein thrombosis.


Consider splenectomy for treatment.
20 Chapter 2

Vignette 4: Rave
A 20-year-old college student is brought by friends to the emergency depart-
ment with complaints of nausea, vomiting, and anxiety. He has no significant
past medical history, no recent travel, and no unusual food consumption. He
denies alcohol use, but his friends mention that he frequently goes to “raves.”
He is noted to be anxious and diaphoretic in the emergency department with a
temperature of 104°F.
On examination, he is jaundiced and has tender hepatomegaly. He is also
noted to have jaw-clenching and bruxism (teeth-grinding). A lumbar puncture
is performed and is unremarkable. Blood tests reveal the following: INR = 1.5,
ALT = 1590, AST = 1440, total bilirubin = 4.2, creatinine = 1.0, WBC = 8.1, hemo-
globin = 14.2, platelets = 190, CK = 50, pH = 7.40; blood cultures are pending.

` What is the most likely diagnosis?


` How can this diagnosis be confirmed?
“Tough Stuff” Vignettes 21

Vignette 4: Answer
This is a case of methylenedioxymethamphetamine (MDMA, also known as
“ecstasy”)-induced hepatotoxicity. If you have a college student presenting with
hyperthermia, anxiety, nausea, vomiting, cramping, and muscular rigidity with
bruxism and diaphoresis, think of illicit drug use. That seems like a no-brainer,
but don’t miss it. If the symptoms coincide with acute liver failure, then think
of recreational use of ecstasy. The diagnosis can be confirmed with most urine
toxicology screening tests.
The pathogenesis of acute MDMA-induced liver injury is multifactorial.
Immune mechanisms and reactive metabolites play a role. When combined
with hyperthermia, these metabolites can accelerate hepatocellular injury (see
Vignette 41 for more on hyperthermia and liver injury). In fact, hyperpyrexia
may persist for several hours and patients can become dehydrated, often with
complaints of polydipsia. These patients require aggressive cooling, rehydration,
and supportive care, and they need to be watched closely for evidence of acute
liver failure.
MDMA was first compounded about 100 years ago, but its psychotropic effects
were not known until the 1970s when it was used in psychotherapy (until becom-
ing illegal). In the 1980s, MDMA became popular in some nightclub scenes. Soon
thereafter, it became—and still is—a common recreational drug in the “rave”
culture among adolescents and young adults in the Western world. MDMA is
also prevalent on college campuses, where many students hold the misconcep-
tion that it’s a safe drug.
Why Might This Be Tested? MDMA-induced acute liver failure is a well-
known problem in young adults using recreational drugs. Public education
campaigns are starting to raise better awareness that ecstasy is not a safe drug.
Board examiners will expect you to be informed as well.

Here’s the Point!

Hyperthermia + Acute hepatitis + College student + Bruxism = Think “ecstasy”


22 Chapter 2

Vignette 5: Right Upper Quadrant Pain in a Young Woman


A 19-year-old college student presents to the student health clinic with a
chief complaint of abdominal pain for 3 days, along with progressive fevers and
sweats. The pain is constant, sharp, 6 out of 10 in intensity, and located in the
right upper quadrant. The pain is worse with deep inspiration and radiates along
her right flank and thorax, but not to her right scapula or shoulder. She feels
nauseous but has not vomited. She does not report changes in stool frequency or
form, has not experienced GI bleeding, and reports no skin rashes or jaundice.
Her past medical history is unremarkable, and she does not take medications.
She does not report using illicit drugs or herbal preparations. She drinks alcohol
socially but reports no intake over the past week. Her last menstrual period was
2 weeks prior to presentation.
On examination, she is febrile with a temperature of 100.9°F. She is not jaun-
diced and shows no stigmata of chronic liver disease. Her lungs are clear bilater-
ally. Her abdomen is diffusely tender with voluntary guarding but no rebound
tenderness. The abdominal tenderness is most prominent in the right upper
quadrant but is also present in the bilateral lower quadrants. Rectal examination
reveals brown, heme-negative stool.
She is transferred to the emergency department of the University Hospital,
where she undergoes further evaluation with laboratory testing, which reveals
the following: WBC = 14 with 92% PMNs, hemoglobin = 13.4, platelets = 212,
creatinine = 1.2, AST = 161, ALT = 189, ALP = 56, total bilirubin = 1.3, INR = 1.1,
albumin = 3.4, amylase/lipase = normal, beta-HCG = negative, informal bedside
right upper quadrant ultrasound is negative.

` What is the next diagnostic step?


` Why?
“Tough Stuff” Vignettes 23

Vignette 5: Answer
This vignette is meant to trick you. If you said the next diagnostic step should
be abdominal CT scanning or some other form of diagnostic imaging, then you
are wrong. Of course, some kind of imaging may need to be done eventually in
this case, but it should not be the very next step.
The correct answer is to perform a pelvic exam. In GI and hepatology we some-
times tend to forget that the pelvis exists, and that can spell trouble—especially
in a young woman with abdominal pain. Testing the beta-HCG is a nice start,
but that is no substitute for a proper pelvic examination. If you were to perform
a pelvic exam on this woman, you would find cervical motion tenderness and
adnexal tenderness with bimanual examination. That’s because this woman had
pelvic inflammatory disease (PID) from Chlamydia and has secondary perihepa-
titis as a consequence of intra-abdominal spread of the infection.
The complex of PID and perihepatitis is also called Fitz-Hugh–Curtis syn-
drome. Fitz-Hugh–Curtis was first described with gonococcal salpingitis, but it
can also occur with Chlamydial infections. The pelvic infection spreads to the
abdomen, involves the peritoneal surfaces, and consolidates along the perito-
neal reflections in the right upper quadrant around the liver. This gives rise to
a purulent, fibrinous exudate around the liver, which looks like so-called violin
string adhesions (Board buzzword) on direct visualization. This presents clini-
cally with severe right upper quadrant abdominal pain. In fact, the abdominal
pain can become so severe that it dominates the presentation, even though the
source of infection is in the pelvis itself. In the case described in this vignette,
all eyes were focused on the upper abdomen rather than the pelvis, to the point
of bypassing an otherwise vital pelvic examination. The pain can be pleuritic, as
seen here. Because it typically involves the anterior surface of the liver, it may
not radiate to the shoulder, as seen with biliary colic (although it can radiate
to the shoulder). Here the pain radiated around the flank and was worse with
breathing—signs of pleurisy. In addition, Fitz-Hugh–Curtis can present with
elevated aminotransferase levels in about half of cases.
The bottom line is that in a young woman with right upper quadrant pleuritic
pain, fevers, elevated WBC, and elevated liver tests, you need to think about
Fitz-Hugh–Curtis and perform a pelvic examination. The CT can wait. In fact,
the CT findings in Fitz-Hugh–Curtis are often nonspecific vis-à-vis the liver
itself, although the pelvic portion of the test should find some changes sugges-
tive of PID. The CT may reveal hepatic capsular enhancement along the anterior
surface of the liver in the arterial phase. In any event, a reasonable case can be
made for holding on the CT, sparing this young patient the radiation exposure,
and instead starting aggressive IV antibiotic therapy for PID as an inpatient
while monitoring her carefully for clinical decrements. Would she really escape
the emergency room without having a CT performed? Probably not. But don’t let
that hold you back from performing (or at least recommending, if you are the
consulting GI) a pelvic examination.
Why Might This Be Tested? Board examiners want you to think about
the entire physical examination, including the pelvic region. Fitz-Hugh–Curtis
is a classic diagnosis everyone learns in med school, and including it on the
exam provides an opportunity for examiners to see if you have totally forgotten
about the pelvis or not. Shooting straight past a pelvic exam can spell trouble;
24 Chapter 2

examiners want to ensure that you are thinking about the pelvic exam as you
develop a full differential diagnosis for abdominal pain in women.

Here’s the Point!

Sexually active woman + Fever + Pleuritic right upper quadrant pain = Think
about Fitz-Hugh-Curtis ... Check pelvic exam
“Tough Stuff” Vignettes 25

Vignette 6: Polycythemia Vera and Liver Abnormalities


A 46-year-old woman with polycythemia vera presents to the emergency
department after awakening to a sense of fullness in her abdomen along with
severe right upper quadrant pain. She had neither symptom prior to going to
bed. She feels nauseous but has not vomited. There is no diarrhea, melena, or
hematochezia. She does not report feeling chills or fever.
On physical examination, she is found to be afebrile, normotensive, slightly
tachycardic (heart rate = 106), and tachypneic (respiratory rate = 18) with a nor-
mal oxygen saturation of 98% on room air. Her body mass index is 26. She is
anicteric but has conjunctival injection. There are extensive excoriations on her
skin. Her lungs are clear to auscultation, and her heart is tachycardic but with-
out other arrhythmias. Her abdomen is slightly distended. There is no significant
shifting dullness appreciated. The liver edge is tender and palpable 3 cm below
the right costal margin and has a total vertical span of roughly 16 cm by percus-
sion. There is voluntary guarding but no rebound tenderness. She has 1+ pit-
ting edema in the bilateral ankles. There are no other significant physical exam
findings. Labs include the following: hemoglobin = 18.6, AST = 480, ALT = 302,
ALP = 361, total bilirubin = 4.6, albumin = 3.4, creatinine = 1.3.

` What is the most likely diagnosis?


` How should this be treated?
26 Chapter 2

Vignette 6: Answer
This is Budd-Chiari syndrome (BCS) secondary to polycythemia rubra vera.
BCS is the result of an obstruction to outflow through the hepatic vein (see
Figure 3-2) with subsequent hepatic congestion. This manifests with the classic
triad of abdominal pain, hepatomegaly, and ascites. A long list of conditions can
cause BCS, but all of these conditions have in common an ability to cause venous
thrombosis or obstruction. In this case, the patient has polycythemia vera, which
is one of the most common underlying conditions to trigger BCS. The polycythe-
mia is not well treated in this case, as evidenced by the conjunctival injection,
excoriations, and elevated hemoglobin. Poorly treated polycythemia can lead to
hyperviscosity and ultimately hepatic vein thrombosis. Other myeloprolifera-
tive disorders, such as essential thrombocythemia and myelofibrosis, are also
associated with BCS but at a lower rate than polycythemia vera. Other common
associations include pregnancy, medications (including oral contraceptives, in
particular), hepatocellular cancer, and a range of prothrombotic conditions such
as factor V Leiden mutation and antithrombin III deficiency, among others.
There are some notable epidemiologic characteristics of BCS that might help
with Board exam vignettes. In particular, BCS tends to occur in younger patients,
with a mean age of presentation of 35 years (although it certainly can be seen
in older patients as well, as evidenced in this vignette). BCS is about twice as
prevalent in women than men, partly because pregnancy is an important risk
factor for BCS. So keep BCS in mind whenever you hear about hepatomegaly and
abdominal pain in a pregnant woman, or simply any younger patient, especially
when there is concurrent ascites. Remember, you need to know about pregnant
women for the Board exam. Test writers seem to love posing questions about
pregnant women. We have included several other pregnancy-related vignettes in
the pages ahead—stay tuned.
The clinical course of BCS can be variable. In this case the onset was acute;
there was not even enough time for ascites to form. Without timely treatment,
acute liver failure is possible. In other cases, the syndrome can evolve over
months and fly under the clinical radar for a while. In some instances, BCS
presents as the initial sign of underlying cirrhosis; in other cases, BCS is discov-
ered in patients with known cirrhosis and refractory ascites only after a fruitless
search for other causes of refractory ascites.
Liver tests are usually abnormal in BCS, although the range of liver test
abnormalities varies widely. Aminotransferases typically range from around
100 to 600+. Alkaline phosphatase can be in the 200 to 500 range. Total biliru-
bin can certainly go up, but usually not much beyond 7 to 10 (unless there is
acute liver failure).
The injury is most pronounced in zone 3 of the portal lobules. This is a good
opportunity to stop and ask yourself why this is so. A review of the anatomy of
the portal triads, lobules, and related metabolic zones will provide the answer.
So put your first-year med school hat on for a second and take a look at
Figure 6-1. This is a stylized version of hepatic acini and portal triads, along with
a depiction of the metabolic zones. Yeah, that’s right, we made the figure in
PowerPoint—it’s pretty sweet.
As shown in the figure, the hepatocytes are arranged in roughly hexagonal
structures that are bordered by a series of portal triads. The portal triads consist
of 3 structures (by definition): the bile ductules, hepatic arterioles, and portal
“Tough Stuff” Vignettes 27

Figure 6-1. Portal circulation and related anat-


omy.

Figure 6-2. Budd-Chiari syndrome with pas-


sive congestion in zone 3. (Reprinted with
permission of Charles Lassman, MD, UCLA
Medical Center.)

venules. The bile ductules form the beginning of the biliary system; they coalesce
into the hepatic ducts, which ultimately merge to form the common bile duct.
The portal venules receive inflow from the portal vein, and the hepatic arterioles
deliver oxygen from the larger hepatic artery. Oxygenated blood from the hepatic
arterioles merges with blood from the portal venules. The admixture courses
through hepatic sinusoids (not pictured) and collect in the central vein. This ana-
tomic arrangement creates a metabolic gradient comprising zone 1 (periportal
zone—maximally oxygenated), zone 2 (intermediate zone), and zone 3 (pericentral
zone—minimally oxygenated).
BCS leads to obstructed outflow, so there is backflow and pericentral con-
gestion in zone 3 (Figures 6-1 and 6-2). This makes it even harder for zone 3
to become oxygenated because blood from the triads fights its way upstream
against the pressure gradient. When the outflow is acute or severe, zones 1 and
2 become affected as well, which can lead to acute liver failure. In less acute
settings, there can still be progressive injury with development of fibrosis within
2 weeks following the initial injury. Left untreated, BCS can progress to bridging
fibrosis and, with time, outright cirrhosis. Of note, in BCS there is often a com-
pensatory caudate lobe hypertrophy (Figure 6-3), since flow is not obstructed in
this portion due to the accessory hepatic veins that drain directly into the infe-
rior vena cava (IVC). This collateral circulation produces the classic “spider web”
appearance on venography. In chronic BCS, the caudate lobe can enlarge enough
to cause a secondary IVC obstruction.
28 Chapter 2

Figure 6-3. Budd-Chiari syndrome with character-


istic caudate lobe hypertrophy. See text for why
this occurs.

In any event, if you suspect BCS, you should move quickly to clinch the diag-
nosis. Doppler ultrasound remains a cheap, easy, noninvasive, accurate way to
diagnose most cases of BCS. The key is to demonstrate obstructed or attenuated
outflow through the hepatic vein. If the study is negative but your pretest likeli-
hood for BCS remains high, then it’s reasonable to employ magnetic resonance
venography. Computerized tomography does not have a usual role in the diagno-
sis of BCS. Venography by the interventional radiology service provides the gold
standard diagnosis (and can also quantify the pressure measurements) but is
cumbersome, expensive, and invasive.
Initial treatment of symptomatic BCS includes anticoagulation, typically with
heparin. Longer-term therapy with warfarin is often indicated. If the thrombosis
is acute and severe, or if initial anticoagulation is unsuccessful, then catheter-
directed thrombolysis may be warranted. Other approaches include transjugular
intrahepatic portosystemic shunts (TIPS) and surgical shunts. Liver transplanta-
tion may be necessary if there is acute liver failure or if there is secondary cir-
rhosis in the chronic setting. Of course, it’s also crucial to identify the underlying
cause of the BCS and, where possible, treat the underlying disorder (in this case,
polycythemia vera).
Why Might This Be Tested? First off, as we noted earlier, Board examiners
seem to love any question that involves a pregnant patient—a point we will make
repeatedly in this book. Although this particular vignette did not feature preg-
nancy, BCS is an important consideration in a pregnant woman with abdominal
pain and a big liver. Second, testing on BCS allows the examiners to check your
understanding of vascular anatomy, as with Vignette 3. Third, examiners love
figuring out if you can connect common internal medicine diagnoses (in this
case, polycythemia vera) with GI and liver conditions. They want to make sure
you have not forgotten your basic internal medicine knowledge. And last, they
could use this as an opportunity to confirm your knowledge about the micro-
anatomy and metabolic zones of the liver.

Here’s the Point!

Ascites + Hepatomegaly + Abdominal pain = Budd-Chiari syndrome (especially


in polycythemia vera, pregnancy, and other prothrombotic conditions).
“Tough Stuff” Vignettes 29

Vignette 7: Can’t Get the Mail


A 48-year-old woman with alcoholic cirrhosis complicated by previous variceal
hemorrhage now presents with shortness of breath over the past several months.
She “can’t get the mail anymore” due to marked dyspnea. The dyspnea seems to
improve when she lies down. She does not have a previous history of smoking,
cardiovascular disease, or lung disease. She has had no chest pain, cough, or
fever.
On exam, there are prominent spider angiomas on the upper torso, facial
telangiectasias, cyanosis, and clubbing with trace pitting edema in the extremi-
ties. Her lungs are clear to auscultation and heart sounds are normal. Lab tests
include the following: Na = 138, K = 3.7, creatinine = 1.0, ALT = 38, AST = 46,
total bilirubin = 1.0, ALP = 100, albumin = 3.4, INR = 1.2, platelets = 80. An
arterial blood gas reveals the following values: pH = 7.47, PO2 = 63 mm Hg,
PCO2 = 34 mm Hg. Pulmonary function tests reveal normal lung volumes but a
DL CO of only 50% predicted.

` What is the diagnosis?


30 Chapter 2

Vignette 7: Answer
This is hepatopulmonary syndrome (HPS). The history and physical exam
alone point to the diagnosis even without knowing the results of the other tests.
HPS is strongly suggested by the presence of platypnea (improvement in dys-
pnea when she lies down), coupled with the physical exam findings of clubbing,
cyanosis, prominent spider angiomas, and facial telangiectasias. Orthodeoxia
(hypoxemia improved when lying down compared with the upright position) can
also help you reach the diagnosis.
HPS probably results from the circulation of vasodilators (such as nitrous
oxide), leading to intrapulmonary vasodilation. This, in turn, causes hypoxemia
by creating a functional A-a gradient from increasing the distance from erythro-
cytes to the oxygen source as they pass through the lungs. Although 5% to 10%
of patients presenting for liver transplant evaluation will have HPS, it seems to
be an under-recognized condition. A patient with chronic liver disease or portal
hypertension will qualify as having HPS by demonstration of both hypoxemia
and intrapulmonary shunting. HPS can be diagnosed when the arterial blood
gas reveals a PaO2 <70 mm Hg and an A-a gradient >20 mm Hg. Of note, a
patient with a PaO2 >80 mm Hg is unlikely to have HPS.
Evidence of intrapulmonary shunting is typically noted with contrast echo-
cardiography with agitated saline. A positive test for HPS results when contrast
is seen in the left atrium more than 3 cardiac cycles after the original right ven-
tricular opacification. If the contrast is noted sooner than 3 cardiac cycles, then
an intracardiac shunt is likely. Furthermore, the intrapulmonary shunt can be
quantified with a technetium macroaggregated albumin (TcMAA) lung perfusion
scan to determine uptake in other organs. Under normal conditions, technetium
macroaggregates of albumin get trapped in the lung after passing through the
right ventricle and do not reach the peripheral circulation. However, with the
intrapulmonary shunting of HPS, the macroaggregates can pass into the brain
and uptake can be quantified. Thus, a brain uptake of greater than 5% of TcMAA
suggests intrapulmonary shunting.
Thus far, medical treatments for HPS have not provided much benefit.
However, liver transplantation can reverse the condition in mild to moder-
ate cases. Complete reversal of the intrapulmonary shunting can take up to
1 year, so the effect of liver transplant is not always immediate. Not all patients
with HPS do well after liver transplantation, especially those with severe intra-
pulmonary shunting. The following risk factors can be used to identify HPS
patients with severe shunting who may not reverse after liver transplantation:
(1) PaO2 <50 mm Hg (severe hypoxemia); (2) inability to correct hypoxemia with
100% O2; and (3) TcMAA >40% brain uptake (high shunt fraction).
Why Might This Be Tested? Similar to portopulmonary hypertension (stay
tuned for more on that later), the history and physical examination can point to
this diagnosis if you understand the pathophysiology. That is why Board exam-
iners love this kind of stuff. Be sure that you don’t confuse HPS with portopul-
monary hypertension.
Clinical Threshold Alert: PaO2 must be <70 mm Hg and A-a gradient
>20 mm Hg to diagnose HPS. When the PaO2 falls below 50 mm Hg in HPS,
it predicts a poor outcome with liver transplantation. Brain uptake of >5% of
TcMAA indicates intrapulmonary shunting in HPS. Liver transplantation is
unlikely to be successful when the TcMAA brain uptake exceeds 40%.
“Tough Stuff” Vignettes 31

Here’s the Point!

Liver disease + Hypoxemia + Intrapulmonary vasodilation = Think HPS

Here’s the Point!

Consider liver transplantation for HPS with PaO2 50 to 70 mm Hg

Here’s the Point!

Do not consider liver transplantation if TcMAA >40%


( ↑ intrapulmonary shunting)

Here’s the Point!

Cyanosis and clubbing are common in HPS


(in contrast to portopulmonary hypertension)
32 Chapter 2

Vignette 8: Pregnant Woman With Cirrhosis


A 32-year-old woman is referred to you by her high-risk obstetrician for risk
assessment at 13 weeks’ gestation. She has well-compensated hepatitis C cir-
rhosis and has been a nonresponder to therapy in the recent past. She had
no significant varices noted on upper endoscopy last year. She feels well and
has no symptoms related to liver disease. Her laboratory tests reveal the fol-
lowing: albumin = 3.8, total bilirubin = 1.0, AST = 38, ALT = 25, ALP = 90,
creatinine = 0.7, INR = 1.1, WBC = 4.2, hemoglobin = 13.1, platelets = 108, HCV
RNA = 800,000 IU/mL. The patient is concerned about transmitting hepatitis C
to the baby.

` What test(s), if any, should you order?


“Tough Stuff” Vignettes 33

Vignette 8: Answer
This is a tough one, but it highlights several crucial points. First, you should
order a Doppler ultrasound of the abdomen to look for a splenic artery aneurysm.
There is a 3% to 5% risk of rupture of splenic artery aneurysms in pregnant
women with cirrhosis, and a rupture carries up to a 75% maternal mortality
rate and a 90% fetal mortality rate. So this can be catastrophic if not identi-
fied in advance. Sometimes the initial bleed may be contained in the lesser sac,
which leaves time for emergency intervention. Splenic artery aneurysm tends
to occur in the third trimester and is multifactorial in etiology. The enlarg-
ing uterus can compress the aorta as gestation proceeds, leading to enhanced
flow through alterative branches including the splenic artery. Furthermore, the
plasma volume is increased in pregnant women with cirrhosis, as is hormone-
related weakness of the vasculature. These conditions create a perfect storm for
a catastrophe. Thus, if the aneurysm is >2 cm, endovascular or surgical therapy
needs to be considered. A question on the Board exam might feature a pregnant
woman with cirrhosis presenting with left upper quadrant discomfort, a pulsatile
left upper quadrant mass, or a left-sided abdominal bruit (see Vignettes 45 to 49
for more on abdominal bruits in liver disease).
Second, you should also perform an upper endoscopy in this patient. There is
a 25% risk of variceal bleeding during pregnancy in women with cirrhosis, and
the risk of hemorrhage rises to 75% in pregnant women with large varices. As
noted, such hemorrhage could have fatal consequences for both the fetus and
the mother. Remember that portal pressures are increased in cirrhosis due to
increased plasma volume. For this reason, you should also avoid excessive fluid
administration during any admission throughout the pregnancy. Furthermore,
there may be increased vascular resistance from external IVC compression by the
enlarging uterus. These factors create the greatest risk of variceal hemorrhage in
the second trimester and during labor (from the Valsalva maneuver). Therefore,
an upper endoscopy should be performed right away in this patient since she
is in the second trimester, even though she did not have varices on endoscopy
last year. If large varices are found, then action needs to be taken for primary
prophylaxis. There are no controlled trials evaluating the safety and efficacy of
endoscopic band ligation versus beta-blockade in pregnant patients. Thus, the
choice is at the discretion of the treating physicians and is generally made after
discussion between the GI and obstetrician. It’s worth mentioning that beta-
blockers have an FDA pregnancy category C in the first trimester but have a
class D rating with risk of intrauterine growth retardation and fetal bradycardia
in the second and third trimesters. Specific questions on the direct treatment
of varices are unlikely to appear on the exam due to poor consensus of opinion.
However, avoidance of labor by caesarean section delivery is recommended to
circumvent the elevations of portal pressure and risk of variceal hemorrhage in
women with large varices.
Last, you should order a human immunodeficiency virus (HIV) test, especially
since the patient wants to know the risk of vertical transmission of hepatitis C. In
general, the risk of HCV transmission is quite low (approximately 5%). However,
the risk of HCV transmission is increased if there is coinfection with HIV, an
elevated HCV RNA level, and/or active intravenous drug use. Transmission
rates are not affected by route of delivery. However, there is an increased risk
34 Chapter 2

of HCV transmission with use of fetal scalp monitoring or if there is a long


duration between membrane rupture and delivery. Breastfeeding does not seem
to increase transmission, unless there are cracked, abraded, or bleeding nipples.
After delivery, HCV antibody testing should not be checked in newborns of moth-
ers with hepatitis C until well after the first year. Earlier testing can lead to a
false-positive diagnosis in the newborn due to transplacental transfer of mater-
nal antibodies. Therefore, HCV RNA should be obtained if needed by the pediatri-
cian in the first year of life.
Why Might This Be Tested? Although many women with cirrhosis are
amenorrheic, pregnancy does occur, especially in those with well-compensated
disease. Such women can have catastrophic bleeding from a splenic artery aneu-
rysm and/or varices. Prevention is paramount to avoid a disaster. Testing for
HCV is now routine during the first obstetrical visit, and positive results will, in
turn, lead to consultations for management. You should anticipate that this will
be on the exam.

Here’s the Point!

Pregnant woman with cirrhosis → Look for splenic artery aneurysm


and varices

Here’s the Point!

HCV has a low risk for vertical transmission unless other risk factors are
present, including HIV infection, intravenous drug use, or very high HCV
viral load.
“Tough Stuff” Vignettes 35

Vignette 9: Transjugular Intrahepatic Portosystemic Shunt


Request
A 58-year-old man with chronic hepatitis C cirrhosis is brought into the emer-
gency department with abdominal distention, malaise, and lethargy. He has
required frequent paracentesis for recurrent ascites and has been intolerant to
increasing doses of diuretics.
On examination he is icteric with spider nevi, muscle wasting, splenomegaly,
hepatic fetor, and tense ascites. Ultrasound shows a shrunken, nodular liver
without a mass. His laboratory tests in the emergency department reveal the
following: WBC = 5, hemoglobin = 11.5, platelets = 71, bilirubin = 7.2, AST = 96,
ALT = 52, ALP = 128, albumin = 2.7, creatinine = 1.5, INR = 1.9, and AFP = 5.
The emergency department physician has consulted interventional radiology to
evaluate the patient for a Transjugular Intrahepatic Portosystemic Shunt (TIPS)
placement. However, the interventional radiologist has asked for your consulta-
tion prior to TIPS placement.

` Should this patient receive a TIPS?


` Why or why not?
36 Chapter 2

Vignette 9: Answer
This patient has a MELD score equal to 25. A score of 25 means there is about
a 25% chance he will die within 3 months without a liver transplant. In some-
one with this degree of liver dysfunction, placing a TIPS could have disastrous
consequences. In patients with poor hepatic synthetic function, TIPS is indicated
only when the patient exhibits acute, life-threatening consequences of portal
hypertension, such as active variceal hemorrhage. With the advanced degree of
liver disease in this patient, there is the added risk of further progression due
to portal diversion of blood flow (not to mention all of the other associated TIPS
complications, such as encephalopathy, bleeding, sepsis, hemolysis, fistulas,
hepatic infarction, hemobilia, and stent dysfunction). The best treatment would
include large-volume paracentesis with intravenous albumin in combination
with an expedited liver transplant evaluation. TIPS would not be advised in this
case unless the patient were in dire straits.
This vignette provides an opportunity to briefly review the MELD scoring sys-
tem. MELD was initially developed in a multicenter study for patients with cir-
rhosis undergoing TIPS. It was found to have about an 85% accuracy for predict-
ing death in these patients. After a mandate from the Department of Health and
Human Services, MELD replaced the previous Child-Turcotte-Pugh (CTP)-based
system. On February 27, 2002, the MELD score was used for liver allocation by
the United Network of Organ Sharing (UNOS). I remember being on service when
the change occurred. I can just say that it made for quite an interesting day.
The MELD score, which ranges from 6 to 40, has distinct advantages over
the CTP system by limiting subjective parameters (such as amount of ascites
and encephalopathy) and providing a continuous scale, thereby eliminating the
“floor and ceiling” effect that occurred with the CTP system. Of note, INR is the
best marker of liver function; therefore, it’s the most heavily weighted variable
in the MELD scoring system. The predictive value of MELD is independent of
common cirrhosis complications, including bacterial peritonitis, variceal hemor-
rhage, ascites, and encephalopathy. Because these clinical events do not provide
an incremental predictive value, MELD is currently calculated with only 3 lab
tests (serum INR, total bilirubin, and creatinine) to obtain a score for estimation
of 3-month mortality without liver transplantation. At a MELD score of 15 or
greater, survival is enhanced 1 year after liver transplant versus remaining on
the waiting list. Therefore, most transplant centers designate a MELD score of
15 as the minimal listing score for liver transplantation (although most trans-
plants occur at higher values).
Use of MELD has led to significant improvements in the process of liver trans-
plantation, namely, a reduction in waiting time for transplantation, fewer new
patients registered for the waiting list, and the ability to prioritize sicker patients
with a greater need for transplantation. Compared to the pre-MELD era, the
MELD era has resulted in a diminished mortality rate for those on the transplant
list. Post-transplant survival has not decreased, despite transplanting sicker
patients. Plus, the use of an evidence-based score makes everyone feel better
about making difficult allocation decisions.
However, nothing is perfect. There are some conditions where the MELD
score may not accurately predict mortality, and a higher score can be requested
through an appeal process to UNOS; such inaccuracy may occur in patients with
“Tough Stuff” Vignettes 37

hepatocellular carcinoma, hepatopulmonary syndrome, and familial amyloido-


sis, among other conditions. These patients can have progression of disease with
a high mortality rate while their MELD score may not necessarily increase in
lockstep with their disease severity (which could lead to death while on the list).
Furthermore, a subset of patients with a low MELD score have hyponatremia,
and these patients may benefit from incorporating serum sodium into the MELD
(termed “MELD-Na”). However, the use of diuretics and intravenous fluids can
cause marked changes in the serum sodium concentration. Although MELD-Na
might be used in the future—as well as some other potential slight revisions of
the current MELD scoring system—the core MELD will be the basis for any future
prognostication system in the foreseeable future. Therefore, gastroenterologists
need to be familiar with applying MELD, and this vignette provides a practical
example where the MELD score has implications not only for transplantation but
for other decisions as well (in this case, whether to place a TIPS).
Why Might This Be Tested? MELD has replaced the use of the CTP score,
which is now considered outdated, for decisions regarding liver transplantation.
MELD is being used in other aspects of liver disease, too. So, MELD is here to
stay. You simply need to become very familiar with MELD for the exam and for
clinical practice.
Clinical Threshold Alerts: A MELD of 15 is the minimal listing score used by
most transplant centers. The MELD ranges from 6 to 40 points—this range pre-
dicts 3-month survival without liver transplantation in patients with cirrhosis.

Here’s the Point!

Avoid TIPS in patients with a high MELD score (25 or more) unless it’s a
life-threatening emergency or liver transplantation is unlikely.
38 Chapter 2

Vignette 10: Going Vertical


A 25-year-old primigravid Asian woman in her 34th week of pregnancy is
referred to you by her obstetrician for management of hepatitis B virus (HBV)
infection. The patient was diagnosed with HBV 3 years ago but has steadfastly
refused treatment, as she fears long-term side effects of “chemicals.” However,
she is concerned about passing HBV to her child and wants to know if she should
reconsider therapy during the peripartum period. There has been no history of
hepatic decompensation. Her pregnancy has been uneventful through 34 weeks
of gestation.
Laboratory tests include the following: WBC = 5.5, hemoglobin = 13.1,
platelets = 195, ALT = 44, AST = 30, total bilirubin = 1.0, INR = 1.0,
albumin = 3.5, creatinine = 0.7, HBeAg = positive, HBeAb = negative,
HBV DNA = 4.1 million IU/mL, HBV genotype = C.

` What should you recommend?


“Tough Stuff” Vignettes 39

Vignette 10: Answer


The best strategy is to start treatment now with an oral antiviral agent. Then,
within 12 hours following delivery, the newborn should receive hepatitis B immu-
noglobulin (HBIG) and HBV vaccination; this strategy will significantly decrease
the risk of vertical transmission of HBV.
This patient is at very high risk for vertically transmitting HBV. The main
risk factors include HBeAg positivity and a high viral load (HBV DNA over
1 million IU/mL). With both of these being positive, the patient has a nearly
90% chance of passing HBV to her newborn—a sobering statistic. Transmission
usually occurs during delivery. However, caesarean section does not seem to
decrease the risk. Treatment with an oral nucleoside/nucleotide analog should
be commenced in the third trimester at approximately 34 weeks since deliv-
ery typically occurs anytime thereafter. Upon delivery, combining active with
passive vaccination in the newborn will further reduce the risk and is about
95% effective. Active vaccination consists of HBV vaccination at delivery and
again 1 and 6 months postpartum. Passive vaccination consists of administer-
ing HBIG to the newborn at delivery. Of note, HBeAg-negative mothers have a
much lower transmission risk (about 20%), but immunoprophylaxis should be
given nonetheless to decrease this risk further. Breastfeeding does not bestow an
increased risk for HBV transmission.
Several drugs are approved for treatment of HBV infection, including a
growing list of nucleoside/nucleotide drugs and pegylated interferon. During
pregnancy, where safety is of paramount importance when considering treat-
ment options, interferon would not be advisable due to its side effect profile and
pregnancy category C rating. Therefore, a nucleoside or nucleotide analog would
be the most appropriate type of therapy to reduce the HBV DNA levels in this
patient. You should choose between tenofovir and telbivudine, both of which
have an FDA pregnancy category B rating (lamivudine, adefovir, and entecavir
are class C). Tenofovir (which has a very low long-term resistance rate) might be
favored in this case, since the patient would require long-term therapy given her
HBeAg-positive status. However, remember that this patient only wanted to take
therapy to decrease her transmission rate. So, if she is not willing to take long-
term therapy, then telbivudine (which has an approximate 25% resistance rate
at 2 years) would be a reasonable choice to take to the end of pregnancy, since
a short course of therapy is unlikely to confer viral resistance. Due to reported
exacerbations of liver disease, all HBV-infected women should be closely followed
after delivery as well.
While we are on the subject of HBV, here are a few more pearls (this stuff is
huge for the Boards, so read carefully!). Along the lines of HBV prophylaxis, keep
in mind that chronic HBV patients can acutely “flare” or decompensate when
anti-TNF therapy or immunosuppression is administered. Such flares or decom-
pensation occurs up to 50% of the time and can lead to death. On the Board
exam, this could be another scenario for linking gastroenterology and hepatology
knowledge. For example, the case may involve an HBV carrier with a low DNA
level who develops fistulizing Crohn’s disease and is planning to start anti-TNF
therapy. Regardless of the DNA level, prophylactic therapy with a nucleoside/
nucleotide analog should be given during the entire course of anti-TNF or immu-
nosuppressive therapy and should be continued for 6 months after completion.
40 Chapter 2

A similar situation might occur upon titrating prednisone for various conditions.
Stay tuned for another vignette on this topic later in this book.
Here’s another fact: Acute liver failure from HBV warrants treatment. It was
previously thought that treatment may be futile in the setting of acute HBV liver
failure. However, data now indicate that timely anti-HBV treatment can improve
mortality rates in acute HBV liver failure. Furthermore, even if liver transplant is
inevitable, a decreased viral load is important to diminish the risk of post-trans-
plant recurrence. For the same rationale, patients with cirrhosis and any level of
viremia require treatment. Both entecavir and tenofovir would be good choices
due to their rapid viral suppression and extremely low long-term resistance rates.
In fact, treating patients with decompensated HBV cirrhosis can improve hepatic
synthetic function. In some instances, patients can have a dramatic response
with marked improvement of their MELD score, and the treatment can render
them well enough to “de-list” for liver transplantation. So there really is a lot of
benefit to treating HBV, even when it might seem like a desperate situation of
diminishing returns.
There are currently 8 known genotypes for HBV (genotypes A through H). Of
these, 4 predominate in the United States: A, B, C, and D. Genotype A is the
most common among African-Americans (remember: AA has A) and has the best
response to pegylated interferon treatment with up to a 50% e antigen seroconver-
sion rate (as opposed to 30% with the other genotypes). In fact, compared to oral
agents, pegylated interferon might be a more effective and cost-effective choice
in an HBV genotype A patient with a high ALT and a low viral load. Genotypes B
and C are predominantly found among Asians. The patient in this vignette had
genotype C, which tends to be a more aggressive form with increased inflamma-
tion, rate of progression, and incidence of HCC compared to genotype B (remem-
ber: among Asians, genotype B is better and C is crummy). Genotype D tends to
be more commonly found in patients from Eastern Europe and has a generally
less favorable prognosis than does genotype A.
Certain subsets of HBV carriers have an increased risk for the development of
HCC. In particular, HCC screening should be performed in African Americans
over age 20, Asian men over age 40, and Asian women over age 50.
Why Might This Be Tested? There are an estimated 350 million persons
worldwide with HBV infection. With increased immigration over the past few
decades from endemic regions into the United States, plenty of HBV patients are
flying under the radar. They will present in various clinical scenarios (including
pregnancy), and you will need to know how to manage them as the treatment is
constantly evolving.

Here’s the Point!

High HBV DNA + HBeAg-positive → Highest risk for vertical transmission


“Tough Stuff” Vignettes 41

Here’s the Point!

HBV carrier + Anti-TNF therapy → Use nucleoside/nucleotide analog for


prophylaxis (DNA level does not matter)

Here’s the Point!

Acute HBV liver failure → Need to treat with nucleoside/nucleotide analog,


even if there is a lack of clinical improvement

Here’s the Point!

HBV genotype A (African-Americans) → Best response to interferon

Here’s the Point!

In Asians, HBV genotype B has more indolent disease course than genotype C
42 Chapter 2

Vignette 11: Bubbles


This isn’t really a vignette, but take a look at Figure 11-1.

Figure 11.1.

` What is this called?


` What is it used for?
“Tough Stuff” Vignettes 43

Vignette 11: Answer


This is a “trail test,” or number connection test (NCT), used to diagnose sub-
clinical (aka “minimal”) hepatic encephalopathy (HE). Diagnosing and treating
overt HE on a timely basis is especially vital. Not only is undiagnosed HE com-
mon in patients with otherwise seemingly compensated cirrhosis, but it’s also
expensive. Overt HE ultimately affects 20% of patients with cirrhosis, requiring
more than 55,000 hospitalizations annually and costing over $1 billion per year
in health care expenditures. Once HE develops, mortality reaches 35% after
5 years. Surviving patients suffer from diminished health-related quality of life,
increased rates of work absenteeism, and decreased work productivity. Patients
with HE are also at high risk of motor vehicle accidents and other delirium-relat-
ed injuries. An additional 50% to 60% of patients with cirrhosis have evidence of
subclinical or minimal HE that interferes with cognition and behavior, and one
third of these patients ultimately develop overt HE within 2 years of diagnosis.
The bottom line is that HE is common and expensive, and you need to know how
to diagnose and treat it on a timely basis. You also need to recognize that mini-
mal HE is a legitimate disorder with real-life implications, despite being subclini-
cal or “covert.” In fact, there is a fine line between “covert” and overt HE.
Too often we rely on insensitive markers that occur late in the process of HE to
make the diagnosis, such as asterixis, hyperreflexia, ataxia, bradykinesia, and
overt confusion. By the time someone has, say, asterixis, he or she has already
progressed to at least grade 1-2 HE (on the 0 to 4 scale; Table 11-1). Although
having asterixis is certainly specific for HE in the setting of cirrhosis, it’s by no
means sensitive for early disease. We cannot rely on asterixis alone to diagnose
HE; we need to do better. Quite frankly, it can be a travesty to evaluate a patient
with seemingly compensated cirrhosis, screen for asterixis in the clinic, docu-
ment “no asterixis” in the chart, confirm a normal serum ammonia level (despite
the poor correlation between venous ammonia and clinical symptoms of HE), and
then send the patient on his or her way without having diagnosed HE—only to
learn that the patient crashed his car into a family crossing the street 3 weeks
later. These things do happen. So it’s not good enough to write “no asterixis” in
the chart and leave it at that. Ruling out asterixis means only that the patient
does not have advanced HE yet; it does not rule out earlier HE, which is still
potentially deadly—if not for the patient himself, then for the people around him,
especially if he drives a car or operates heavy machinery. Diagnosing early HE
is serious business.
Screening for signs of HE is worth the time and effort, especially since the
effort is relatively small and the benefit is potentially huge. A first step is to ask
the patient about sleep, especially variations in sleep-wake cycles. Patients with
early HE may develop insomnia at night and hypersomnia during the day, and
these diurnal variations may precede the more commonly sought neurologic
signs, such as asterixis. Sometimes primary care providers do not recognize
this problem and prescribe sleeping pills, which exacerbate the encephalopa-
thy. Patients may also have difficulty concentrating. You can ask this straight
up, and many patients will acknowledge that they are having more and more
trouble focusing; some describe “brain fog” and other sorts of cloudy thinking.
Of course, some patients with HE may not recognize that they are developing
forgetfulness or concentration problems, namely because they have HE. So it’s
44 Chapter 2

Table 11-1.

STAGES OF HEPATIC ENCEPHALOPATHY


Stage Description
0 This is also called minimal hepatic encephalopathy, or MHE. The older term was
subclinical encephalopathy. These patients exhibit a seemingly normal personal-
ity but have minimal changes in their cognitive function, including memory and
concentration. This can be picked up with the NCT and other psychometric tests.
Otherwise, there are no significant neurologic sequelae.

1 Now things get more involved. These patients have a shorter attention span
and reduced ability to perform simple math, like serial 7s. They often have
reversal of day-night sleep patterns; diminished awareness and concentration;
and signs of irritability, depression, or even euphoria. NCT times get slower.
Electroencephalographic (EEG) abnormalities are now detectable. May begin to
show asterixis.

2 Now things start getting more serious. Asterixis sets in, and patients experience
disorientation, drowsiness, and significant personality changes. These patients
can become disoriented and very easily confused. The EEG is clearly abnormal at
this point with diffuse slow waves. The patients are hyperreflexic and can exhibit
clonus. This is generally obvious to the astute clinician.

3 At this point the patients become sleepy yet arousable. They are totally disori-
ented regarding time and place. They forget lots of things, have emotional out-
bursts, and slur their speech. So-called triphasic waves show up on the EEG. If
you cannot recognize these clinical features in patients with cirrhosis, then it’s
time to go back to med school!

4 By stage 4, the patient has basically slipped into a coma. The response to painful
stimuli is minimal or nonexistent. If this slips your clinical detection, then it’s time
to find another line of work.

useful to objectively document evidence of diminished concentration, when pres-


ent. It’s also helpful to elicit history from a family member or spouse who lives
with the patient. In the clinic, I like to toss a tennis ball at the patient (gently,
of course!) to see if they can catch it, or at least quickly move their hands into
position to make the grab. This may also help to make the patient realize that
they have HE.
Now let’s get back to the image in the vignette. The NCT is a validated tech-
nique that can provide objective evidence of HE. The test works by staggering
numbers or letters in an arbitrary pattern on a piece of paper, as depicted in
Figure 11-1. The patient is asked to link the numbers together as fast as pos-
sible by drawing lines between sequential integers. Rather than asking patients
to “sequence integers,” it’s better to use these standardize directions: “On this
page you see the numbers from 1 to 25. They have been all scattered about. Your
task is to order the numbers by drawing a line between them with a pencil (or
pen), starting with the smallest one. You start with the number 1 and draw a
straight line from there to 2, then to 3, and so forth. Do this as fast as you can.”
“Tough Stuff” Vignettes 45

A person without HE or other cognitive dysfunction should be able to do this


within 30 seconds (try it, it’s not that easy to do within 30 seconds but is cer-
tainly doable); taking longer suggests HE. Specifically, 31 to 50 seconds tends to
correlate with stage 0 to 1 HE, 51 to 80 seconds with stage 1 to 2 HE, and 81 to
120 seconds with stage 2 to 3 HE. Forced termination of the test suggests stage
3. You will not be asked these values on the Board exam, but it’s convenient to
know them in any event.
If HE is identified, whether overt or minimal, the next step is to consider
whether an underlying process might be triggering the HE. There is usually
something causing the problem that can be identified and treated. It’s not good
enough to simply start therapy (more on that shortly) without also considering
the underlying cause. Board examiners might want to test this, and we can
imagine them developing an HE vignette in which there are clues for an underly-
ing precipitant. Classic examples include overdiuresis from furosemide (leading
to a contraction alkalosis, which can precipitate HE); use of sedatives, hypnotics,
or opioids; underlying infection (eg, spontaneous bacterial peritonitis); hypogly-
cemia; gastrointestinal bleeding; electrolyte abnormalities (especially hypokale-
mia); and a new hepatocellular carcinoma. Be sure to look for these things, both
on Board exams and in real life.
Therapy for HE has been difficult and limited in long-term efficacy. In addition
to identifying and treating precipitating factors of HE, clinicians traditionally
rely on nonabsorbable disaccharides, such as lactulose and lactitol, as the cor-
nerstone of treatment. However, the primary disadvantage of these agents is the
high incidence of poorly tolerated adverse events with their use, such as cramp-
ing, diarrhea, and flatulence. Moreover, although a Cochrane Systematic Review
found that lactulose is more effective than placebo in resolving symptoms of HE,
the analysis found no statistically significant difference when limited to studies
of high methodological quality. The review concluded that there are insufficient
data to support the use of lactulose in the management of HE given the exist-
ing data in the literature. However, from clinical experience, it’s obvious that
lactulose does work in many patients. Taken together, these data indicate that
although lactulose may be effective for patients, its effect is not always robust,
and compliance is often limited by side effects.
An alternative therapy to nonabsorbable disaccharides is the poorly absorbed
oral antibiotic neomycin. Although neomycin has been used in HE for over
3 decades, there are few data to support its efficacy. In fact, no controlled studies
have found neomycin to be more effective than standard treatments, and data
from one randomized trial found no difference between neomycin and placebo.
Moreover, the long-term use of neomycin is limited by nephrotoxicity and ototox-
icity, and the incidence of these side effects is even higher in patients with renal
insufficiency—a common comorbidity in patients with advanced cirrhosis.
More recently, the minimally absorbed oral antibiotic rifaximin has demon-
strated efficacy in maintaining remission of HE. Rifaximin demonstrates high
antimicrobial activity against common gut flora in vivo, achieves high gut con-
centrations, and is negligibly absorbed into the systemic circulation. Randomized
controlled trial data reveal that rifaximin is superior to lactulose in the treat-
ment of HE. In addition, rifaximin has lower risks of side effects and better oral
tolerability among patients. It was FDA approved for the prevention of recurrence
of HE in 2010 after showing a reduced recurrence of overt HE and reduced num-
ber of HE-related hospitalizations.
46 Chapter 2

Why Might This Be Tested? HE is prevalent and expensive. Moreover, people


with undiagnosed HE can harm themselves and others if not treated in a timely
manner. HE has gained increasing attention as a serious disorder with impor-
tant consequences such as poor driving performance on simulated tests and
increased risk for motor vehicle accidents. Thus, it’s important to advise patients
with HE to avoid driving. Board examiners will want to know that you have heard
of minimal HE and that you understand how to identify HE without relying on
overt signs like asterixis.
Clinical Threshold Alert: If a patient with cirrhosis takes longer than 30 sec-
onds on an NCT, underlying HE is suggested. This threshold is unlikely to show
up on a Board exam, but it’s good to have a sense of how to interpret the NCT.

Here’s the Point!

Cirrhosis + Sleep trouble + Slow NCT = Hepatic encephalopathy (even if there


is no asterixis)
“Tough Stuff” Vignettes 47

Vignette 12: Fatty Liver Consult


An 18-year-old student without previously known medical problems is referred
to you by his family physician for fatty liver on an ultrasound. He saw his family
doctor due to persistent mild discomfort in the right upper quadrant after being
kicked by a goat at their farmhouse 9 months ago. His mother tells you that his
performance at school has also declined in the past 2 years despite tutoring. He
was previously a “straight-A student,” and now she has trouble reading his small
handwriting. His family physician has diagnosed this patient with attention defi-
cit disorder. His mother mentions that she had relatives who “died from liver fail-
ure” at a young age. The patient reports no history of alcohol use. His body mass
index (BMI) is 21. Laboratory tests reveal the following: total bilirubin = 1.0,
albumin = 3.9, ALT = 52, AST = 48, hemoglobin = 12.1, platelets = 160, INR = 1.0,
iron saturation = 20%.

` What is the diagnosis?


` What test(s) could you order to confirm the diagnosis?
` What else should you tell the patient to decrease the risk of
progression?
48 Chapter 2

Vignette 12: Answer


This is Wilson disease (WD), or hepatolenticular degeneration. WD was
described by Kinnear Wilson about 100 years ago. It’s an autosomal recessive
condition marked by impaired biliary secretion of copper as a result of a muta-
tion in the ATP7B WD gene on chromosome 13; this leads to accumulation of
copper in the liver with resulting injury, almost always occurring before 30 years
of age. As excess copper is released into the bloodstream, it’s deposited into other
organs such as the brain, cornea, and kidneys. In the kidneys the copper typi-
cally deposits in the renal tubular cells, which leads to a form of Fanconi syn-
drome. Of note, the ATP7B is the same gene responsible for the copper-deficient
state found in Menkes disease, which leads to growth failure, coarse and sparse
hair, and a deterioration of the nervous system. Menkes disease is found in early
childhood. Therefore, you will not see Menkes disease on the exam, but you can
use this knowledge to impress your colleagues!
Patients with WD may also present with neuropsychiatric symptoms.
Micrographia (as seen in this case) is a classic symptom of WD and can also be
seen in Parkinson’s disease. In fact, WD can present with other Parkinsonian
symptoms, including the tremor, drooling, rigidity, and risus sardonicus, or
uncontrollable grinning. Patients with neurologic symptoms tend to present at a
younger age and may show a decline in school performance that can be miscon-
strued as attention deficit disorder.
Fatty liver is commonly seen in WD, and the patient can be misdiagnosed as
having nonalcoholic steatohepatitis (NASH). However, this patient had a normal
BMI and did not have other significant medical history, which undermines the
diagnosis of NASH. In fact, there is quite a bit of histologic and clinical varia-
tion in WD. Histologically, patients can present with interface hepatitis similar
to autoimmune hepatitis. So, if a young patient is not responding to steroids for
autoimmune hepatitis, think of WD as a distinct possibility. Patients with WD
can also present with acute liver failure, which is usually accompanied by hemo-
lytic anemia due to the excess free copper in the circulation and a markedly low
serum ALP (less than 40 IU/L).
Serum ceruloplasmin is the first test that should be ordered. A low level (less
than 20 mg/dL) is considered positive, but further testing is needed for confirma-
tion of the diagnosis. A very low level (less than 5 mg/dL) can clinch the diagnosis
with the right clinical picture. A 24-hour urine copper level should be checked,
and the diagnosis is confirmed when the copper level is more than 100 mcg.
Alternatively (or as an adjunct test), you might order a slit-lamp examination for
Kayser-Fleischer (KF) rings. The greenish-brown color of these rings is due to
copper deposited in a granular complex with sulfur in Descemet’s membrane of
the cornea. KF rings are not pathognomonic for WD (they can also be found in
other chronic cholestatic conditions such as primary biliary cirrhosis). KF rings
are present in about half of WD patients with solely hepatic manifestations;
however, they are present in at least 90% of cases with neurologic involvement
(such as in this case). Of note, the sunflower cataract (copper deposits in the
lens) is another ocular manifestation of WD. A liver biopsy can also be helpful;
a hepatic copper concentration of 250 mcg/g dry weight can seal the diagnosis
(Figure 12-1). However, as with KF rings, other chronic cholestatic diseases can
also cause an elevation in the hepatic copper concentration.
“Tough Stuff” Vignettes 49

Figure 12-1. Rhodanine stain of the liver in Wilson dis-


ease. Red granules of copper are seen in the hepatocytes.
(Reprinted with permission of Charles Lassman, MD, UCLA
Medical Center.)

Treatment for WD is lifelong and consists of using a metal chelating agent


such as trientine or penicillamine to remove accumulated copper. Zinc can also
be used to prevent future reaccumulation and to maintain clinical remission
(zinc interferes with GI absorption of copper). Twenty-four-hour urinary copper
levels can be followed to assess response and compliance to therapy. Response to
therapy is usually excellent, provided the patient is diagnosed before the onset of
decompensated cirrhosis or acute liver failure. However, neuropsychiatric mani-
festations may not resolve completely. Liver transplantation provides a cure for
the liver disease since it corrects the metabolic defect in the liver. Thus, patients
who undergo liver transplantation do not need chelation or zinc therapy.
Foods that contain a large amount of copper should be avoided to minimize
further copper accumulation. These include chocolate, nuts, organ meats (espe-
cially liver—perhaps that is a little too obvious), shellfish, and mushrooms. The
patient in this vignette lives on a farm and likely is drinking well water, which
may be entering the house through copper pipes. The home’s water supply will
need to be checked for copper content. If it’s elevated, then the family should find
other forms of portable water or else invest in a home water purification system.
Also, the family should be mindful to avoid cooking or storing food or water in
copper containers or cookware.
Why Might This Be Tested? WD is an uncommon disease with plenty of
extrahepatic manifestations and unique diagnostic values, making it a favorite of
the examiners. You just have to know this stuff. For that reason we will reinforce
some of this information later on in this book.
Clinical Threshold Alerts: The following values all suggest the diagnosis of
WD:
1. Serum ceruloplasmin <20 mg/dL
2. 24-hour urinary copper excretion >100 mcg
3. Hepatic copper concentration >250 mcg/g dry weight
4. Serum ALP level <40 IU/L in acute liver failure

Here’s the Point!

Neurologic symptoms with liver disease in a young patient =


Think Wilson Disease
50 Chapter 2

Vignette 13: Screening for Varices in Cirrhosis


A 48-year-old man with compensated cirrhosis from chronic hepatitis C is
referred for consideration of screening for esophageal varices. The patient has
not previously received an upper endoscopy. He has a MELD score of 14; meets
criteria for Child classification A disease; and has not developed variceal bleed-
ing, encephalopathy, or ascites.
On physical examination, his heart rate is 86, blood pressure 142/86, respira-
tory rate 14, and temperature 98.8ºF. He is found to have minimal but present
scleral icterus, spider angiomas on the anterior chest, a firm liver edge 2 cm
below the right costal margin, a palpable spleen tip, and 1+ pitting edema at the
ankles. There is no abdominal distention, shifting dullness, or caput medusae.
There is no asterixis, and the patient readily passes office-based psychometric
testing with a trail test, cube reproduction test, and serial 7s.
Laboratory testing reveals the following: hemoglobin = 12.2, WBC = 3.4,
platelets = 72, INR = 1.2, total bilirubin = 2.4, creatinine = 1.3, albumin = 3.1,
AST = 38, ALT = 56, ALP = 47, AFP = 8. Right upper quadrant ultrasonography
reveals a nodular-appearing liver with a portal vein diameter of 2 cm and a por-
tal vein thrombus.

` What specific features raise the risk of underlying varices, other


than having cirrhosis itself?
` Do guidelines recommend screening for varices in this situation?
“Tough Stuff” Vignettes 51

Vignette 13: Answer


Up to one-half of patients with compensated cirrhosis have moderate or large
underlying esophageal varices. Once esophageal varices form, the risk of vari-
ceal bleeding within 2 years is 20% to 35%, and the risk of dying from an initial
variceal bleed is around 20% (historically as high as 50%). In light of the sub-
stantial human and economic costs of variceal hemorrhage, several strategies
have been developed for primary prophylaxis against an initial bleed, including
beta-blocker therapy, combination beta-blocker and nitrate therapy, and endo-
scopic band ligation.
However, the clinician evaluating a patient with compensated cirrhosis is often
unaware of whether there are underlying varices and is faced with the dilemma
of whether or not to perform screening upper endoscopy. Proponents of screen-
ing endoscopy contend that the procedure allows for the detection of esophageal
varices, targeted endoscopic treatment in patients with esophageal varices, and
reduction of unnecessary therapy in patients without esophageal varices. The
American College of Gastroenterology (ACG) and the American Association for
the Study of Liver Diseases (AASLD) recommend universal screening endoscopy
in patients with compensated cirrhosis, followed by active therapy for those
patients with esophageal varices. Opponents argue that a strategy of universal
screening endoscopy requires an excessive use of resources to identify the subset
of at-risk patients, and that empiric medical therapy may be a more cost-effective
approach that can be carried out in the primary care setting, without reliance
on specialist care.
In deference to both strategies, another approach is to perform screening
endoscopy only in patients at high risk for underlying esophageal varices. Several
studies have identified 4 clinical variables that significantly correlate with the
presence of varices including platelet count <88,000/mL, elevated INR, spleno-
megaly, and portal vein diameter >13 mm on ultrasonography. Each of these
predictors is independently associated with underlying varices. Of note, the pres-
ence of ascites, encephalopathy, and level of bilirubin elevation have not been
found to be independently predictive of varices. The patient in this vignette has
thrombocytopenia and portal vein dilatation, both of which indicate an elevated
HVPG and therefore a higher-than-average risk of varices. These clinical pre-
dictors may allow physicians to select the subgroup of patients with cirrhosis
who are most likely to have underlying varices and therefore are most likely to
benefit from screening endoscopy. But it turns out that the cost-effectiveness of
screening endoscopy does not get any better if we use these rules; they are good
to know about, but in the end, the guidelines basically recommend screening
for varices in all patients with compensated cirrhosis, not just those with a low
platelet count, big portal vein, splenomegaly, and elevated INR.
In passing, it’s worth reviewing the role of HVPG monitoring in cirrhosis. Data
indicate that variceal formation increases once the HVPG exceeds 12 mm Hg.
Furthermore, the higher the HVPG, the higher the portal pressure and higher
the risk of variceal bleeding. The goal of beta-blocker therapy is to drop the HVPG
below 12 mm Hg. Of course, that does not happen in many people on beta-block-
er therapy, even if they are compliant. The failure of beta-blocker therapy to drop
the HVPG below 12 mm Hg partly explains why the risk reduction for variceal
hemorrhage is only around 50% with beta-blockers (which isn’t bad, but isn’t
52 Chapter 2

great either). In contrast, prophylactic endoscopic band ligation drops the risk
of bleeding by up to two-thirds and can work even if the HVPG remains above
12 mm Hg. In general, HVPG monitoring is invasive, expensive, and not routinely
performed in the United States. Some argue that it’s most useful in patients on
the waiting list for a transplant in order to maximize therapy. In fact, it has been
shown that routine HVPG monitoring is cost-effective in this high-risk group
awaiting transplantation. But otherwise, guidelines do not recommend routine
HVPG testing in patients with cirrhosis, even if they have varices. The surrogate
markers for beta-blocker therapy are to monitor blood pressure and heart rate
with a goal to drop the heart rate to 55 to 60 beats per minute and to drop the
systolic blood pressure by 25% as tolerated. Patients who cannot meet those
goals despite compliance with therapy should be moved to another approach,
such as endoscopic band ligation.
It’s also worth taking a moment to review how to manage (or not manage, as
the case may be) portal vein thrombosis. Here’s the deal: Portal vein thrombosis
is quite common in cirrhosis. Its presence confirms hepatofugal flow and high
portal pressures. It’s typically an epiphenomenon of underlying portal hyperten-
sion, not a disease unto itself in need of active therapy. Heparin is generally not
indicated for portal vein thrombosis in the setting of cirrhosis, unless there is
extensive thrombosis that could preclude liver transplantation. Follow-up imag-
ing is recommended to ensure stability of the thrombus and to exclude hepato-
cellular carcinoma. More importantly, you should focus on the basic principles of
cirrhosis care, which are described throughout this book. Stay tuned for another
vignette on this topic that tells the tragic story of unnecessarily treating a portal
vein thrombus.
Why Might This Be Tested? Varices are common, so you should know about
their basic management principles. This vignette contains several important and
highly testable points about variceal screening and management. First, remem-
ber that current guidelines recommend routine screening for varices in basically
all cirrhotics, whether you agree with that or not. Second, remember that you
can determine whether patients are at higher-than-average risk for varices using
readily available clinical data. Third, remember that the point of beta-blocker
therapy is to drop the HVPG below 12 mm Hg—a clinical threshold that might be
useful to know on the Board exam. Fourth, remember that beta-blocker therapy
should drop a patient’s heart rate to 55 to 60 and systolic blood pressure by 25%;
these are surrogate markers for a decrease in HVPG. And finally, don’t go nuts
when you see a portal vein thrombus in a patient with cirrhosis—it’s usually a
sign of high portal pressures, but not a disease unto itself. All of these facts are
eminently testable.
Clinical Threshold Alert: If the HVPG exceeds 12 mm Hg, then the risk of
underlying varices goes up significantly; the goal of beta-blocker therapy is to
drop the HVPG below this level. Clinical surrogate markers of diminished portal
pressure include a heart rate of 55 to 60 and a systolic blood pressure drop of
25% upon initiation of beta-blocker therapy. While this is admittedly redundant
of what has just been said, it bears repeating, and these thresholds are well
worth memorizing. In addition, when the portal vein diameter exceeds 13 mm on
ultrasonography or when the platelet count falls below 88,000, the risk of varices
rises independent of other clinical factors.
“Tough Stuff” Vignettes 53

Here’s the Point!

Independent predictors of underlying varices in cirrhosis:


z Portal vein diameter >13 mm
z Splenomegaly
z Platelet count <88,000
z Elevated INR
54 Chapter 2

Vignettes 14 to 22: DILI Delight


It seems like virtually every medication and herbal supplement can affect the
liver. Obviously that is an exaggeration, but there is a definitely a core set of med-
ications and herbals that are especially known for causing drug-induced liver
injury, or DILI. You can bet that DILI will show up on the exam; DILI is big time,
and it’s something we’re always dealing with. For that reason, we have prepared
a series of mini-vignettes that feature classic DILI scenarios. For each question,
identify the culprit medication or herbal, and specify the related adverse event
described in the scenario.

14. A woman began using a dietary supplement touted for its weight loss ben-
efits and for relieving menopausal symptoms. After beginning to use the supple-
ment, she became jaundiced and rapidly encephalopathic. Liver biopsy revealed
hepatic necrosis. She subsequently required liver transplantation.

15. A patient is found to have liver test abnormalities 2 months after begin-
ning an antiarrhythmic for atrial fibrillation. Relevant labs include the following:
AST = 52, ALT = 86, ALP = 110, total bilirubin = 1.4, INR = 1.0, albumin = 3.4.
Liver biopsy reveals phospholipid-laden lysosomal lamellar bodies.

16. A patient is treated with an antibiotic for a urinary tract infection (UTI).
She develops icterus. Liver tests reveal AST = 35, ALT = 59, ALP = 620, total bili-
rubin = 2.6, INR = 1.1, albumin = 3.5.

17. A gymnast begins taking an anti-inflammatory medication for muscu-


loskeletal pain. Eight weeks later she is found to have an elevated liver test as
part of a routine physical examination. She has the following liver test values:
AST = 225, ALT = 280, ALP = 109, total bilirubin = 1.9, INR = 1.3, albumin = 3.4.
Hint: What commonly prescribed anti-inflammatory received an FDA warning
for this in 2009?

18. A patient with inflammatory bowel disease (IBD) begins to take a steroid-
sparing agent. Routine liver testing 4 weeks later reveals an elevated AST and
ALT. The patient is otherwise asymptomatic.

19. A patient with ulcerative colitis begins to take a steroid-sparing agent.


Four weeks later he is hospitalized with ascites, marked weight gain, and right
upper quadrant abdominal pain. He is found to have an enlarged and tender
liver on examination, and laboratory tests reveal hyperbilirubinemia (total
bilirubin = 4.2) and thrombocytopenia (platelets = 105).
“Tough Stuff” Vignettes 55

20. A bodybuilder develops jaundice. Physical examination reveals an


enlarged liver. Labs include ALP = 462, GGT = 387, AST = 82, ALT = 90, total
bilirubin = 1.8.

21.A health freak develops elevated liver tests and is found to have lipid-filled
stellate cells on liver biopsy.

22. A patient with metabolic syndrome is prescribed a statin. The


aminotransaminases elevate to twice the upper limit of normal, and the
patient’s primary care physician sends the patient to you for manage-
ment. The patient is otherwise asymptomatic. Should you stop the statin?
56 Chapter 2

Vignettes 14 to 22: Answers


14. This is severe liver injury from kava kava, a botanical product obtained
from the roots of the Piper methysticum shrub indigenous to the South Pacific.
Although knowing about the arcane Piper methysticum bush will provide you
with nothing more than championship-level minutiae, recognizing kava kava and
understanding its risks can be vital information. Kava kava is an herbal dietary
supplement that has been used throughout the world for several indications,
including anxiety, sleep disturbances, weight loss, and menopausal symptoms. It
may have an appealing name, but what it can do to the liver is not appealing—in
fact, it can be downright deadly. There are hundreds of case reports of severe
hepatotoxicity related to kava kava; in some of these patients, the hepatotoxicity
led to acute liver failure resulting in transplantation and/or death. The hepato-
toxic effect of kava kava is unpredictable. In case series, the time to adverse event
varied dramatically, ranging from just a few weeks to several years following
initiation of the supplement. It remains unknown exactly how kava kava causes
liver damage. The histologic pattern of injury is also somewhat variable and
includes hepatic necrosis, lobular hepatitis, and a cholestatic pattern.
Because of the severe injury related to kava kava, the FDA issued a consumer
warning in 2002 regarding its hepatotoxicity. The FDA advises special caution
in patients who have pre-existing liver disease and further recommends that
“health-care providers should consider questioning patients with evidence of
hepatic injury about the use of dietary supplements and herbal products” in gen-
eral. While it may seem obvious to ask about herbals, all too frequently health-
care providers neglect to do this. If you don’t ask about herbals, it’s time to start.
Make it routine. You can save lives by advising patients to stop potentially hepa-
totoxic agents if you catch it early enough. So the next time you have a patient
with elevated aminotransferase levels, be sure to ask about herbal supplements
as part of the usual workup.
Some traditional Chinese herbal supplements, in particular, are known to
cause liver test abnormalities or even acute liver failure. However, because these
supplements are often a blend of many agents, it can be difficult to pinpoint the
culprit. For that reason, take caution whenever a patient is taking unnamed or
blended herbal supplements, especially when he or she has evidence of liver test
abnormalities. A full accounting of all the known herbals and botanicals associ-
ated with liver injury is outside the scope of this little review, but other culprits
include ma huang (Ephedra), mistletoe, alkaloid-containing herbal teas, german-
der blossoms (used for abdominal pain, by the way), and chaparral, among many
others.

Here’s the Point!

Liver test abnormalities + Any herbal = Suspect the herbal until proven
otherwise

Think about kava kava, ma huang (Ephedra), mistletoe, alkaloid-containing


teas, germander, and chaparral, among many others.
“Tough Stuff” Vignettes 57

15. This is amiodarone toxicity. When a patient presents with heart arrhyth-
mias and liver test abnormalities, think about amiodarone along with what-
ever else might be on your mind (eg, right heart failure, hemochromatosis with
cardiomyopathy). Amiodarone can be an effective antiarrhythmic, but it’s also
known to cause liver injury. Case series indicate that roughly 1 in 4 patients
develop asymptomatic elevations in liver enzymes after starting amiodarone, and
guidelines suggest discontinuing amiodarone if the liver test values double early
in the therapeutic course. Clinically significant liver injury is less frequent but
still occurs in a fair number of patients—roughly 3% develop hepatitis. Long-
term amiodarone therapy can lead to chronic liver injury in a smaller group of
patients, with cirrhosis and liver failure described after cumulative doses begin
to mount (although there is no magical cumulative dose beyond which liver
injury is definitely known to occur). Because long-term amiodarone therapy
increases the risk of chronic liver injury, guidelines suggest using the lowest
dose of amiodarone possible and monitoring liver tests both at baseline and at
6-month intervals. Interestingly, amiodarone-induced liver injury is marked by
Mallory bodies along with steatosis, phospholipidosis, intralobular inflamma-
tion, and—when advanced—fibrosis. A great Board buzzword is “phospholipid-
laden lysosomal lamellar bodies,” which can be found on electron microscopy in
amiodarone-induced liver injury. Because other conditions are also associated
with Mallory bodies (see Vignettes 34 to 37 for details), not the least of which is
alcohol-induced liver injury, some have suggested testing for phospholipidosis to
help distinguish amiodarone from alcohol in the arrhythmic alcoholic on amio-
darone (the “triple A”) with liver test abnormalities. Bear in mind, however, that
amiodarone is not alone in causing phospholipidosis; other culprits include ami-
triptyline, chloroquine, chlorpromazine, and thioridazine, among others.

Here’s the Point!

Arrhythmias + Liver test abnormalities + Phospholipid-laden lysosomal


lamellar bodies = Amiodarone toxicity

16. This is most likely cholestasis from trimethoprim-sulfamethoxazole (TMP-


SMX). TMP-SMX can cause an acute cholestatic injury, typically seen biochemi-
cally with an ALP greater than twice the upper limit of normal or an ALT/ALP
ratio of <2. This is in contrast to acute hepatocellular injury, in which the ALT
is at least twice the upper limit of normal and the ALT/ALP ratio is >5. Drug-
induced cholestasis is traditionally divided into the “bland” type of pure cana-
licular injury, in which there is primarily an elevated ALP out of proportion to the
liver enzymes, and so-called hepatocanalicular injury, in which the liver enzymes
are elevated along with the ALP. The latter injury can be more serious than the
pure canalicular pattern, since it often involves hepatocyte necrosis and destruc-
tive cholangitis. Another commonly used antibiotic, erythromycin, is especially
known to cause the mixed hepatocanalicular pattern. But erythromycin is not
58 Chapter 2

commonly employed for the treatment of urinary tract infections, making it less
likely to be the culprit in this particular case. However, if you see a patient with
gastroparesis who is now developing cholestasis, you might consider erythro-
mycin, which is sometimes used as a prokinetic. Amoxicillin-clavulanate and
tetracycline are also known to cause cholestatic injury, including the mixed
hepatocanalicular pattern. You might have thought about nitrofurantoin, which
is indeed used for urinary tract infections and is also known to cause liver inju-
ry. However, nitrofurantoin is most commonly associated with a hepatocellular
injury marked by elevations in the AST and ALT, and with an autoimmune-type
of chronic hepatotoxicity; it’s less commonly associated with an acute cholestatic
injury pattern.

Here’s the Point!

UTI + Cholestasis = Think TMP-SMX


URI + Cholestasis = Think amoxicillin-clavulanate
Gastroparesis + Cholestasis = Think erythromycin

17. This is most likely acute hepatocellular injury from diclofenac. Diclofenac
is a nonselective nonsteroidal anti-inflammatory drug (NSAID) that is known to
cause liver injury. In fact, the relationship between diclofenac and liver injury is
strong enough for the FDA to have issued a public warning in December 2009
regarding cases of diclofenac-induced hepatotoxicity. There have been instances
of diclofenac causing acute liver failure requiring transplantation or leading to
death. For some reason the relationship between diclofenac use and liver injury
seems stronger in women than in men. The FDA suggests that providers monitor
liver enzymes “periodically” in patients receiving long-term diclofenac therapy,
and definitely monitor liver enzymes within the first 4 to 8 weeks of therapy.
The acute liver injury caused by diclofenac can be idiosyncratic; it’s hard to
predict when, and in whom, it will occur. But acute liver failure is thankfully rare
(the true incidence is not clear, since it’s hard to know the exposed denomina-
tor). Asymptomatic liver enzyme elevations greater than 3 times the upper limit
of normal are more common and better described, occurring in around 1% to 4%
of patients receiving diclofenac.
It’s notable that diclofenac is also associated with an autoimmune-type chronic
hepatitis in addition to the acute hepatotoxicity-type injury. This form of chronic
liver injury is similar to the type I, or classic, form of autoimmune hepatitis. The
histologic appearance is also similar to autoimmune hepatitis, and patients with
diclofenac-induced autoimmune hepatitis are also more likely to have underlying
autoimmunity, including antinuclear antibody (ANA) positivity or the presence of
smooth muscle antibodies. As an aside, other drugs associated with the auto-
immune-type of hepatotoxicity include methyldopa, minocycline, nitrofurantoin
(see Vignette 16), phenytoin, and propylthiouracil, among others.
“Tough Stuff” Vignettes 59

By the way, if you were thinking about acetaminophen when you read the
vignette, you were wrong—acetaminophen is not an anti-inflammatory, and
warnings about its liver toxicity were issued way before 2009. But more on acet-
aminophen-induced liver damage in Vignette 27.

Here’s the Point!

NSAID use + Liver trouble = Think diclofenac

Here’s the Point!

Autoimmune-type chronic liver injury from medications, think:


z Diclofenac
z Minocycline
z Nitrofurantoin
z Phenytoin
z Propylthiouracil

18. This is hepatocellular injury from azathioprine (AZA) or 6-mercaptopu-


rine (6-MP). For a full explanation of the metabolic by-products and associated
adverse events of AZA/6-MP, refer to the first Acing book; there we provide a
detailed account of how AZA/6-MP is metabolized and how to interpret variations
in the levels of its by-products. In short, AZA is nonenzymatically converted into
6-MP. This, in turn, can be broken down into several by-products, one of which is
6-methylmercaptopurine (6-MMP), an inactive metabolite that can cause hepato-
toxicity. The breakdown of 6-MP to 6-MMP is catalyzed by the enzyme thiopurine
methyltransferase (TPMT). This enzyme is important because genetic variations
in TPMT have big implications in AZA/6-MP dosing and toxicity. Ninety percent
of the population has high TPMT enzyme activity, in which there is shunting to
high production of 6-MMP. These patients can have hepatotoxicity with doses of
AZA/6-MP that would normally be therapeutic. Levels of 6-MMP above 5700 are
typically associated with liver toxicity and should be avoided.
60 Chapter 2

Here’s the Point!

Patient with IBD bumps liver enzyme after starting AZA/6-MP = Think about
low TPMT enzyme activity versus supratherapeutic dosing

19. This is veno-occlusive disease (VOD), also known as sinusoidal obstruc-


tion syndrome (SOS), from AZA. So AZA strikes again. VOD is marked by a non-
thrombotic obstruction of the central hepatic venules with resulting sinusoidal
dilation and congestion throughout the liver. It’s caused by a diffuse endothelial
injury of unclear etiology. Although the exact pathogenesis remains unclear,
risk factors for VOD are well recognized. AZA is a classic culprit, although the
absolute risk of developing VOD from AZA is exceedingly small, especially when
AZA is used in inflammatory bowel disease (there are case reports, but they are
very rare). Other more common culprits include bone marrow transplantation,
in which VOD complicates up to 50% of cases, and various forms of chemo-
therapy (especially busulfan and cyclophosphamide) and Jamaican herbal teas.
Clinically, patients with VOD typically have a conjugated hyperbilirubinemia
>2 mg/dL, ascites, painful hepatomegaly, and marked weight gain over a short
period. Thrombocytopenia may occur as well. Liver biopsy is not necessary to
make the diagnosis, but when it’s performed (usually through a transjugular
approach given the thrombocytopenia), it reveals widening of the subendothe-
lium in the central venules and sinusoidal congestion. Whereas treatment for
VOD is very difficult in the setting of bone marrow transplant, case reports of
AZA-induced VOD often reveal biochemical and symptomatic resolution following
discontinuation of the offending agent.

Here’s the Point!

Patient with IBD gets painful hepatomegaly and has high bilirubin
level after starting AZA = Think about rare but serious veno-occlusive disease

20. This is peliosis hepatis from anabolic steroids. Peliosis hepatis is a rare
liver disorder marked by multiple blood-filled cystic spaces throughout the liver
parenchyma (not to be confused with VOD, as described in Vignette 19). The cysts
of peliosis hepatis do not have an endothelial lining but instead communicate
directly with the hepatic sinusoidal system. Peliosis is associated with a range of
conditions, including HIV, where it’s most commonly a consequence of Bartonella
infection, but it can also be found in nonimmunosuppressed patients.
Peliosis has been described in users of anabolic steroids, as well users of aza-
thioprine (there it is again), oral contraceptives, vitamin A, and hydroxyurea,
among many other medications. Rather than memorize this tedious list, you
“Tough Stuff” Vignettes 61

should focus on remembering anabolic steroids and HIV as the most common
and most important associations. Peliosis usually presents with fever, weight
loss, and jaundice in the setting of an enlarged liver. Because peliosis involves
cystic lesions in the liver, the ALP and GGT are elevated and out of proportion to
the AST and ALT (which are elevated to a lesser degree). The bilirubin is charac-
teristically normal or near normal. CT scans will show an enlarged and heteroge-
neous liver, and may also reveal a large spleen and abdominal lymphadenopathy.
Treatment is to remove the culprit agent. However, when peliosis occurs in the
setting of acquired immunodeficiency syndrome (AIDS), you should treat with
erythromycin 2 g daily (which hopefully will not cause a cholestatic hepatocana-
licular injury) to empirically cover Bartonella infection.

Here’s the Point!

AIDS + Elevated ALP/GGT + Fever + Weight loss + Enlarged liver + Bartonella =


Peliosis hepatis

21. This is hypervitaminosis A. Vitamin A (retinoic acid) can be a good thing,


but too much of a good thing can cause problems. There are high concentra-
tions of vitamin A in certain foods, including liver, kidney, and egg yolks. Beta-
carotene, in contrast, is a pro-vitamin A substance. Whereas vitamin A is found
in animal products, beta-carotene is usually found in green, leafy vegetables.
Another important difference is that vitamin A can cause direct liver injury,
whereas beta-carotene does not—it needs to be converted to vitamin A before
wreaking havoc. So it’s less common to develop liver injury from beta-carotene
overdose than from vitamin A overdose. Most commonly, however, hypervita-
minosis A occurs from taking too much retinoic acid supplementation. This is
especially common among “health nuts” who view vitamin A as an important
part of their overall health and well-being. Vitamin A can cause proliferation of
the stellate cells with lipid infiltration and, over time, is even associated with cir-
rhosis and veno-occlusive disease.

Here’s the Point!

Health freak + Lipid-filled stellate cells = Vitamin A toxicity

22. No, you should not stop the statin. This mini-vignette provides an excuse
to review the hepatotoxicity (or lack thereof) of statin therapy. There is no ques-
tion that statins are associated with liver test abnormalities, but that does not
mean that statins are dangerous or need to be stopped at the earliest sign of liver
62 Chapter 2

trouble. Around 10% of statin users will have mild elevations in their ALT levels.
Usually these elevations are pretty minimal, although in up to 1% to 3% of users
the elevations exceed 3 times the upper limit of normal (ULN). However, some
large, randomized controlled trials of statin versus placebo have found virtually
no difference in the prevalence of elevated ALT between statin and placebo. One
study evaluated lovastatin versus placebo in over 6500 subjects in a 5-year fol-
low-up study and found that ALT elevations exceeding 3 times ULN were virtu-
ally the same between the lovastatin (0.6%) and placebo (0.3%) groups. Similarly,
multiple studies in thousands of patients have found no difference in ALT levels
between patients using simvastatin versus placebo. The same thing has been
found with pravastatin and atorvastatin. Fluvastatin may have a slightly higher
risk than placebo, but even there the risk is small.
You get the point—statins probably don’t do much to the liver (if anything),
and they end up getting a bad rap. Yet gastroenterologists and hepatologists
are bombarded by consults asking them to comment on or manage the risk of
statin-induced hepatotoxicity. Hepatologists, in particular, usually wrinkle their
brows at the consult and mumble something about statin-induced hepatotoxicity
being overblown. In fact, a recent review of the topic in the American Journal of
Gastroenterology by T. Bader started like this: “Statin-induced hepatotoxicity is
a myth. ‘Myth’ is used here to mean a false collective belief that, despite factual
contradiction, endures as suspicion.” Bader went on to emphasize that the “eleva-
tion of serum ALT is not a disease. At worst, the ephemeral out-of-range ALT val-
ues represent adaptation to exposure to statins by the different organs involved
in ALT regulation. In the liver this is done by alteration of metabolic enzyme and
transporter systems to process the drug. When a statin is continued, despite
elevations of ALT, the ALT eventually returns to normal unless some other cause
for liver disease exists.” Notably, transient ALT elevations have also been observed
in other classes of anticholesterol agents, suggesting that transient rises may
actually reflect that the drug is working and that the elevation is, in fact, the
result of an agent lowering cholesterol, not a direct cause of parenchymal dam-
age. In fact, statins are being studied by hepatologists in the setting of nonal-
coholic fatty liver disease (NAFLD) as a therapeutic approach to managing the
metabolic syndrome. So, if liver docs are comfortable using a statin even in the
setting of liver disease, internists should probably be comfortable using the drug
in settings without liver disease, even if there is a transient minor rise in amino-
transferase levels. Perhaps surprisingly, there are even data that the efficacy of
anti-HCV therapy is enhanced in the setting of statin therapy, and simvastatin
has even been employed as a potential therapy for portal hypertension.
So what are the recommendations for liver test monitoring in patients on
a statin? The current label for lovastatin no longer requires routine liver test
monitoring in the absence of known liver disease. The label suggests monitoring
liver tests only if there is a history of liver disease or if there are clinical signs or
symptoms of liver disease. For simvastatin and pravastatin, the guidance is simi-
lar—just follow liver tests “if clinically indicated.” For fluvastatin, atorvastatin,
and rosuvastatin, the guidance is to check liver tests before and 12 weeks after
the initiation of therapy and “periodically (e.g., semiannually)” thereafter. But
there is no reason to stop the drug, especially if the ALT elevations remain below
3 times ULN.
“Tough Stuff” Vignettes 63

Here’s the Point!

If you are asked to stop a statin because of hepatotoxicity concerns, the


answer is more often than not “No.”
64 Chapter 2

Vignette 23: Big Knuckles


A 39-year-old Caucasian woman presents with complaints of hand discomfort
with “big and swollen knuckles.” She enjoys fishing and has had more difficulty
using her reel. She states that she had an “early menopause” and has been
amenorrheic for the past 4 years. She emphasizes that she has a very healthy
lifestyle and diet. She loves to eat fresh seafood and takes an assortment of vita-
mins daily to keep herself “healthy.”
On examination, you find that her skin appears tanned, there is no hepato-
splenomegaly, and there are no stigmata of chronic liver disease. Her BMI is 32.
Hand X-rays reveal degenerative arthritis of her second and third metacarpopha-
langeal (MCP) and proximal interphalangeal (PIP) joints. Fasting laboratory tests
reveal an iron saturation of 75%, ferritin = 432, INR = 1.0, total bilirubin = 0.9,
creatinine = 0.8, glucose = 130, albumin = 4.1, ALT = 19, AST = 23, ALP = 98,
WBC = 5.8, hemoglobin = 13.9, platelets = 225, with negative autoimmune and
viral hepatitis serologies.

` What is the diagnosis?


` What testing should be performed to confirm the diagnosis?
` Will treatment help her knuckles?
` What can you tell the patient about life expectancy?
` What lifestyle adjustments should you recommend?
“Tough Stuff” Vignettes 65

Vignette 23: Answer


This clinical scenario is consistent with hereditary hemochromatosis, an auto-
somal recessive disorder that leads to excessive intestinal iron absorption due
to genetic mutations (most commonly in the HFE gene). You probably knew that
already, which is good. But the Board exam probably will not ask you simply to
diagnose hereditary hemochromatosis; it will probably ask some more nuanced
questions, and that is why we are taking this opportunity to review the classic
clinical pearls of hemochromatosis.
The clinical manifestations of hemochromatosis arise from excessive iron
deposition in the liver, pancreas, heart, and pituitary gland. This woman pre-
sented with skin hyperpigmentation, which is present in many patients with
hemochromatosis (even at the early stages), prominently on sun-exposed regions
of the skin. The skin changes result from iron deposition and increased mela-
nin synthesis in melanocytes. The patient also presents with diabetes, which
can be caused by iron accumulation in the pancreatic beta cells in the islets of
Langerhans and is present in up to half of hemochromatosis patients. Remember
the classic “bronze diabetes” sign of hemochromatosis that you learned in medi-
cal school? Both components of “bronze diabetes”—namely, cutaneous hyper-
pigmentation and hyperglycemia—can improve with phlebotomy. However, phle-
botomy does not generally help arthropathy, decompensated cirrhosis, or hypo-
gonadism from excess iron deposition in the pituitary. Therefore, this patient’s
menses are unlikely to resume after phlebotomy, and her arthropathy probably
won’t improve either. Furthermore, keep in mind that the arthropathy does not
correlate with the severity of liver disease.
Serum HFE mutation genotype testing is the next step in diagnosing this
patient. Since the diagnosis can be readily established in this case with a simple
laboratory test (when other options are possible such as liver biopsy), this makes
for a great test question. Liver biopsy was previously the gold standard for diag-
nosis, but it’s no longer needed in all patients.
You might be asking, “But how do we know she does not have cirrhosis?” You
might feel comfortable diagnosing hemochromatosis without a biopsy, but how
can you rule out cirrhosis without performing a biopsy? It turns out that cirrho-
sis is very rare in hemochromatosis if the AST is normal, there is no hepatomeg-
aly, the ferritin is less than 1000 ng/mL, and the patient is less than 40 years
old. Thus, it’s unlikely that this patient has cirrhosis, and a liver biopsy and its
inherent risks can be avoided.
There are further practical implications as well. For example, this patient may
be denied life insurance on the basis of hemochromatosis. However, you can help
appeal to the life insurance company that her life expectancy is similar to that
of age-matched controls, assuming she is compliant with phlebotomy, if she does
not have advanced fibrosis on biopsy. So, after all, you might end up getting a
liver biopsy to prognosticate for her life insurance purposes.
Contrary to this case, women tend to present later than men with symp-
tomatic hemochromatosis, since menses provide a route of iron depletion. This
patient presented with “early menopause”; however, she was likely amenorrheic
due to pituitary iron deposition. A classic Board factlet is that hemochromatosis
often remains undetected in women until after menopause. Men tend to develop
sequelae far earlier than women do.
66 Chapter 2

Iron in the liver is primarily deposited in periportal hepatocytes in hemochro-


matosis. As the disease progresses, iron is deposited in the pericentral hepato-
cytes, macrophages (Kupffer cells), and bile duct cells. In contrast, in a secondary
hemosiderosis condition, iron is primarily deposited in macrophages rather than
in the hepatocytes. See Vignette 86 for more on the distinction between hemo-
chromatosis and hemosiderosis from secondary iron overload (Figure 23-1).
Of note, there is at least a 20-fold increased risk of hepatocellular carcinoma
in patients with hemochromatosis-related cirrhosis (as opposed to the risk in
patients with autoimmune hepatitis-related cirrhosis, where hepatocellular car-
cinoma has only a 1% annual risk). Furthermore, recent data have shown excel-
lent post-transplant survival with hemochromatosis-related cirrhosis (86% at
1 year), similar to that with other causes of cirrhosis. This is in contrast to earlier
reports in which the outcomes were not as favorable. However, patients can still
develop cardiovascular complications (cardiomyopathy, congestive heart failure,
and arrhythmias from cardiac iron deposition) following liver transplantation.
Important lifestyle changes should be recommended for this patient. She
should avoid supplemental vitamin C (ascorbic acid) intake, which can increase
free iron, leading to oxidative injury and even rapidly fatal cardiomyopathy when
ingested in large amounts. She should also avoid uncooked seafood and ensure
that her food is properly cleansed, since certain bacteria (Vibrio vulnificus,
Yersinia enterocolitica, Yersinia pseudotuberculosis, Listeria monocytogenes) can
flourish in an iron-rich serum, which can lead to fatal septicemia.
Why Might This Be Tested? Hereditary hemochromatosis can present with
a number of signs and symptoms and can be effectively treated if caught early
in its course. It also has important lifestyle implications, and Board examiners
want to know that you are thinking beyond the algorithmic approach to this
condition and also know about key lifestyle advice for the patient. This condition
creates a perfect recipe for a variety of potential Board questions. Nearly every
sentence of this answer is testable, so know this section well.
Clinical Threshold Alert: A patient under 40 years of age with a ferritin level
of less than 1000 ng/mL is unlikely to have cirrhosis, and liver biopsy can often
be avoided.

Figure 23-1. T-2 weighted MRI image shows


the low signal intensity in the liver compared
with the normal spleen signal intensity in
hemochromatosis.
“Tough Stuff” Vignettes 67

Here’s the Point!

“Bronze diabetes” + Arthropathy + High ferritin = Hereditary hemochromatosis


68 Chapter 2

Vignette 24: Abdominal Rash


A 52-year-old man is receiving interferon and ribavirin therapy for chronic
hepatitis C infection. Four weeks into his therapeutic course, he presents for fol-
low-up and is found to have a new lesion on his abdomen, pictured in Figure 24-1.
No other lesions are noted, and his mucous membranes are unremarkable.

Figure 24-1. Abdominal lesion for Vignette


24.

` What is this?
` What are you going to do about it?
“Tough Stuff” Vignettes 69

Vignette 24: Answer


This is a cutaneous reaction to interferon (IFN) injections. You might have
known that right off the bat if you’ve seen it before, but serious injection reac-
tions to IFN are relatively unusual. You might have thought about other HCV-
related skin abnormalities, such as porphyria cutanea tarda (PCT) or mixed
cryoglobulinemia with a leukocytoclastic vasculitis. However, those reactions
look different, as will be described later in this discussion.
Tissue damage at the site of injection is observed with all of the different forms
of IFN, although at different rates. The reaction is most common in patients
who always use the same injection site; for this reason it’s important to instruct
patients to rotate their injection site and never use the same site twice in a row.
Patients should also avoid injection sites near the waistline, where irritation is
more common. The injection site in this photograph is actually just fine—between
the navel and waistline.
Injection site reactions occur in up to 60% of patients on IFN, but they are
usually not severe. The most common reaction is local erythema. More signifi-
cant damage involves local tissue damage or necrosis (as pictured), which can be
further complicated by localized cellulitis. In this case, the patient did have evi-
dence of surrounding cellulitis and was treated with cephalexin. The IFN was not
discontinued—he did fine after rotating his site and being treated for cellulitis.
However, the spectrum of cutaneous adverse effects goes beyond localized
necrosis and cellulitis. In fact, the package insert for pegylated IFN alfa-2a states
that “serious skin reactions including vesiculobullous eruptions, reactions in
the spectrum of Stevens Johnson syndrome (erythema multiforme major) with
varying degrees of skin and mucosal involvement and exfoliative dermatitis
(erythroderma) have been rarely reported in patients receiving PEG-IFN alfa-2a
with and without ribavirin.” The insert goes on to say that “patients developing
signs or symptoms of severe skin reactions must discontinue therapy.” With less
severe reactions such as the one pictured, discontinuation is not mandatory. The
patient should start by rotating the site of injection, continuing to stay away from
the waistline. If site reactions persist despite this maneuver, and especially if
there is persistent cellulitis or other complications, then discontinuation is gener-
ally warranted, although that decision is usually a case-by-case determination.
While we are in the neighborhood of HCV and skin lesions, we should review
a few other important dermatologic associations of HCV and its treatment. For
example, ribavirin is also strongly associated with various skin reactions. The
dermatological side effects of ribavirin usually occur early in the treatment period
and may be due to histamine-like reactions to the agent. Patients describe vari-
ous types of ribavirin-related rashes spanning from localized erythema to con-
fluent erythematous rashes and vesicular lesions. Some patients also describe
itchiness, nasal stuffiness, and even asthma-like symptoms—also probably his-
tamine related. Because the rashes are thought to be histamine mediated, some
recommend using topical therapies such as 1% hydrocortisone or triamcinolone.
If such interventions do not work, then discontinuation of ribavirin is warranted,
which usually leads to clearing of the skin lesions.
As for PCT, this topic could fill a book. But the short story is that PCT is the
most common of the various hepatic porphyrias. There are both sporadic and
autosomal dominant inherited forms of PCT. In both forms, there is reduced
70 Chapter 2

activity of uroporphyrinogen decarboxylase (also known as UROD), which leads


to a buildup of uroporphyrinogen in the blood. We could put a figure in this
vignette to show you the relevant metabolic pathways, but we’re guessing you
either already know it (through some miracle of brute memorization or hands-on
research in the area), or will never know it. Most people fall in the latter cat-
egory, so we’re just being realistic by keeping that figure out of the book since
it’s unlikely to show up on the Boards. Our focus here is on the characteristic
dermatologic manifestations that can occur when uroporphyrinogen backs up
in the blood. Patients with PCT classically have skin reactions that occur in
response to sun exposure. The typical PCT lesion consists of vesicles and bullae
on the dorsal aspects of the hands. Other sun-exposed areas, such as the face,
legs, dorsa of the feet, and forearms, are frequently involved.
Leukocytoclastic vasculitis, another HCV-related skin abnormality, typically
presents with palpable purpura in the lower extremities in the setting of underly-
ing mixed cryoglobulinemia. Hepatitis C is the most common cause of cryoglobu-
linemia. Up to half of patients with hepatitis C have detectable cryoglobulins,
although most patients do not suffer consequences from these temperature-sen-
sitive, circulating immunoglobulins. Cryoglobulins precipitate out of solution at
temperatures below 98.6°F and can redissolve when rewarmed. This is relevant
because cryoglobulins can precipitate out of solution while in blood vessels,
leading to a wide range of seemingly disparate consequences, including arthri-
tis, glomerulonephritis, neuropathy, abdominal pain from bowel vasculitis, and
a leukocytoclastic vasculitis. Cryoglobulinemia is also marked by low C4 levels
and a positive rheumatoid factor. So the presence of any of these features plus
some skin lesions in HCV should make you think of cryoglobulinemia.
Why Might This Be Tested? Board examiners seem to love testing on derma-
tologic manifestations of underlying illness, and they love testing on hepatitis C.
So the combination makes for a perfect storm.

Here’s the Point!

Hepatitis C + Interferon treatment + Infraumbilical necrotic lesion/cellulitis =


Interferon injection adverse reaction

Here’s the Point!

Hepatitis C + Interferon treatment + Confluent erythematous rashes or vesicles


+ Pruritus = Ribavirin adverse reaction
“Tough Stuff” Vignettes 71

Here’s the Point!

Hepatitis C + Bullae and vesicles on sun-exposed skin =


Porphyria cutanea tarda

Here’s the Point!

Hepatitis C + Lower extremity purpuric rash + Paresthesias + Positive rheuma-


toid factor + Low complement + Renal disease = Mixed cryoglobulinemia
72 Chapter 2

Vignette 25: Liver Lesion With Central Scar


A 24-year-old woman presents with persistent right upper quadrant (RUQ)
discomfort for the past several months and tells you that she “feels something
underneath” her ribs. There has been no jaundice, weight loss, or fevers. She has
no other significant medical history. Laboratory tests, including liver panel and
AFP, are normal. Cross-sectional imaging reveals a noncirrhotic liver with an
8 cm lesion with calcifications within a central scar, as depicted in Figure 25-1.

Figure 25-1. Abdominal CT of the patient


in Vignette 25. (Reprinted with per-
mission of Barbara Kadell, MD, UCLA
Medical Center.)

` What is the diagnosis?


` What should you recommend?
“Tough Stuff” Vignettes 73

Vignette 25: Answer


The best answer is probably not the most obvious: fibrolamellar hepatocellular
carcinoma (FLHCC). Your first thought may have been that this is a benign diag-
nosis of focal nodular hyperplasia (FNH). The “central scar” is the usual Board
buzzword for FNH. However, it’s extremely rare to have calcifications with FNH,
even if there is a central scar. Calcification with a central scar highly suggests
the more ominous lesion of FLHCC. Also, on T2-weighted images on MRI, the
central scar of FLHCC is dark or hypointense, whereas the central scar of FNH
is bright or hyperintense. Steady enlargement of this lesion, if resection were
delayed, would also imply the diagnosis of FLHCC.
This patient represents a typical clinical scenario seen with FLHCC (a young
patient with vague and persistent RUQ pain, normal synthetic function, and a
large lesion found on imaging). It’s uncommon (less than 20%) to have an elevated
AFP with FLHCC, so the patient’s normal AFP value also supports this diagnosis.
This lesion is found equally in both young men and women. Treatment consists of
aggressive surgical resection, which provides a 5-year survival of around 75%—a
value that is surprisingly similar to that for the usual HCC. However, resection is
typically more feasible in FLHCC (as opposed to HCC) due to the normal underly-
ing synthetic function of the liver. Figure 25-2 demonstrates a surgical specimen
of FLHCC. Look closely: The surrounding liver appears pretty normal; it’s not
cirrhotic. In contrast, a typical HCC often occurs in the setting of background
cirrhosis. Notice that the central scarring is apparent even on the macroscopic
appearance of the lesion. Figure 25-3 shows the microscopy of the FLHCC lesion.
Compared to HCC, FLHCC has bigger, more granular cells, along with so-called
pale bodies and fibrous bands.
FNH is the second most common benign tumor of the liver behind heman-
giomas. FNH consists of complexes of benign hepatocytes separated by fibrous
bands with prominent bile duct proliferation and malformed blood vessels typi-
cally within a central scar. This lesion rarely requires resection unless markedly
symptomatic. Since more than half of FNH lesions have a central scar, you can
bet that an FNH on the exam will have a central scar. Figure 25-4 depicts an
FNH.

Figure 25-2. Resected fibrolamellar


hepatocellular carcinoma (FLHCC).
Notice the lack of cirrhosis in the sur-
rounding, normal-appearing parenchy-
ma. Whereas typical HCC occurs in
the setting of cirrhosis, FLHCC usu-
ally occurs without underlying cirrhosis.
There is a central scar with calcification
that is consistent with the CT imaging in
Figure 25-1. (Reprinted with permission
of Charles Lassman, MD, UCLA Medical
Center.)
74 Chapter 2

Figure 25-3. Microscopy of FLHCC


lesion demonstrating large, granu-
lar cells with intervening fibrous
bands. (Reprinted with permission of
Charles Lassman, MD, UCLA Medical
Center.)

The differential diagnosis could include the following:


z Hemangioma
z Hepatic adenoma
z Hepatocellular carcinoma (usual type)
z Cholangiocarcinoma
z Metastasis

Figure 25-4. Arterial phase from triphasic


CT scan of a focal nodular hyperpla-
sia lesion with a characteristic central
scar. This is a very different lesion from
FLHCC. (Reprinted with permission of
Javier Casilllas, MD, University of Miami.)
“Tough Stuff” Vignettes 75

However, the other lesions on this differential typically do not have a central
scar and thus would not be good choices to select on a multiple-choice question
on a Board exam.
Why Might This Be Tested? This is a case where the imaging provides guid-
ance for therapy. This was cancer and required resection—a very different course
of action than what might occur in FNH. Liver lesions are one of the most frequent
reasons for consultation with a gastroenterologist. Thus, you need to know this
for real life and are very likely to encounter liver lesions on the Board exam.

Here’s the Point!

Calcification with a central scar suggests FLHCC rather than FNH.


76 Chapter 2

Vignette 26: Alcohol Binge and a Portal Vein Thrombus


A 26-year-old Latina woman presents to the emergency department with
weakness, fatigue, and progressive jaundice over a 2-week period. She reports
over 10 years of “heavy” alcohol use since she was a young teenager. She recently
went on a drinking binge that culminated in nonbloody vomiting and abdominal
swelling (but not pain), in addition to her other chief complaints.
On examination in the emergency room, she is found to be alert to person,
place, and time but easily distracted. Her vital signs reveal a temperature of
99.9°F, blood pressure of 128/72, heart rate of 112, and respiratory rate of 18 with
98% oxygen saturation on room air. She has scleral icterus, mild conjunctival
pallor, and sublingual icterus. There is no cervical lymphadenopathy. Her lungs
are clear, and her heart is tachycardic but without murmurs. Her abdomen
reveals moderate distention but no fluid wave or shifting dullness on percussion.
Her abdomen is otherwise soft and nontender. Rectal exam yields brown stool.
There is 1+ lower extremity pitting edema. There are no spider angiomas and
no palmar erythema. Neurologic examination is nonfocal; there is no asterixis.
Laboratories include the following: WBC = 33.4 with 92% PMNs, INR = 1.8, pro-
thrombin time = 15.4, creatinine = 2.1, albumin = 2.8, total bilirubin = 19.8,
AST = 151, ALT = 74, ALP = 164. Ultrasound performed in the emergency depart-
ment reveals a large fatty liver, enlarged spleen, and evidence of hepatofugal flow
with a portal vein thrombus. There are no stones in the gallbladder, and the
common bile duct is 4 mm in diameter. She is now admitted to your service from
the emergency department for further inpatient management.

` What is the most likely diagnosis?


` What will your treatment plan be?
` How are you going to handle the portal vein thrombus?
“Tough Stuff” Vignettes 77

Vignette 26: Answer


This is acute alcoholic hepatitis (AH) in the setting of underlying chronic liver
disease. Although this patient is young, she still can have chronic liver disease
or even outright cirrhosis. When this patient arrived in the emergency depart-
ment, there was confusion about what was going on. Was this just acute AH?
Or was there underlying cirrhosis as well? Wasn’t she too young to have cirrho-
sis? And what about this portal vein thrombus? Did the thrombus need to be
actively treated? Was there something else altogether underlying this pattern of
liver injury, like autoimmune hepatitis, acute viral hepatitis, and so forth? (The
patient had no records on file at the time of her emergency room visit.) Or was
this just an impacted gallstone?
Before we get to those questions, it’s important to review the spectrum of
alcohol-related liver damage. The usual range of liver injury begins with simple
steatosis, which could evolve to steatohepatitis, which could ultimately lead to
early fibrosis and outright cirrhosis. But these stages are not mutually exclusive
or distinct. The spectrum of liver injury does not follow a perfectly linear path,
but instead can be erratic and overlapping. Some people exhibit different forms
of liver injury at once. In this patient, who unfortunately died after a prolonged
inpatient hospitalization, the autopsy revealed evidence of alcoholic liver dis-
ease (ie, Mallory bodies, megamitochondria, inflammatory infiltrate) and early
perisinusoidal fibrosis, but not cirrhosis (ie, no evidence of regenerative nodules
with bridging fibrosis). Yet when she presented to the emergency room, there was
evidence of portal hypertension, as indicated by the ascites, hepatofugal flow on
ultrasound, and a portal vein thrombus (more on that later). Alcohol is known to
transiently increase portal pressures, so even if a patient does not harbor out-
right cirrhosis, she can appear to have cirrhosis in the setting of acute alcohol
toxicity. The woman in this vignette had portal hypertension, fibrosis (but not
outright cirrhosis), and evidence of acute AH, supported by the high AST:ALT
ratio (well over the traditional 2:1 ratio seen in acute AH), leukocytosis, and high
Maddrey discriminant function (DF) score driven by the elevated INR and total
bilirubin (more on the DF score later).
Fatty liver is common in people who drink a lot of alcohol. This can show up on
ultrasonography when there is more than 30% steatosis in the liver. In fact, up
to 90% of people who consume more than 60 g of alcohol per day have steatosis.
In the United States, the standard “drink,” which includes a 12-oz can of beer,
5-oz glass of wine, or 1.5-oz shot of hard liquor, contains roughly 12 g of alcohol
by weight. (By the way, Japan tends to have the “strongest” drinks by average
alcohol content.) So 60 g of daily alcohol ingestion equates to around 5 drinks per
day. Almost everyone who drinks this much alcohol has hepatic steatosis.
The progression to advanced liver injury depends on several variables. Gender
is an especially strong predictor of alcohol-induced liver injury; women are gen-
erally more susceptible to the hepatotoxic effects of alcohol than men due to
gender-related enzymatic differences. Whereas it often takes 60 g of alcohol per
day for around 10 years for men to progress to chronic injury, it may take only
20 g per day to achieve the same effect for women. This may well explain how
this patient, who was only 26 years old, had already developed advanced fibrosis.
Another factor that predicts liver injury is the pattern in which alcohol is ingest-
ed. Data indicate that binge drinkers (like this woman) have a higher incidence of
78 Chapter 2

advanced injury and mortality compared to people who drink only during meals,
even if the same amount of alcohol is ingested over the long run. So there may
be something about getting the hit all at once that really kick-starts the injury
cascade. Race may also play a role in predicting liver injury from alcohol. Data
reveal that rates of cirrhosis are higher in African-Americans and Latinos than
in Caucasians, and alcohol-related mortality is highest in the Latino population,
independent of the amount of alcohol consumed. This may reflect racial varia-
tions in alcohol dehydrogenase, among other factors both known and unknown.
This patient happened to be a Latina woman, so she carried two independent
risk factors for accelerated liver injury from alcohol.
Simple steatosis is often fully reversible and usually (but not always) subsides
after 6 weeks of abstinence. Unfortunately, even with abstinence some people (up
to 15% in some series) still progress to fibrosis and cirrhosis. Once steatosis has
set in, it does not take that much more drinking to keep the damage coming.
Only 40 g per day of continued drinking (around 3 drinks per day) is enough to
progress to fibrosis and cirrhosis in approximately one third of individuals with
alcoholic steatosis.
Acute AH itself can range from mild injury to catastrophic, life-threatening
illness, as occurred here. It’s important to remember that acute steatohepatitis
often occurs in the setting of underlying chronic liver disease. The mere pres-
ence of acute AH suggests a strong history of drinking and is a surrogate marker
for underlying chronic liver injury, not just acute injury. Of note, in the setting
of cirrhosis, the traditional AST:ALT ratio of 2:1 does not always hold, so you
should still keep AH in mind even if the ratio is different. It’s useful to calcu-
late the Maddrey DF score when you suspect AH. The DF is a disease severity
score developed years ago to predict mortality. The Board exam will probably not
ask you to calculate a DF, and most people have a calculator for this on their
handheld devices. But with that said, here is the equation: DF = 4.6 * (Patient’s
Prothrombin Time – Control Prothrombin Time) + Total Bilirubin. The main
thing to remember is that the prothrombin time and total bilirubin are the
2 biochemical factors that predict mortality in AH, at least by the Maddrey DF
standard. The risk of mortality shoots up when the score exceeds 32. The MELD
score, which is generally used for allocating liver transplants (see Vignette 9),
can also be useful to predict outcomes in acute AH; the prognosis is worst when
the MELD exceeds 18. Both the DF and MELD scores include prothrombin time
(or INR) and total bilirubin levels, although the MELD also includes creatinine.
Aminotransaminases are not part of any validated prognostic scores in hepatol-
ogy, namely because they are not liver function tests, despite commonly being
referred to as LFTs. See Vignette 1 for more on that.
If a patient’s DF exceeds 32 or MELD exceeds 18 (as in this case), it’s time to
begin timely medical interventions. Whereas supportive therapy is important
for everyone with acute AH (ie, nutritional support with vitamins, monitoring
for alcohol withdrawal), more aggressive therapy is needed for patients with a
poor prognosis. The usual decision is whether to begin a systemic steroid, such
as prednisolone, or anti-TNF therapy, such as pentoxifylline. Both are accept-
able, and it would be hard to imagine the Board exam allowing one as a correct
answer but not the other. That said, there are a few nuances to know about
these different approaches. Regarding steroids, the effect on mortality is small
and somewhat controversial (different results in different studies), but the overall
“Tough Stuff” Vignettes 79

effect from meta-analysis is positive, with a number needed to treat of 5 (ie, for
every 5 people with AH that you treat with steroids instead of placebo, there is
1 additional survivor) for a short-term mortality benefit. This benefit appears
highest in patients with concurrent spontaneous hepatic encephalopathy. Most
of the data have been with prednisolone (40 mg per day for 4 weeks followed
by a taper), not prednisone. Since the liver is responsible for the conversion of
prednisone to the active metabolite prednisolone, which may be challenging in
a patient with poor synthetic function, prednisolone has been used more often.
A final point is that steroids are associated with more infections than placebo,
which undermines their benefit.
As for pentoxifylline, this oral phosphodiesterase inhibitor with anti-TNF activ-
ity is associated with a significantly lower risk of mortality than placebo, and
is especially effective in the setting of hepatorenal syndrome (HRS). Thus some
physicians prefer this therapy when the creatinine is elevated and there is suspi-
cion for underlying HRS (see Vignette 75 for more on HRS). It remains uncertain
whether combining both steroids and pentoxifylline is any better than either
therapy alone.
What about the portal vein thrombosis? In general, the answer is usually
nothing. Portal vein thrombosis is typically an epiphenomenon of high intrahe-
patic pressures in cirrhosis and hepatic fibrosis and is a surrogate marker for
hepatofugal (away from the liver) flow. When the intrahepatic pressure rises, the
pressure within the portal vein begins to rise in lockstep. After a while, the blood
can’t flow into the liver (see Figure 3-2), and it starts to back up, become static,
and then clot. But the clot, in and of itself, is not necessarily something to target
with therapy in this setting. It’s one thing if a clot organizes and extends down
the mesenteric system. It’s also a different situation if there is an underlying pro-
thrombotic condition giving rise to the clot (eg, factor V Leiden). But if the clot is
limited to the portal vein, it does not need treatment. In this case, the admitting
team began heparin, and after a traumatic paracentesis that lanced a recanaliz-
ing umbilical vessel, the patient exsanguinated and required emergency surgery
to find the bleeding source and achieve hemostasis. Suffice it to say anticoagu-
lation was not the answer for this particular portal vein thrombus. The answer
is abstinence from alcohol (obviously), medical therapy for acute AH, nutritional
support, watching for alcohol withdrawal, imaging to ensure that there isn’t a
complicating HCC with portal vein invasion, and monitoring (and hoping) for
biochemical evidence of improvement. Heparin did not have a role in this set-
ting. The admitting diagnosis from the emergency department was not alcoholic
hepatitis; it was portal vein thrombus. By mistaking an epiphenomenon as the
reason for admission, the entire focus of the admission was on the clot itself. The
patient did not receive steroids or pentoxifylline. She was very ill, so it remains
unclear if the therapy would have helped, but the point is to make the diagnosis
quickly, begin appropriate therapy right away in the setting of an elevated DF or
MELD score, and provide supportive therapy throughout.
Why Might This Be Tested? Everyone manages patients with acute AH, but
not everyone is good at diagnosing it. For such a common problem, it’s surpris-
ing how frequently acute AH is misdiagnosed and mismanaged. In this case, for
example, the patient had a low-grade fever and persistent leukocytosis with a left
shift that triggered an extensive workup. The workup not only included blood
cultures and other expeditions for underlying infection, but even included a bone
80 Chapter 2

marrow biopsy to screen for myeloproliferative disorders! In fact, this patient


was also treated empirically with broad-spectrum antibiotics. Yet leukocytosis
and low-grade fevers are classic features of acute AH; they are part of the illness
itself.
Another common situation is for a patient with AH to have an elevated ALP
along with the high total bilirubin, as occurred here. This almost inevitably leads
to an ultrasound and request for an endoscopic retrograde cholangiopancrea-
tography (ERCP) to screen for an impacted stone. Although acute AH frequently
presents with a high ALP and total bilirubin, the presentation is otherwise
totally different from that with an impacted stone, where there would typically
be a higher ALT than AST (see Vignette 32), and where there would be a charac-
teristic pattern of biliary colic. The bottom line is that it should not be hard to
distinguish acute AH from an impacted gallstone. If this vignette were on a Board
exam and you were asked for the diagnosis, the answer would be acute AH, as it
would usually be in real life also. So just make the AH diagnosis and get started
with appropriate therapy (while, of course, keeping a broad differential diagnosis
in the back of your mind).
Clinical Threshold Alerts: If the AST:ALT ratio exceeds 2:1 in the setting of
recent alcohol ingestion, think about acute AH. A “standard drink” in the United
States contains roughly 12 to 14 g of alcohol by weight. Ingesting 60 g of alcohol
per day for 10 years can lead to cirrhosis in men, whereas ingesting only 20 g
per day can lead to cirrhosis in many women. If the Maddrey DF score exceeds
32, or if the MELD exceeds 18, then initiate either prednisolone or pentoxifylline
in acute AH. If there is more than 30% hepatic steatosis, it can be detected by
ultrasonography.

Here’s the Point!

↑ WBC + ↑ Total bilirubin + ↑ ALP + Low-grade fever + ↑ Alcohol = Think


about acute alcoholic hepatitis, not just a gallstone or cholangitis!

Here’s the Point!

Portal vein thrombus in cirrhosis or acute alcoholic hepatitis? Treat the


underlying disease, not the epiphenomenon.
“Tough Stuff” Vignettes 81

Vignette 27: Empty Pill Bottle


A 23-year-old woman is brought into the emergency department by paramedics
after she was found lying on the floor next to an empty acetaminophen bottle in
her apartment. Her father had not heard from her after speaking to her 2 nights
ago when she was upset after a recent breakup with her boyfriend. She is noted
to be jaundiced and lethargic with unremarkable vital signs. Laboratory tests
reveal the following: pH = 7.23, INR = 3.1, total bilirubin = 16.1, creatinine = 2.8,
AST = 7210, ALT = 6590, albumin = 3.2, glucose = 52; acetaminophen level is at
the minimally detectable range.

` What should you do?


82 Chapter 2

Vignette 27: Answer


This patient is in serious trouble. She has acute liver failure (ALF) and is at
high risk of death given her pH of less than 7.3 (she meets King’s College Criteria,
as shown in Table 27-1). Although many patients with acetaminophen-induced
ALF recover with supportive care, this patient is unlikely to survive without
orthotopic liver transplantation. You need to take action quickly, and the patient
needs to be referred to a liver transplant center immediately. Transportation can
be dangerous, in and of itself, if there is increased intracranial pressure and/or
severe coagulopathy. Also, the liver transplant center will need time to perform
an expedited transplant evaluation. So when the patient is in bad shape, you
(and the patient) need to move quickly!
Okay, so the patient is getting admitted to the ICU and has been referred to a
transplant center. But, she will need treatment. Remember that acetaminophen
is a dose-dependent toxin that can cause ALF typically when more than 7.5 to 8
g are ingested. However, the hepatotoxicity is enhanced in patients who are con-
comitantly taking isoniazid, rifampin, phenytoin, or carbamazepine, and in alco-
holics, due to induction of the cytochrome P450 system. Furthermore, there will
be more damage in a fasting or malnourished state due to glutathione depletion.

Table 27-1.
KING’S COLLEGE HOSPITAL CRITERIA FOR LIVER
TRANSPLANTATION
Acetaminophen-Induced Acute Liver Failure
Arterial pH <7.3 (irrespective of the grade of encephalopathy)

OR

Creatinine >3.4 and INR >6.5 and grade 3 or 4 encephalopathy

Non-Acetaminophen-Induced Acute Liver Failure


INR >6.5 (irrespective of the grade of encephalopathy)

OR

Any 3 of the following:


1. Age <10 years or >40 years
2. Etiology: Idiosyncratic drug reactions, halothane hepatitis, non-A hepatitis, non-B hepa-
titis
3. Duration of jaundice before encephalopathy >1 week
4. INR >3.5
5. Bilirubin >18 mg/dL
* You don’t need to memorize this table (that would be fair game for the transplant hepatology
examination, not the GI exam). However, you should know the basics.
“Tough Stuff” Vignettes 83

Therefore, lesser amounts can also cause ALF in some patients. In the United
States there are also analgesic medications that combine acetaminophen (a dose-
dependent hepatotoxin) with an addictive narcotic in a single tablet (eg, Vicodin,
Percocet). Doesn’t really make sense to bundle them, does it? Taking too many of
these tablets (due to excessive pain or addiction) can accidentally lead to ALF—the
so-called therapeutic misadventure. Even though 48 hours may have transpired
since this patient ingested the acetaminophen, there may still be a benefit to using
N-acetylcysteine (NAC) therapy. Depending upon the clinical scenario, NAC can
be administered either orally or intravenously per standard protocol. Risk of inju-
ry can be estimated with the acetaminophen nomogram, shown in Figure 27-1,
but this is not something that needs to be memorized for the GI Board exam. You
can also throw the nomogram out the window if you have an alcoholic patient or
if they are on medications that can induce the cytochrome p450 system. That’s
really important to remember: The nomogram is uninterpretable in an alcoholic
with acetaminophen toxicity.

Figure 27-1. Acetaminophen toxicity nomogram. (Reprinted with


permission of Dheeresh J. Patel, MA.)

Ipecac is not indicated in this patient due to risk of aspiration with emesis,
since she is obtunded, and also because this was not a recent ingestion. But,
if the time of ingestion would have been in the past few hours, then activated
charcoal would have been indicated. However, if charcoal is administered, then
it must be given well before oral NAC to avoid problems with absorption.
ALF is defined as acute severe liver injury of less than 6 months’ duration
with impaired synthetic function, coagulopathy, and encephalopathy in a patient
with a previously normal liver or well-compensated liver disease (such as Wilson
disease, hepatitis B, or autoimmune hepatitis). Previously used terms such as
fulminant hepatic failure and hyperacute or subacute liver failure are not typically
used any longer since they do not help predict prognosis. The etiology is much
more important than the length of illness. For example, “hyperacute” liver fail-
ure from acetaminophen generally has a better prognosis than “subacute” liver
failure from an idiosyncratic drug reaction.
84 Chapter 2

Acetaminophen is the most common cause of ALF in the United States


and Europe (where it’s called paracetamol). You should also know that
certain etiologies tend to do better than others; liver failure from acetaminophen,
hepatitis A, and hepatitis B all have a better prognosis than liver failure from
Wilson disease, autoimmune hepatitis, idiosyncratic drug-induced liver injury, or
indeterminate etiology.
Why Might This Be Tested? Acute liver failure is one of the classic emergen-
cies for which the gastroenterologist can be awakened in the middle of the night
by a call from the emergency department. Acetaminophen is the most common
cause of ALF, so you are bound to see this on an exam and in real life.
Clinical Threshold Alert: A pH of <7.3 in acetaminophen-induced ALF is an
indication for liver transplantation listing. And be aware of the various King’s
College Criteria (more on this later in this book as well).

Here’s the Point!

Very high INR + Severe hepatic encephalopathy → Severe synthetic


dysfunction → Dismal prognosis without liver transplantation
“Tough Stuff” Vignettes 85

Vignette 28: Elevated Aminotransaminases on a Routine


Physical Exam
A 41-year-old Caucasian woman is referred for evaluation of persistently ele-
vated liver tests. She was initially found to have an ALT of 92 and an AST of 83 on
routine tests as part of a physical examination. Her liver function tests, including
INR, total bilirubin, and albumin, have all been normal. She underwent evalua-
tion for various forms of hepatitis, which has been negative to date. Specifically,
there is no biochemical evidence of infection with hepatitis A, B, or C, and her
ANA is negative. A right upper quadrant ultrasound was normal.
The patient’s past medical history is remarkable for iron-deficiency anemia
thought to be from menorrhagia as well as irritable bowel syndrome (IBS). She
has not received a colonoscopy or upper endoscopy. She takes intermittent ibu-
profen for aches and pains. She does not use any herbal therapies or alternative
medicinal preparations.
A review of systems reveals intermittent painful oral ulcers, recurrent lower
abdominal cramping that improves with bowel movements, intermittent diarrhea,
and chronic fatigue. She has no children, although she tried for years to conceive
and had an infertility workup that did not identify an underlying cause.
Physical exam reveals a thin patient with a BMI of 18. There is no icterus or
stigmata of chronic liver disease. There are scattered aphthous ulcerations along
the bilateral buccal mucosa. There is no lymphadenopathy. Abdominal exam
does not reveal hepatosplenomegaly, masses, or tenderness. Rectal examination
reveals brown loose stool that is heme-negative. The remainder of her examina-
tion is unremarkable.

` What diagnostic test should you order next?


86 Chapter 2

Vignette 28: Answer


This patient needs to be screened for celiac sprue, either with an anti-tissue
transglutaminase IgA or anti-endomysial IgA antibody. Celiac sprue is one of
those conditions that seems to do just about everything; you can put it on virtu-
ally any morning report differential diagnosis and be right. It should be tested
here because of its association with elevations in aminotransferase levels. The
exact mechanism for this association remains unclear, but it’s well known that
more than 40% of patients with celiac sprue have abnormally elevated amino-
transferase levels. In contrast, celiac patients do not typically have abnormal
liver function tests, such as total bilirubin, INR, or albumin (unless there is a
concurrent liver process accompanying the sprue). Furthermore, up to 10% of
patients with unexplained elevations in AST and ALT have underlying celiac
sprue. All of this supports testing for sprue in this patient. The elevated liver tests
usually normalize after starting a gluten-free diet.
In this case there are many reasons to suspect underlying celiac sprue, in
addition to the lab values. The patient is Caucasian, which is the first tip-off. In
addition, she has long-standing iron-deficiency anemia, recurrent aphthous oral
ulcerations, a diagnosis of IBS (celiac disease occurs in around 1% to 4% of IBS
patients), and infertility. Plus her physical exam reveals a thin body habitus,
which suggests possible nutrient malabsorption. An abdominal ultrasound may
be consistent with fatty liver and will also show hyposplenism (small spleen) in
celiac disease, supported by the presence of Howell-Jolly bodies on the periph-
eral smear. Given all of this, it would be a major mistake to forget about testing
for sprue. Making the diagnosis of sprue could have large positive impact on the
patient’s overall quality of life, not just her liver test abnormalities.
Why Might This Be Tested? It’s estimated that around 1 in every
133 Americans has celiac sprue. Whether or not that precise number is true, it’s
clear that sprue is a prevalent condition that is frequently missed. Board exam-
iners will want to know that you can make this diagnosis on a timely basis. But
it would be too easy to just show a Marsh 3 lesion in this patient and ask for
the diagnosis; instead, the examiners will likely want to see if you can recog-
nize less common (although still common) presentations of celiac sprue—in this
case, elevated aminotransferase levels. Finally, Board examiners have a number
of content “check boxes” they need to check off when putting an exam together.
This topic works well because it covers several areas, including both luminal GI
and liver disease. With one question they could potentially test across several
content areas.

Here’s the Point!

Unexplained elevated aminotransferases + Any of the following:


z Iron-deficiency anemia
z Infertility
z Osteoporosis
z Itchy elbow rashes (dermatitis herpetiformis)
z IBS symptoms
z Etc, etc, etc…
Think celiac sprue!
“Tough Stuff” Vignettes 87

Vignette 29: Still Yellow


You are asked to evaluate a 39-year-old man who developed hepatitis A virus
(HAV) infection 3 months ago. At the time, he had returned from a month-long
beach vacation, during which he ate raw seafood. He initially presented with low-
grade fever, malaise, diarrhea, anorexia, right upper quadrant discomfort, and
jaundice. His blood tests at the time of acute infection included the following:
total bilirubin = 6.8, ALT = 1987, and AST = 1492. He felt better soon thereafter
but noticed that his skin was yellow despite otherwise feeling markedly better.
He saw his primary care provider in follow-up yesterday, who advised him to
see you urgently. The patient mentions that his doctor “was freaking out since I
was still yellow and he wanted me to see you again to do more tests right away.”
Laboratory tests from yesterday are notable for a total bilirubin = 8.9, ALT = 1987,
and AST = 70.

` What is going on here?


` What test(s) should you order?
88 Chapter 2

Vignette 29: Answer


This is prolonged cholestasis (aka cholestatic hepatitis) from HAV infection and
is one of the often forgotten, atypical manifestations of HAV (notice the term acute
is not needed before HAV since there is no “chronic” HAV infection). In typical
HAV infection, patients may develop a prodrome of anorexia, malaise, fever, nau-
sea, diarrhea, and right upper quadrant pain. Days later they can have pruritus
and icterus, and diagnosis is confirmed with serum HAV IgM. Of note, jaundice
occurs in about 30% of adults but is much less common in children with HAV.
The rise in bilirubin occurs after the initial marked elevation of the ALT and
AST. Typically, the bilirubin gradually falls to normal within 3 months. However,
some patients may develop this prolonged cholestasis variant of HAV infection,
where the bilirubin can be markedly elevated (more than 10 mg/dL) before finally
returning to normal spontaneously over the next 3 to 5 months. Patience is
the key here, since the deep jaundice will resolve on its own without long-term
sequelae. A short course of steroids (prednisolone) could be an option to shorten
the cholestatic picture if the patient were having marked pruritus, weight loss,
or anorexia. However, there have been reports of steroid use precipitating HAV
relapse (more on relapse in a moment). Regardless, steroids are not indicated for
this patient since he is asymptomatic; expectant care is best. Remember that old
clinical saying, “Jaundice [by itself] never kills anyone,” or the common variant,
“Jaundice [by itself] is merely cosmetic.”
Regarding further testing, an HAV IgM can be ordered (which often would still
be positive) if you are unsure of the original diagnosis, but is not absolutely nec-
essary to make the diagnosis. You might also consider a right upper quadrant
ultrasound to exclude biliary obstruction. Beyond this, nothing else should be
needed diagnostically. It’s reasonable in this case to recommend to the primary
care physician that you would just sit tight, let nature take its course, and expect
a complete recovery. In fact, invasive testing, such as liver biopsy or ERCP, could
cause unnecessary complications and expense.
Relapsing hepatitis is another atypical clinical manifestation of HAV infection
with a self-explanatory name. After HAV seemingly resolves with normalization
of liver tests and symptoms, there can be a “relapse” a few weeks later with symp-
toms and laboratory tests similar to those in the initial phase, including HAV IgM
in the serum and even HAV isolated in the stool. The general rule is that this also
resolves spontaneously. HAV is one of the success stories in public health. Due to
improvements in public health measures such as sanitation and immunization,
the incidence of HAV has been decreasing steadily in the United States.
Why Might This Be Tested? HAV is a classic illness, but atypical manifesta-
tions of the disease are often forgotten and lead to unnecessary tests that have
their own inherent risks and expenses. Therefore it’s prudent to recognize this
atypical presentation of HAV.

Here’s the Point!

Prolonged cholestasis and relapsing hepatitis are atypical manifestations of


HAV and resolve spontaneously.
“Tough Stuff” Vignettes 89

Vignette 30: Trouble During a Steroid Taper


A 52-year-old man presented to his primary care physician with painful ery-
thematous nodules on his anterior shins and recurrent abdominal pain. After
evaluation, he was found to have polyarteritis nodosa (PAN) complicated by
abdominal and renal vasculitis. He was started on high-dose prednisone, which
led to symptomatic improvement. After steroid induction was complete, the patient
began a slow taper off steroids. Four weeks later, he noticed that his eyes were
yellow, which was quickly followed by yellow skin and overwhelming fatigue. He
presented to the emergency department, where he was found to be deeply jaun-
diced with tender hepatomegaly on examination. Laboratory values included the
following: AST = 1208, ALT = 1633, LDH = 521, total bilirubin = 22.3, INR = 3.8,
creatinine = 1.6, albumin = 3.4. He was admitted to the hospital, where his con-
dition rapidly deteriorated with the development of advanced encephalopathy,
eventually culminating in coma and death.

` What happened here?


` What diagnostic test should have been done prior to starting
steroids?
90 Chapter 2

Vignette 30: Answer


This is acute liver failure resulting from acute HBV infection in the setting of
immune reconstitution. How would you know that this is probably HBV when
that was not mentioned in the vignette? You may recall that polyarteritis nodosa
(PAN) is frequently associated with HBV and in some cases may be a second-
ary reaction to chronic infection with HBV. That’s an important relationship to
remember. Hepatitis C infection has also been associated with PAN, but hepatitis
C does not typically lead to acute liver failure in the setting of immune reconsti-
tution. In contrast, dormant HBV is known to spring to life, so to speak, in the
setting of immune reconstitution following a period of immune suppression. This
has been described in a variety of conditions, including PAN, during withdrawal
of steroids. During immune suppression, there is enhancement of HBV replica-
tion with diffuse involvement of the hepatocytes. The HBV is primed and ready to
attack. As the immune system reconstitutes following steroid withdrawal (or, say,
completion of chemotherapy in a different setting), there is a resurgence of cell-
mediated immunity that leads to direct injury of the hepatocytes harboring the
HBV. This is a natural response to try to rid the body of the rampant HBV, but
is quite literally a major overkill. It’s like taking a bazooka to an anthill. So the
HBV itself does not directly damage the hepatocytes; instead, the cell-mediated
immunity itself damages the liver. This injury can range from transient hepato-
cellular damage to acute liver failure, as seen here. The typical time course is
within 2 to 6 weeks following withdrawal of the immune-suppressing agent (eg,
steroids, chemotherapy, or anti-TNF therapy). Because HBV is so closely linked
to PAN, it’s vital to screen for latent HBV prior to beginning steroids, because the
withdrawal phase can lead to serious problems if the HBV has not been properly
handled.
Why Might This Be Tested? A vignette like this could test several different
facts at once, which makes for a good potential Board question. Not only do you
need to remember the relationship between PAN and HBV, but you also need to
recognize that immune suppression in untreated HBV can lead to severe liver
injury and even acute liver failure. Finally, remember that the damage from HBV
is not directly caused by the virus, but is indirectly caused by the virus hijack-
ing the innate immunity to wreak havoc on the hepatocytes. For Board exams
and life in general, remember that it’s important to check for latent HBV prior to
immune suppression.

Here’s the Point!

Polyarteritis nodosa on a GI Board exam? Think hepatitis B.

Here’s the Point!

Acute liver failure upon withdrawal of immune suppression = Think immune-


mediated damage orchestrated by HBV
“Tough Stuff” Vignettes 91

Vignette 31: Abnormal Liver Tests


You are asked to see a 31-year-old woman for abnormal liver tests. She has
had malaise and generalized weakness progressively worsening over the past
4 months without fever. She is now so fatigued that she has trouble getting out
of her chair. She does not have a history of liver disease, viral hepatitis, jaun-
dice, illicit drug use, prescription drug use, herbal drug use, over-the-counter
drug use (including acetaminophen products), alcohol use, or previous abnormal
liver tests. She has had liver tests performed annually at her routine health care
maintenance visits with her primary care provider, and they have always been
normal. Her laboratory tests now reveal elevations in her AST and ALT confirmed
on 2 samples. Therefore, you have been asked to provide consultation and per-
form a liver biopsy.
When you see the patient, physical examination shows no stigmata of chronic
liver disease and no rashes. Labs reveal the following: AST = 980, ALT = 490,
total bilirubin = 1.0, albumin = 3.4, ALP = 108, WBC = 11.1, hemoglobin = 13.2,
platelets = 394, ESR = 42.
Last week the patient had the following tests with normal results: ANA, smooth
muscle antibody, anti-liver/kidney microsomal antibody, soluble liver antigen,
ceruloplasmin, hepatitis A IgM, hepatitis B core IgM, hepatitis C RNA, hepatitis
E IgM, HSV IgM, EBV IgM, CMV IgM, and abdominal ultrasonography.

` What is the diagnosis?


` What test should you order first?
92 Chapter 2

Vignette 31: Answer


A simple blood test with serum creatinine kinase (CK) would help to yield
the diagnosis, which is polymyositis. In this case, the patient had been seeing
her internist for 2 months before we made the diagnosis upon seeing her on the
inpatient service. When we tested the patient’s CK, it was over 3000. The CK test
avoided the need for a liver biopsy. A muscle biopsy confirmed the diagnosis of
polymyositis, and the patient dramatically responded to steroid therapy.
Muscle injury of any cause, including rhabdomyolysis, can give a false-posi-
tive indication of “acute hepatitis.” Muscle contains both AST and ALT, and with
injury these can be elevated, with AST typically elevated more than ALT. It’s
important to think about the patient in a broad sense (including the history)
when you are evaluating abnormal liver tests. You are the expert consultant, so
you can’t have tunnel vision. A liver biopsy is reasonable if the diagnosis is in
doubt and can certainly diagnose autoimmune hepatitis. However, a simple blood
test (CK) in this case helped to establish the diagnosis and spare the potential
risks of a liver biopsy.
Why Might This Be Tested? Since the ABIM creates the examination, any-
thing under the umbrella of internal medicine is fair game for a test question.
“Abnormal LFTs” is one of the most common reasons to consult a gastroenterolo-
gist. You should know all the potential etiologies of elevated AST and ALT, includ-
ing nonhepatic etiologies.

Here’s the Point!

Elevated AST and ALT can be due to muscle injury and


not necessarily to hepatitis.
“Tough Stuff” Vignettes 93

Vignette 32: Pancreatitis and Elevated Alanine


Aminotransferase
A 38-year-old woman presents to the emergency room with a chief com-
plaint of abdominal pain, nausea, and vomiting. The following labs are noted:
amylase = 836, lipase = 562, AST = 132, ALT = 214, ALP = 86, total biliru-
bin = 1.2.

` Without knowing anything else, what is the most likely cause of the
pancreatitis?
94 Chapter 2

Vignette 32: Answer


This is most likely acute gallstone pancreatitis. The usual debate in acute
pancreatitis concerns how best to distinguish between its two most common eti-
ologies: gallstones versus alcohol. Obviously, a history of recent alcohol ingestion
is a key fact to support alcohol over gallstones (as compared to the many other
etiologies of pancreatitis), but patients’ stories are often suboptimal or less than
clear-cut. Here you don’t have a history to review, so the vignette is really testing
your ability to predict gallstone versus alcoholic pancreatitis using biochemical
parameters alone (an admittedly artificial exercise, but useful nonetheless).
There are 2 laboratory parameters that are helpful. The first is the serum ALT
level. Data indicate that when the ALT exceeds 150 IU/L (roughly 3 to 5 times
the upper limit of normal) with elevated pancreatic enzyme levels, the positive
predictive value for gallstone pancreatitis is 95%. Said another way, when the
ALT is over ~150 in the setting of acute pancreatitis, there is a 95% chance of
underlying gallstone pancreatitis, regardless of anything else. In contrast, nei-
ther the ALP nor the total bilirubin level is predictive of gallstone pancreatitis. A
common misconception is that the ALP and total bilirubin rise before the ALT in
the setting of a stone obstructing the common bile duct and/or pancreatic duct.
Yet it’s the ALT—not the ALP or total bilirubin—that jumps up first. The AST can
also be elevated, although its sensitivity and specificity for acute gallstone pan-
creatitis lag slightly behind those for ALT. The ALP rise can be slow in all forms
of acute biliary obstruction because the elevation is from induction and new syn-
thesis of ALP rather than spillage of stored enzymes. So the rise in ALP can take
a little time after acute biliary obstruction, whereas aminotransferases tend to
rise much faster. Also keep in mind that patients can have gallstone pancreatitis
even if an abdominal ultrasound is unrevealing; in acute pancreatitis, there can
be overlying bowel gas from ileus that obscures the view. A normal ultrasound
should not stop the hunt for a stone, especially when the ALT is elevated.
The second laboratory parameter is the ratio of lipase to amylase. Elevated
lipase to amylase ratios, especially when greater than 2.0, are predictive of
alcoholic rather than gallstone pancreatitis. The sensitivity and specificity of
this rule for predicting alcoholic pancreatitis are 91% and 76%, respectively. For
the patient described in this vignette, the ratio is less than 1.0, arguing against
alcoholic pancreatitis. But this rule is not carved in stone—it’s more of a clini-
cal pearl that is supported by some data but is not widely proven. So it’s best
to focus mainly on the high ALT. When the ALT is 150 or above in pancreatitis,
think gallstones—don’t be dissuaded by a normal ALP or total bilirubin.
Why Might This Be Tested? A vignette like this one provides a good way to
see if you know a time-tested clinical threshold. Plus it tests whether you have
fallen prey to the misconception that a near-normal ALP and/or total bilirubin
are incompatible with ductal obstruction. They are not… at all. Board examiners
are almost certain to test your general knowledge in interpreting liver tests, and
this is a good way to test your knowledge of a sometimes underappreciated use
of the ALT, in particular.
Clinical Threshold Alert: If a patient with pancreatitis has an ALT exceeding
150, then he or she probably has gallstone pancreatitis (positive predictive value
of 95% based on meta-analysis of multiple studies). In contrast, if the lipase-
to-amylase ratio exceeds 2.0, then alcoholic pancreatitis is likely, not gallstone
pancreatitis.
“Tough Stuff” Vignettes 95

Here’s the Point!

Pancreatitis + Elevated ALT = Think gallstone pancreatitis


96 Chapter 2

Vignette 33: Primary Biliary Cirrhosis


A 38-year-old woman with primary biliary cirrhosis (PBC) presents for a fol-
low-up examination in the office after the diagnosis was made 6 months ago. She
had a percutaneous liver biopsy showing stage 2 fibrosis with a positive antimi-
tochondrial antibody; this was initially performed to evaluate an asymptomatic
elevation in her ALP level. She was placed on ursodeoxycholic acid therapy and
has remained on a 1000 mg total daily dose. She has been doing well without
fatigue, pruritus, abdominal distention, weight loss, confusion, or bleeding, and
currently has no complaints. She weighs 70 kg and physical examination is
unremarkable. Laboratory tests from earlier today reveal the following: Na = 140,
creatinine = 0.8, AST = 17, ALT = 19, total bilirubin = 1.0, ALP = 90, GGT = 35,
INR = 1.0, albumin = 4.0, WBC = 6.9, hemoglobin = 13.2, platelets = 124.

` What tests should you recommend?


“Tough Stuff” Vignettes 97

Vignette 33: Answer


This patient should have an upper endoscopy despite the lack of cirrhosis on
biopsy. Patients with PBC, unlike most other liver diseases, are at risk to develop
presinusoidal portal hypertension due to nodular regenerative hyperplasia, even
without cirrhosis. Portal hypertension can occur with a normal platelet count,
but it is even more likely when the platelet count starts to fall. Thus, an upper
endoscopy to screen for esophageal varices is recommended in this patient even
though she is not cirrhotic (PBC without the “C”).
Surveillance for hepatocellular carcinoma is not recommended in PBC in the
absence of cirrhosis. This patient should, however, undergo bone mineral den-
sitometry testing. Osteoporosis occurs in up to one third of patients with PBC.
The etiology of osteoporosis in PBC is not entirely clear; sometimes it occurs from
vitamin D deficiency resulting from fat malabsorption, which tends to occur late
in the disease course. Nonetheless, all patients with PBC should receive supple-
mental calcium (at least 1200 mg per day) and vitamin D (at least 800 IU per day).
The replacement dose of vitamin D is much higher for those with a deficiency in
this vitamin (which can be determined by checking levels of 25-hydroxyvitamin
D). Osteoporosis is usually asymptomatic and is not associated with any par-
ticular laboratory abnormalities. Therefore, bone mineral densitometry should
be performed upon diagnosis and at periodic intervals thereafter (eg, every 2 to
3 years). If patients are osteoporotic, then bisphosphonate therapy should be
considered.
About 20% of PBC patients develop autoimmune-related hypothyroidism. In
fact, thyroid disease can be detected several years before the PBC diagnosis is
made. Thus, a TSH level should be checked on this patient as well. Identification
and treatment of hypothyroidism can help reverse many signs and symptoms,
including the neurologic and depressive symptoms that can often be confused
with encephalopathy in patients with late-stage PBC. Sicca syndrome (which can
be part of Sjögren’s syndrome and includes dryness of the mouth [xerostomia],
eyes [xerophthalmia], and vagina) and CREST syndrome (Calcinosis, Raynaud’s,
Esophageal dysfunction, Sclerodactyly, Telangiectasias) are more commonly
found in PBC patients than in controls. Keep this in mind when autoimmune
diseases present on exam. A patient with severe reflux symptoms with CREST
syndrome, for example, may have an abnormal ALP, which would suggest the
possibility of PBC.
Most patients with PBC also have hyperlipidemia. Yet despite this comorbid-
ity, PBC patients are not at increased risk for atherosclerosis or cardiovascular
events in the early disease course, probably because the HDL is typically high.
However, LDL may increase in the late stages with a lowering of the HDL. You
should check a lipid profile and consider lipid-lowering agents if the patient has
risk factors or symptoms of coronary artery disease.
The patient in this vignette is on the correct dose of ursodeoxycholic acid at
13 to 15 mg/kg/day, which has been proven to delay the progression to end-stage
disease, delay the need for liver transplantation, and increase survival. Higher
doses of ursodeoxycholic acid do not have incremental efficacy compared with
the 13 to 15 mg/kg/day dose. Unfortunately, no other single therapy has been
consistently found to be helpful to delay the progression of this disease.
98 Chapter 2

PBC has among the best survival rates after liver transplantation with more
than a 90% 1-year survival. Recurrence does happen, but it typically does not
progress rapidly enough to require retransplantation.
Why Might This Be Tested? PBC is a “classic,” slowly progressive liver dis-
ease with several interesting manifestations. You will see patients who have PBC
in your office and you should know how to diagnose them, but you should also
know how to manage these patients for the long term. Board examiners have
started to emphasize outpatient management of chronic diseases, not simply
diagnostic decision making early in the disease course. That makes PBC a good
bet to show up on the exam.
Clinical Threshold Alert: A dose of 13 to 15 mg/kg/day is the target range
for therapeutic ursodeoxycholic acid for the treatment of PBC.

Here’s the Point!

PBC → Think of nodular regenerative hyperplasia, osteoporosis,


and hypothyroidism
“Tough Stuff” Vignettes 99

Vignettes 34 to 37: Mallory Body Potpourri


You’ve been hearing about Mallory bodies since you were in medical school
(and especially if you trained at Boston City Hospital, where the famed Mallory
Institute of Pathology resides). Anyway, you know that Mallory bodies occur in
alcoholic liver disease, but they show up elsewhere, too. This section includes
a series of mini-vignettes of patients with Mallory bodies found on liver biopsy.
Your goal is to figure out the underlying diagnosis. What are Mallory bodies,
anyway? What are they made of? These questions and more are addressed in
the answer section.

34.A 68-year-old man with atrial fibrillation develops liver test abnormalities
after starting a new medication. Liver biopsy reveals phospholipid-laden lyso-
somal lamellar bodies along with Mallory bodies.

35.A 46-year-old woman with pruritus, progressive jaundice, and an elevated


ALP undergoes liver biopsy revealing Mallory bodies and granulomatous destruc-
tion of the bile ducts with infiltration by lymphocytes and plasma cells.

36. An 18-year-old man presents with acute hepatitis with elevated amino-
transferases but with a depressed ALP and uric acid levels. After recovery, a liver
biopsy reveals periportal inflammation, interface hepatitis, and Mallory bodies.

37. A 58-year-old woman with diabetes and hyperlipidemia is found to have


abnormal liver tests in labs performed as part of a routine physical. Laboratory
values included the following: AST = 123, ALT = 159, ALP = 126, total bili-
rubin = 1.3. She consumes less than 30 g of alcohol per week. Liver biopsy reveals
steatosis, lobular inflammation, ballooning hepatocytes, and Mallory bodies.
100 Chapter 2

Vignettes 34 to 37: Answers


Mallory bodies are cytoplasmic inclusions within hepatocytes that are com-
posed of cytokeratin intermediate filaments and ubiquitins, which are small
regulatory proteins that are ubiquitously expressed in eukaryotes (hence the
name ubiquitin). Mallory bodies have a characteristic “twisted rope” appearance
on light microscopy (Figure 34-1). Although they are most common in alcoholic
hepatitis, where they have a prevalence of around 65%, they are also found in a
variety of other conditions, as described in the answers that follow.

Figure 34-1. Classic Mallory body inclusion within a hepatocyte (arrow).


The Mallory body has a characteristic twisted-rope appearance. This
image reveals the inclusion within a ballooning hepatocyte, which is char-
acteristic of both alcoholic and nonalcoholic fatty liver disease. (Reprinted
with permission of Charles Lassman, MD, UCLA Medical Center.)

34. This is amiodarone toxicity. Refer to Vignette 15 for more details.

Here’s the Point!

Phospholipid-laden lysosomal lamellar bodies + Mallory bodies =


Amiodarone toxicity
“Tough Stuff” Vignettes 101

35. This is primary biliary cirrhosis. PBC is discussed in more detail in


Vignette 33 and elsewhere, but the short story here is that Mallory bodies can be
found in up to one quarter of liver biopsies from patients with PBC. If we had said
the patient was positive for the antimitochondrial antibody (AMA), this exercise
would have been too easy! AMA is positive in 95% of cases of PBC but can be
negative on occasion, which is when a liver biopsy is typically useful. The natu-
ral history of PBC features 4 histologic stages. Stage 1, called the “portal stage,”
is marked by granulomatous destruction of the bile ductules. This is called a
“florid duct lesion,” which is a Board buzzword you should remember. The duct
damage is accompanied by lymphocytic and plasma cell infiltration of the portal
tracts (thus the “portal stage” designation of stage 1). In stage 2, called the “peri-
portal stage,” there is loss of the bile ducts (also called “ductopenia”) along with
extension of the inflammation to the periportal areas. In stage 3, there is septal
fibrosis between adjacent portal tracts, and in stage 4, this organizes into cir-
rhosis. Mallory bodies can be found throughout these stages. Figure 35-1 reveals
a characteristic micrograph of PBC with granulomatous infiltration of the bile
ducts and a florid duct lesion. It’s important to remember that Mallory bodies
can occur in PBC, but it’s even more important to remember the granulomatous
duct destruction of PBC, since that is a buzzword.

Figure 35-1. Micrograph of primary biliary cirrhosis with granu-


lomatous infiltration of the bile ducts and a florid duct lesion.
(Reprinted with permission of Charles Lassman, MD, UCLA
Medical Center.)

Here’s the Point!

Florid granulomatous duct lesion + Mallory bodies = PBC


102 Chapter 2

36. This is Wilson disease. WD was discussed previously in more detail in


Vignette 12, but for now let us review a few pearls about this condition (you
can’t go over WD enough). WD is an autosomal recessive condition marked by
inadequate excretion of hepatic copper into the bile ducts (resulting from absent
or reduced function of a gene called ATP7B—props to you if you can memorize
that!). Bad things happen when copper builds up in the liver and blood. Liver
manifestations include acute hepatitis, as occurred here, acute liver failure, and
chronic disease marked by fibrosis and ultimately cirrhosis. In its earlier form,
the liver biopsy reveals periportal inflammation, interface hepatitis, and bridging
fibrosis. Mallory bodies are found in 25% to 50% of patients.
While we are on the topic of WD, here are the top 10 championship-level, clini-
cally useful factlets to know (in no particular order):
z The acute hepatitis and acute liver failure phases of WD are often marked
by an abrupt hemolytic anemia.
z Low ALP levels (not high ones) and low serum uric acid levels may occur
in symptomatic liver or neurologic disease due to Fanconi syndrome of the
proximal renal tubules.
z WD happens in young people, so if a Board question features a patient over
60 years of age, it’s essentially incompatible with WD—don’t be fooled.
z The presence of neurologic findings is almost synonymous with cirrhosis—it’s
very unusual to have neurologic findings in the absence of cirrhosis.
z Kayser-Fleischer (KF) rings occur from deposition of copper in the Descemet’s
membrane of the inner cornea, whereas the characteristic “sunflower cata-
ract” is from copper deposition in the lens capsule.
z In the setting of classic neurologic symptoms from deposition of copper in
the basal ganglia (eg, dystonia, chorea, dysarthria, Parkinsonian features),
the presence of KF rings on slit-lamp examination is pathognomonic for WD,
but the absence of KF rings with neurologic symptoms nearly excludes WD
(ie, high negative predictive value of absent KF rings when there are neuro-
logic symptoms).
z Although 95% of patients with WD have depressed serum ceruloplas-
min levels (<20 mg/dL) and 85% have elevated urine copper excretion
(>100 mcg/24 hrs), low ceruloplasmin and high urine copper are sensitive but
not entirely specific for WD, because both are seen in some asymptomatic
heterozygotes, among other conditions.
z The gold standard for diagnosis remains quantification of hepatic copper,
where a concentration above 250 mcg copper/g dry tissue is diagnostic of
WD—yet copper distribution may be heterogeneous, so a negative study
does not entirely rule out WD, but a positive study is diagnostic in the right
clinical setting.
z It’s vital to screen first-degree family members of affected individuals with
slit-lamp examination and measurement of serum ceruloplasmin levels.
z Both neurologic symptoms and liver tests may initially worsen after start-
ing therapy (more so with D-penicillamine than trientine), but subsequent
improvement typically occurs within 6 months of starting therapy (ie, it
takes a while for the medicine to work, so don’t discontinue it if the symp-
toms initially worsen unless there is a serious side effect).
“Tough Stuff” Vignettes 103

Here’s the Point!

Hepatitis + Depressed ALP + Mallory bodies = Wilson disease

Here’s the Point!

Know the “top 10” facts of Wilson disease—Go back and read them again!

37.This is nonalcoholic fatty liver disease. NAFLD is covered elsewhere in this


book, but this is a good opportunity to review some key facts about this astonish-
ingly prevalent and important condition. By now you probably know that NAFLD
is part of the metabolic syndrome, so it’s commonly coupled with diabetes, hyper-
lipidemia, and of course obesity. A full accounting of NAFLD’s pathophysiology is
outside the scope of this book, but the quick and dirty point is that the metabolic
syndrome predisposes to NAFLD through insulin resistance; this results in the
subsequent accumulation of fatty acids in hepatocytes. When coupled with a
“second hit” of oxidative stress, some people develop steatohepatitis and beyond.
NAFLD is generally defined as hepatic steatosis (with our without steatohepati-
tis) exceeding 5% to 10% of the liver mass in the absence of significant alcohol
consumption. How much alcohol can a patient consume and still be counted as
having NAFLD? That is somewhat debatable, but a very conservative rule is less
than 40 g per week, which isn’t much. Recall from Vignette 26 that a standard
alcoholic drink contains roughly 12 g of ethanol, so this rule means the patient
has fewer than 4 drinks per week. Furthermore, in NAFLD the AST:ALT ratio is
usually less than 1, in contrast to the typical 2:1 ratio in alcoholic hepatitis.
The liver biopsy in NAFLD depends on the grade of the disease, as follows:
Grade 1 is basically steatosis alone without much inflammation; grade 2 is
steatosis with lobular inflammation and ballooning hepatocytes but not much
portal inflammation (Figure 37-1); grade 3 is steatosis, lobular inflammation,
ballooning hepatocytes, plus portal inflammation. Of note, the diagnosis of
nonalcoholic steatohepatitis, or NASH, which lies in the spectrum of NAFLD,
is confirmed within these grades not only by the neutrophilic infiltration and
hepatocyte ballooning, but also with the development of “chicken-wire” fibrosis
(Figure 37-2). Mallory bodies can be seen throughout these various stages, but
especially in more advanced stages.
104 Chapter 2

Figure 37-1. Ballooning degeneration with fatty


infiltration in NASH. (Reprinted with permission of
Alton B. Farris, MD, Emory University.)

Figure 37-2. Nonalcoholic steatohep-


atitis with evidence of “chicken-wire”
fibrosis investing the liver paren-
chyma on trichrome staining. The
blue fibrous strands course between
the ballooning hepatocytes in this
advanced stage of disease. (Reprinted
with permission of Charles Lassman,
MD, UCLA Medical Center.)

Here’s the Point!

Metabolic syndrome + Low AST:ALT ratio + Mallory bodies = NAFLD


“Tough Stuff” Vignettes 105

Vignette 38: Cough and Abdominal Pain


A 39-year-old man returns from a mission trip to rural Brazil. He has not
been feeling well for the past 2 weeks. He has developed progressive symptoms
of fever, right upper quadrant discomfort, right shoulder pain with inspiration,
and coughing. He also reports fatigue for the past week without jaundice or
weight loss. There has been no diarrhea or vomiting. He was not ill and had
no medical problems while he was in Brazil for the past 6 months. Laboratory
tests now reveal the following: WBC = 14 with left shift and 0% eosinophils,
hemoglobin = 12.9, platelets = 290, total bilirubin = 1.1, ALP = 238, AST = 68,
ALT = 80, albumin = 3.4, creatinine = 1.0, and INR = 1.0. Chest radiograph is
remarkable for an elevated right hemidiaphragm. An abdominal CT scan is per-
formed, shown in Figure 38-1.

Figure 38-1. Abdominal CT for the patient in


Vignette 38. (Reprinted with permission of
Barbara Kadell, MD, UCLA Medical Center.)

` What is the diagnosis?


` What do you do about it now?
106 Chapter 2

Vignette 38: Answer


This is an amebic liver abscess, which is the most common extraintestinal
manifestation of amebiasis. The CT scan reveals a large, round, low-attenua-
tion lesion with an enhancing rim measuring 6 cm in the right hepatic lobe.
Although not seen in this image, right hemidiaphragm elevation is seen on chest
X-ray (seen in about half of patients with amebic liver abscess). By the way, “right
hemidiaphragm elevation” is a Board buzzword that often means “amebic liver
abscess.” For the patient in the case presented here, the next step is to perform
serum testing for amebiasis, which will be positive about 95% of the time after
1 week of illness in a previously unexposed patient. Thus, serologic testing would
be the best way to clinch the diagnosis. Aspiration, which carries a risk for
subcapsular peritonitis, would not be a good idea and is not required for either
diagnosis or routine treatment. In uncomplicated abscesses, antiamebic therapy
alone is as effective as the combination of aspiration and antiamebic therapy.
However, there are certain instances, as shown in Table 38-1, where aspiration
or drainage of amebic liver abscesses is indicated (eg, when there is risk for
imminent rupture or when a pyogenic abscess needs to be excluded). In these
situations, close monitoring of the patient is critical, since rupture of the abscess
with extension into the peritoneum, pleural cavity, or pericardium (which can
occur with left lobe abscess) can be disastrous and requires emergent surgical
therapy. Thankfully, most amebic abscesses occur in the right lobe, are solitary,
and respond well to anti-amebic therapy with an ~1% mortality. When aspira-
tion is performed, it might yield the classic “anchovy paste” substance; this is a
chocolate-colored fluid composed of an admixture of blood and liver tissue (usu-
ally without visualization of the trophozoites on microscopy). We’ve never seen
anchovy paste before, actually, so we’ll just have to take their word for it.
This intestinal parasitic infection begins with ingestion (via the fecal-oral
route) of contaminated water or food containing Entamoeba histolytica cysts.
Excystation occurs in the intestinal lumen, where trophozoites can form new
cysts in the mucin layer, resulting in an asymptomatic infection. This propagates
the infectious cycle as the cysts are passed in stool. However, the trophozoites
can occasionally invade the colonic lumen, causing an inflammatory response
and the classic “flask-shaped” ulcerations, and extraintestinal spread can allow
passage to the liver and other sites. Surprisingly, an antecedent diarrheal illness
is not common and stool studies are typically negative in patients with amebic
liver abscess. Leukocytosis without eosinophilia and an elevated ALP are com-
mon with an uncomplicated amebic liver abscess. However, hypoalbuminemia
and elevated bilirubin are independent risk factors for increased mortality. The

Table 38-1.
INDICATIONS FOR ASPIRATION OR DRAINAGE OF SOLITARY
AMEBIC LIVER ABSCESS
1. Lack of clinical improvement within 3 to 5 days
2. Seronegative abscess
3. Left lobe abscess
4. Thin rim of tissue (<1 cm) surrounding large abscess
“Tough Stuff” Vignettes 107

abscess typically responds well to a 10-day course of metronidazole. Radiographic


resolution can take several months despite clinical improvement. Even if stool
studies are initially negative, a luminal amebicide (such as paromomycin) should
be given after treatment of the liver abscess. Furthermore, follow-up stool speci-
mens should be obtained upon completion of therapy to ensure against coloniza-
tion and risk of recurrence. Unfortunately, infection does not confer immunity.
Therefore, reinfection and recurrence of amebiasis are possible. Amebiasis is
most often encountered in the United States in people who have traveled to tropi-
cal regions of Central and South America, southern Asia, and Africa.
It’s important to distinguish amebic from pyogenic liver abscesses. Although
pyogenic abscesses can appear radiographically similar to amebic abscesses,
they can be distinguished clinically and radiographically in several ways. Unlike
amebic abscesses, pyogenic liver abscesses typically derive from an intra-abdom-
inal source of infection (such as appendicitis, diverticulitis, or cholangitis) or are
secondary infections of underlying inflammatory bowel disease or malignancy.
Pyogenic abscesses are often multiple and caused by bacterial seeding through
the portal venous system, which feeds the liver with the bacteria. Therefore,
unlike most amebic liver abscesses, they do require aspiration for diagnosis to
guide antibiotic therapy, in addition to drainage and/or surgical therapy. In most
circumstances, pyogenic liver abscesses also tend to occur in older patients, and
have a slower recovery and worse prognosis than amebic liver abscesses. Table
38-2 summarizes the differences between pyogenic and amebic liver abscesses.
Be sure that you don’t confuse either of these with echinococcal cysts, which are
something altogether different (more to come on that)!
Why Might This Be Tested? With increased globalization and without an
effective vaccination, amebiasis tends to be commonly found in travelers to
endemic regions. You should be able to recognize how this type of abscess differs
clinically from pyogenic liver abscesses. Moreover, the luminal-liver correlations
provide fodder for GI Board exam questions.

Table 38-2.
AMEBIC VERSUS PYOGENIC LIVER ABSCESSES
Characteristic Amebic Abscesses Pyogenic Abscesses
Typical number of lesions Single Multiple
Gender Male predominance Equal
Historical association Traveler Abdominal infection
Biliary involvement Sterile Infection common
Clinical response to therapy Faster Slower
Overall prognosis Better Worse

Here’s the Point!

Right-lobe abscess + Right hemidiaphragm elevation + Recent travel =


Amebic liver abscess (aspiration usually not mandatory)
108 Chapter 2

Vignette 39: Ascites and Painful Gynecomastia


A 63-year-old man with alcoholic cirrhosis is started on 40 mg of furosemide
and 100 mg of spironolactone once daily for control of ascites. Along with sodium
restriction, the diuretics provide a good therapeutic response. A random urine Na
is 25 mmol/L, and urine K is 8 mmol/L. Relevant serum laboratories include the
following: Na = 135, K = 3.9, creatinine = 1.4. However, the patient reports painful
gynecomastia after several weeks of receiving therapy.

` What is the next therapeutic step?


` What if that step fails?
“Tough Stuff” Vignettes 109

Vignette 39: Answer


This is spironolactone-induced gynecomastia. Painful gynecomastia, or mas-
talgia, is one of the most common side effects of spironolactone in patients with
cirrhosis. The decision about whether to discontinue spironolactone should be
based on a determination of how effective the therapy for ascites has been, how
severe the mastalgia really is, and whether the gynecomastia can be effectively
treated while maintaining spironolactone on board. We will address this in a
moment.
First, though, this is a good opportunity to review some time-tested pearls of
wisdom regarding ascites management in cirrhosis. This is a big topic—much
bigger than we can cover here—but the following are some of the high-yield
facts.
The cornerstone of treatment in ascites is strict sodium restriction—not diuret-
ics (although diuretics come in a close second). Patients should restrict their
daily sodium to 2 g (88 mmol) or less. Of note, strict fluid restriction is generally
not indicated in cirrhosis. This point seems perennially lost on inpatient teams
who place a knee-jerk order to strictly limit fluids, especially when the sodium
is a bit low. Fluid restriction certainly has a role when sodium gets really low in
cirrhosis, but the general approach is to restrict sodium for managing cirrhotic
ascites, not to strictly limit fluids.
Regarding diuretics, the usual approach is to begin with furosemide 40 mg
and spironolactone 100 mg, both once daily. Split dosing is not necessary for
these drugs in the management of ascites. This ratio of 40:100 is optimal to
maintain normokalemia, since furosemide leads to hypokalemia, and spirono-
lactone causes hyperkalemia. If the 40:100 ratio doesn’t work, then move next
to 80:200, and so forth, until you reach a maximum of 160:400. If that doesn’t
work (and assuming the patient is somewhat compliant with sodium restriction),
then the patient has diuretic-resistant refractory ascites. The term refractory
ascites is reserved for patients who have persistent ascites despite maximum
diuretic therapy and sodium restriction (diuretic-resistant), or who have persis-
tent ascites and can no longer tolerate diuretic therapy due to the side effects
(diuretic-intractable; eg, if there is dose-limiting hyponatremia, azotemia, or
renal insufficiency).
Along the way it’s useful to check a random urine sodium level. The goal of
diuretic therapy is to induce natriuresis, defined by a urine sodium exceeding
10 mmol/L; a urine sodium below this threshold suggests insufficient natriure-
sis, which is common in cirrhosis due to systemic vasodilation and upregulation
of the renin and angiotensin. This process leads to release of antidiuretic hor-
mone (ADH) with subsequent reabsorption of sodium and free water. In theory, a
24-hour urine collection is superior to a spot urine. The goal of diuretic therapy
is to have more than 78 mEq/day of sodium in the 24-hour collection. But col-
lecting a 24-hour urine is onerous and time-consuming, so most people use the
random urine sodium as a barometer of whether there is adequate natriuresis.
The ratio of urine Na to K has also been evaluated, and a ratio >1.0 (ie, more
Na than K) is strongly predictive of >78 mEq/day of sodium in a 24-hour collec-
tion.
So, in this patient, things were going well until the gynecomastia. All it
took was a starting dose of furosemide and spironolactone in the 40:100 ratio
110 Chapter 2

to induce adequate natriuresis (based on urine Na and urine Na:K ratio) and
achieve a good clinical effect. Because furosemide alone is often inadequate for
treating ascites in decompensated cirrhosis and can cause dangerous hypoka-
lemia, the decision to discontinue spironolactone is not straightforward. The
decision is further complicated by the fact that the traditional replacement for
spironolactone, amiloride (another distal tubule agent), is less effective. If the
mastalgia is enough to impact quality of life, then you can switch to amiloride
at doses of 5 to 20 mg daily. If that doesn’t work, then you can consider adding
tamoxifen 20 mg twice daily to the aldactone. Randomized controlled trial data
reveal that tamoxifen is highly effective and well tolerated for combating the spi-
ronolactone-induced gynecomastia.
Why Might This Be Tested? Ascites is a great topic for the GI Board exam
because it’s something everyone should know. But the GI exam is different from
the ABIM internal medicine exam because the level of detail is greater and the
expectation for knowledge is higher. So the exam might ask you to interpret a
serum–ascites albumin gradient (SAAG), for example, but most everyone knows
that already. It gets harder when the exam asks detailed management questions
about rare but real problems, like mastalgia from spironolactone, or interpreting
urine sodium levels.
Clinical Threshold Alert: If a patient on diuretics has <10 mmol/L of urine
sodium on a random spot urine, or <78 mEq/day of sodium over 24 hours, then
there is inadequate natriuresis and the diuretics should be further optimized.
The same applies if the ratio of urine Na:K is less than 1.0.

Here’s the Point!

Spironolactone can cause painful gynecomastia. If that happens, you can


switch to amiloride or consider adding tamoxifen.
“Tough Stuff” Vignettes 111

Vignette 40: An Unusual Case of Hepatitis


You are consulted to evaluate a patient in order to assist with the diagnosis of
new-onset hepatitis. The patient is a 38-year-old Latino man who works as a chef
in a local restaurant. Three days prior to admission, he developed chills, fever,
and abdominal discomfort. He reported to his primary care doctor, who found
that the patient was febrile (temperature = 101.2°F), had a pulse of 62, and had
a blood pressure of 108/70. Outpatient laboratories included an AST of 192 and
an ALT of 220. He was subsequently admitted for further evaluation of possible
hepatitis.
On exam now he is febrile (temperature = 103.4°F) with a pulse of 66 and a
blood pressure of 110/68. There is no orthostatic hypotension. Skin exam reveals
faint, diffuse, salmon-colored macules across the trunk. He is not jaundiced,
and there are no stigmata of chronic liver disease. He has diffuse tenderness in
all quadrants with voluntary guarding but no rebound tenderness. Rectal exam
reveals brown stool. Laboratories now include the following: hemoglobin = 10.6,
WBC = 18.3 (90% PMNs), platelets = 288, amylase = 12, lipase = 15, ESR = 9,
CRP <1.0, AST = 212, ALT = 246, LDH = 182. ALP = 204, total bilirubin = 1.8,
INR = 1.1, albumin = 3.4.

` What is the most likely diagnosis?


` Why?
112 Chapter 2

Vignette 40: Answer


This is typhoid fever from infection with either Salmonella typhi or Salmonella
paratyphi. Typhoid fever is a classic Board-type condition because it can easily
mimic other acute intra-abdominal processes. Plus, it can cause hepatitis and
can easily be confused with viral hepatitis.
Typhoid fever usually begins within 7 to 21 days after ingestion of the culprit
organism. It’s rarely encountered in the United States and is most commonly
contracted from ingestion of contaminated food or water in foreign countries
(especially in those who neglected to get vaccinated for typhoid ahead of time).
The prevalence of S. typhi and S. paratyphi is especially high in Nepal—a classic
location for developing typhoid fever. This patient is not from Nepal, but had he
just come back from Nepal with hepatitis, you would have to think about this.
But, he is a chef, and you need to think about food contamination as a possible
mode of transmission.
The condition usually begins with a complex of severe abdominal pain and
“stepwise fevers,” in which the fever curve follows a step-like pattern of progres-
sive elevations. Another characteristic feature of the fever is a “temperature-pulse
dissociation,” in which the heart rate is bradycardic in relation to the fever. In
this case, the pulse of 66 is quite low for such a high fever—even if this patient
were an athlete. The abdominal pain of typhoid fever can be severe, and it’s often
in the right lower quadrant from underlying terminal ileitis or cecitis. In severe
cases, the bowel can perforate, leading to a surgical emergency. In the second
week of the illness the patient may develop so-called rose spots, which are faint
salmon-colored macules across the trunk, as mentioned here.
The key for this vignette is that patients may also develop a hepatitis-like
picture with elevated transaminases and bilirubin. The red herring in a Board
exam would be to confuse this with simple viral hepatitis. Certainly hepatitis A is
high on the differential for a chef coming in with a fever and biochemical hepati-
tis. But there are too many clues here to point away from simple viral hepatitis.
The temperature-pulse dissociation is one clue; another is the presence of “rose
spots.” In addition, the ratio of ALT to LDH has been cited as a distinguishing
feature, where a ratio of <4 is common in salmonella hepatitis, whereas a ratio of
>4 to 5 is more common in acute viral hepatitis (see Vignette 1 as well). Data also
reveal that compared to viral hepatitis, salmonella hepatitis is associated with
higher peak fevers, higher ALP levels, and a higher white count with a left shift.
The bottom line is that viral hepatitis can be clinically indistinguishable from
salmonella hepatitis, but the biochemical patterns can help distinguish between
them, especially when coupled with physical findings of relative bradycardia and
characteristic skin rashes. Oh, and don’t forget to treat the Salmonella, typically
with a fluoroquinolone.
Why Might This Be Tested? Board examiners love anything that may affect
the terminal ileum, and this condition is one of the classics. It’s also full of Board
buzzwords, like “temperature-pulse dissociation,” “rose spots,” and abdominal
pain and fevers after being in Nepal. Finally, it affects both the intestines and
the liver, so it can be “counted” in both bins as test-makers ensure sufficient
coverage of all content frontiers. In short, it seems like a good condition to know
about for the exam.
“Tough Stuff” Vignettes 113

Here’s the Point!

“Stepwise fever” + Temperature-pulse dissociation + “Rose spots” + Hepatitis


= Typhoid fever
114 Chapter 2

Vignette 41: Liver Failure During a Marathon


A 38-year-old man was running a marathon on a hot day (82°F) when he col-
lapsed at mile 18. Emergency medical technicians arrived to find the patient
stuporous. Vital signs in the field were as follows: heart rate = 162, blood pres-
sure = 98/62, respiratory rate = 22, O2 saturation = 96%, temperature = 104.8°F.
Cooling measures and intravenous fluids were initiated, and the patient was
urgently transferred to the emergency department, where he arrived unconscious
with a blood pressure of 102/70 and a rectal temperature of 103.6°F. Admission
laboratories were as follows: AST = 78, ALT = 118, LDH = 451, ALP = 80, INR = 1.2,
total bilirubin = 1.8, CK = 812, creatinine = 2.0, Na = 150, phosphate = 1.1. Over
the next 24 hours he received intensive care with continued cooling measures
and intravenous hydration. His mental status slowly improved, and he regained
consciousness but remained drowsy. His blood pressure stabilized, and he
was never hypotensive. The core temperatures normalized after 24 hours. By
48 hours, however, he became rapidly jaundiced and was found to have the fol-
lowing laboratories: AST = 1004, ALT = 1200, LDH = 899, ALP = 120, INR = 2.9,
total bilirubin = 16.4, CK = 1002, creatinine = 1.6, Na = 139, phosphate = 1.1,
AFP = 10.2.

` What happened here?


` How can these specific lab values assist in determining prognosis?
“Tough Stuff” Vignettes 115

Vignette 41: Answer


This is acute liver failure resulting from exertional heat stroke (EHS). EHS is
a diagnosis well known to paramedics and emergency medicine physicians, but
we should know about it too because it can affect the liver. In fact, it can ham-
mer the liver to the point of acute liver failure requiring transplantation. EHS
occurs when environmental heat overcomes the body’s ability to dissipate heat.
This naturally leads to an increased core temperature, which, before long, pre-
cipitates a cascade of multiorgan damage. Cardiovascular collapse is the most
common and feared complication of EHS, but liver damage is nearly universal as
well. In most cases, hepatic injury is mild or moderate and associated with tran-
sient increases in the liver tests. But in some cases, the EHS can cause extensive
hepatocellular injury, acute liver failure, and death. The injury is not necessar-
ily from hypotension, as can be seen with ischemic hepatitis (although in both
cases the LDH tends to be quite high and the ALT:LDH ratio low, as seen here),
but from a variety of mechanisms including endotoxinemia (so called heat sepsis)
and high blood concentrations of cytokines and acute-phase proteins.
The prognosis of acute liver failure in EHS can be poor—but not always. The
good news is that many patients with this condition are young and otherwise
healthy, so survival is often better than expected. In general, patients under
30 years of age have a better prognosis in acute liver failure (of any kind) than
older patients do. And yes, being in your 30s is considered “old” when it comes
to acute liver failure. This patient is 38, which makes him old enough for bad
things to happen. But there are some bright spots in his biochemical profile. In
particular, his AFP is elevated at 10.2. The AFP is usually employed to screen for
hepatocellular cancer, but it has a prognostic role in acute liver failure as well.
When the AFP is above 3.9 ng/mL the day after the ALT peaks, the likelihood of
recovery is significantly improved. AFP is a surrogate marker for liver regenera-
tion in the setting of acute liver failure. Another good sign is the hypophospha-
temia. Low phosphate means the liver is kicking into gear and regenerating—a
process that requires cellular uptake of phosphate to keep the machinery run-
ning under stress. So the combination of elevated AFP and low phosphate pre-
dicts a favorable prognosis in this otherwise very sick patient. Some people also
measure factor V levels in acute liver failure because low factor V indicates a poor
prognosis and diminished functional capacity of the remaining liver.
While we’re on the topic of biochemical prognosticators in acute liver failure,
it’s worth remembering that rapid drops in aminotransferase levels are a bad
thing. When you see the AST and ALT quickly rise and then quickly drop (some-
times to very low levels), it may suggest a “burnout” of the liver from overwhelm-
ing necrosis (see Vignette 1).
Why Might This Be Tested? Published case series of EHS reveal a surpris-
ingly high prevalence of liver injury, yet this is not as widely appreciated as it
probably should be. The severity of liver injury is often not appreciated until it’s
too late. Sometimes, these patients need a liver transplant, yet when the focus
is on correcting body temperature, renal function, and cardiovascular function,
it can become easy to forget about the liver. Once those systems are restored,
the liver often remains dysfunctional, and the focus is only then fixated on the
liver injury. The bottom line is that liver injury with EHS really happens and is
often not realized until it’s too late. The Board exam might also use a vignette
116 Chapter 2

like this as a way to see how well you know the biochemical prognosticators of
acute liver failure.
Clinical Threshold Alert: If the AFP exceeds 3.9 ng/mL a day after the ALT
peaks in acute liver failure, a favorable outcome is predicted.

Here’s the Point!

Really hot + Liver trouble = Hepatic injury from exertional heat stroke…
Watch out for acute liver failure!

Here’s the Point!

Favorable biochemical indicators in acute liver failure include:


z Hypophosphatemia
z Elevated AFP
z Elevated factor V
“Tough Stuff” Vignettes 117

Vignette 42: Persistent Hypotension in Acute Liver Failure


A 42-year-old woman is admitted for management of acute liver failure
from acetaminophen toxicity. She is admitted to the intensive care unit, where
she is intubated and mechanically ventilated for advanced encephalopathy.
After 72 hours of multidisciplinary management including treatment with N-
acetylcysteine, she becomes progressively hypotensive, requiring vasopressor
support. The hypotension continues for several days, during which time she is
evaluated for cardiogenic shock and sepsis. Blood, urine, and sputum cultures
are unremarkable, and cardiac evaluations are not consistent with a cardio-
genic source of hypotension. An intracranial pressure (ICP) monitor is placed,
which reveals an ICP of 10 mm Hg and a cerebral perfusion pressure (CPP) of
70 mm Hg. Despite fluid resuscitation and continued vasopressor support, the
patient remains persistently hypotensive.

` What should you look for next?


118 Chapter 2

Vignette 42: Answer


This is most likely adrenal insufficiency in the setting of acute liver failure.
Did you get it? If not, you’re not alone. It’s easy to miss adrenal insufficiency as
a cause of persistent hypotension in critically ill patients. But this is not just
any critically ill patient—this patient has acute liver failure, and adrenal insuffi-
ciency is especially common in acute liver failure. One series from King’s College
in Britain (the source of the King’s College Criteria described in Vignette 27)
found that nearly two thirds of patients with acute liver failure have biochemical
evidence of adrenal insufficiency, which is especially common in acetaminophen-
induced liver failure, although it’s certainly found in other etiologies as well.
This is obviously important because adrenal insufficiency can mimic some of
the hemodynamic anomalies seen in acute liver failure, including hyperdynamic
cardiovascular collapse. The work from King’s College has shown that patients
with adrenal insufficiency are sicker, have more severe hemodynamic instabil-
ity, are more likely to require liver transplantation, and are more likely to die
compared to those without adrenal insufficiency. It remains unclear whether
this is a cause-and-effect relationship or simply a marker of illness. But the key
is to think about adrenal insufficiency early, test for it actively (typically with a
cosyntropin stimulation test), and treat with IV corticosteroids on a timely basis.
In fact, in this case it would be reasonable to empirically treat with IV steroids
while awaiting the result of the stimulation test.
This vignette provides a good excuse to review ICP treatment goals in acute
liver failure. In this case, the patient was evaluated for persistently elevated ICP
despite hyperventilation and was found to have an ICP of 10 mm Hg and a CPP
of 70 mm Hg. Both of those are right where you would want them to be. As an
important aside, the need for ICP monitoring remains debatable—it’s not nec-
essarily standard of care to place ICP monitors in all patients with acute liver
failure. The monitors themselves can cause problems, including intracranial
bleeding and infection.
But assuming the ICP monitoring device is in and is working, it’s good to
remember a few numbers. The goal of therapy is to maintain ICP below 20 mm
Hg. The CPP, in turn, should be higher than 50 to 60 mm Hg. The CPP is cal-
culated by subtracting the ICP from the mean arterial pressure (MAP). So the
higher the ICP, the lower the CPP, and the harder it is to get oxygenated blood
to the brain—a bad combination. But if the ICP can remain low enough while
the MAP is maintained at adequate levels, then the CPP will be higher and the
patient’s cerebral oxygenation will be improved. If the CPP falls and the ICP goes
up, the usual first-line therapy is to start intravenous mannitol.
Why Might This Be Tested? Acute liver failure is eminently testable. And
this vignette is great Board fodder because it simultaneously requires knowl-
edge about acute liver failure and general internal medicine. Finally, if you miss
this diagnosis, you might significantly worsen outcomes by failing to identify a
treatable problem. Treating the problem may or may not save a life, but failing
to make this diagnosis could definitely hurt a patient. Board examiners want to
make sure that doesn’t happen.
“Tough Stuff” Vignettes 119

Clinical Threshold Alert: For elevated ICP in acute liver failure, the goal is
to maintain ICP below 20 mm Hg and increase CPP above 50 to 60 mm Hg. And
remember that CPP = MAP – ICP.

Here’s the Point!

Persistent unexplained hypotension in acute liver failure =


Think adrenal insufficiency
120 Chapter 2

Vignette 43: Abnormal Liver Tests (Yes, Again)


A 38-year-old Caucasian man is referred to you by his primary care provider
for further evaluation of asymptomatic liver test elevations. He feels fine and does
not have pruritus, fatigue, jaundice, or abdominal pain. Physical examination,
BMI, and vital signs are all unremarkable. Laboratory tests show the following:
ALT = 42, AST = 40, ALP = 180, GGT = 102, total bilirubin = 1.0, albumin = 4.2,
creatinine = 1.0, INR = 1.0, WBC = 5.2, hemoglobin = 14.6, platelets = 259. You
order a magnetic resonance cholangiopancreatography (MRCP), as shown in
Figure 43-1. Liver biopsy is performed showing “fibrous obliteration of bile ducts
with connective tissue replacement concentrically.”

Figure 43-1. MRCP of the patient in Vignette


43. (Reprinted with permission of Barbara
Kadell, MD, UCLA Medical Center.)

` What is this?
` Are any other tests needed?
“Tough Stuff” Vignettes 121

Vignette 43: Answer


This is small-duct primary sclerosing cholangitis (PSC). Unlike in typical
large-duct PSC, where liver biopsy is rarely needed since cholangiography is
diagnostic, liver biopsy is essential in small-duct PSC since cholangiography is
normal. So, if the MRCP in Figure 43-1 looked normal to you, you were right.
Small-duct PSC can be diagnosed only with liver biopsy, not by cholangiography.
In this case, the biopsy describes the classic “onion-skin” pattern of fibrosis,
which clinches the diagnosis. That is, there is fibrous obliteration of the bile duct
with concentric connective tissue replacement. About 5% of all patients with PSC
have small-duct PSC, and these patients tend to have a more indolent disease
course than patients with large-duct PSC; around 20% progress to large-duct
disease and very few develop cholangiocarcinoma.
Even though the patient in this vignette is asymptomatic, a colonoscopy with
biopsies should be undertaken to rule out concomitant inflammatory bowel dis-
ease (IBD), which is present in the majority of cases (see Vignette 93 for more on
this).
While we are on the topic of PSC, let’s focus on large-duct PSC for a moment. In
patients with large-duct disease, especially in those without evidence of IBD, an
IgG4 level should be checked. An elevation of IgG4 may signify an IgG4-associ-
ated cholangitis that can mimic PSC (and is sometimes associated with scleros-
ing of the pancreatic ducts and autoimmune pancreatitis). However, unlike PSC,
IgG4-associated cholangitis can respond to a course of corticosteroids. Therefore,
it’s important to rule out IgG4-associated cholangitis in these patients.
Here are some interesting associations and tidbits related to PSC:
1. IBD is prevalent in the majority of patients with PSC; however, PSC is
present in less than 10% of patients with IBD (more in Vignette 93).
2. IBD can occur de novo after liver transplantation for PSC.
3. PSC can occur de novo after proctocolectomy for ulcerative colitis (UC).
4. Patients with UC and PSC are at increased risk for colorectal neoplasia
compared with patients with UC but not PSC. Furthermore, when
colorectal neoplasia is found, it’s in the right colon about 75% of the
time. This stresses the importance of a full colonoscopy with a good
bowel preparation regimen.
5. Ursodeoxycholic acid therapy is not effective to halt the progression of
PSC and has increased side effects at the higher doses. Thus, it is not
recommended routinely for PSC. However, it may be considered in select
patients with PSC who have a particularly increased risk for colorectal
neoplasia (due to family history, etc), as some retrospective studies have
shown a decreased risk for colorectal neoplasia in patients with PSC
taking ursodeoxycholic acid.
6. Cholangiocarcinoma occurs in about 10% of patients with PSC.
However, as opposed to colorectal malignancy in UC, which tends to
present well after the initial diagnosis of UC, cholangiocarcinoma is
present within 1 year of the time of diagnosis of PSC in about half of the
patients who develop it.
7. Gallbladder polyps in PSC are frequently malignant. Thus, cholecystec-
tomy needs to be considered in PSC patients with significant gallbladder
polyps.
122 Chapter 2

Why Might This Be Tested? PSC and UC provide a great combination


of gastroenterology and hepatology diseases occurring in the same patient.
Furthermore, there are several associations with malignancy that are clinically
relevant. This provides a perfect setup for the GI Board examiner. Expect to see
some of this on the exam.

Here’s the Point!

Small-duct PSC → Normal cholangiogram → Liver biopsy needed for


diagnosis

PSC without IBD → Think of IgG4-associated cholangitis

And know the 7 PSC factoids listed in the discussion!


“Tough Stuff” Vignettes 123

Vignette 44: Biliary Obstruction After Eating Dirt


A 21-year-old man is referred for ERCP in the setting of recurrent biliary
colic. He emigrated from El Salvador 1 year ago and, upon careful question-
ing, acknowledged episodes of pica with eating dirt. He initially presented
8 months prior to this visit with classic biliary colic, and he similarly presented
again 2 months before the current visit. He is now asymptomatic and does not
report any GI symptoms, including fever, chills, nausea, vomiting, abdominal
pain, or change in bowel habits. Physical examination is significant only for an
enlarged liver. Labs include the following: hemoglobin = 15.7, AST = 92, ALT = 72,
ALP = 113, total bilirubin = 1.8. Pre-ERCP ultrasonography revealed a liver span
of 19.9 cm, diffuse fatty infiltration, tortuous tracks through the liver paren-
chyma, and a common bile duct dilated at 11 mm. There were multiple echogenic
foci in the gallbladder and common bile duct. ERCP with sphincterotomy did not
show stones, but instead revealed multiple flat parasites, as pictured in Figures
44-1 and 44-2.

Figure 44-1. Flat parasites in the duodenum. (Reprinted


with permission of Stanley Dea, MD, Olive-View
UCLA Medical Center.)

Figure 44-2. Flat parasites collected from


patient after ERCP. (Reprinted with permis-
sion of Stanley Dea, MD, Olive-View UCLA
Medical Center.)

` What is the diagnosis?


` How should this be treated?
124 Chapter 2

Vignette 44: Answer


This is fascioliasis, which is an infection with either Fasciola hepatica or
Fasciola gigantica. That’s right, there is actually something called Fasciola gigan-
tica. Yikes! This just shouldn’t happen... but it does. There’s nothing good about
fascioliasis. Both Fasciola hepatica and gigantica are liver flukes, or trematodes,
that occur all over the world. These flukes are most prevalent around sheep. In
fact, they infect sheep and cause something called “liver rot.” Like I said, there’s
just nothing good about this. Humans are an accidental intermediate host in the
complex life cycle of these critters. Fascioliasis typically occurs in people who
live in sheep-rearing communities most commonly in the temperate climates of
Central America, South America, the Middle East, parts of Africa, and China.
It even happens in the U.S. every so often, albeit rarely. Humans are usually
infected by eating contaminated food products or drinking unboiled water con-
taminated by the parasites. In this particular case, the patient was not a sheep
herder, but had a curious pica habit that led him to eat dirt. That probably
caused him to become ill, but it’s hard to say for certain.
Once these buggers are ingested, they eventually take up lodging in the bili-
ary tree where they lay their nasty little eggs. These ultimately shed through
the ampulla of Vater and are expelled in feces. They tend to hang out in water
where they mature until a snail comes along and ingests them. They grow within
the snail, get pooped out again, and then start hanging out on various aquatic
vegetation. Eventually a sheep will come along and eat the vegetation and get
sick. If a human eats these contaminated fresh water plants or if the water is
directly consumed, then the infective forms are ingested (called metacercariae,
if you really want to know). These next turn into the larvae in the duodenum
(see Figures 44-1 and 44-2). It gets even nastier, because these larvae next eat
through the wall of the small bowel and enter the peritoneum. From there, they
head over to the liver and penetrate the capsule, where they proceed to bur-
row through the liver tissue. That’s right—they can burrow right through your
liver (kind of like a TIPS placement). Just look at the mouths of these things
under microscopy (Figure 44-3). Once they penetrate the capsule, subcapsular
hematoma can occur. The burrows themselves are large enough to show up as

Figure 44-3. Histologic section of


Fasciola parasite showing the mouth-
like area. This parasite eats its way
through the intestinal wall, into the
peritoneum, through the liver capsule,
and through the liver parenchyma
before finally taking residence in the
biliary tree. (Reprinted with permission
of Stanley Dea, MD; Olive-View UCLA
Medical Center.)
“Tough Stuff” Vignettes 125

“tortuous tracks” on imaging. These tracks become necrotic, leading to liver rot.
The trematodes then head over to the biliary tree where they start laying their
eggs. They can totally plug up the biliary tree because they are so large—at least
by parasite standards. (Not sure if they are really gigantica, but they are gigan-
tica enough to plug up a biliary tree!) And then the cycle repeats itself. I mean,
you’ve got to be kidding!
Treatment for fascioliasis is a little tricky because some of the usual therapies
are not effective against this liver fluke. Specifically, praziquantel, mebendazole,
and albendeazole don’t seem to work. Nitazoxanide does have some efficacy,
however, with a cure rate approaching 80%. In this case, the patient was treated
with nitazoxanide 500 mg twice daily for 7 days and had complete resolution of
symptoms. He may still be eating dirt, however.
Why Might This Be Tested? Clearly this is a rare infection in the United
States, at least. But, it happens. Plus, we tend to focus too much on viral infec-
tions of the liver so it becomes easy to forget about the various nonviral infec-
tions that befall it. Board examiners may reserve the right to trailblaze off the
viral infection highway and explore some less frequented, yet still important,
liver infections. Other classic nonviral infections of the liver include pyogenic
liver abscesses, amebic liver abscesses, schistosomiasis, clonorchis sinensis, and
everyone’s favorite—echinococcosis. Table 44-1 provides some board buzzwords
you can remember to quickly sort out the diagnosis and treatment for the usual
suspects. This table alone is probably enough to get you most of the way through
questions about these infections.

Table 44-1.
BOARD BUZZWORDS FOR LIVER PARASITES
Board Buzzword/Association Associated Condition(s)
Sheepherder Fascioliasis, echinococcosis
Strong male predominance Amebiasis
Serum sickness (aka Katayama fever) Schistosomiasis
Solitary hepatic cyst of right lobe Echinococcosis, amebiasis
Portal vein granulomas and fibrosis Schistosomiasis
Hepatic abscess with elevation of right Amebiasis
hemidiaphragm with adhesions obliterat-
ing the costophrenic angle
Transverse myelitis Schistosomiasis
Anaphylaxis after cystic rupture Echinococcosis
Abscess filled with “anchovy paste” Amebiasis
Epilepsy Schistosomiasis
Smooth, round cyst with internal septa- Echinococcosis
tions
Undercooked fish Clonorchiasis
(continued)
126 Chapter 2

Table 44-1 (continued).

BOARD BUZZWORDS FOR LIVER PARASITES


Board Buzzword/Association Associated Condition(s)
Portal hypertension without stigmata of Schistosomiasis
chronic liver disease
Gallstones Clonorchiasis
Biliary ductal fibrosis Clonorchiasis
Liver cyst with eggshell calcifications Echinococcosis
Contaminated freshwater plants Fascioliasis
Cyst filled with “hydatid sand” Echinococcosis
Tortuous tracks in liver on CT Fascioliasis
Tsetse fly African trypanosomiasis (aka
sleeping sickness)
Cyst surrounded by “daughter cysts” Echinococcosis
Reduviid bug (aka kissing bug) American trypanosomiasis
(aka Chagas disease)
Sandfly Leishmaniasis (kala-azar)

Here’s the Point!

Sheepherder + Liver with “tortuous tracks” on CT + Biliary obstruction =


Fascioliasis (usually from F. hepatica, less commonly from F. gigantica)
“Tough Stuff” Vignettes 127

Vignettes 45 to 49: Hepatic Bruit Potpourri


Hepatic bruits. Have you ever really heard one? If you listen long enough to
enough patients, you will hear one eventually. Of course, knowledge of hepatic
bruits tends to be more valuable on Board exams than in real life, but it also
makes for great roundsmanship to pick up a bruit. The following mini-vignettes
describe patients in whom a hepatic bruit was auscultated. Try to figure out the
most likely cause of the bruit in each vignette.

45. A patient is brought into the emergency department by paramedics after


being found lying unconscious on a sidewalk. On exam, there are no stigmata of
liver disease, but the patient is jaundiced and there is a hepatic bruit. The labo-
ratories include an AST = 212 and an ALT = 89.

46. A patient with hepatitis B cirrhosis and HBsAg and HBeAg double posi-
tivity presents with progressive hepatic encephalopathy. Exam reveals a new
hepatic bruit.

47. A patient presents with recurrent right upper quadrant abdominal pain.
Physical exam reveals no stigmata of liver disease but a hepatic bruit. Ultrasound
reveals a normal gallbladder and biliary tree, but there is a cystic mass in the
gastroduodenal ligament between the pancreatic head and left liver lobe. A CT
scan confirms this lesion and reveals vascular contrast enhancement within the
cystic structure.

48. A patient with cirrhosis and progressive ascites develops prominent umbil-
ical and periumbilical vessels followed by progressive improvement of the ascites.
Auscultation over the abdominal vessels reveals a constant humming sound
(okay, not quite a hepatic bruit, but a periumbilical bruit, so to speak).

49. A patient is stabbed in the abdomen. Exploratory laparotomy reveals liver


lacerations, which are then sutured closed. The patient recovers from surgery
and is discharged. He returns several weeks later with right upper quadrant
pain. Physical exam reveals a continuous hepatic bruit and stool positive for
occult blood.
128 Chapter 2

Vignettes 45 to 49: Answers


45. This is acute alcoholic hepatitis (AH). Refer to Vignette 26 for much more
on AH. In this case, AH is likely given the AST:ALT ratio exceeding 2:1 coupled
with the patient being found unconscious (probably drunk). The point here is
that patients with AH can present with a hepatic bruit on physical exam. The
bruit probably results from an elevated hepatic artery diameter and increased
arterial flow in the setting of acute AH. These findings can be seen on duplex
Doppler ultrasound, although the diagnosis is usually clinched without the need
for this test.

46. This is concerning for hepatocellular carcinoma (HCC). Patients with hepa-
titis B cirrhosis are at especially high risk for cancer, in general, so you always
need to be on the lookout for HCC in this population. The risk is especially high
in patients who are both HBsAg and HBeAg positive. Those with double positivity
have a 60 times higher risk of developing HCC compared to patients who are dou-
ble negative for HBsAg and HBeAg. Those who are positive for HBsAg alone are
around 10 times more likely to develop HCC than patients who are double nega-
tive, which is still high, but it’s the HBeAg positivity that really confers the major
risk. This patient is double positive, so the risk of HCC is especially high. And
HCC, in turn, is a hypervascular lesion that can sometimes generate a hepatic
bruit on auscultation. Although this is not the usual way that HCC is identified,
there are some instances when HCC is detected by auscultating a hepatic bruit.
Another pearl of wisdom is that new-onset hepatic encephalopathy should also
trigger a search for underlying HCC if there are no other obvious causes for the
encephalopathy (see Vignette 11).
It’s nearly impossible to overstate the epidemiologic importance of HCC.
Whereas the incidence of cancer has been slowly dropping across most major
cancer diagnoses, the same cannot be said for HCC. With the rising tide of
patients with chronic hepatitis C and NAFLD, we are now witnessing a tidal
wave of new HCC diagnoses. HCC is the third leading cause of cancer death and
the fourth most prevalent malignancy worldwide. According to the World Health
Organization, over 500 million people globally die each year from primary liver
cancer. So you can rest assured that HCC will be on the Board exam—if it isn’t,
then the examiners are not doing their job.

47. This is an extrahepatic hepatic artery aneurysm (HAA). Extrahepatic


HAA most often occurs in the setting of atherosclerosis, but it also has been
described with medial degeneration, trauma, and mycotic infection. Intrahepatic
HAA, in contrast, is usually a false aneurysm resulting from trauma. Although
less relevant for a general GI examination, anastomotic complications from
orthotopic liver transplantation can also lead to HAA.
HAA can be very dangerous (perforation and exsanguination in up to 20% of
patients), but it’s hard to diagnose since most people are asymptomatic. Patients
sometimes have symptoms that are biliary in nature, like this patient, who
presented with recurrent right upper quadrant abdominal pain. Ultrasound
imaging may demonstrate a cystic mass, although this can be mistaken for a
tumor. Follow-up CT is more reliable for demonstrating the vascular contrast
“Tough Stuff” Vignettes 129

enhancement within an HAA (Figure 47-1). Extrahepatic HAA often occurs in


the gastroduodenal ligament between the pancreatic head and the left liver lobe,
as was seen here. This type of HAA is treated surgically, typically with proximal
and distal ligation, since the gastroduodenal artery serves as a collateral ves-
sel to help supply the liver. But an HAA that arises distal to the gastroduodenal
artery requires a more extensive repair. When an HAA is intrahepatic, things get
more difficult—in some cases partial liver resection is necessary to remove the
aneurysm.

Figure 47-1. Hepatic artery aneurysm with calcifi-


cations appearing as a cyst-like structure near the
porta hepatis. (Reprinted with permission of Barbara
Kadell, MD, UCLA Medical Center.)

48. This is the Cruveilhier-Baumgarten murmur from an underlying sponta-


neous portosystemic shunt due to portal hypertension. This is the ultimate in
roundsmanship—virtually every hepatologist asks unsuspecting students, resi-
dents, and fellows about this sign. In fact, they have been known to look at others
in amazement if they don’t know this eponym, almost as if the person didn’t go to
medical school. Although it’s not really vital to know this sign, it’s eminently test-
able. The Cruveilhier-Baumgarten murmur occurs when there is recanalization
of the umbilical vein and reversal of blood flow from the liver into the abdominal
veins, which can give rise to a caput medusae, or “Medusa’s head.” In some cases
this coincides with improvement of the underlying ascites, likely because the new
portosystemic shunt helps to mobilize fluid by bypassing the portal blockade.

49. This is a traumatic arteriovenous (AV) fistula, possibly between the hepatic
artery and the portal vein. The vignette does not give you enough information
to know for sure, and this could even be an HAA, as seen in Vignette 47. But
the persistent symptoms several weeks after an operation suggest a persistent
and previously unrecognized AV fistula from the trauma. Normally AV fistulas
lead to high-output heart failure, but this is less common when the AV fistula
is intrahepatic. Based on experiments in dogs going back to the 1960s, fistulas
between the portal vein and hepatic artery are dampened by resistance in the
surrounding hepatic sinusoids, so patients do not fall into heart failure the way
they might with AV fistulas in other parts of the body. But they often do have
persistent abdominal pain or discomfort, and they may even have low-grade
hemobilia from leakage into the biliary system (as seen here). Treatment is usu-
ally to fix these surgically.
130 Chapter 2

Vignette 50: Coffee and the Liver


Let’s say you drink a lot of coffee.

...That’s the whole vignette.

` Does this habit protect you from liver problems?


` If so, which problems, and why?
“Tough Stuff” Vignettes 131

Vignette 50: Answer


There has been a lot of talk about the relationship between coffee consumption
and liver disease—so much that it’s now reasonable to expect a Board question
on this topic. This relationship is more than an epidemiologic oddity at this point;
data have been mounting for years that coffee consumption may be protective
against certain forms of liver disease. Indeed, meta-analyses have looked at the
protective benefits of coffee consumption in this regard.
The strongest data pertain to protection against hepatocellular cancer, in par-
ticular. Overall, drinking 2 cups of coffee per day is associated with a roughly
40% reduction in liver cancer compared to drinking less coffee. This protective
effect is not only in those with pre-existing liver disease, such as cirrhosis, but
even in those who do not have cirrhosis—although the benefits are larger in
the former group (about 45% reduction in those with liver disease, and 30% in
those without liver disease). There also seems to be a dose-response relationship
between coffee consumption and hepatocellular cancer (ie, the more coffee you
drink, the higher the protection). The protection is not just for liver cancer, but
also for earlier forms of liver injury. Coffee consumption has been inversely asso-
ciated with aminotransaminase levels in studies from around the world, with an
especially strong benefit in those who drink a lot of coffee. So, coffee consump-
tion appears to be protective against developing cirrhosis in the first place, even
prior to developing liver cancer. In addition, recent data suggest that high levels
of coffee consumption (more than 3 cups per day) improves virologic response to
peginterferon plus ribavrin in patients with hepatitis C. Finally, there is evidence
that coffee consumption can reduce the progression of disease in patients with
chronic hepatitis C virus infection.
The way in which coffee might protect people from liver injury remains unclear.
It seems to be more than the caffeine alone, although caffeine may be part of
the story. The bottom line is that nobody really knows, so you would not need to
know the mechanism for a Board exam.
You should, however, know that the relationship between coffee consump-
tion and liver disease is an association, not a cause-and-effect relationship.
Understanding the difference between causation and association is important.
In fact, this is fair game for a Board exam, since the exam includes a variety
of questions on study design, evidence-based medicine, and epidemiology. In
order for something to be causal, several things need to line up, including the
following: (1) There needs to be biologic plausibility for the relationship; (2) there
should be a consistent association among many different studies with differ-
ent populations and designs; (3) there should be a temporal relationship, such
that the exposure comes first and the outcome follows; and (4) there should be
a dose-response relationship, so that the more someone is exposed to the risk
factor, the more likely he or she is to develop the outcome of interest. In the case
of coffee and liver disease, there appears to be some biologic plausibility, the
findings seem reproducible among many studies, and there is a dose-response
relationship. However, there is not a clear temporal relationship—demonstrating
that would ultimately require a prospective, randomized, controlled trial. It’s also
possible that the relationship is confounded such that patients with liver disease
might end up drinking less coffee because of their illness—not that the coffee
protected them. For now you should at least recognize that there is a pretty good
132 Chapter 2

case that coffee is somewhat protective against elevated aminotransferase levels,


hepatitis C progression, cirrhosis, and even hepatocellular cancer.
Why Might This Be Tested? Liver cancer, in particular, is a hot topic. And
the epidemiology of liver cancer is also increasingly important. Since coffee is
now widely considered to be a protective factor for liver cancer, this epidemiologic
observation is eminently testable.

Here’s the Point!

Coffee consumption is epidemiologically protective against elevated AST/ALT,


cirrhosis, and hepatocellular cancer.
“Tough Stuff” Vignettes 133

Vignette 51: Mushroom Madness


A 51-year-old American woman developed colicky abdominal pain and watery
diarrhea 1 day after eating a mushroom she found while hiking on vacation in
Ireland. She nonetheless boarded a plane to return to the United States and devel-
oped progressive nausea and vomiting while en route. Upon arrival she reported
to her local hospital, where she was found to be delirious, jaundiced, hypotensive,
and tachycardic. Lab tests included the following: total bilirubin = 8.2, INR = 4.2,
AST = 190, ALT = 210, albumin = 3.2, and creatinine = 2.1.

` What happened here?


` How should this be treated?
134 Chapter 2

Vignette 51: Answer


This is acute liver failure from mushroom poisoning with Amanita phalloides.
The Amanita mushroom, also known as the “death cap,” is found across many
parts of Europe, including Ireland (as in this case). In contrast, the death cap
is not commonly found in the United States. All it takes is 30 g, or about half a
normal mushroom cap, to kill an adult. The mushroom is full of toxins, including
amatoxins and phallotoxins. These toxins inhibit RNA polymerase II, which ulti-
mately shuts down the synthesis of essential proteins. The effect is most marked
in the liver, since that is where the toxins are first encountered after ingestion.
The toxins can also cause a hemolytic anemia and can often lead to kidney dam-
age as well. When the toxicity is severe, acute liver failure and death can occur.
The treatment for Amanita-related liver failure is similar to the treatment
for other forms of acute liver failure. In particular, it’s important to screen for
evidence of elevated intracranial pressure, monitor acid-base and renal status,
ensure an intensive level of care, and triage for liver transplantation when nec-
essary. It’s also worth performing gastric lavage if the ingestion was known to
be recent. Unfortunately, the patient typically presents days after the initial
ingestion. For the Board exam, remember that these patients should also receive
continuous infusions of penicillin G, although the mechanism of action remains
unclear. In addition, extract of silymarin, or milk thistle, has been used success-
fully. The milk thistle extract is thought to prevent uptake of the amatoxins by
the hepatocytes; it also may stimulate RNA polymerases, thereby kick-starting
RNA synthesis and helping to reverse the effects of the Amanita toxins.
Why Might This Be Tested? Acute liver failure is eminently testable. So
you should know acute liver failure 6 ways to Sunday, so to speak. This is just
another in a long list of unusual presentations of acute liver failure. Moreover, it
has a specific antidote that could be easily tested on an exam (milk thistle and
penicillin G). Be prepared for unusual etiologies of acute liver failure, such as
exertional heat stroke (Vignette 41), latent hepatitis B reactivated during immune
reconstitution (Vignette 30), kava kava ingestion (Vignette 14), and so forth.

Here’s the Point!

Mushroom ingestion + Acute liver failure = Amanita (death cap) poisoning.


Start penicillin G and silymarin (milk thistle extract).
“Tough Stuff” Vignettes 135

Vignettes 52 to 55: Vitamin Power!


Vitamins are organic compounds that cannot be made by humans but are
required as dietary nutrients to prevent metabolic disorders. Fat-soluble vitamin
deficiencies are common in cholestatic liver disease, especially after the develop-
ment of jaundice. It’s important to know how to identify these vitamin deficien-
cies. The following mini-vignettes describe signs and symptoms of patients with
end-stage cholestatic liver disease. For each, identify the culprit vitamin defi-
ciency and specify the supplementation strategy.

52. Problems with balance while ambulating.

53. Difficulty driving at night.

54. Persistent bleeding after polypectomy.

55. Hip pain with osteopenia on DEXA scan.


136 Chapter 2

Vignettes 52 to 55: Answers


52. This is vitamin E (tocopherol) deficiency, with classic symptoms of ataxia
and gait disturbance. These patients can also have peripheral neuropathy and
hyporeflexia. If you think about it, this scenario would also be typical for a case
of overt hepatic encephalopathy, which highlights the importance of checking lev-
els of vitamin E when you encounter neurologic symptoms like this in a patient
with cholestatic liver disease. Vitamin E is an antioxidant and is responsible for
the protection of cell membranes from oxidative damage. It’s found in vegetable
oils, meat, and eggs. Under normal conditions, we need just a touch of vitamin
E to avoid neurologic sequelae from deficiency; only 15 mg (approximately 22 IU)
is recommended on a daily basis. However, greater amounts are required for
supplementation in vitamin E deficiency states (typically 1000 IU per day).

Here’s the Point!

Cholestatic liver disease + Ataxia = Think about vitamin E deficiency

53. This is vitamin A (retinol) deficiency. A sophisticated people well ahead of


their time, the ancient Egyptians fed liver to people with night blindness—pretty
smart! More recently, we have learned that liver is an excellent food source for
vitamin A. Retinol is also found in egg yolks, butter, and carrots. Other symp-
toms of vitamin A deficiency include hyperkeratosis and xerophthalmia with cor-
neal and conjunctival xerosis. If a patient has symptoms of vitamin A deficiency,
serum retinol levels should be checked. Levels less than 20 mcg/dL should
prompt replacement of vitamin A. The recommended daily allowance of vitamin
A is 900 mcg, or 3000 IU. Higher doses are needed in deficient states (typically
10,000 to 25,000 IU per day). However, care should be taken to avoid overdos-
ing, because chronic toxicity can occur when over 50,000 IU are taken daily
(see Vignette 21). This has been known to occur in some “health nuts” who take
excessive amounts of vitamins, including vitamin A. Overdosing can lead to cir-
rhosis with liver biopsy showing Ito (fat-storing) cell hyperplasia with vacuoles.

Here’s the Point!

Cholestatic liver disease + Night blindness = Think about vitamin A deficiency


“Tough Stuff” Vignettes 137

54. This is vitamin K deficiency. Mucosal bleeding (which occurred after


polypectomy here) and easy bruising are hallmarks of vitamin K deficiency. As
you probably recall from medical school, vitamin K-dependent factors (II, VII,
IX, X and protein C) are intricately involved in coagulation. Vitamin K is found
in leafy vegetables and can also be produced by gut bacteria (which can be
undermined by antibiotics). Thus, when you treat an infection in a patient with
cirrhosis and you notice that the patient has a drastic increase in the MELD
score, the increase may not necessarily be from hepatic deterioration. That’s a
great setup for a Board question—a patient with PBC who develops bleeding and
an elevated INR after starting antibiotic therapy for hepatic encephalopathy. The
elevated INR might simply be due to vitamin K deficiency, not from progression
of the liver disease. Administration of 10 mg of vitamin K can often correct the
INR and can be given through multiple routes of administration, depending on
the clinical scenario.

Here’s the Point!

Patient with cirrhosis starts antibiotics and MELD peaks =


Think vitamin K deficiency

55. This is vitamin D deficiency. It seems like everyone is vitamin D deficient


these days. Well, not everyone—but vitamin D deficiency is much more common
than most people realize. For the GI Board exam, remember that vitamin D defi-
ciency is particularly common in inflammatory bowel disease and celiac sprue.
Serum testing with measurement of the 25-hydroxyvitamin D level can aid in
the diagnosis and treatment of these disorders with the goal of a 25-hydroxyvi-
tamin D level of greater than 30 ng/mL. Vitamin D is found in fish, eggs, and
certain mushrooms (but not the kind of mushroom featured in Vignette 51!). It
is also manufactured by skin, provided there is adequate exposure to sunlight.
Replacement can be given in various forms in high doses and then tapered to
reach the desired level. However, in malabsorptive conditions, the patient may
require 10,000 IU of vitamin D with high doses of calcium (up to 4000 mg).
Prevention of osteoporosis and fractures is extremely important in these patients
because disabling fractures and lack of mobility can limit candidacy for liver
transplantation. Thus, bisphosphonate therapy should be considered in patients
with osteopenia to prevent osteoporosis if other risk factors for fracture are pres-
ent.

Here’s the Point!

Cholestatic liver disease + Osteopenia = Think about vitamin D deficiency


138 Chapter 2

Why Might This Be Tested? Vitamin deficiencies are Board review fodder,
both for internal medicine and subspecialty Board examinations. While you
may think it will be easy to spot some of these deficiencies in an exam question,
sometimes the most obvious answer will not be at the top of your mind during the
stress of an exam. So, it’s good to review all of the fat-soluble vitamin deficiency
states while preparing for the test. In clinical practice, these deficiencies can be
diagnosed and treated readily by the observant physician.
“Tough Stuff” Vignettes 139

Vignette 56: Fatigue and an Elevated Antinuclear Antibodies


A 32-year-old woman presents to her primary care physician with a chief
complaint of progressive fatigue. She does not recall signs of icterus and has
not experienced a recent viral illness. She drinks alcohol on occasion, reporting
only 1 to 2 glasses of wine on weekends. She is not using any medications, does
not report herbal exposures, and is not using illicit drugs. Her past medical his-
tory is unremarkable. On physical exam, there are no stigmata of chronic liver
disease, there is no icterus and no jaundice, and the abdominal exam is normal.
Laboratory tests reveal a normal blood count, INR = 1.0, total bilirubin = 1.2,
albumin = 4.1, ALT = 206, AST = 189, and ALP = 151. Further laboratories reveal
a negative HBsAg and negative anti-HCV antibody. Antinuclear antibodies (ANA)
is elevated at 1:640 and AMA is elevated at 1:320. Anti-smooth muscle antibody
(anti-SMA) is 1:40. Serum ferritin is 280, and ceruloplasmin levels are normal.

` What is the most likely diagnosis?


` What might a liver biopsy reveal?
140 Chapter 2

Vignette 56: Answer


This is an overlap syndrome of autoimmune hepatitis (AIH) and PBC. AIH can
present in both typical and atypical forms, and it can be a difficult diagnosis to
pin down. The diagnostic dilemma is compounded by the fact that AIH some-
times overlaps with other conditions, namely PBC and PSC.
This gets even more confusing because the AIH-PBC overlap syndrome can
come in 2 different forms. One form includes patients who are positive for AMA
with histologic evidence of AIH but without histologic evidence of PBC. Some call
this condition AMA-positive AIH. The clinical course and natural history of AMA-
positive AIH are very similar to those of the pure, type 1 AIH. The second form
of AIH-PBC overlap includes patients who are positive for ANA and/or anti-SMA
with histologic evidence of PBC but without histologic evidence of AIH. This is
often referred to as autoimmune cholangiopathy. Others think this is just AMA-
negative PBC—not really an overlap syndrome.
The main thing to understand is that AIH and PBC, in particular, can overlap,
and this can have both diagnostic and therapeutic implications. Around 10% to
15% of patients initially diagnosed with PBC end up having biochemical and/or
histologic evidence of overlapping AIH. The converse is true as well—around the
same percentage of patients with AIH demonstrate some evidence of comorbid
PBC. Sometimes these disorders are found together at diagnosis, but oftentimes
the natural history is that one disorder occurs first, and the other follows in
time.
The patient in this vignette has a markedly elevated ANA and AMA, along
with elevated aminotransferases and an elevated (but not too elevated) ALP. The
anti-SMA is not particularly high. The histology is not described, but there are
clear biochemical markers of both AIH and PBC. The picture is suggestive of
AMA-positive AIH; it’s less suggestive of autoimmune cholangiopathy. Since the
natural history of AMA-positive AIH is similar to that of type 1 AIH, treatment
should follow the usual AIH guidelines. In contrast, patients with autoimmune
cholangiopathy, or AMA-negative PBC, should be treated with ursodeoxycholic
acid (UDCA) and often glucocorticoids.
Why Might This Be Tested? You know that the Board exam will feature a
number of questions about AIH, PBC, and PSC. But examiners will want to see
if you know the next step, which is how to identify and manage common overlap
syndromes across these conditions. The AIH-PBC overlap, in particular, is a good
one to test since it has 2 different forms with different natural histories and treat-
ment approaches. This can be confusing, and what better than a confusing area
to cull the wheat from the chaff among test-takers?

Here’s the Point!

ANA positive + AMA positive + Elevated aminotransferases = AMA-positive AIH

ANA positive + Anti-SMA positive + AMA negative + Elevated ALP =


AMA-negative PBC, also known as autoimmune cholangiopathy
“Tough Stuff” Vignettes 141

Vignette 57: Holy Pilgrimage


A 38-year-old woman, recently divorced, went to Surat, India, for a spiritual
awakening to “find herself.” She received the usual vaccinations from a travel
clinic prior to leaving. She has just returned to the United States and discovered
that she is 6 weeks pregnant. She was promptly referred to see you because of
complaints of fatigue, fever, anorexia, jaundice, dark-colored urine, and light-
colored stools for the past 2 weeks. Abdominal ultrasound is significant for
hepatomegaly. Laboratory tests reveal the following: ALT = 2310, AST = 1640,
LDH = 1290, total bilirubin = 3.2, WBC = 9.1, INR = 1.4, platelets = 160.

` What is the most likely diagnosis?


` Can this be prevented?
142 Chapter 2

Vignette 57: Answer


This is hepatitis E virus (HEV) infection, which can be confirmed with an IgM
anti-HEV titer. Like HAV infection, there is no need to call it “acute” since there
is no chronic form of this infection. Travel to an endemic region should tip you off
to this diagnosis. HEV was first recognized in India more than 30 years ago and
is not endemic in the United States, but international travelers to Asia, Africa,
Central America, and the Middle East could become infected. In fact, there was
an outbreak in Surat (which is where one of the authors of this book was born)
that was reportedly due to a contaminated drinking water pipeline. This was cor-
rected and led to marked improvements in sanitation thereafter. The incubation
period for HEV is approximately 1 to 2 months before the classic symptoms of
jaundice, anorexia, nausea, abdominal pain, and fever are declared. Since acute
viral hepatitis is the most common cause of jaundice in pregnancy, you should
also check serologies for HAV, HBV, HCV, hepatitis D virus (HDV), Epstein-Barr
virus (EBV), and cytomegalovirus (CMV). By the way, you should know that CMV
hepatitis can give a distinct histological appearance with an “owl’s eye” due to
the surrounding halo of the intranuclear inclusion (Figure 57-1). HAV infection
is less likely in this case since the patient should have been vaccinated against
HAV at the travel clinic.
Similar to HAV infection, HEV is an RNA virus spread via the fecal-oral route
and is generally self-limited. But HEV can be more severe than HAV infection
with up to a 4% mortality rate, and among pregnant women in the third trimester,
the mortality rate is a sobering 20%. Acute herpes simplex virus (HSV) hepatitis
(Figure 57-2) also has a marked increase in mortality in pregnancy. Therefore,
empiric treatment with acyclovir should be considered until the diagnosis is con-
firmed. In contrast, HAV does not have an increased mortality in pregnancy.
Treatment for HEV consists of supportive care with referral to a liver trans-
plant center if severe. There are 4 classified genotypes of HEV, and epidemiologic
studies are in progress to better understand the differences. Furthermore, a
recombinant well-tolerated vaccine shows promise for the future. Prevention is

Figure 57-1. CMV hepatitis. An aggre-


gate of neutrophils surrounds an
infected hepatocyte [white arrow],
containing an intranuclear inclusion
with a surrounding halo, also known
as an “owl’s eye” inclusion. In the
inset, CMV immunohistochemis-
try stains an infected hepatocyte
[black open arrow], confirming the
presence of CMV. (Reprinted with
permission of Alton B. Farris, MD,
Emory University.)
“Tough Stuff” Vignettes 143

the key for now. International travelers should be wary of drinking water and/or
ice of unknown purity and avoid uncooked seafood and unpeeled/uncooked
fruits and vegetables that are not prepared by the traveler.
Why Might This Be Tested? There are more people traveling globally these
days, leading to more infectious diseases coming home. We have said before
that pregnancy is a favorite topic on Board exams; examiners especially like to
include conditions for which there is a marked difference in pregnancy.

Figure 57-2. HSV hepatitis. The hepatic


parenchyma shows extensive necro-
sis with scattered hepatocytes with
glassy nuclear inclusions [arrows] and
occasional multinucleated cells [open
arrows] amidst the necrosis. (Reprinted
with permission of Charles Lassman,
MD, UCLA Medical Center.)

Here’s the Point!

Traveler to Indian subcontinent with acute hepatitis → Think hepatitis E

Here’s the Point!

HEV and HSV hepatitis → Increased mortality in pregnancy

Here’s the Point!

Intranuclear “owl’s eye” inclusions → Think CMV


144 Chapter 2

Vignette 58: Big Liver Cysts


A 41-year-old pilot with autosomal-dominant polycystic kidney disease
(ADPKD) presents to your office for initial consultation, having recently moved
to your area. She brings her abdominal MRI taken last year for you to review
(Figure 58-1). Although her kidney cysts have not been problematic, she has
encountered persistent problems with innumerable, diffuse, large liver cysts, as
pictured. She describes chronic pain and recurrent infections from these lesions
despite having received fenestration procedures for palliation. She was told by
her surgeon that resection was not feasible due to the diffuse nature of her large
hepatic cysts. In fact, her condition has limited her mobility and she is no longer
able to fly planes. She now presents to you with anorexia and early satiety with
a resultant 50-pound unintentional weight loss over the past 6 months. Her BMI
has fallen to 19. Laboratories include the following: ALT = 21, AST = 22, total
bilirubin = 1.0, ALP = 110, albumin = 3.2, BUN = 7, creatinine = 0.6, INR = 1.0,
WBC = 8.2, hemoglobin = 13.8, and platelets = 190. Due to her signs and symp-
toms and the concern for gastric malignancy, you perform an esophagogastro-
duodenoscopy (EGD), which is remarkable only for the finding in the esophagus
shown in Figure 58-2.

Figure 58-1. T2-weighted abdominal MRI of this patient.


(Reprinted with permission of Barbara Kadell, MD, UCLA
Medical Center.)
“Tough Stuff” Vignettes 145

Figure 58-2. Esophageal lesions found on upper


endoscopy.

` What is going on here?


` What should you recommend for long-term treatment?
146 Chapter 2

Vignette 58: Answer


Those plump esophageal varices are indirectly due to autosomal-dominant
polycystic kidney disease (ADPKD). Portal hypertension and the formation of
esophageal varices can occur through a few different mechanisms in ADPKD,
even in the absence of cirrhosis. The causes of noncirrhotic portal hypertension
in ADPKD include portal vein compression by the massive hepatic cysts (thus,
without ascites), hepatic vein or inferior vena cava compression by the massive
hepatic cysts (thus, often with ascites), and congenital hepatic fibrosis in asso-
ciation with ADPKD (rare). Take a look at Vignette 3 for a refresher on portal
hypertension.
Liver cysts are the most frequent extrarenal manifestation of ADPKD. The liver
cysts tend to be more common in women than in men with ADPKD. Usually,
liver cysts (with or without ADPKD association) do not pose much of a prob-
lem and rarely cause symptoms. As an aside, benign liver cysts in general (not
associated with ADPKD) are also relatively common and occur in 4% of normals
with a female predominance and increasing prevalence with advancing age.
With benign liver cysts (whether ADPKD related or not), the liver parenchyma
itself does not increase in size (therefore hepatic synthetic function is usually
normal), but cystic enlargement can lead to massive hepatomegaly. This causes
pain through stretch of the liver capsule and can also trigger other symptoms
related to mechanical compression. The patient in this vignette had secondary
gastric outlet obstruction from gastric compression, manifesting with early sati-
ety and weight loss. In addition, compression of the venous system can lead to
extrahepatic portal hypertension, as seen here. Unfortunately, this patient did
not respond to cyst fenestration and was unable to undergo resection due to her
diffusely large cysts.
Primary prophylaxis of variceal hemorrhage is indicated for this patient (see
Vignette 13 for more on that). She should also be referred for consideration of
liver transplantation before malnourishment poses a problem. Liver transplanta-
tion has its own set of risks, but the benefits would probably outweigh the risks
in this particular case. If she is accepted onto a liver transplantation waiting list,
the transplant center would need to appeal to UNOS for a higher MELD excep-
tion score, since her intrinsic MELD score (6) is normal. See Vignette 9 for more
on MELD. So, this is a rare case where liver transplantation is reasonable even
though there is no cirrhosis or liver synthetic dysfunction—seems like a nice
setup for a Board question.
This particular patient should also be screened for an intracranial aneu-
rysm, which occurs in 6% of patients with ADPKD without a family history of
intracranial hemorrhage (16% with a family history). Screening for intracranial
hemorrhage may not be appropriate for all patients with ADPKD. However, since
the patient in this case is a pilot, it would be prudent to avoid the devastating
consequences of an intracranial aneurysm rupture. Thus, screening with brain
magnetic resonance angiography is indicated.
Why Might This Be Tested? Liver involvement is the most frequent extrarenal
manifestation of ADPKD. The consequences of massive hepatomegaly can be dif-
ficult to manage. Liver transplantation can offer a last resort to select patients
if other options do not help. The anatomic relationships of this condition are
important to understand and fair game for the exam.
“Tough Stuff” Vignettes 147

Here’s the Point!

Massive liver cysts in ADPKD → Massive hepatomegaly → Portal hyperten-


sion and gastric outlet obstruction

Here’s the Point!

Massive liver cysts + Intracranial aneurysm = Autosomal-dominant polycystic


kidney disease (ADPKD)
148 Chapter 2

Vignette 59: Autosomal-Dominant Polycystic Kidney Disease


Case, Continued
The patient with ADPKD in Vignette 58 undergoes liver transplantation. She
makes an excellent recovery, returns to flying (of course, after a screening brain
MRA was negative), and enjoys spending time at the beach with her family. She
has been doing well for 7 years after liver transplantation and is now 48 years
old. However, today she comes into the emergency department with fever and sud-
den, severe, generalized abdominal pain with voluntary guarding and rebound
tenderness on examination.

` What known complication of ADPKD is this consistent with?


` What else does she need to worry about in the long term?
“Tough Stuff” Vignettes 149

Vignette 59: Answer


This is colonic perforation secondary to acute diverticulitis. Patients with
ADPKD have increased frequency of diffuse colonic diverticulosis and compli-
cations related to diverticular disease. In addition, patients with ADPKD have
an increased risk of abdominal wall hernias (45%) compared to controls (4%).
Therefore, a patient with ADPKD who presents with abdominal pain needs evalu-
ation for these possibilities as well as for renal cyst bleeding, infection, and/or
rupture.
This vignette provides an opportunity to review some long-term complications
of liver transplantation. Skin cancer is by far the most common malignancy that
occurs de novo after transplantation. In particular, squamous cell carcinoma
is the cancer with the highest incidence following liver transplantation. In con-
trast, basal cell carcinoma comprises 90% of skin cancers in immunocompetent
patients. Thus, annual full-body skin examination is an extremely important
preventive modality for routine health care maintenance in liver transplant
recipients. Furthermore, it’s important to advise patients to avoid prolonged sun
exposure, especially during peak sun hours (9 AM until 4 PM), and to wear long
sleeves and a hat when possible. Sunscreen is also important to help block both
UV-A and UV-B radiation. In this case, the patient needs to know that prolonged
sun exposure at the beach, although enjoyable, may increase her risk for devel-
oping skin cancer following liver transplantation.
Cardiovascular disease is also problematic in the long term after liver trans-
plantation due to a combination of risk factors that develop. These include
hypertension, hyperlipidemia, renal failure, and diabetes mellitus, which are in
part due to immunosuppressive medications (especially calcineurin inhibitors,
such as tacrolimus and cyclosporine). Chronic rejection is becoming much less
of a problem. In fact, there has been a shift to using lower immunosuppression
doses in the long term to decrease these secondary complications. Lowering
immunosuppression should also help with reducing infections and malignancy,
which are important causes of mortality in these patients. Furthermore, up to
40% of patients meet criteria for obesity within 1 year after transplantation,
which contributes to cardiovascular complications over the long term as well.
Vigilant management of these modifiable risk factors should help reduce long-
term mortality.
Why Might This Be Tested? Survival has steadily increased following liver
transplantation, which means that there are a larger number of post-transplant
patients requiring long-term care and routine health care maintenance. Liver
transplant centers are often returning these patients to the referring gastroen-
terologist for follow-up. It’s important to recognize the possible complications that
may arise so that preemptive therapy can be instituted.

Here’s the Point!

Colonic diverticulosis and its complications are more common in ADPKD.


150 Chapter 2

Here’s the Point!

Squamous cell skin cancer is the most common malignancy after liver
transplantation.

Here’s the Point!

Calcineurin inhibitors increase risk for post-transplant hypertension,


hyperlipidemia, diabetes, and renal failure.

Here’s the Point!

Obesity is common following transplant and contributes to


cardiovascular risk.
“Tough Stuff” Vignettes 151

Vignettes 60 to 63: Low Alkaline Phosphatase Potpourri


High alkaline phosphatase (ALP) levels should conjure up a broad differential
diagnosis. You can find a series of vignettes dealing with isolated elevated ALP in
the first Acing book. Here we are focusing on the flip side: low ALP. The differen-
tial diagnosis of low ALP is relatively narrow, so what better excuse to tie together
a few questions featuring this unusual lab finding? The following mini-vignettes
describe patients with a low ALP. Try to figure out the most likely diagnosis for
each one.

60. Ascites with SAAG >1.1, ascitic total protein >2.5, fatigue, constipation,
hair changes, and a low ALP.

61.Developed gray hair early in life, now with dyspepsia, macrocytic anemia,
and a low ALP.

62. A 20-year-old woman presents with acute hepatitis, hemolytic anemia,


and a depressed ALP level.

63. A patient with Crohn’s disease develops diarrhea and a scaling vesiculo-
pustular plaque-like rash on the legs and face. The ALP is diminished.
152 Chapter 2

Vignettes 60 to 63: Answers


60.This is hypothyroidism. On occasion, severe hypothyroidism can present
with ascites and a low ALP. The ascites is typically characterized by a high ascitic
total protein count and a high serum–ascites albumin gradient, or SAAG. An
elevated SAAG is due to a transudative process such as cirrhotic ascites, acute
liver failure, alcoholic hepatitis, or cardiac ascites from congestive heart failure.
A SAAG <1.1 suggests an exudative process such as infection or malignancy.
This case features a rare form of an elevated SAAG without portal hyperten-
sion and severe hypothyroidism (usually outright myxedema) due to an uncer-
tain mechanism. The myxedema secondary to the hypothyroidism produces an
elevated ascitic fluid total protein in combination with the high SAAG (similar to
cardiac ascites). The low ALP observed in some patients with hypothyroidism is
of unclear etiology. Some think the decreased ALP results from low serum zinc
and magnesium levels in hypothyroidism. In any event, the ALP usually returns
to normal with treatment. Refer to Figure 60-1 for a review of how to interpret
the SAAG and ascitic fluid total protein.

Figure 60-1. Diagnostic algorithm for combining serum–ascites albumin gradient (SAAG) with
total protein of ascitic fluid.

Here’s the Point!

Low ALP + High SAAG, High total protein ascites = Think hypothyroidism
“Tough Stuff” Vignettes 153

61.This is pernicious anemia. Pernicious anemia is an autoimmune disorder


marked by antiparietal cell antibodies, low intrinsic factor, and diminished vita-
min B12 absorption with a macrocytic anemia. These patients often have comor-
bid autoimmunity. For example, early graying of the hair has been described in
patients with pernicious anemia, which is probably an autoimmune phenom-
enon. The low ALP of pernicious anemia it thought to occur because osteoblast
activity depends on cobalamin, and when there is cobalamin deficiency, bone
metabolism is perturbed and ALP is diminished.

Here’s the Point!

Low ALP + Macrocytosis + Early graying = Think pernicious anemia

62. This is acute Wilson disease (see Vignettes 12 and 36). There can be a
marked depression in ALP levels in the acute phase of WD. The “Here’s the Point!”
for this vignette is the same as in Vignette 36—a point so pertinent we repeat it
here.

Here’s the Point!

Hepatitis + Depressed ALP + Mallory bodies = Wilson disease

63. This is zinc deficiency. Zinc deficiency is especially common in patients


with Crohn’s disease and is also seen in alcoholics (beer drinkers, in particular,
as beer contains virtually no zinc). In the past, zinc deficiency was also seen
in patients on total parenteral nutrition (TPN) therapy, when zinc was not rou-
tinely added to the formulation. Zinc may also be sequestered in the liver during
episodes of physiological stress, including trauma, infection, and other acute
systemic inflammatory conditions. In these instances, there is not a true defi-
ciency but a functional deficiency, as the zinc is maldistributed. The deficiency in
Crohn’s disease appears to be due to poor zinc absorption, but it may also arise
from underlying inflammation with a functional deficiency. Zinc deficiency leads
to a characteristic acral-predominant skin lesion that appears as an erythema-
tous, scaling, vesicopustular plaque on the legs and face. It’s also associated with
diarrhea, as occurred here. The lesions can rapidly improve with appropriate
zinc supplementation. ALP is a zinc-metalloenzyme that requires magnesium for
activity, so deficiencies of either zinc or magnesium can lead to diminished ALP
activity in serum and tissue.
154 Chapter 2

Here’s the Point!

Crohn’s disease + Low ALP = Zinc deficiency

In addition to the conditions described in Vignettes 60 to 63, diminished ALP


has been associated with a number of other clinical conditions and settings. For
example, low ALP is commonly observed following cardiac surgery with cardio-
pulmonary bypass. The mechanism of diminished ALP in this setting is unclear,
but it may have something to do with the bypass itself. In addition, it has been
reported in vitamin C deficiency (ie, scurvy), in folic acid deficiency, and with low
levels of phosphorus. Some patients with celiac sprue have a low ALP (probably
from vitamin and micronutrient malabsorption), and it has also been seen in
chronic anemia (mechanism also unclear). Table 63-1 provides a full list of condi-
tions that have been associated with diminished ALP levels.

Table 63-1.
CONDITIONS ASSOCIATED WITH LOW
ALKALINE PHOSPHATASE LEVELS
(IN NO PARTICULAR ORDER)
Zinc deficiency
Magnesium deficiency
Vitamin C deficiency/scurvy
Folic acid deficiency
Vitamin B6 deficiency
Excess vitamin D ingestion
Low phosphorus levels, as seen in hypophosphatasia
Celiac disease
Pernicious anemia
Protein-calorie malnutrition
Hypothyroidism
Acute Wilson disease
Cardiac surgery and cardiopulmonary bypass
Severe anemia
Milk-alkali syndrome
Hypoparathyroidism
“Tough Stuff” Vignettes 155

Vignette 64: Poor Response to Interferon/Ribavirin


A 54-year-old man with genotype 1 hepatitis C virus (HCV) is started on
combination therapy with pegylated interferon and ribavirin. The patient is
compliant with the medication. HCV RNA is measured at week 12 and remains
detectable. There has been a 1-log reduction in HCV RNA from the pretreatment
baseline level to the week 12 level.

` How do you classify this response?


` Should you continue therapy?
156 Chapter 2

Vignette 64: Answer


This patient has a null response at 12 weeks, and treatment should be dis-
continued. This vignette provides an excuse to review the frequency for measur-
ing HCV RNA levels during treatment for HCV, and interpretation of the viral
kinetics. Early virologic response, or EVR, is defined by a 2-log drop or more in
HCV RNA level at week 12. This week 12 milestone has a high predictive value
for achieving sustained virologic response (SVR), defined as absent HCV RNA
6 months after treatment cessation. If a patient with genotype 1 HCV achieves
EVR, then the SVR rate is around 66%, which is pretty good (Table 64-1). In con-
trast, if a patient with genotype 1 HCV fails to achieve EVR, then the SVR rate is
virtually nil. For this reason, failure to achieve EVR is called a “null response.”
Patients failing to achieve EVR should not continue therapy. This is one category
of not responding to therapy.
The EVR has now been supplemented with another assessment, the rapid
virologic response, or RVR. The first benchmark to check following the initiation
of HCV treatment, RVR is defined as having an undetectable HCV RNA at week
4 of treatment. If a patient with genotype 1 HCV achieves an RVR at week 4, then
the SVR rate approaches a blistering 90%. The same can be said for genotypes
2 and 3. So, if a patient achieves an RVR, he or she should obviously continue the
therapy—for a total of 48 weeks for genotype 1 and 24 weeks for genotypes 2 and
3. RVR is achieved in around 25% of patients with genotype 1 and in upwards of
66% of patients with genotypes 2 and 3.
What if patients with genotype 1 achieve an EVR but still have detectable virus
by week 24? These patients are also called nonresponders because even though
they had a 2-log drop at the 12-week milestone, they did not clear the virus at the
critical 24-week milestone. In this circumstance, there is virtually no chance of
achieving SVR by week 48, and these patients need to stop therapy. By the way,
a “complete EVR” is defined as undetectable virus (not just a 2-log drop) at the
12-week mark and predicts about a 70% SVR after 48 weeks of therapy, which is
almost as encouraging as an RVR.
Let’s take this one step further. A “partial EVR” is defined as having at least
a 2-log drop in virus at 12 weeks but still having detectable HCV RNA until the
24-week mark, where the virus is cleared. These patients, called slow responders,
have double the relapse compared with the “complete EVR” patients. Therefore, it
is now recommended to extend therapy for 72 weeks in these patients. To repeat:
if a patient is a slow responder but does finally have negative HCV RNA by week
24, you should extend therapy to 72 weeks instead of the usual 48 weeks for
genotype 1 HCV.
Figure 64-1 provides a visual overview of the major patterns of HCV response.
Table 64-1 provides more information on how to manage these patients.
“Tough Stuff” Vignettes 157

Table 64-1.
INTERPRETATION AND TREATMENT APPROACH FOR HCV RNA
LEVELS DURING TREATMENT WITH INTERFERON AND RIBAVIRIN*
Week First
Became
HCV RNA % of Relapse SVR Treatment
Undetectable Description Patients Rate Rate Strategy
GENOTYPE 1
4 Rapid 15 10 90 Keep patients
response on treatment for
48 weeks even
if have to reduce
dose. If shorten
therapy, SVR still
very high
12 Early response 35 33 66 Complete
48 weeks of
treatment
24 Slow to 15 55 45 Extend therapy to
respond 72 weeks
Never Null response 20 0 0 Stop treatment as
soon as pattern
recognized at
12 weeks
Never Nonresponse 15 0 0 Stop treatment as
soon as pattern
recognized at
24 weeks
GENOTYPES 2 AND 3
4 Rapid 66 10 90 Keep patients
response on treatment for
24 weeks even
if have to reduce
dose. If shorten
therapy, SVR still
very high
After week 4 Early response 33 50 50 Consider extend-
ing therapy to
48 weeks
Never Nonresponse 3 0 0 Stop treatment as
soon as pattern
recognized at
12 weeks
*Adapted from Schiffman, M. (2008), Curbside Consultation of the Liver. Thorofare, NJ: SLACK
Incorporated.
158 Chapter 2

Figure 64-1. Patterns of HCV RNA in patients receiving interferon and riba-
virin. (Adapted from Schiffman, M. [2008], Curbside Consultation of the
Liver. Thorofare, NJ: SLACK Incorporated.)

Why Might This Be Tested? You know that viral hepatitis will be on the
exam. These viral kinetics and treatment milestones are perfect for a Board
exam because they are tried and true and they set up numerous opportunities
for clinical threshold questions. You simply need to memorize this information;
every so often the Board exam rewards pure memorization.
Clinical Threshold Alert: EVR requires at least a 2-log drop in HCV RNA ver-
sus baseline by week 12 of interferon and ribavirin therapy. Anything less than
a 2-log drop by week 12 is a null response. RVR is assessed at 4 weeks. SVR
is assessed 6 months after cessation of treatment. Genotype 1 slow responders
should receive 72 weeks of therapy, not 48 weeks.

Here’s the Point!

If a patient with genotype 1 HCV does not achieve EVR, stop therapy.
Also, just know Table 64-1!

Here’s the Point!

Genotype 1 slow responders should be treated for 72 weeks.


“Tough Stuff” Vignettes 159

Vignette 65: Oral Contraceptives and a Liver Mass


A 32-year-old woman presents to her primary care provider with dyspepsia.
Her initial workup includes an abdominal ultrasound, which reveals a 7 x 6 cm
mass at the edge of the liver. A follow-up MRI confirms the lesion and demon-
strates that it’s situated beneath the liver capsule (Figure 65-1). The mass lights
up on the arterial phase of imaging and is otherwise homogeneous. There is
no evidence of cirrhosis on imaging, physical examination, or biochemical test-
ing. AFP level is normal. The patient has been taking an oral contraceptive for
5 years.

Figure 65-1. Subcapsular liver lesion


drooping off the right lobe of the
liver.

` How should you manage this lesion?


160 Chapter 2

Vignette 65: Answer


This is a hepatic adenoma. You probably knew that. The key features of hepatic
adenoma include the exposure to oral contraceptives and the enhancement on
arterial phase of MRI. In addition, hepatic adenomas are frequently (but not
universally) subcapsular in location. The lack of a central scar argues against
something like focal nodular hyperplasia or fibrolamellar hepatocellular cancer
(see Vignette 25 for more on those lesions).
The question asked, however, is how you should manage the lesion, which is
a little more tricky. Hepatic adenomas are benign, but because they are often
subcapsular and highly vascular, they can spontaneously rupture. When they
rupture, they can lead to massive exsanguination into the peritoneal cavity. The
risk of hemorrhage increases to about 30% as the adenoma enlarges, and the
bleeding itself has a 6% mortality rate. In addition, hepatic adenomas can trans-
form into cancer. For these reasons, surgical resection is the treatment of choice
when feasible.
Hepatic adenomas are strongly associated with oral contraception use (as
mentioned earlier) and also anabolic steroid use (as is peliosis hepatis—see
Vignette 20), so the offending agent must be discontinued when this type of lesion
is identified. In fact, these lesions were exceedingly rare before the advent of the
birth control pill and anabloic steroids. Although hepatic adenomas can regress
after discontinuation of oral contraceptives, there are reports of rare malignant
transformation in some hepatic adenomas that regress. Thus, if they are reach-
able, they should be resected. However, some adenomas can be in difficult spots,
like the porta hepatis. In such cases, it may be reasonable to follow the smaller
adenomas (ie, <5 cm) after oral contraceptives have been discontinued. Because
that is controversial, it’s unlikely to be the subject of an exam question. But you
certainly should know the diagnosis and its association with oral contraceptives.
And for larger lesions (>5 cm), there is little debate that surgical resection is war-
ranted even if oral contraceptives are discontinued.
So for the patient in the current vignette, there are 2 good reasons to remove
the lesion. First, it’s symptomatic and, second, it’s large; discontinuing the oral
contraceptive is not enough. It’s important to advise patients with hepatic adeno-
mas to avoid becoming pregnant. These adenomas can grow during pregnancy
with the risk of massive hemorrhage. For that reason, all adenomas should be
resected prior to a planned pregnancy.
Why Might This Be Tested? Liver masses, in general, are nearly guaranteed
to be on the Board exam. So we’ve included a variety of liver masses throughout
this book. In addition, you can find several other vignettes pertaining to liver
masses in the original Acing the GI Board Exam book. The hepatic adenoma is a
classic. The trick here is to not just accurately identify this fairly obvious diag-
nosis, but to figure what to do with it; the latter is what we call a “second-level”
question that goes beyond the mere diagnosis.

Here’s the Point!

Oral contraceptive + Subcapsular liver mass = Hepatic adenoma


“Tough Stuff” Vignettes 161

Vignette 66: Can’t Lose Weight


A 35-year-old man is referred to you for management of elevated aminotrans-
ferases in the setting of diabetes and hyperlipidemia. He began treatment for
diabetes 8 years ago and is now insulin dependent. He first developed elevated
aminotransferases 5 years ago, at which time his BMI was 41. He attempted to
lose weight on several occasions, including under a supervised program, but
his current BMI is 42. He is adamant that he “can’t lose weight” despite focused
attempts at weight loss. His only medications are simvastatin and insulin. He
does not drink alcohol. His most recent liver tests include the following values:
AST = 96, ALT = 125, ALP = 63, total bilirubin = 1.3, and INR = 1.0. You now
obtain a liver biopsy, which is pictured in Figure 66-1.

Figure 66-1. Liver biopsy for patient in


Vignette 66. (Reprinted with permission of
Alton B. Farris, MD, Emory University.)

` How should this be managed?


162 Chapter 2

Vignette 66: Answer


This is nonalcoholic fatty liver disease, or NAFLD. We provided an overview of
NAFLD in Vignette 37, including the grades of steatohepatitis. Remember that
NAFLD is part of the metabolic syndrome, a condition marked by insulin resis-
tance with hyperglycemia, hyperlipidemia, hypertension, and atherosclerotic
cardiovascular disease. The metabolic syndrome is usually diagnosed clinically
on the basis of these comorbidities, and NAFLD runs with the pack. But meta-
bolic syndrome can also be diagnosed more formally by measuring insulin resis-
tance using the homeostatic model assessment (HOMA), which is a mathematical
model that yields a score based on fasting plasma glucose and insulin concentra-
tions. The details of HOMA are beyond the scope of this mini-review, but suffice it
to say that HOMA works well as an inexpensive and easily calculated alternative
to the more laborious euglycemic glucose clamp method of estimating insulin
resistance.
Whether diagnosed clinically, by HOMA, or by both means, insulin resistance
causes big-time trouble. NAFLD is one of the various complications of insulin
resistance. Although NAFLD may occur in nonobese patients, it’s overwhelmingly
found in obese patients with metabolic syndrome.
Recall that NAFLD is defined as hepatic steatosis (with or without steatohepa-
titis) exceeding 5% to 10% of the liver mass in the absence of significant alcohol
consumption. The patient in this vignette has more than NAFLD—he has NASH.
His liver biopsy with trichrome stain, depicted in Figure 66-1, reveals steatosis,
lobular inflammation, ballooning hepatocytes (steatohepatitis) within a mesh of
“chicken-wire” fibrosis. Cirrhosis may be right around the corner if this contin-
ues.
This scenario is incredibly common. The Framingham heart study revealed
that around 50% of the American populace is now overweight, and 1 in
4 Americans is obese. Obesity is an overwhelming public health issue, and gas-
troenterologists and hepatologists are on the front lines of this epidemic. You can
bet that the Board exam will include questions about the management of obesity,
especially in patients with advancing liver disease. You might not have signed
up to become a dietitian or endocrinologist, but—like it or not—that is now part
of your identity.
The good news is that weight loss may abate the progression of NASH to cir-
rhosis in this patient. But despite supervised attempts at weight loss, he has not
been able to shed the pounds. He has a BMI of 42, which is class III obesity—oth-
erwise known as severe, extreme, or morbid obesity. Table 66-1 provides the full
classification for BMI as supported by the National Institutes of Health (NIH) and
the World Health Organization. It’s amazing how many people consider a BMI of
30 to be “normal.” As shown in the table, a BMI of 30 is actually class I obesity.
A normal BMI is between 18.5 and 24.9.
“Tough Stuff” Vignettes 163

Table 66-1.

BODY MASS INDEX CLASSIFICATIONS ADOPTED BY THE


NATIONAL INSTITUTES OF HEALTH AND THE
WORLD HEALTH ORGANIZATION
BMI Range Classification
<18.5 kg/m2 Underweight
>18.5 to 24.9 kg/m2 Normal
>25 to 29.9 kg/m2 Overweight
>30 to 34.9 kg/m2 Obesity Class I
>35 to 39.9 kg/m2 Obesity Class II
>40 kg/m2 Obesity Class III (extreme, severe, or morbid obesity)

The BMI is preferred over body weight as a measure of obesity because it bet-
ter correlates with the percentage of body fat, in particular. Plus, the BMI is
much more predictive of obesity complications, including mortality, than weight
alone. One caveat about the BMI is that it’s less accurate in patients who are big
yet muscular—these bodybuilder types have much less body fat than the BMI
would predict. For the rest of us, though, the BMI works pretty well as a quick
estimate of body fat mass. Another caveat is that the thresholds in Table 66-1
were developed from data on Caucasians. Different races should probably have
different cutoffs. For example, in people of South Asian decent, the risks of obe-
sity begin to mount at much lower BMI cutoffs than in Caucasians. This partly
explains why NAFLD is more common in nonobese Asians compared to nonobese
individuals of other races. In contrast, obesity-related complications accumulate
at higher BMI thresholds in African-Americans than in Caucasians.
While we are on the topic of obesity assessment, it’s important to note that the
BMI is only part of the evaluation process. The United States Preventive Services
Task Force (USPSTF) and The National Institues of Health (NIH), among others,
suggest measuring waist circumference in addition to BMI. The idea behind
waist circumference is that it estimates abdominal fat, in particular—and cen-
tral adiposity (also called “android” adiposity) is classically associated with an
increased risk of diabetes, high blood pressure, cardiovascular disease, and
hyperlipidemia. In patients who are overweight or have class I obesity, the risk
of obesity complications rise when the waist circumference exceeds 40 inches in
men and 35 inches in women. These are good numbers to keep in mind for clini-
cal practice, although we doubt they will be directly tested on the GI Board exam.
But certainly know that waist circumference correlates with central adiposity,
which, in turn, is highly related to cardiovascular complications of obesity. Once
patients reach class II obesity or beyond (eg, BMI >35), the waist circumference
does not provide much additional explanatory power; at that point the BMI alone
tells the story.
164 Chapter 2

So, this patient is morbidly obese, and he has remained morbidly obese despite
supervised attempts at weight loss. He is young and has evidence of advanced
complications of obesity, including NASH. This vignette asked how this patient
should be managed. Given his age, failure to lose weight, and mounting com-
plications, you should recommend bariatric surgery. Medical therapy (which is
still evolving) and diets just won’t cut it. If he doesn’t lose weight, he will likely
die much sooner than he otherwise should. This is a tragedy waiting to hap-
pen, so you must be prepared to discuss bariatric surgery and refer him to the
appropriate consultants. This vignette underscores the utility of a liver biopsy.
Despite the risks associated with liver biopsy, such as bleeding, the diagnostic
value supersedes the risk in this situation. Just checking this patient’s liver tests
annually would not help.
Bariatric surgery is effective. Although there are a variety of possible compli-
cations, bariatric surgery is the most effective approach for managing morbid
obesity. In patients who are morbidly obese, surgery improves survival and
enhances overall quality of life. Bariatric surgery is indicated for patients with
a BMI >40 or a BMI >35 with comorbid complications. Importantly, bariatric
surgery has recently been shown to improve steatosis, necroinflammation, and
even hepatic fibrosis in NASH. Although every patient does not experience these
results, most exhibit improvements in histologic parameters as long as the weight
loss is not too rapid.
Not all bariatric surgeries are created equal. In the past, surgeons would
fashion a jejunoileal bypass in which the intestinal anastomosis was placed
just proximal to the ileocecal valve. This was highly effective for weight loss, as
you can imagine, but was rife with complications. In particular, many patients
developed liver failure following the operation, so the jejunoileal bypass is now
long gone.
Laparoscopic adjustable gastric banding (LAGB) has become very popular
because of its relative ease of placement and low risk of complications. In the
LAGB operation, a circumferential band is placed around the stomach to create a
restrictive pouch. This works pretty well for weight loss but is not as effective for
reversing the metabolic complications of obesity compared to other operations.
This is likely attributed to the fact that LAGB does not change hormonal profiles
the way other approaches can.
The Roux-en-Y gastric bypass (RYGB), which is now the operation of choice
for morbidly obese patients and those with advanced metabolic complications of
obesity, provides robust weight loss and improvements in metabolic parameters.
In the RYGB, a 15- to 20-mL restrictive pouch is created in the stomach, and the
Roux loop is brought up to the pouch. As a result, the duodenum and proximal
jejunum are bypassed. This duodenal bypass may be the magic ingredient for
reducing metabolic complications, possibly because duodenal passage of gastric
contents triggers a cascade of regulatory peptides. The LAGB, in contrast, does
not alter the hormonal milieu in the same manner, since duodenal transit is
retained.
In any event, this young man needs surgery. Without it he could die young, but
with an effective operation coupled with postoperative counseling and oversight,
“Tough Stuff” Vignettes 165

he might reverse the NAFLD, cure his diabetes, improve his quality of life, and
even prolong his life span.
Why Might This Be Tested? You can expect to see this on the exam. You
should know all about NAFLD as the hepatic manifestation of metabolic syn-
drome, and you should know all about the obesity epidemic.
Clinical Threshold Alert: A BMI between 25 and 29.9 is “overweight.” A BMI
above 30 is “obese.”

Here’s the Point!

Bariatric surgery is the most effective therapy for a morbidly obese


patient with NASH who is unable to lose weight via traditional methods.
166 Chapter 2

Vignette 67: Pregnancy Consultation


A 28-year-old primigravid woman at 30 weeks’ gestation presents to you for
further evaluation. She has been feeling particularly fatigued over the past
2 weeks and has noticed skin changes. She also has mild nausea but has been
eating small amounts without vomiting. She complains of polyuria. She had ter-
rible nocturnal heartburn earlier in this pregnancy and had been under the care
of another gastroenterologist (until she appeared at your office today). In review
of those records, you find that an upper endoscopy was performed in the sec-
ond trimester that showed small esophageal varices without red signs, a sliding
hiatal hernia, and no evidence of erosive esophagitis. Recent laboratory workup
for liver disease was unremarkable, and there is no family or personal history of
liver disease. Her retrosternal pyrosis has resolved with the use of a proton pump
inhibitor daily. There has been no hematochezia, melena, diarrhea, hematemesis,
or weight loss. She has not had fevers, rigors, pruritus, or jaundice.
She saw her primary care physician yesterday, and laboratory tests were
ordered. She was referred to see you the very next day.
Her vital signs show a temperature of 98.8°F, blood pressure of 92/60, heart
rate of 100, and respiratory rate of 18. Physical examination reveals bilateral
palmar erythema, a few spider angiomas, a gravid abdomen, trace pitting edema,
and varicosities of the lower extremities. No other remarkable findings are noted
on examination. Laboratory tests reveal the following: albumin = 3.3, total bili-
rubin = 1.0, ALT = 21, AST = 18, GGT = 22, ALP = 290, BUN = 5, creatinine = 0.5,
glucose = 80, WBC = 4.9, hemoglobin = 11.2, MCV = 92, platelets = 190, normal
serum bile acids, and normal urinalysis. Doppler ultrasound of the abdomen
reveals a normal-appearing liver, spleen, and vasculature without ascites.

` What is the diagnosis?


` What needs to be done now?
“Tough Stuff” Vignettes 167

Vignette 67: Answer


I know what you’re thinking: pregnancy, pregnancy, pregnancy. You might
as well be preparing for the obstetrics Board examination. As we’ve said before,
pregnancy seems to come up over and over on the GI Board exam. Remember
that most adult women get pregnant and have the potential to face some liver
issues during this time. Obstetricians will refer these challenges to gastroenter-
ologists (and rightly so). So pregnancy presents dilemmas that commonly face
the GI practitioner. Quite simply, you need to know this stuff cold.
So, back to the vignette. Don’t be fooled here; this is a normal pregnancy. The
patient should be followed expectantly by her obstetrician, and no further tests
are needed at this time. A certain level of paranoia is understandable since you
are dealing with two lives: mother and fetus. However, this case demonstrates
typical and expected findings with normal pregnancy. The physiologic changes
that occur with pregnancy are important to understand, and are worth review-
ing here, since you need to be able to recognize them when bad things really do
happen.
Let’s break this case down further. This patient’s fatigue is most likely from
the increased energy demands of pregnancy. Although nausea is nonspecific
and can have many causes, here it’s most likely due to gastric compression from
an enlarging uterus. The patient is compensating by eating smaller amounts.
Polyuria is probably from urinary bladder compression, also from the enlarg-
ing uterus. Furthermore, the uterine enlargement interferes with venous return
from the lower extremities and is leading to her edema and varicose veins. There
is up to a 50% increase in cardiac output in pregnancy, mostly due to increased
plasma volume, which raises the heart rate as the gestation advances. This
increase in plasma volume causes hemodilution and results in the relatively
low albumin, hemoglobin, BUN, and creatinine levels. The slightly low blood
pressure is due to systemic vasodilation; however, the blood pressure tends to
gradually increase further along in the gestation. Palmar erythema and spider
angiomas are commonly found during pregnancy due to the increased estrogen
levels. The elevation of the ALP results from the addition of placental ALP (notice
that the GGT level was normal). The presence of small esophageal varices can
be found in up to half of healthy pregnant patients due to increased flow in the
veins of the azygos system (which feed esophagogastric varices). However, these
varices do not typically bleed. We could have also thrown in an elevated AFP
level (which is also normal in pregnancy), but we thought we had confused you
enough already. Therefore, you have a normal pregnant patient here and you can
advise her primary care provider to relax and not worry at this point in time.
Why Might This Be Tested? You have to know the basic physiologic changes
of pregnancy to understand when there is real pathology. There is no need to be
intimidated when you see a question about a pregnant patient or when you get a
consult on a gravid patient in real life. If you know your stuff, you will be more
confident and able to answer the query right away. Remember, confidence breeds
success (so long as it’s not false confidence)!
168 Chapter 2

Here’s the Point!

Elevated ALP in pregnancy is due to the addition of placental ALP.

Here’s the Point!

Palmar erythema and spider angiomas can be seen in normal pregnancy due
to the elevated serum estrogen levels.

Here’s the Point!

Small esophageal varices can form during pregnancy due to increased flow in
the azygos system—these do not typically bleed.
“Tough Stuff” Vignettes 169

Vignette 68: The Itchy and Scratchy Show


A 38-year-old man is referred to you for further evaluation of abnormal liver
tests. Review of systems reveals flushing with alcohol ingestion, intermittent
abdominal pain, and bouts of diarrhea. Upon further questioning, you learn
that he can literally write on his skin during periods of flushing, leaving visible
red streaks where he scratches with his fingernails. Physical exam is notable for
hepatosplenomegaly. Labs reveal an elevated ALP and GGT.

` What is the most likely diagnosis?


170 Chapter 2

Vignette 68: Answer


This is systemic mastocytosis. This condition is a Board favorite because it
presents in many different ways and affects nearly every major organ system.
Although mastocytosis often infiltrates the liver, it’s a systemic disorder char-
acterized by mast cell infiltration of several systems including lymph nodes,
spleen, skin, bone marrow, central nervous system, and the GI tract. The high
ALP occurs because of diffuse infiltration of the liver with mast cells. Recall that
anything that diffusely infiltrates the liver can drive up the ALP, often out of
proportion to the aminotransferases. The mast cell infiltrates secrete histamine,
which, in turn, can lead to hypersecretion of acid in the stomach, peptic ulcers,
and acid reflux disease. This is a rare hyperacidic syndrome that can be treated
as effectively with a histamine-2 receptor blocker as with a proton pump inhibi-
tor. Mastocytosis can lead to periodic flushing (particularly with alcohol inges-
tion), abdominal pain, diarrhea with malabsorption, paresthesias, low blood
pressure (histamine mediated), and just about anything else! A fun fact is that
it’s associated with Darier’s sign, which is visible urticaria from scratching the
skin (ie, dermatographism). By the way, if you don’t recognize the source of this
vignette’s title, it’s time to bone up on your Simpson’s trivia.
Why Might This Be Tested? Because it’s extremely rare, and the Board
examiners seem to love testing rare diagnoses; knowledge of rare diagnoses
extracts the true “acers” from the rest of the crowd. Plus Board examiners seem
to love questions on dermatologic manifestations of GI disorders.

Here’s the Point!

Big liver + Flushing + Dermatographism + High ALP = Mastocytosis


“Tough Stuff” Vignettes 171

Vignette 69: D-Penicillamine Trouble


A patient with recently diagnosed Wilson disease is started on D-penicillamine
therapy to manage serum copper levels. The patient develops a fever and cutane-
ous skin eruptions within days. He presents for evaluation and is found to have
new lymphadenopathy on physical exam. Laboratories reveal new thrombocyto-
penia and neutropenia.

` What happened here?


` How should this patient be managed, both acutely and chronically?
172 Chapter 2

Vignette 69: Answer


This is an early hypersensitivity reaction to D-penicillamine, which is known
to occur in up to 10% of patients started on this therapy. D-penicillamine is an
orally administered chelating agent that helps reduce copper levels by promot-
ing copper excretion in the urine. It’s an amazing drug because it can reverse
neurologic symptoms in Wilson disease, slow overall disease progression, and
significantly extend overall survival. But these remarkable benefits come with a
price; there are many serious side effects of D-penicillamine (which is the main
reason that another chelator with fewer side effects, trientine, has gained favor).
The adverse events of D-penicillamine are traditionally divided into early versus
late-occurring consequences. The current vignette demonstrates the most seri-
ous early adverse event, which is a hypersensitivity reaction marked by fever,
skin rash, lymphadenopathy, thrombocytopenia, and neutropenia. This reaction
usually occurs within 3 weeks of starting therapy. When hypersensitivity occurs,
the drug must be stopped immediately. Some advocate restarting the therapy
with the addition of steroids, but that is a matter of personal style, the severity
of the initial reaction, and knowing the patient. Late adverse events are multiple
and include Goodpasture’s syndrome, pemphigoid lesions, lichen planus, and a
lupus-like syndrome, among others. Also keep in mind (as noted in Vignette 36)
that both neurologic symptoms and liver tests may initially worsen after starting
D-penicillamine, but subsequent improvement typically occurs within 6 months
of starting therapy (ie, it takes a while for it to work, so don’t stop the medicine if
the symptoms initially worsen unless there is a serious side effect like a hyper-
sensitivity reaction).
Since this patient did not tolerate D-penicillamine, the next step is to use an
alternative chelating agent or zinc salts (or both). Trientine is less potent than
D-penicillamine but has a much more favorable side-effect profile, and in many
patients trientine is as effective as D-penicillamine. Zinc salts have a different
mechanism of action than the chelators. Rather than binding up copper, zinc
blocks intestinal absorption of copper and basically traps the copper within
the enterocytes. Also keep in mind that patients with Wilson disease should
avoid copper-rich foods (as discussed in Vignette 12), such as those listed in
Table 69-1.
Why Might This Be Tested? The emphasis of a disease on the Board exam
often seems to be inversely proportional to its population prevalence. Since
Wilson disease is very rare, you had better know it well! There are just too many
nuances and pearls to think that Wilson disease will not be on the exam.
“Tough Stuff” Vignettes 173

Table 69-1.
COPPER-RICH FOODS TO AVOID IN
WILSON DISEASE
(IN NO PARTICULAR ORDER)
Chocolate Navy beans
Sesame seeds Garbanzo beans
Raw cashews Soybeans
Sunflower seeds Cooked barley
Poppy seeds Oysters
Liver (go figure!)

Here’s the Point!

D-penicillamine causes many side effects. Most patients can be treated with
trientine and/or zinc salts.
174 Chapter 2

Vignettes 70 to 75: Cirrhosis/Renal Throw Down


It’s hard to find a patient with decompensated cirrhosis who does not have,
or is not soon to have, some kidney trouble. It can be vexing to sort out what’s
causing elevated creatinine levels and azotemia in these patients. The differential
diagnosis is extensive; it includes not only the usual causes of renal failure, but
also the cirrhosis-specific etiologies. This can get confusing, so what better topic
to put on a Board exam? Let’s just mix it up and jump in. This renal throw down
will help you see where you are with your renal knowledge vis-à-vis cirrhosis. In
each mini-vignette in the following series, identify the most likely cause of the
elevated creatinine.

70.A 72-year-old patient with cirrhosis and a MELD score of 13 complicated


by diuretic-controlled ascites is found to have grade 1 hepatic encephalopathy at
a clinic visit. The patient normally receives furosemide 80 mg daily and spirono-
lactone 200 mg daily for control of ascites, which has been effective. Baseline
creatinine is 1.0. Sodium and potassium balance has been well preserved. At
the clinic visit, the serum bicarbonate is 27 mEq/L. A spot urine revealed a
sodium:potassium ratio exceeding 1.0 and a spot urine sodium of 20 mmol/L.
The patient is started on oral neomycin, which helps the encephalopathy.
However, on follow-up lab testing, the creatinine has risen to 2.3. There has been
no interval change in diuretic dosing. What happened here? And what else do you
need to screen for to cover the bases? (This is a “read our mind” or “Do you hear
us?” type of question, but think about it for a second…)

71. A patient with tense ascites undergoes paracentesis that yields 18 L of


fluid. She is then started on strict sodium restriction and diuretic therapy,
beginning with furosemide 40 mg daily and spironolactone 100 mg daily. The
ascites does not improve, and a spot urine sodium is 5 mmol/L, with a urine
sodium:potassium ratio below 1.0. The diuretics are increased, and she is con-
currently treated with serial large-volume paracenteses. But these measures do
not help—the ascites persists. The diuretic dose is gradually pushed to 160 mg
daily of furosemide and 400 mg daily of spironolactone. At this level, the creati-
nine rises from a baseline of 1.0 to its current level of 2.3. The fractional excretion
of sodium (FENa) is low, and the bicarbonate is elevated at 32. The patient is also
noted to be increasingly encephalopathic in parallel with the increase in diuretic
dosage. What is the most likely reason for the elevated creatinine, and what is
the next step in managing this patient?

72. A patient with well-compensated cirrhosis and a MELD score of 7 has a


normal baseline creatinine of 1.0. He injures his ankle and reports to the emer-
gency department (ED) for evaluation, where he is ruled out for a fracture and is
placed on a course of therapy to help manage a severely sprained ankle. A week
later he undergoes routine lab testing by his primary care provider. His creati-
nine is now 2.3. What is your best guess about what happened here? Imagine
that you are on the phone right now with the primary care provider, who has
“curbsided” you.
“Tough Stuff” Vignettes 175

73. A 61-year-old with well-compensated cirrhosis from chronic hepatitis C


develops new-onset ankle edema. He presents to his primary care physician,
who orders labs that reveal the following: creatinine = 2.3 (baseline = 1.0); albu-
min = 1.8 (baseline = 3.1), AST = 48, ALT = 53, total bilirubin = 1.6, INR = 1.3. The
patient is referred to his gastroenterologist, who checks a 24-hour urine protein
(4.6 g), serum complement levels (low), and a rheumatoid factor (positive). What
is the most likely explanation for this picture?

74.A patient with cirrhosis and ascites develops low-grade fevers and abdomi-
nal pain. Paracentesis is performed and reveals a total PMN count of 550.
Cultures grow out Escherichia coli. Laboratories are checked, which reveal a
creatinine of 2.3 (baseline = 1.0). What might be causing the acute renal insuf-
ficiency here?

75. The patient in Vignette 71 starts to require weekly paracentesis with


intravenous albumin. The diuretics are discontinued to avoid further contrac-
tion alkalosis. However, the creatinine begins to rise further. Within 2 weeks,
the creatinine has risen to 3.2, and the creatinine clearance is estimated to be
10 mL/min. The urine output begins to drop, culminating in oliguria. The urine
sediment is bland without evidence of casts or erythrocytes. There are no other
obvious nephrotoxins on board. The patient does not have evidence of spontane-
ous bacterial peritonitis, other infections, GI bleeding, or shock. What is the most
likely diagnosis? How should this be treated?
176 Chapter 2

Vignettes 70 to 75: Answers


70. This is neomycin-induced renal toxicity. Although neomycin historically
has been used for managing the symptoms of hepatic encephalopathy, it’s not
commonly used anymore given the availability of safer alternatives (see Vignette
11 for more details). Neomycin can cause renal toxicity; it even has a black box
warning from the FDA regarding this adverse event. Several risk factors raise the
likelihood of developing renal failure from neomycin. These include advanced age
(as seen here), pre-existing renal impairment (not seen here), and use of diuretic
therapy and/or volume depletion (seen here). This patient is on a reasonably high
dose of diuretic therapy, and although it has been effective, there is evidence of
some contraction alkalosis (bicarb slightly elevated). This patient seemed okay
on the regimen before the neomycin was given; that is, the patient was otherwise
stable, the ascites was controlled, and the creatinine was normal. Also, there
was adequate urine sodium excretion, based on the urine indices (see the other
mini-vignettes in this series for more on interpreting the urine indices). But this
patient became a perfect setup for kidney trouble due to his age, underlying cir-
rhosis (which itself is a setup for renal injury, as discussed further below), and
diuresis. Neomycin probably should not have been used here. As for what else you
should screen for, the answer is that you need to screen for ototoxicity by asking
about hearing and tinnitus symptoms. Although nephrotoxicity and ototoxicity
do not go hand in hand, you need to screen for ototoxicity nonetheless.

Here’s the Point!

Predictors of neomycin-induced kidney injury in cirrhosis:


z Advanced age
z Preexisting renal insufficiency
z Volume depletion
z Use of diuretics

71.This is contraction alkalosis and renal insufficiency resulting from overdi-


uresis. This patient has diuretic-resistant, refractory ascites defined as persistent
ascites despite sodium restriction and maximal doses of oral diuretics. It has
been nearly 50 years since investigators determined that the maximum physi-
ologic rate of peritoneal fluid absorption is 500 mL per day. That is an impor-
tant number to keep in mind because it indicates a physiologic ceiling for how
fast ascitic fluid can be mobilized. When there is a lot of extravascular volume
marked by peripheral edema, diuretics work fine to keep the fluid moving along.
But once that fluid is mobilized, the ascites becomes the main focus. So it then
becomes a race to see if the fluid removal from the peritoneum (physiologically
capped at about 500 mL per day) can outpace the rate of fluid collection in the
“Tough Stuff” Vignettes 177

peritoneum. As cirrhosis gets worse and worse, portal hypertension and systemic
arterial vasodilation increase in tandem. This leads to renal hypoperfusion with
compensatory sodium and free water retention by the kidney. So the race begins
as the body absorbs more and more sodium and water, and the diuretics try
to dump the sodium and water out. In the meantime the ascites just keeps on
accumulating, and at some point the rate of accumulation exceeds the physi-
ologic ceiling of reabsorption. So what happens then? Well, if you keep flogging
with diuretics, it will just serve to totally dry out the patient. Since the ascitic
fluid just can’t be sucked out any faster than 500 mL per day, other fluid spaces
get drained instead (namely diuresis from the intravascular space coupled with
the increasing systemic vasoconstriction). And then the patient experiences
even more intravascular volume depletion, develops a contraction alkalosis, and
bumps the creatinine as a consequence of prerenal azotemia. This is marked by
a low FENa and low urine sodium (below 10 mmol/L). Patients may also develop
worsening hepatic encephalopathy in the setting of contraction alkalosis, as
occurred here.
So the treatment is not to push the diuretics any further. Instead, you should
stop the diuretics, acknowledge pharmacologic defeat, and replete the intravas-
cular volume with intravenous albumin or other colloid. As far as treatment for
the ascites is concerned, serial large-volume paracentesis or TIPS can be con-
sidered. Ultimately liver transplantation is the only treatment that can improve
survival when this chain of events begins to unfold.

Here’s the Point!

Understand Figure 71-1, below.


178 Chapter 2

72.It is hard to know for sure what happened here because the vignette does
not provide quite enough information. But if you think about it, the ED probably
started this patient on a NSAID to manage the sprained ankle. Under normal
circumstances, it’s not a big deal to begin a NSAID in someone with an ankle
sprain. But in a patient with cirrhosis you always need to think about what
medicines you are giving and what the consequences might be. The neomycin
example in Vignette 70 is a classic; this is another. NSAIDs can cause acute kid-
ney injury even in healthy individuals, but patients with cirrhosis are especially
prone to acute kidney injury even if they are otherwise well. That does not mean
that NSAIDs can never be used in a patient with cirrhosis (although you better
be watching very closely if such a patient is on an NSAID); but the NSAID should
definitely be stopped if there is a bump in the creatinine.

73.This is mixed cryoglobulinemia from hepatitis C. See Vignette 24 for more


information about cryoglobulinemia. Clues here include the low complement
levels, proteinuria, and positive rheumatoid factor. Other clues that are not seen
here include a purpuric rash from leukocytoclastic vasculitis, paresthesias from
peripheral neuropathy, and abdominal pain from vasculitis. The renal involve-
ment in cryoglobulinemia is variable. Here the patient exhibits a nephrotic syn-
drome, but patients can also present with acute nephritic syndrome, membra-
noproliferative glomerulonephritis, chronic kidney disease without an obvious
glomerulonephropathy, or even acute renal failure. A GI Board exam is unlikely
to ask you about the details of the renal injury. Just be aware that it can vary.

Here’s the Point!

Hepatitis C + Positive rheumatoid factor + Low complement + Lower extremity


purpuric rash + Paresthesias + Renal disease = Mixed cryoglobulinemia

74. This is renal insufficiency from spontaneous bacterial peritonitis (SBP).


How can SBP have anything to do with the kidneys? Well, as these vignettes all
point out, the kidneys are especially susceptible in cirrhosis. Patients with cir-
rhosis already have some underlying splanchnic vasodilation, so the kidneys are
already a little underperfused. They can handle this if there are no perturbations
and life stays the same. But the kidneys can be quickly boxed in if anything
disrupts the delicate vascular balance. Infections like SBP can do just that. In
SBP there is an accumulation of endotoxins from the smoldering infection, and
these can further exaggerate systemic vasodilation. Just a little bit of extra vaso-
dilation might be enough to tip the balance and greatly diminish effective renal
perfusion. So, think about a new-onset infection as one of the possible culprits
(in addition to everything else that might happen, such as GI bleeding leading
to systemic hypotension) whenever the creatinine goes up in a patient with cir-
rhosis. Along the same lines, ongoing GI bleeding can precipitate hypotension
“Tough Stuff” Vignettes 179

and lead to creatinine elevations as well, so it’s important to think about all the
different things that can box in the kidneys.

Here’s the Point!

Think about a new infection (like SBP) when the creatinine bumps in a patient
with cirrhosis (in addition to everything else that causes this!).

75. This is hepatorenal syndrome, or HRS. We could fill the next 30 pages
writing about HRS, but we won’t. Let’s just cover the basics for now. HRS is basi-
cally the end result of the cascade depicted in Figure 71-1. To review, as cirrhosis
gets worse and worse, systemic vasodilation becomes more prominent (a result
of different circulating vasodilators, like nitric oxide, in particular). The effec-
tive circulating blood volume drops, there is less blood flow to the kidneys, and
renal function declines. The kidneys respond by aggressively retaining sodium
and, secondarily, free water. This helps to fill the intravascular space a bit, but
not for long; the fluid just starts spilling into extravascular spaces, including the
soft tissues (edema) and peritoneum (ascites). Renal function drops even more,
the kidneys become even more desperate, and sodium and water reabsorption
continue at full throttle. The vicious cycle continues. Finally, the kidneys can’t
keep up anymore and just give out. But it gets worse. As systemic pressures drop,
the body starts to route blood to where it really matters—the brain and heart.
Hypotension triggers the renin-angiotensin system and the sympathetic nervous
system, and this leads to compensatory vasoconstriction in different vascular
beds, including the femoral and renal arteries. That is bad news for the kidneys,
not only because they are seeing less perfusion and suffering as a consequence,
but also because the afferent arterioles are clamping down at just the wrong
moment. The kidneys go from being boxed in to being strangulated by intense
afferent vasoconstriction. Now the kidneys maximize sodium and free water
absorption, urine sodium levels fall below 10 mEq per day, and the patient devel-
ops oliguria (as seen here). Administering a vasopressin analog, like terlipressin
(not yet available in the United States), can partly correct the perturbed vascular
system by serving to increase the mean arterial pressure, drop renin levels, and
improve renal perfusion and glomerular filtration. The net result is to loosen the
vice on the kidney, which is evidenced by increased urine sodium concentrations.
Combination therapy with midodrine (a selective alpha-1 adrenergic agonist) and
octreotide (a somatostatin analog) can also serve this role. Midodrine can work
to increase vascular tone and pressures, and octreotide reduces concentrations
of splanchnic vasodilators. Further, infusions of albumin can help to increase
the effective circulating volume. Basically, however, this is an “all hands on deck”
situation, and while these therapies may temporize the situation, only liver trans-
plant will dictate survival.
180 Chapter 2

To make things more complicated, HRS is divided into type I and type II. Just
remember that type I is the serious type, and type II is the less serious (but
still not great) type and is commonly seen in refractory ascites. In type I HRS
things happen fast—there is a >50% drop in creatinine clearance to a value
below 20 mL/min in a <2 week period, or there is a 2-times increase in serum
creatinine levels, with a level exceeding 2.5 mg/dL (lots of numbers here—just
remember that it happens fast and furious). Type II HRS is a slow-growing phe-
nomenon and is basically the situation in Figure 71-1 before things go haywire;
it’s having refractory ascites and seeing the creatinine rise in the process. So,
type II HRS is similar to the patient described in Vignette 71, although some
argue that diuretics cannot precipitate HRS. We won’t delve into that physiologic
debate here, but will simply say that type II HRS is the end result of refractory
ascites, whereas type I involves a rapid decompensation. In both types of HRS
you need to rule out other obvious causes of intrinsic renal failure (ie, you need to
determine that there is no evidence of glomerulonephritis in urinalysis, no obvi-
ous nephrotoxins, no post-renal obstructions, no infections like SBP, no bleeding
or shock, etc). That is, HRS is a diagnosis of exclusion. The kidneys in HRS are
actually fine—the problem is the prerenal state coupled with the afferent vaso-
constriction. Remember that patients with type I HRS are going downhill fast
and will receive higher prioritization for liver transplantation (the MELD score
rises with the increasing creatinine). Furthermore, they usually do not require
a combined liver and kidney transplantation since the kidneys should return to
normal function after the liver transplant.

Here’s the Point!

Type I HRS = Fast and furious renal decompensation (see text for details)
Type II HRS = Basically refractory ascites with a rising creatinine

(In both cases there is no evidence of intrinsic kidney disease, no obvious


nephrotoxins, and no shock, bleeding, or infection.)
“Tough Stuff” Vignettes 181

Vignette 76: Lots and Lots of Stones


A 48-year-old man who recently immigrated from Mexico is referred for evalu-
ation of jaundice and abnormal liver tests, including the following: ALP = 802,
ALT = 135, AST = 141, albumin = 3.4, total bilirubin = 12.8, creatinine = 1.5,
INR = 1.1, WBC = 7.3, platelets = 202. Beside noting jaundice, he has been
asymptomatic. On examination, he is found to be afebrile with otherwise unre-
markable vital signs. The liver and spleen are not palpable, and there is no
shifting dullness on abdominal exam. He is icteric, but otherwise there are no
stigmata of chronic liver disease and no peripheral edema. Abdominal CT reveals
intrahepatic biliary dilatation with multiple intrahepatic stones (Figure 76-1). A
follow-up ERCP reveals innumerable stones in the common bile duct and intra-
hepatic ducts without cholelithiasis (Figure 76-2).

Figure 76-1. Multiple round lucencies


in the liver consistent with intrahepatic
stones. (Reprinted with permission of
Francisco Durazo, MD, Ronald Reagan
UCLA Medical Center.)

Figure 76-2. ERCP with cholangio-


gram revealing innumerable space-fill-
ing objects in the common bile duct
and intrahepatic ducts. (Reprinted
with permission of Francisco Durazo,
MD, Ronald Reagan UCLA Medical
Center.)

` What is the most likely diagnosis?


182 Chapter 2

Vignette 76: Answer


This is recurrent pyogenic cholangitis, also known as Oriental cholangitis. For
a full review of this condition, refer to the original Acing book. In short, pyogenic
cholangitis is most commonly seen in Southeast Asians, with a particularly
high prevalence in rural areas of China and Hong Kong. However, it also has
been diagnosed in Latino immigrants to the United States (as in this case), so
its association with Asia is not exclusive (one more reason why the term Oriental
cholangitis is unfortunate).
The cause of this unusual condition remains unknown. It has been variably
blamed on bacterial infections, parasitic infections with Clonorchis sinensis and
Ascaris lumbricoides, and abnormal biliary stasis. This is a chronic disease
marked by oftentimes innumerable intrahepatic pigment stones and biliary
strictures, with an absence of disease within the gallbladder. Remember that key
Board buzzword element—a crazy amount of stones within the biliary tree but
with a normal gallbladder. Thus, the gallbladder is clearly not the origin of the
stones. Instead, the stones must originate within the biliary system itself, often
in the hepatic portion of the biliary system. The left hepatic system is classically
affected for unclear reasons—possibly due to tighter angulations of the left duc-
tal takeoffs compared to the right. In this case, there were innumerable stones
stacked one after another up the biliary system and into the liver, as shown in
Figure 76-2.
Management of acute attacks of cholangitis involves administering intrave-
nous fluids and antibiotics, and attempts to remove as many stones as possible
through ERCP. ERCP can also be used to dilate the strictures that inevitably
form with recurrent attacks. Unfortunately, ERCP alone is rarely adequate, and
it’s often coupled with percutaneous T-tube drainage or even surgical resection of
affected hepatobiliary segments. UDCA is often used, but it’s probably of no real
benefit for most patients because it does not effectively dissolve these calcium
bilirubinate pigment stones (in contrast to its efficacy for cholesterol stones).
Therapy for this complex disorder typically requires multimodal approaches
including endoscopic, radiographic, and surgical therapies.
Why Might This Be Tested? Recurrent pyogenic cholangitis is a rare condi-
tion that has a nearly pathognomonic presentation.

Here’s the Point!

Lots of intrahepatic and biliary pigment stones + Normal gallbladder =


Recurrent pyogenic (Oriental) cholangitis
“Tough Stuff” Vignettes 183

Vignette 77: Acute Liver Failure With Super-High AST/ALT


A 59-year-old man with AIDS (CD4 count of 220) presents to the emergency
room with progressive jaundice, nausea, malaise, abdominal pain, and fevers
over a 1-week period. Four weeks prior to admission, he suffered a bout of severe
Pneumocystis carinii (aka P. jirovecii) pneumonia, which required an inpatient
hospitalization including treatment with glucocorticoids and trimethoprim-sul-
famethoxazole (TMP-SMX). The patient tolerated the treatment well and eventu-
ally recovered from the illness. He was recently tested for both hepatitis B and C
occult infection due to mild elevations of the AST and ALT (both 2.5 times upper
limit of normal as an outpatient), with negative results. He has refused treatment
with highly active antiretroviral therapy (HAART).
He now presents with 101.3°F fever in the emergency department. His exam
reveals no icterus and moderate upper abdominal tenderness on palpation. The
liver edge is not palpable, and there are no stigmata of chronic liver disease.
There is no ascites or edema. Neurologic exam reveals hyperreflexia but no
asterixis. Laboratory values include the following: WBC = 3K, hemoglobin = 12.6,
platelets = 152, creatinine = 1.3, AST = 5210, ALT = 3526, ALP = 202, INR = 1.5,
total bilirubin = 1.6, albumin = 3.1.

` In addition to usual supportive care for acute liver injury, what other
specific therapy should be considered for this patient?
184 Chapter 2

Vignette 77: Answer


You should assume this patient has acute HSV hepatitis until proven oth-
erwise, and you should start acyclovir immediately due to the high mortality
risk. A prompt liver biopsy (check out Figure 57-2 again) should be ordered to
make the definitive diagnosis, but treatment should not be held for the results.
Both HSV-1 and HSV-2 can cause run-of-the-mill hepatitis, but they can also
precipitate acute liver failure. This can occur in immunocompetent people, but
it should be highly considered in immunocompromised patients presenting with
liver injury, such as this individual. The patient’s recent exposure to steroids
might have triggered this event, although it’s hard to know for sure (it’s generally
argued that using glucocorticoids for acute pneumocystis pneumonia does not
trigger opportunistic infections).
There are some clues to support HSV in this patient. First, patients with acute
HSV hepatitis characteristically have very high aminotransferase levels, often in
the 3000+ range, without jaundice. Second, patients with HSV hepatitis often
have a relatively low white count with fever and abdominal pain—also seen in
this case. Third, patients with HSV hepatitis can have coexistent HSV pneumo-
nitis or encephalitis. Of course, those clues are nonspecific in and of themselves,
but when coupled with the immunocompromised state and recent glucocorticoid
use, you have to think of HSV.
Again, the key is to think of HSV early and start empiric acyclovir right away;
don’t wait to confirm HSV because it may be too late. Acute HSV hepatitis can
cause acute liver failure and confers a high mortality rate. This especially holds
true for pregnant women, in whom prompt treatment can often avert the need for
urgent delivery if the diagnosis is made in time. Case series of HSV-induced acute
liver failure consistently reveal that acyclovir is usually started too late or not at
all. So the emphasis must be on early and aggressive treatment, in addition to
everything else you would normally do for acute liver failure.
Why Might This Be Tested? Board examiners want to make sure you
employ potentially life-saving therapies early, especially when the decision to
start therapy must occur before you have a confirming diagnosis. In this case,
the risk-benefit ratio of starting empiric acyclovir is overwhelmingly in favor of
treatment—you just need to think about it and then take the plunge. Hesitation
could spell trouble, and examiners want to know that you won’t hesitate with
something fundamental like this, even if the situation is uncommon.

Here’s the Point!

Super-high AST/ALT without jaundice + Immunocompromised + Fever +


Abdominal pain + Leukopenia = Think about acute HSV hepatitis, and start
empiric acyclovir
“Tough Stuff” Vignettes 185

Vignette 78: Kidney Stone and a Liver Mass


A 50-year-old man with history of nonalcoholic steatohepatitis and GERD
presents with complaints of fatigue and an unintentional 20-pound weight loss
over the past 3 months along with polydipsia, polyuria, and mild constipation.
He mentions that he recently saw his primary care physician after he “passed
a kidney stone” and had a negative evaluation for both diabetes mellitus and
prostate problems. There has been no pruritus, jaundice, rigors, abdominal pain,
increased abdominal girth, confusion, or memory problems. There is also no his-
tory of gastrointestinal bleeding. He reports undergoing a screening colonoscopy
and upper endoscopy earlier this year that were “normal except for small veins
in the esophagus.” Laboratory tests reveal the following: ALT = 42, AST = 54,
total bilirubin = 1.6, creatinine = 0.9, albumin = 3.4, calcium = 10.9, INR = 1.2,
WBC = 3.9, hemoglobin = 12.2, platelets = 80, AFP = 108, glucose = 72, hemo-
globin A1C = 5.0%, prostate specific antigen = 1.0 ng/mL. An abdominal CT was
performed, an image from which is shown in Figure 78-1. A bone scan and chest
CT scan were both negative.

Figure 78-1. Arterial phase of tri-


phasic abdominal CT for patient in
Vignette 78. (Reprinted with permis-
sion of Barbara Kadell, MD, UCLA
Medical Center.)

` What is the diagnosis?


` What should you recommend?
186 Chapter 2

Vignette 78: Answer


This is hepatocellular carcinoma with a paraneoplastic syndrome due to
secretion of parathyroid hormone-related protein (PTHrP) by the tumor cells.
The patient has a liver mass with characteristic radiographic findings along
with symptoms of hypercalcemia, including polydipsia, polyuria, constipation,
and nephrolithiasis. A serum PTHrP level would be elevated if checked (normal
is <1.5 pmol/L). If the patient responds to therapy for the HCC, the serum AFP,
calcium, and PTHrP levels should all decrease. The lack of osteolytic lesions
on bone scan essentially rules out bony metastasis as a cause of the hypercal-
cemia. In addition to hypercalcemia, HCC is associated with a variety of other
paraneoplastic conditions related to the synthesis of active proteins. These con-
ditions may present with symptoms and signs of erythrocytosis, hypoglycemia,
diarrhea, or rashes, such as glycogenic acanthosis, among other paraneoplastic
phenomena.
HCC is an international public health concern; it’s the fourth most common
type of malignancy worldwide. In the United States, HCC is the most common
primary liver cancer (metastases are the most commonly overall liver malignancy
in the United States). Moreover, the incidence of HCC is expected to rise dramati-
cally over the next 20 years due to the increase of nonalcoholic fatty liver disease
and hepatitis C cirrhosis.
In patients with HCC and cirrhosis, the best outcomes with surgical resection
occur with well-preserved liver function; namely, a total bilirubin level <1.0 and
absence of advanced portal hypertension, meaning an HVPG of 10 or less. Since
this patient has evidence of portal hypertension (esophageal varices on upper
endoscopy along with hypersplenism) and a total bilirubin >1.0, orthotopic liver
transplantation (OLT) should be considered. If the HCC characteristics fit within
the Milan criteria (Table 78-1), then OLT has a 5-year survival rate of greater
than 70%, which is comparable to other indications for OLT. Therefore, UNOS has
adopted the Milan criteria for allocation and prioritization for OLT in HCC and
assigns an initial MELD score of 22 for patients with HCC that meet the criteria.
For the patient in this vignette, a MELD score of 22 would lead to a decreased
waiting time on the transplant list, since his intrinsic MELD score is considerably
lower (you should be able to tell this by “eyeballing” the labs, even if you have not
memorized the MELD regression equation). Unfortunately, the time to OLT may

Table 78-1.

MILAN CRITERIA FOR LIVER TRANSPLANTATION


1. One lesion ≤5 cm
2. Up to 3 lesions, all ≤3 cm
3. No vascular invasion
4. No distant metastases
“Tough Stuff” Vignettes 187

be prolonged (depending upon the particular region’s waiting list), and patients
with HCC can develop tumor progression while awaiting transplant. Therefore,
if transplant is unlikely to occur within 6 months, locoregional therapy is often
recommended (either radiofrequency ablation [RFA] or transarterial chemoembo-
lization [TACE]) as a “bridge” for the patient while awaiting transplantation.
The choice of RFA versus TACE usually depends on local expertise. However,
RFA is generally used for smaller lesions that do not involve the dome of the liver
due to risk of tumor seeding, which is thought to be approximately 3%. TACE of
feeding arteries is typically used for larger lesions as long as there is adequate
hepatic reserve, since there is an increased risk for hepatic failure with TACE
versus RFA; this is especially true if there is concomitant portal vein thrombosis.
Remember that TACE uses the hepatic artery, and if there is a portal vein throm-
bosis, you essentially can cut off all significant blood supply to a large part of the
liver. This may very well lead to rapid deterioration of hepatic function and death.
So, you need to select the appropriate therapy on an individual basis.
Investigators have shown that if the tumor burden exceeds the Milan crite-
ria, locoregional therapy can be used for some select tumors for “downstaging”
prior to OLT. However, these patients require at least a 3-month hold to ensure
that biologically favorable tumors (which have shown response to therapy) are
selected for OLT.
If OLT is not possible, then locoregional therapy should be offered depending
on the tumor burden and hepatic reserve. For those patients with extrahepatic
metastases and/or large tumor burden who are not candidates for locoregional
therapy, sorafenib (a tyrosine kinase inhibitor) can prolong survival, provided
that the patient has a good performance status and adequate hepatic synthetic
function. For terminal-stage patients with markedly deteriorating physical sta-
tus, comfort care is recommended to avoid undue suffering.
In passing, it’s worth noting other conditions, besides HCC, that are associ-
ated with an elevated AFP level. These include cholangiocarcinoma (which is
also more common in patients with cirrhosis), some gastrointestinal metastatic
tumors, pregnancy (yes, there it is again!), and gonadal tumors. Because elevated
AFP can occur in conditions other than HCC, the diagnosis of HCC should not
rely too heavily on AFP levels. Guidelines suggest diagnosing HCC on the basis
of imaging and/or histology; AFP levels are now relatively de-emphasized in the
diagnostic algorithm (although an AFP level of over 200 ng/mL remains highly
suspicious for HCC in a patient with cirrhosis).
To diagnosis HCC without a biopsy (to avoid the risk of tumor seeding), a
contrast-enhanced study is needed to show a ≥2 cm lesion with specific imag-
ing characteristics. This can include either a triphasic CT scan or an MRI. HCC
enhances more intensely than the surrounding hepatic parenchyma in the arte-
rial phase (see Figure 78-1) and less than the surrounding liver in the venous
phase, producing the “washout” appearance. If the patient in this vignette did
not have a lesion on abdominal imaging, testicular ultrasound could have been
performed to rule out testicular cancer as a cause of the elevated AFP.
Why might this be tested? HCC is a major health concern with a continued
projected increase over the coming years. Therefore, you will encounter HCC
more often and will need to know how to diagnosis and treat patients with this
188 Chapter 2

type of lesion, especially as therapies continue to evolve. It is also important to


recognize that paraneoplastic syndromes are associated with HCC.
Clinical Threshold Alert: Patients within Milan criteria (see Table 78-1)
receive an automatic MELD score of 22 independent of other MELD parameters.
If OLT is unlikely for 6 months or more, then locoregional bridge therapy is war-
ranted.

Here’s the Point!

Hypercalcemia + Cirrhosis + Liver lesion + Elevated AFP =


Hepatocellular carcinoma with elevated PTHrP
“Tough Stuff” Vignettes 189

Vignettes 79 to 86: Liver Histology Throw-Down


You are probably not a pathologist. But for purposes of the Board exam, you
will need know a slew of pathology buzzwords to navigate through many of the
questions. This collection of mini-vignettes provides a set of classic pathology
buzzwords with minimal surrounding information. Most of these diagnoses are
featured elsewhere in this book along with characteristic micrographs. However,
the buzzwords alone may be enough for you to determine the diagnosis. Read
each vignette, and then make the diagnosis. By the way, we thought the “owl’s
eye” intranuclear inclusion finding was too much of a gimme. You remember that
one, right? If not, better go back to Figure 57-1.

79. An obese patient undergoes liver biopsy for persistently elevated amino-
transferase levels. The AST:ALT ratio has always been less than 1.0. Liver biopsy
reveals steatosis with lobular infiltration and ballooning hepatocytes.

80. A patient with cholestasis has a liver biopsy revealing a “florid duct lesion”
marked by bile duct destruction, “ductopenia,” and granulomas.

81. A patient with cholestasis has a liver biopsy revealing periductal “onion-
skinning fibrosis.”

82. A patient with elevated transaminases and an elevated gamma globulin


has interface hepatitis with a plasma cell infiltrate and lobular inflammation on
liver biopsy.

83. Nutmeg liver (yep, that’s all you get).

84. A patient with elevated transaminases has PAS-positive diastase-resis-


tant globules in hepatocytes on liver biopsy.

85. A patient with long-standing elevations in AST and ALT has “ground-glass
hepatocytes” on liver biopsy.

86. A former intravenous drug user is referred to you for persistently elevated
liver enzymes (AST = 89, ALT = 110) and transferrin saturation (85%). Recent
liver biopsy revealed the presence of hemosiderin in Kupffer cells but not in hepa-
tocytes.
190 Chapter 2

Vignettes 79 to 86: Answers


79.This is NASH. Clinical clues include obesity and the AST:ALT ratio below
1.0 (whereas alcoholic liver disease typically has a ratio above 2.0). See Vignette
37 for more information on NASH and NAFLD. Board buzzwords for NASH include
lobular inflammation, ballooning hepatocytes, and portal inflammation. Other
features include Mallory bodies, fibrosis, and ultimately cirrhosis (Figure 79-1).

Figure 79-1. Nonalcoholic steatohepatitis with bal-


looning hepatocytes. (Reprinted with permission of
Alton B. Farris, MD, Emory University. )

Here’s the Point!

NASH Histopathology Board Buzzwords:


z Lobular inflammation
z Ballooning hepatocytes
z Portal inflammation
z Mallory bodies

80. This is primary biliary cirrhosis. PBC is a progressive, destructive disease


that decimates the bile ducts over time. Early on, it produces a so-called florid
duct lesion marked by bile duct destruction, a severe lymphoplasmacytic infil-
trate in the portal tracts, and granulomas. Refer to Vignette 35 and Figure 35-1
for more information about PBC, and to Vignette 56 for information on the PBC
overlap syndrome with autoimmune hepatitis.

Here’s the Point!

High ALP + Florid duct lesion + Granulomas = PBC


“Tough Stuff” Vignettes 191

81. This is primary sclerosing cholangitis. PSC can sometimes produce a


similar appearance to PBC, but it’s not associated with granulomas. Instead, the
classic feature of PSC is a fibrous obliterative cholangitis that looks like “onion
skinning” around the bile ducts, as shown in Figure 81-1 (although any condition
with chronic duct obstruction can cause onion-skinning fibrosis). Both PBC and
PSC can lead to “ductopenia” or to a lack of bile ducts altogether in the burnt-
out stage.

Figure 81-1. Primary sclerosing


cholangitis with “onion-skinning”
obliterative fibrosis on trichrome
staining. (Reprinted with permis-
sion of Charles Lassman, MD,
UCLA Medical Center.)

Here’s the Point!

High ALP + Onion skinning = Think PSC


192 Chapter 2

82. This is autoimmune hepatitis. AIH is classically associated with elevated


transaminases in association with elevated gamma globulin levels. Characteristic
histologic changes include interface hepatitis with a plasma cell infiltrate (Figure
82-1). In particular, the limiting plate of the portal tract is invaded by the infil-
trating plasma cells, which, in turn, extend into the lobule.

Figure 82-1. Autoimmune hepatitis with


plasma cell infiltrate and interface hepati-
tis. (Reprinted with permission of Charles
Lassman, MD, UCLA Medical Center.)

Here’s the Point!

Interface hepatitis + Plasma cell infiltrate = Autoimmune hepatitis


“Tough Stuff” Vignettes 193

83. This is congestive hepatopathy. The term nutmeg liver is a fanciful one,
sort of like many terms cooked up by pathologists who spend too much time look-
ing through microscopes in a sub-sub basement. “Nutmeg” refers to the speckled
appearance of the liver from passive congestion; evidently this looks something
like a grated nutmeg kernel on gross examination. Microscopically, there are
dark areas from dilated hepatic venules that are full of blood, and light areas
that are unaffected surrounding parenchyma (Figure 83-1). Nutmeg liver most
commonly results from right-sided heart failure or Budd-Chiari syndrome.

Figure 83-1. Congestive hepatopa-


thy. (Reprinted with permission
of Charles Lassman, MD, UCLA
Medical Center.)

Here’s the Point!

Nutmeg liver = Congestive hepatopathy from right-heart failure or


Budd-Chiari syndrome
194 Chapter 2

84. This is alpha-1 antitrypsin (A1AT) deficiency. A1AT deficiency is an auto-


somal codominant disorder that leads to a range of problems, including liver
disease, pulmonary emphysema, panniculitis, and arterial aneurysms. The liver
manifestations of A1AT deficiency arise from a defect in protein secretion from
the endoplasmic reticulum. The A1AT proteins get “stuck” and accumulate in the
hepatocyte endoplasmic reticulum, and are classically detected as PAS-positive
“globules” that are “diastase resistant” (Figure 84.1).

Figure 84-1. A1AT deficiency with PAS stain


demonstrating “diastase-resistant” intrahe-
patic globules. (Reprinted with permission
of Charles Lassman, MD, UCLA Medical
Center.)

Here’s the Point!

PAS-positive diastase-resistant gobules in Hepatocytes =


Alpha-1 antitrypsin (A1AT) deficiency
“Tough Stuff” Vignettes 195

85.This is chronic infection with hepatitis B virus. Ground-glass hepatocytes


are a hallmark of HBV infection. Ground glass is another fanciful term from a
weary pathologist that describes the hazy and dull appearance of hepatocyte
cytoplasm in chronic HBV infection (Figure 85-1). The ground glass is composed
of surface antigens (sAg) from the HBV; the surface antigens collect in the endo-
plasmic reticulum of the hepatocytes to create this visual effect on microscopy.
The ground-glass appearance does not occur in acute HBV—only chronic HBV.
Ground-glass hepatocytes also have been described in some other conditions (eg,
drug induced, Lafora disease, fibrinogen storage disease, type IV glycogenosis),
but HBV is by far the most common cause.

Figure 85-1. Hepatitis B virus with


ground-glass hepatocytes. (Reprinted
with permission of Charles Lassman,
MD, UCLA Medical Center.)

Here’s the Point!

Ground-glass hepatocytes = Chronic infection with hepatitis B virus


196 Chapter 2

86. This is hemosiderosis, probably from underlying chronic viral hepatitis.


The key here is to distinguish hemochromatosis from hemosiderosis. The former
leads to iron deposition in the hepatocytes, and the latter leads to iron deposi-
tion in the Kupffer cells, which are the macrophages of the liver. Both can be
associated with an elevated transferrin saturation. Hemochromatosis may lead
to transaminase elevations, but hemosiderosis, in and of itself, is less likely to
cause transaminases to rise to 3 times the upper limit of normal, as occurred
here. So we need to posit an underlying condition that can lead to both hemo-
siderosis and transaminemia. There are 3 common liver conditions that fit the
bill: chronic viral hepatitis, nonalcoholic fatty liver disease, and alcoholic liver
disease. This former intravenous drug user is at risk for viral hepatitis, in par-
ticular, so that should be high on the differential diagnosis. Hemolytic anemia
is the most common non-liver cause of secondary iron overload. Whereas phle-
botomy is the standard management of hemochromatosis, it has no role in the
management of hemosiderosis. Iron chelation therapy is often used in patients
with hemosiderosis due to hemolytic anemia, but this is not typically a decision
overseen by a hepatologist or gastroenterologist.

Here’s the Point!

Iron in hepatocytes = Hemochromatosis


Iron in Kupffer cells = Hemosiderosis
“Tough Stuff” Vignettes 197

Vignette 87: Genetic Variations in Hepatitis C Treatment


Response
Chronic infection with HCV is currently treated with combination therapy
using pegylated interferon-alpha (peg-IFN-alfa) and ribavirin (RBV). Treatment
responses are especially low in genotype 1 HCV (around 45%) as compared
to genotypes 2 and 3 HCV. Moreover, the genotype 1 SVR is usually lower in
patients of African ancestry compared to the SVR in those of European or Asian
ancestry. A recent biological discovery has revealed a major genetic reason why
responses to IFN and RBV vary so much by ethnicity.

` What is the reason?


198 Chapter 2

Vignette 87: Answer


Ethnic variations in treatment response are partly determined by genetic poly-
morphisms near the interleukin-28B (IL-28B) gene, which encodes interferon-
lambda-3 (IFN-λ-3). You may think this is too arcane to show up on a Board
exam, but it’s not. This topic is prime time, and there is no time like the present
to understand this fascinating biological story with immediate clinical implica-
tions.
So what’s the deal here? It is known that Caucasians and Asians tend to have
better treatment response rates for HCV than Latinos or people of African descent,
and there is now a commercially available test that can predict nearly half the
variation in HCV treatment response rates between these patients of different
ancestry. This test is based on the results of several genome-wide association
studies that have found a single nucleotide polymorphism (SNP) just upstream
from the IL-28B gene on chromosome 19 that significantly affects response rates
to peg-INF-alfa and RBV. As noted, the IL-28B gene normally codes for IFN-λ-3
(the “lambda form” of interferon), which has its own antiviral activity. So if this
intrinsic interferon is not produced as a consequence of a genetic polymorphism
of the gene, then the treatment effect of peg-IFN-alfa and RBV will be under-
mined. The polymorphism consists of CC, CT, and TT genotypes of IL-28B. The
CC genotype leads to higher levels of IFN-λ-3 production. In contrast, the CT and
TT genotypes lead to lower levels of IFN-λ-3 production. Those patients with the
CC genotype have a 2-fold higher response rate to peg-IFN-alfa and RBV than
those with the CT and TT genotypes. And the CC genotype is much more com-
mon in patients of Asian and European ancestry, whereas it’s less prevalent in
patients of African and Latino ancestry.
This is pretty fascinating, but it’s also clinically relevant. It has been shown
that the presence of the CC IL-28B genotype predicts >2-fold rate of RVR, com-
plete EVR, and SVR compared to the presence of the CT and TT IL-28 geno-
types (recall the details on the basic viral kinetics of HCV from Vignette 64).
Furthermore, in those patients who did not achieve an RVR, the CC genotype
also had a >2-fold increase in achieving an SVR compared with the CT and TT
genotypes. Because treatment for genotype 1 HCV is extremely expensive, oner-
ous, and rife with adverse events, knowing a patient’s IL-28B genotype might
help to make or break the decision to treat. As mentioned earlier, if a patient has
the favorable CC genotype, then treatment is much more likely to work. If the
patient does not have this genotype, the likelihood of achieving an SVR is consid-
erably lower, especially if there are other factors undermining success (discussed
more later in this vignette). In fact, the IL-28B polymorphism is the most potent
pretreatment predictor of SVR in genotype 1 patients. Of course, achieving an
RVR is the strongest overall predictor for attaining an SVR.
Recently, the fight against HCV got a lot more interesting with the arrival
of direct acting anti-viral agents (DAAs) previously called specifically targeted
anti-viral therapy for HCV (STAT-C). Two linear protease inhibitors (PIs) directed
against the NS3/4a protease (telaprevir and boceprevir) are approved for use in
a combination “triple therapy” regimen (peg-IFN-alpha + RBV + PI) for genotype
1 HCV. This triple therapy regimen has improved SVR rates and can shorten
the treatment period. However, there are increased drug interactions and side
effects, including rash and anemia, in particular. Furthermore, the development
“Tough Stuff” Vignettes 199

of resistance can blow it for an entire class of DAAs. For this reason, the same
class of PIs should not be used together or as monotherapy and vigilance is
required to monitor for lack of response or virological breakthrough. Data is
accumulating in regards to the IL-28 genotype and DAA triple therapy to help
prognosticate SVR rates further. Get ready for even more DAAs, since polymerase
inhibitors and cyclophilin inhibitors will be joining the armamentarium in the
years to come!
While we are on the topic of predicting treatment response in HCV, it’s worth
listing other clinical factors known to adversely impact the efficacy of INF and
RBV. These include higher baseline viral load, higher age, male sex, higher body
mass index, presence of insulin resistance, hepatic steatosis, and hepatic fibro-
sis. We know genotypes 2 and 3 have much higher rates of response than geno-
type 1 HCV. In addition, genotype 2 tends to have a higher treatment response
than genotype 3. Furthermore, genotype 3 is associated with steatosis, which
tends to improve with treatment.
Why Might This Be Tested? This story is so biologically interesting, and the
IL-28B genotype is such a strong predictor of SVR, that it’s hard to imagine this
won’t show up on an exam soon. The availability of a commercially available test
makes this a clinically relevant discovery. And this polymorphism explains much
of the variance in response between patients of different ancestry. You probably
will not need to know too much about this polymorphism, other than it exists
and what it’s called.

Here’s the Point!

Genetic polymorphisms in IL-28B significantly predict treatment success from


peg-INF-alfa and RBV in chronic HCV. This polymorphism also explains over
half the variation in treatment differences between patients of
different ancestry.
200 Chapter 2

Vignette 88: More Swelling


A 59-year-old man with HCV cirrhosis complains of increased fatigue and dys-
pnea with minimal exertion over the past several months. He has been compliant
with sodium restriction and increasing diuretic therapy but has had “more swell-
ing” in his abdomen and legs. There is no history of smoking. Physical examina-
tion is notable for muscle wasting, marked ascites, and pedal edema. His lungs
are clear to auscultation, and he is noted to have a loud P2 heart sound with
jugular venous distention and a high-pitched holosystolic murmur at the lower
left sternal border accentuated with inspiration. There is no cyanosis.

` What is the diagnosis? (That’s right, you don’t need any labs or
imaging.)
“Tough Stuff” Vignettes 201

Vignette 88: Answer


This is portopulmonary hypertension (PPHTN). The history and physical
examination should clue you in to the diagnosis before you even look at labs or
imaging. In addition to the accentuation of the tricuspid regurgitant murmur,
physical exam may also reveal an increased right ventricular heave with inspira-
tion. An echocardiogram may show right ventricular hypertrophy with elevated
pulmonary artery pressures. However, the gold standard for diagnosis is to per-
form a right heart catheterization with hemodynamic measurements. Of course,
further imaging, laboratory testing, and cardiopulmonary tests would be helpful
to rule out other etiologies for this patient’s clinical deterioration.
Cirrhosis, by itself, causes a hyperdynamic circulation and a volume overload-
ed state, which can cause a minimal increase in pulmonary artery pressures.
However, when there is increased pulmonary vascular resistance due to PPHTN,
this will lead to marked increases in pulmonary artery pressures. To confirm
the diagnosis, a right heart catheterization would need to show a resting mean
pulmonary artery pressure (MPAP) >25 mm Hg and an increased pulmonary
vascular resistance (PVR) >240 dynes/s/cm-5 with a pulmonary capillary wedge
pressure (PCWP) <15 mm Hg.
PPHTN has a poor prognosis with median survival of approximately 15 months.
Several treatments have been used in PPHTN including prostacyclin, phospho-
diesterase inhibitors, endothelin receptor antagonists, oxygen, and anticoagula-
tion. However, liver transplantation provides the best option in select patients
with mild PPHTN whose MPAP can fall below 35 mm Hg with medical therapy,
provided that right heart function is also deemed acceptable. Although PPHTN
occurs in up to 15% of patients undergoing evaluation for liver transplantation,
the severity of PPHTN does not correlate well with the MELD score. Therefore,
exception MELD scores often can be given to these patients if they are approved
for transplant. Liver transplantation is not generally offered to patients with
MPAP >35 mm Hg due to the high perioperative risk.
Why Might This Be Tested? The history and physical examination alone
(as provided in this vignette) are critical to decide which further tests to order
to ascertain this diagnosis. Board examiners love to throw in cardiac physical
examination findings; PPHTN represents a perfect clinical scenario to test your
knowledge.
Clinical Threshold Alert:
1. 25 mm Hg is the minimum MPAP for diagnosis.
2. 35 mm Hg is the maximum MPAP for liver transplant consideration.

Here’s the Point!

Cirrhosis + Loud P2 + Tricuspid regurgitation + Increased edema →


Think PPHTN
202 Chapter 2

Vignettes 89 to 92: More Pregnancy Woes


The following mini-vignettes describe liver diseases that are unique to preg-
nancy. See if you can determine the diagnosis for each one using the minimal
information provided.

89. A 21-year-old nulliparous woman presents with severe nausea, vomiting,


and fatigue at 8 weeks’ gestation. She has had no fever, diarrhea, or ill con-
tacts. Labs include the following: ALT = 340, AST = 290, total bilirubin = 1.9,
WBC = 6.5, hemoglobin = 14.8, platelets = 271, glucose = 78, INR = 1.0, and cre-
atinine = 1.3.

90. A 36-year-old nulliparous woman, who is expecting twins via in vitro


fertilization, presents with pruritus leading to insomnia at 28 weeks’ gestation.
The pruritus started on her palms and soles, and now she describes having to
“scratch myself to death.” Labs include the following: ALT = 56, AST = 51, total
bilirubin = 1.4, WBC = 6.1, hemoglobin = 12.8, platelets = 204, glucose = 90,
INR = 1.7, and creatinine = 0.8.

91. A 28-year-old nulliparous woman at 36 weeks’ gestation presents with mal-


aise, nausea, vomiting, epigastric pain, and jaundice but no fever. Her husband
mentions that she has been sleeping most of the day. Labs include the follow-
ing: ALT = 298, ALT = 268, total bilirubin = 4.9, WBC = 14.8, hemoglobin = 10.9,
platelets = 140, glucose = 59, INR = 2.1, and creatinine = 1.7.

92. A 29-year-old multiparous woman at 31 weeks’ gestation presents with


worsening fatigue, edema, headache, nausea, and vomiting without fever.
However, she now has severe, unrelenting abdominal pain and feels lighthead-
ed. There is no melena, hematochezia, hematemesis, or vaginal bleeding. Labs
include the following: ALT = 166, AST = 190, total bilirubin = 3.8, WBC = 8.2,
hemoglobin = 6.7, platelets = 63, glucose = 80, INR = 1.3, and creatinine = 1.6.
“Tough Stuff” Vignettes 203

Vignettes 89 to 92: Answers


89. This is hyperemesis gravidarum (HG). HG occurs early in pregnancy, usu-
ally at 4 to 10 weeks’ gestation (first trimester), and almost always resolves by
week 20. The exact etiology of HG remains unknown, but several factors have
been studied. A classic fact is that liver biopsy reveals little or no pathology.
Inpatient supportive care with antiemetics, intravenous fluids, vitamin replace-
ment, and bowel rest is recommended. This clinical picture closely resembles
acute viral hepatitis, which needs to be excluded. Urinalysis and right upper
quadrant ultrasound are also important to exclude pyelonephritis and gallstone-
related disease, respectively. The maternal and fetal prognosis for HG is excel-
lent.

90. This is intrahepatic cholestasis of pregnancy (ICP), which is the second


leading cause of jaundice in pregnancy (you know the first, right?). The diagnos-
tic test of choice is serum bile acids, which can be elevated up to 100-fold and
clinches the diagnosis. Higher bile acid levels (especially more than 40 μmol/L)
in ICP carry an increased risk of complications. ICP usually occurs in the third
trimester but may sometimes present at the end of the second trimester. Risk
factors for the development of ICP include having multiple pregnancies, multiple
gestations, and being of advanced age. Furthermore, there is a genetic predispo-
sition for developing ICP, and it’s more common in certain populations, including
persons of Chilean descent. Physical examination may reveal diffuse excoria-
tions. Although the maternal outcome is good, this is not a benign condition for
the fetus. There is a significant fetal risk for premature delivery, fetal distress,
meconium ileus, and stillbirth, with up to a 4% mortality. Furthermore, there
tends to be subclinical steatorrhea associated with the cholestasis, which can
lead to vitamin K deficiency (causing the elevated INR) in the mother and fetus.
Vitamin K replacement can reduce the risk of postpartum hemorrhage, which
can be quite severe. Patients with ICP may also develop cholelithiasis. Treatment
with ursodeoxycholic acid (13 to 15 mg/kg/day) is helpful to minimize the symp-
toms, and prompt delivery is recommended if there is any sign of fetal distress.
ICP tends to recur in more than half of future pregnancies, so patients need to
be informed of this risk before getting pregnant again. Oh, and what is the most
common cause of jaundice in pregnancy? Right, viral hepatitis.

91. This is acute fatty liver of pregnancy (AFLP), which means trouble! AFLP
is quite rare to encounter in real life, occurring in 1 in 10,000 deliveries, but is
much more common on Board exams! The etiology stems from impairments of
intramitochondrial fatty acid oxidation, sometimes involving a mutation causing
long-chain 3-hydroxyacyl-coenzyme A dehydrogenase (LCHAD) deficiency. In
fact, approximately 20% of newborns of women with AFLP will have homozygous
LCHAD deficiency. The inability of the fetus to oxidize long-chain fatty acids
causes these acids to accumulate in the maternal circulation via placental trans-
fer. The heterozygous mother, in turn, has a reduced capacity to oxidize these
long-chain fatty acids, which leads to the microvesicular steatosis, encephalopa-
thy, and liver failure in the mother during the third trimester. If the diagnosis is
unclear, urgent liver biopsy is needed on fresh specimen using oil red O staining
204 Chapter 2

to examine for microvesicular steatosis (Figure 91-1). Both maternal mortal-


ity and fetal mortality are up to 20%. Thus, prompt diagnosis and delivery are
mandated. Furthermore, vigilant intensive care monitoring is necessary for the
mother after delivery. If recovery is not swift, the patient should be urgently
referred to a liver transplant center for transplant evaluation. To compound mat-
ters, the newborn with LCHAD deficiency is at risk for potentially fatal, fasting
nonketotic hypoglycemia over the next several months. Therefore, it’s mandatory
to check the newborn for LCHAD deficiency. The risk of recurrence is low with
future pregnancies, but most women choose not to get pregnant again after AFLP
(which is not terribly shocking after going through the ordeal).

Figure 91-1. Acute fatty liver of preg-


nancy with oil red O stain showing
the microvesicular fatty infiltration
in the hepatocytes. (Reprinted with
permission of Charles Lassman, MD,
UCLA Medical Center.)

92. This is HELLP (Hemolysis, Elevated LFTs, and Low Platelets) syndrome
complicated by hepatic rupture, which is another obstetric emergency! HELLP
occurs within the setting of pre-eclampsia in some cases (a great deal more
often than AFLP does). Thus, hypertension, proteinuria, and edema (the classic
triad of preeclampsia signs) are often present in HELLP. Of note, preeclampsia
on its own merit can also cause liver necrosis in severe cases. However, with
HELLP syndrome there is thrombocytopenia, an elevated LDH level, indirect
hyperbilirubinemia, and low haptoglobin from the microangiopathic hemolytic
anemia associated with the condition. Histologically, there is periportal hemor-
rhage, fibrin deposition in the sinusoids, and focal ischemic necrosis consistent
with the vascular changes that occur with the condition. If there is sudden and
severe abdominal pain with acutely worsened anemia, look for hepatic rupture or
subcapsular hematoma, which carries a mortality rate for the mother and fetus
of up to 60%. Treatment consists of prompt delivery with angiographic emboliza-
tion and/or surgery to stop the bleeding. Because this disaster can also occur
after delivery, intensive care monitoring of the mother is required until recovery.
Preeclampsia or HELLP tends to recur in up to one third of future pregnancies.
“Tough Stuff” Vignettes 205

Why Might This Be Tested? The liver disorders that are unique to pregnancy
can cause significant morbidity and mortality to the mother and fetus. Therefore,
it’s important to diagnose these conditions accurately and efficiently. In addition,
it’s important to exclude other disorders that can mimic some these conditions,
such as acute viral hepatitis or biliary disease, since the treatment can be quite
different. These are challenges for which your obstetrics colleagues will look to
you for guidance.
Clinical Threshold Alert: 13 to 15 mg/kg/day is the target range for thera-
peutic dosing of ursodeoxycholic acid for the treatment of ICP and PBC.

Here’s the Point!

LIVER DISEASES UNIQUE TO PREGNANCY


Approximate
Condition Trimester Key Labs Pathology Incidence
Hyperemesis 1st None are spe- Little to none on 1 in 200
gravidarum cific biopsy
Intrahepatic 2nd/ 3rd Elevated bile Bland cholestasis 1 in 1000
cholestasis of acids (above
pregnancy 10 µmol/L )
Acute fatty liver 3rd or High INR; Low Microvesicular 1 in 10,000
of pregnancy postpar- glucose steatosis
tum
HELLP 3rd or Low platelets Periportal hemor- 1 in 200
postpar- High LDH rhage with focal
tum High indirect ischemic necrosis
bilirubin

Here’s the Point!

ICP → High recurrence rate and is not a benign condition for the fetus
206 Chapter 2

Here’s the Point!

ICP → Increased cholelithiasis

Here’s the Point!

AFLP → Microvesicular steatosis, and can progress after delivery

Here’s the Point!

AFLP → Check for LCHAD deficiency in newborn to avoid fatal hypoglycemia

Here’s the Point!

With HELLP, hepatic rupture or intraperitoneal bleeding can also occur


after delivery.
“Tough Stuff” Vignettes 207

Vignette 93: PSC in UC


A 51-year-old woman with chronic, progressive ulcerative colitis undergoes
total colectomy after lack of response to medical therapy. There is no history of
PSC, and her ALP and GGT levels have been normal. She had read about PSC
and now wants to know if the colectomy will protect her from developing PSC in
the future.

` Has the colectomy removed her risk of developing PSC?


208 Chapter 2

Vignette 93: Answer


No. Unfortunately, colectomy does not remove the risk of developing PSC in
patients with UC. It is important to recognize, however, that the overall risk of
developing PSC in UC is low, regardless of colectomy. Case series reveal that up
to 7.5% of UC patients develop PSC. The converse is not true. That is, patients
with PSC are at much higher risk of developing UC; up to 80% of PSC patients
are ultimately diagnosed with UC. Moreover, liver transplant does not remove the
risk of developing UC. For this reason patients with PSC should undergo routine
surveillance colonoscopy as if they are in a high-risk group for colorectal cancer,
even if they have had a liver transplant; they may already harbor underlying
colonic dysplasia even if a formal diagnosis of UC has not been established. But
patients with UC need not undergo routine MRCP or ERCP to screen for PSC
unless there is some clinical indication of PSC, such as an elevated ALP, GGT,
or bilirubin level.
The relationship between UC and PSC is pretty complicated. For example,
UC patients with PSC have a higher risk of colorectal cancer than UC patients
without PSC. In fact, PSC is an independent risk factor for colorectal cancer,
although the reason is unclear. The risk of colorectal cancer is especially high in
patients who have had UC for at least 10 years and who also have PSC. Another
curious fact is that compared to UC patients without PSC, those with PSC have
a higher incidence of pouchitis following total colectomy with ileal pouch–anal
anastomosis. And the pouchitis will persist even if they undergo liver transplant
for the PSC.
Why Might This Be Tested? Board examiners have a number of content
“check boxes” they need to check off when putting an exam together. This topic
is convenient because it covers several areas, including hepatobiliary disorders,
luminal pathology, and even liver transplant. With one question they could
potentially test across several content areas.

Here’s the Point!

Colectomy does not protect against developing PSC in UC.


Liver transplant does not protect against developing UC in PSC.
“Tough Stuff” Vignettes 209

Vignette 94: Stones


A 40-year-old obese woman at 18 weeks’ gestation presents with intermittent
bouts of postprandial epigastric abdominal pain, nausea, and vomiting for the
past few weeks. Her latest episode was quite severe prompting her to come in
to the emergency department. She is admitted to the hospital with the follow-
ing labs: creatinine = 1.0, total bilirubin = 1.3, AST = 76, ALT = 38, ALP = 130,
lipase = 1658, WBC = 11.8, and hemoglobin = 14.2. An abdominal ultrasound
shows multiple gallstones in the gallbladder without thickening or pericholecys-
tic fluid noted. The common bile duct is reported as normal and 4 mm in diam-
eter; pancreatic views are obscured by bowel artifact. She receives intravenous
fluids and analgesia by the obstetrics service. You are consulted the next day
and note that her pain and nausea have resolved completely. The patient wants
to know when she can eat, since she has been placed on nil per os status. She
feels well now and wants to go home. She states that she will follow up with you
as an outpatient if she has abdominal pain again. Her obstetrician told her that
discharge home today would be fine if it were approved by the GI service. Her
laboratory tests today include the following: creatinine = 0.7, total bilirubin = 1.0,
AST = 32, ALT = 28, ALP = 128, lipase = 430, WBC = 8.4, hemoglobin = 13.1,
platelets = 210.

` Should you agree with the patient’s plan?


210 Chapter 2

Vignette 94: Answer


No to this one, too. This patient has recovered from gallstone pancreatitis
and should now have a surgical consult for laparoscopic cholecystectomy. She
requires timely surgical treatment; delay in treatment carries significant mor-
bidity to the fetus and mother if there is a recurrent episode. Therefore, it would
not be appropriate to follow her expectantly. You should advise against her plan
and should recommend that she not be discharged home without a cholecystec-
tomy. It was previously thought that cholecystectomy should be performed only
in the second trimester due to concerns of fetal demise in the first trimester and
induction of premature labor in the third trimester. However, immediate chole-
cystectomy should be considered whenever there is severe acute cholecystitis not
responding to conservative management, intractable biliary colic, or gallstone
pancreatitis. In these scenarios, there would be too much risk for both the fetus
and mother with delay. Board examinations tend to avoid questions regarding
areas of controversy, which is why this question described a patient in her second
trimester of pregnancy. Nonetheless, any pregnant patient with uncomplicated
symptomatic biliary colic should have a cholecystectomy, ideally in the second
trimester if possible. Of note, the laparoscopic approach involves less uterine irri-
tation and less postoperative analgesia and is favored over the open approach.
ERCP with sphincterotomy can be performed if there is choledocholithiasis.
Of course, care should be taken to minimize the fluoroscopy time, and the fetus
must be covered with a protective shield. In addition to the total amount of radia-
tion exposure, the timing of radiation is of utmost importance. The first week
of gestation is the most risky time to get irradiated (however, most women don’t
even know they are pregnant during this time).
Cholelithiasis is prevalent in pregnant women (about 10%) due to estrogen
increasing cholesterol synthesis and increased gallbladder volumes. This pro-
motes biliary lithogenicity; however, less than 1% have symptomatic cholelithia-
sis. Gallstones need to be considered in the workup of abdominal pain in the
pregnant patient. By the way, this patient fits the (not politically correct) 4 F’s of
gallstones that we all learned in medical school: Female, Fertile, Forty, and Fat.
Why Might This Be Tested? Gallstones are common in pregnancy, and symp-
tomatic disease will often prompt a consult to a gastroenterologist for manage-
ment (before the surgery consult). Everybody will be looking to you to provide
answers. So you need to recommend wisely.

Here’s the Point!

Cholecystectomy for complicated biliary colic during pregnancy


should not be delayed.

Here’s the Point!

Uncomplicated biliary colic should prompt laparoscopic cholecystectomy


(preferably in the second trimester).
“Tough Stuff” Vignettes 211

Vignette 95: Cough and an Elevated ALP


A 68-year-old Mexican-American presents to his primary care provider for
management of a chronic cough. During the course of his workup, laboratories
reveal abnormal liver tests including ALP = 234, AST = 99, ALT = 67, and total
bilirubin = 1.4. He is referred to you for further evaluation. On questioning, you
learn that he lived on a farm growing up where he prepared wool from sheep. He
immigrated to the United States when he was 35. He does not drink alcohol. He
does not have a history of liver disease. Although he has had a chronic cough
for several months, he does not smoke. He is febrile on examination. His liver
is enlarged, spanning 15 cm, and is not tender. There are no other stigmata of
chronic liver disease. The rest of his examination is unremarkable. Testing for
amebiasis is negative.

` What is the most likely diagnosis?


` What is the best way to confirm this diagnosis?
212 Chapter 2

Vignette 95: Answer


This is most likely an echinococcal (aka “hydatid”) cyst of the liver. Echinococcal
liver cysts occur from chronic infection with the Echinococcus granulosus tape-
worm. This tapeworm is especially prevalent in South and Central America,
the Middle East, China, and Sub-Saharan Africa. The life cycle is complex and
involves sheep and dogs, so the classic Board buzzword here is “sheepherder.”
When you hear about a sheepherder, think about echinococcosis and fasciolia-
sis (see Vignette 44). This patient was not a sheepherder, but he certainly hung
around sheep. In any event, humans are an accidental host in the echinococcal
life cycle. Echinococcosis can exist in humans for years without causing symp-
toms, only to show up years later in the form of pulmonary disease or liver cysts,
among many other manifestations. So the other clue in a Board question is hear-
ing about a cough along with some liver trouble. Cough plus liver trouble can be
many things (eg, sarcoidosis, alpha-1 antitrypsin deficiency, coccidioidomycosis),
but cough with a high ALP in someone from Central America should also make
you think about an echinococcal cyst, in particular. Any space-filling lesion of
the liver can cause the ALP to be elevated out of proportion to the other liver
tests; echinococcal cysts are no exception.
When E. granulosus infects the liver, it tends to most commonly affect the right
lobe. This distribution is common among liver infections, since the right lobe is
larger and receives more of the mesenteric vessel drainage than other parts of
the liver. So keep this clue in mind when reading Board vignettes about liver
lesions. The echinococcal liver cysts are often totally asymptomatic and only
detected through biochemical abnormalities, as seen here. However, once the
cysts grow large, they can cause trouble; this typically occurs when the cysts
expand to more than 10 cm in diameter. Once the cyst becomes large, it leads to
hepatomegaly (as seen here) and may cause abdominal pain or discomfort from
stretching of the liver capsule. In severe cases, the cyst can rupture into the
biliary system and trigger a number of problems including biliary obstruction,
cholangitis, or even pancreatitis. The cyst can also compress local vessels includ-
ing the portal vein, hepatic vein, and inferior vena cava. When this occurs the
patient can present with portal hypertension, Budd-Chiari syndrome, or venous
obstruction with lower extremity edema.
The next step in this case is to confirm the diagnosis. Diagnosing echinococ-
cal cysts may include both imaging and serologic studies (echinococcal antibod-
ies), although the imaging is usually done first. Ultrasound is highly sensitive
for detecting these cysts and is often employed as the first diagnostic study. The
ultrasonographic appearance is classically a smooth, round cyst with “daughter
cysts” and internal septations. These cysts can be calcified and have an “eggshell”
appearance due to the calcified wall—another classic Board buzzword. These
cysts may also contain so-called hydatid sand, which consists of various parts
of the tapeworms themselves (literally, the hooks and scolices of the tapeworm
form a granulated, sand-like substance within the cyst). Detailed sonographic
characteristics can be used to classify these cysts as active, transitional, or
inactive, and the classification has treatment implications, as will be described
shortly. CT scanning can reveal the same basic information but with even better
accuracy (Figure 95-1), and is often used for monitoring cyst size over time.
“Tough Stuff” Vignettes 213

Figure 95-1. Typical echinoccocal liver cyst in


the right lobe of the liver.

Sometimes it’s hard to confirm the diagnosis because the imaging is ambigu-
ous, the serology is negative, or both. In such cases percutaneous cyst aspiration
can be used to obtain fluid to evaluate for the worms themselves. This is where
distinguishing active from inactive cysts is helpful; active cysts are under high
pressure, whereas inactive cysts are not. The most worrisome consequence of
aspiration is triggering an anaphylactic reaction to the spilled cyst contents. This
is rare but can be devastating.
As a random aside, an earlier vignette involved cysts that fill up with so-called
anchovy paste (another Board buzzword). Are these the same type of cysts? The
answer is no; anchovy paste is described in abscesses from Entamoeba histo-
lytica (see Vignette 38). This patient also tested negative for amebiasis. Table
44-1 contains Board buzzword associations regarding liver parasites. That table
contains a gold mine of information for the Board exam.
Treatment of echinococcal cysts depends upon several factors and may include
a combination of albendazole, repeated percutaneous aspirations, and surgery.
Surgery is employed when the cysts are really large (ie, >10 cm), complex, or near
the capsule and at risk of rupture regardless of size. Smaller or less complicated
cysts are typically treated with either albendazole or percutaneous aspirations.
The details beyond this get fairly complicated and are not reviewed in depth
here.
Why Might This Be Tested? This diagnosis is chock-full of Board buzzwords,
so it’s a perfect setup for a test question.
Clinical Threshold Alert: Echinococcal liver cyst exceeding 10 cm typically
requires surgery.

Here’s the Point!

Sheepherder + Central or South America + Cough + Liver cyst with septations,


eggshell calcifications, and hydatid sand = Echinococcosis
3
60 HEPATOLOGY BOARD REVIEW
“CLINICAL THRESHOLD VALUES”
Many exam questions require that test-takers have memorized some numeri-
cal threshold value, like: “If an echinococcal cyst exceeds XX cm, then the risk
of rupture is clinically significant and surgery is warranted.” Or: “If the ALT:LDH
ratio exceeds Y:Y in the setting of severe transaminemia, then acute viral hepati-
tis is the most likely diagnosis.” These values have been highlighted throughout
this book. What follows is a “one-stop shop” for all these numerical facts. These
are presented by increasing numerical order—not by a rational taxonomy. So
the resulting list will seem like a pretty random hodgepodge, which is the point.
Exam questions are random too, so just go with the flow.

1.0 = If the ratio of urine sodium to potassium in a spot urine sample is greater
than this value in cirrhosis (ie, more urinary Na than K), then the
patient has a sufficient response to diuretic therapy and is likely to have
>78 mEq Na per day during a full 24-hour urine collection.
1.1 = If the serum–ascites albumin gradient (SAAG) equals or exceeds this
value, then portal hypertensive ascites is 97% likely.
1.5 = If the ALT:LDH ratio exceeds this value in the setting of severe transamin-
emia (eg, ALT and AST in the 1000+ range), then acute viral hepatitis is
likely. If the ratio is lower than this value, consider drug-induced, toxin-
induced, or hypoxemic-induced liver injury.
2 = If the AST:ALT ratio exceeds this value in the setting of biochemical hepati-
tis, and assuming the ALT is below 500 U/L, then alcoholic liver injury
is likely. Of note, cirrhosis due to any cause can also have this ratio, but
typically with lower transaminase levels than in alcoholic hepatitis.
2x ULN = If the ALP exceeds this upper limit of normal (ULN) threshold in the
setting of a culprit medication (eg, erythromycin, estrogen, rifampin,
amoxicillin, chlorpromazine), then drug-induced cholestasis is likely.
Spiegel BMR, Karsan HA.
Acing the Hepatology Questions on the GI Board
215 Exam: The Ultimate Crunch-Time Resource (pp 215-220)
© 2012 SLACK Incorporated
216 Chapter 3

Similarly, if the ALP:AST ratio exceeds 2, then this is a supportive crite-


rion for canalicular (“bland”)-type cholestasis.
2.5 mg/dL = If the ascitic fluid total protein exceeds this level with a SAAG ≥1.1,
then cardiac ascites, Budd-Chiari syndrome, or myxedema from hypo-
thyroidism is in the differential diagnosis.
3 cm = Maximum allowable size for multifocal hepatocellular carcinoma (HCC)
lesions in order to remain eligible for liver transplantation, assuming
there are no more than 3 total lesions and there is no metastatic disease
and no vascular invasion (Milan criterion—see “5 cm” threshold for an
additional Milan criterion).
3x ULN = If ALT is above this threshold in the setting of acute pancreatitis, then
the positive predictive value for a gallstone etiology is 95%.
3.4 mg/dL = If serum creatinine exceeds this level in the setting of acetamino-
phen-induced acute liver failure, then it portends a poor prognosis if
INR >6.5 and there is grade 3 or 4 encephalopathy (per King’s College
criteria).
3.9 mcg/L = An AFP of 3.9 mcg/L on day 1 after peak ALT can be used to pre-
dict survival in acetaminophen-induced liver failure with a sensitivity
of 100%, a specificity of 74%, a positive predictive value of 45%, and a
negative predictive value of 100%.
4 weeks = Undetectable hepatitis C virus (HCV) RNA at this treatment milestone
indicates a rapid virologic response (RVR). The likelihood of treatment
success with sustained virologic response (SVR) is 90% when an RVR is
achieved.
5 = If the ratio of AST:ALP exceeds this threshold in the setting of a culprit
medication, then it suggests a hepatocellular form of drug-induced liver
injury.
5 cm = Maximum allowable size for a solitary HCC lesion in order to remain
eligible for liver transplantation, provided there is no vascular invasion
and no metastatic disease (Milan criterion).
5% = Brain uptake of technetium macroaggregated albumin (TcMAA) exceeding
this amount indicates intrapulmonary shunting and supports a diag-
nosis of hepatopulmonary syndrome (HPS) assuming hypoxemia (PaO2
<70 mm Hg) and an A-a gradient >20 mm Hg.
5x ULN = If the AST exceeds this threshold in autoimmune hepatitis (AIH), and
the gamma globulin concurrently exceeds >2x the ULN, then consider
starting medical therapy. Of course, there are other indications to begin
treatment for AIH—see other thresholds later in this list.
6 = Minimum possible Model for End-Stage Liver Disease (MELD) score.
6 months = If a patient with resectable HCC is unlikely to receive an orthotopic
liver transplantation (OLT) for at least this amount of time, then initia-
tion of locoregional bridge therapy with radiofrequency ablation (RFA)
or transarterial chemoembolization (TACE) is reasonable while awaiting
OLT.
60 Hepatology Board Review “Clinical Threshold Values” 217

6.5 = If INR exceeds this level in the setting of acetaminophen-induced acute


liver failure in conjunction with creatinine >3.4 and grade 3 or 4 enceph-
alopathy, then it portends a poor prognosis (per King’s College criteria).
6 to 12 cm = Normal span of the liver by percussion.
7.3 = If arterial pH falls below this value after fluid resuscitation in the setting
of acetaminophen-induced acute liver failure, then it portends a poor
prognosis (per King’s College criteria).
7.5 g = If more than this amount of acetaminophen is consumed at once, then
acetaminophen can be hepatotoxic even to a patient without pre-existing
liver disease.
10 mmol/L = The goal of diuretic therapy is to induce natriuresis, defined by a
spot urine sodium exceeding this threshold.
10 cm = If an echinococcal liver cyst exceeds this size, then it likely requires
surgery for definitive therapy due to high risk of rupture.
10x ULN = If the AST exceeds this threshold in AIH, then consider starting
medical therapy regardless of the gamma globulin level.
12 g = Quantity of alcohol in a standard alcoholic drink in the United States.
12 mm Hg = If the hepatic venous pressure gradient (HVPG) exceeds this value,
then variceal formation is enhanced. Goal of beta-blocker therapy is to
reduce the pressure to beneath this threshold.
12 weeks = A 2-log drop or more in HCV RNA at this treatment milestone indi-
cates an early virologic response, or EVR. When an EVR is achieved, the
likelihood of achieving an SVR is 66% in genotype 1 HCV.
13 to 15 mg/kg/day = Target range for therapeutic dosing of ursodeoxycholic
acid for management of primary biliary cirrhosis (PBC) and intrahepatic
cholestasis of pregnancy (ICP).
15 = Usual minimal MELD listing score for liver transplantation.
15 mm Hg = Pulmonary capillary wedge pressure must be below this threshold
in order to diagnose portopulmonary hypertension, assuming the mean
pulmonary artery pressure (MPAP) is above 25 mm Hg and the pulmo-
nary vascular resistance (PVR) is above 240 dynes/s/cm-5.
20 mg/dL = Ceruloplasmin levels below this value are sensitive (but not specific)
for Wilson disease.
20 mm Hg = A-a gradient must exceed this threshold in order to diagnose HPS,
assuming there is hypoxemia with PaO2 <70 mm Hg.
20 mm Hg = Goal of treatment in acute liver failure complicated by elevated
intracranial pressure is to drop intracranial pressure below this
threshold.
22 = Automatic MELD score initially assigned by the United Network of Organ
Sharing (UNOS) for a patient with HCC that fits Milan criteria, regard-
less of other MELD parameters.
24 weeks = Usual treatment course for genotype 2 or 3 HCV.
218 Chapter 3

25 = Body mass index (BMI) at or above this threshold defines “overweight.”


25 mm Hg = MPAP must be above this threshold in order to diagnose portopul-
monary hypertension, assuming the PVR is above 240 dynes/s/cm-5 and
the pulmonary capillary wedge pressure is less than 15 mm Hg.
30 = BMI at or above this threshold defines “obesity.”
32 = If the Maddrey discriminant function score (4.6 x ∆ prothrombin time +
total bilirubin) is above this value in acute alcoholic hepatitis, then con-
sider starting steroids or pentoxiphylline.
34 weeks = Weeks of gestation at which an HBeAg-positive mother with elevated
hepatitis B virus (HBV) DNA level should begin oral anti-HBV therapy to
minimize vertical transmission of HBV to the newborn.
35 mm Hg = Maximum MPAP often considered acceptable for liver transplanta-
tion in the setting of portopulmonary hypertension.
40 = BMI at or above this threshold defines “morbid obesity.”
40 = Maximum possible MELD score.
40 μmol/L = When the serum concentration of bile acids exceeds this value in
pregnancy, the risk of developing complications from ICP increases sig-
nificantly.
40% = Brain uptake of TcMAA exceeding this amount is a poor prognostic indi-
cator in HPS indicating a high level of shunting and is a contraindication
to liver transplantation.
48 weeks = Usual treatment course for genotype 1 HCV.
50 = Goal in hereditary hemochromatosis is to drive ferritin below this level.
50 mm Hg = PaO2 below this threshold is a poor prognostic indicator in HPS and
a contraindication to liver transplantation.
50 to 60 mm Hg = Goal in acute liver failure is to keep cerebral perfusion pres-
sure (CPP) above this threshold (CPP = mean arterial pressure – intra-
cranial pressure).
70 mm Hg = PaO2 must fall below this threshold in order to diagnose HPS,
assuming the A-a gradient is above 20 mm Hg.
72 weeks = Length of therapy recommended for hepatitis C genotype 1 patients
who are “slow responders”—not the traditional 48 weeks.
78 mEq/L = Goal of diuretic therapy in cirrhotic ascites is to achieve at least
this amount of sodium excretion over a 24-hour urine collection.
Because a 24-hour collection is often difficult to obtain, most use a spot
urine to estimate what a 24-hour collection might have yielded. A urine
sodium:potassium ratio >1.0 predicts >78 mEq/L of sodium excretion in
a 24-hour collection.
88 = If platelet count is below this value in cirrhosis, then the risk of underlying
varices increases substantially.
100 mcg/24 hours = Urinary copper excretion above this level is found in almost
all symptomatic patients with Wilson disease.
60 Hepatology Board Review “Clinical Threshold Values” 219

240 dynes/s/cm-5 = PVR must exceed this threshold in order to diagnose por-
topulmonary hypertension, assuming the pulmonary capillary wedge
pressure is below 15 mm Hg and the MPAP is >25 mm Hg.
250 = If the PMN count in ascites exceeds this value in cirrhosis, then the
patient likely has spontaneous bacterial peritonitis (SBP).
250 mcg/g = Hepatic copper concentration above this level occurs in Wilson
disease.
500 mL = Physiologically, this is the maximum amount of ascites that can be
absorbed from the peritoneum in 1 day.
1000 ng/mL = A patient under 40 years old with hemochromatosis who has a
ferritin value less than 1000 ng/mL is unlikely to have underlying cir-
rhosis, and liver biopsy can often be avoided.
5700 = 6-methylmercaptopurine levels above 5700 are associated with increased
risk of hepatotoxicity when using azathioprine or 6-mercaptopurine.
4
“CRUNCH-TIME” SELF-TEST
TIME TO GET YOUR GAME ON
This is a rapid-fire “crunch-time” self-test. The questions in this test are loose-
ly based on the “Here’s the Point!” bullet points from the vignettes. These points
represent the distilled essence of potential Board vignettes, so know them well.
Some of the questions, however, are stand-alone questions that do not have a
corresponding vignette in the book. They are included to test overall background
knowledge as opposed to book memorization.
As you read each one-liner, write the answer on the corresponding blank line.
Really … just actively write it right there on the page. Many of the questions
ask you to make a diagnosis based on the information given. Although Board
questions often ask about much more than the mere diagnosis, you will need to
know the diagnosis in order to know what to do next. So, these questions are a
bottom-line test of your basic diagnostic capabilities for the “tough stuff” that
might show up on the exam.
Very few of these questions are true “gimmies.” But if you have carefully
studied the vignettes up to this point, then this should be a relative snap—and
should reaffirm that you are well on your way to acing the tough stuff. Once you
are done, check the answer key on page 237 and score yourself.
Try not to cheat too much as you score your test. If you cheat your way
through this, then you won’t really know how you did and you won’t be able to
interpret your score according to the guide on page 243. Once you have finished
scoring your test, look up the corresponding vignettes for each of the items you
got wrong, and then study those vignettes carefully to fill in your knowledge
gaps.

Spiegel BMR, Karsan HA.


Acing the Hepatology Questions on the GI Board
221 Exam: The Ultimate Crunch-Time Resource (pp 221-236)
© 2012 SLACK Incorporated
222 Chapter 4

Question 1. Arrhythmias + Liver test abnormalities + Phospholipid-laden lyso-


somal lamellar bodies in hepatocytes.
` Diagnosis

Question 2. Health freak develops elevated liver tests and has lipid-filled stellate
cells on liver biopsy.
` Diagnosis

Question 3. Liver lesion that yields “anchovy paste” upon aspiration.


` Diagnosis

Question 4. Isolated fundic varices in the setting of chronic pancreatitis.


` Diagnosis

Question 5. Upper respiratory tract infection + Cholestasis.


` Diagnosis

Question 6. Acute liver failure upon withdrawal of steroids in polyarteritis


nodosa.
` Diagnosis

Question 7. Hepatitis C + Treatment + Infraumbilical necrotic lesion.


` Diagnosis

Question 8. Hepatitis C + Lower extremity rash + Positive rheumatoid factor +


Renal disease.
` Diagnosis

Question 9. Hepatitis C + Treatment + Confluent erythematous rash or pruri-


tus.
` Diagnosis

Question 10. ALT and AST in 1000+ range with ALT:LDH ratio above 1.5 with
fever, malaise, and no abdominal pain.
` Diagnosis #1

` Diagnosis #2
“Crunch-Time” Self Test: Time to Get Your Game On 223

Question 11. Unexplained elevated aminotransferases + Chronic IBS symp-


toms.
` Diagnosis

Question 12. Crohn’s disease + Scaling rash + Low ALP.


` Diagnosis

Question 13. Liver failure during a marathon.


` Diagnosis

Question 14. Acute hepatitis + Hemolytic anemia + Low ALP.


` Diagnosis

Question 15. Hepatitis C + Bullae and vesicles on sun-exposed skin.


` Diagnosis

Question 16. Hepatic bruit + Recent abdominal stab wound.


` Diagnosis

Question 17. Most likely cause of pancreatitis when the ALT is markedly ele-
vated.
` Diagnosis

Question 18. Florid duct lesion + Mallory bodies.


` Diagnosis

Question 19. Hepatitis + Hypouricemia + Mallory bodies.


` Diagnosis

Question 20. Liver problems with risus sardonicus and micrographia.


` Diagnosis

Question 21. Metabolic syndrome + Low AST:ALT ratio + Mallory bodies.


` Diagnosis
224 Chapter 4

Question 22. Mastalgia with ascites treatment.


` Diagnosis

Question 23. Premature gray hair + Dyspepsia + Macrocytosis + Low ALP.


` Diagnosis

Question 24. Acute liver failure with persistent hypotension and no evidence of
sepsis, neurologic, or cardiogenic shock.
` Diagnosis

Question 25. Gastroparesis + Cholestasis.


` Diagnosis

Question 26. Caudate lobe hypertrophy.


` Diagnosis

Question 27. ≥2-log drop in HCV RNA but detectable virus at week 12; undetect-
able RNA at week 24.
` Diagnosis

Question 28. Hepatic abscess in traveler with elevation of right hemidiaphragm


with adhesions obliterating the costophrenic angle.
` Diagnosis

Question 29. Dermatographism + Hepatomegaly + Pruritus + Flushing +


Elevated ALP.
` Diagnosis

Question 30. New-onset continuous periumbilical humming sound in a patient


with cirrhosis with recent improvement of ascites.
` Diagnosis

Question 31. Acute liver failure after eating some botanicals on a nature walk
in Ireland.
` Diagnosis
“Crunch-Time” Self-Test: Time to Get Your Game On 225

Question 32. Hepatic bruit + Sudden high AST:ALT ratio + History of recurrent
pancreatitis.
` Diagnosis

Question 33. Sheepherder with liver cyst with “eggshell calcifications” and sep-
tations.
` Diagnosis

Question 34. ANA positive + SMA positive + AMA negative + Elevated ALP.
` Diagnosis

Question 35. High SAAG, high protein ascites + Fatigue + Constipation + Low
ALP.
` Diagnosis

Question 36. Polycythemia vera + Hepatomegaly.


` Diagnosis

Question 37. Liver mass with central stellate scar without growth on serial
imaging.
` Diagnosis

Question 38. Anabolic steroids + Subcapsular liver mass with arterial


enhancement.
` Diagnosis

Question 39. “Stepwise” fever + Temperature-pulse dissociation + “Rose spots”


+ Hepatitis.
` Diagnosis

Question 40. Undetectable HCV RNA by week 4.


` Diagnosis

Question 41. Undetectable HCV RNA by week 12.


` Diagnosis
226 Chapter 4

Question 42. Undetectable HCV RNA 6 months after completion of treatment


course.
` Diagnosis

Question 43. Sheepherder develops biliary obstruction and hepatomegaly, and


has “tortuous tracks” in liver on CT.
` Diagnosis

Question 44. IBD patient gets painful hepatomegaly, ascites, and a high biliru-
bin after starting azathioprine.
` Diagnosis

Question 45. Spider web collaterals on hepatic venography.


` Diagnosis

Question 46. Chronically unexplained elevation of aminotransferases + Vesicular


elbow rash + Iron deficiency anemia.
` Diagnosis

Question 47. Hepatic bruit + Long-standing hemochromatosis + New-onset


encephalopathy.
` Diagnosis

Question 48. A bodybuilder develops a rare liver tumor (not hepatocellular car-
cinoma).
` Diagnosis

Question 49. Cause of biliary ductal fibrosis and gallstones in someone who eats
lots of undercooked seafood.
` Diagnosis

Question 50. Worms in the biliary tree after eating contaminated freshwater
plants.
` Diagnosis

Question 51. Lots and lots of biliary and intrahepatic pigment stones, but a
normal gallbladder.
` Diagnosis
“Crunch-Time” Self-Test: Time to Get Your Game On 227

Question 52. Anicteric acute liver failure with fever, leukopenia, and abnormal
chest X-ray.
` Diagnosis

Question 53. Genetic polymorphisms of this gene explain variations in HCV


treatment success among patients with different ancestry.
` Gene

Question 54. “Owl’s eye” intranuclear hepatocyte inclusions.


` Diagnosis

Question 55. Sexually active woman with fever, right upper quadrant pain,
abnormal aminotransferase with enhancement of the anterior liver capsule
on arterial phase of the CT undergoes laparoscopy notable for “violin string”
perihepatic adhesions.
` Diagnosis

Question 56. Acute liver failure in HBV carrier with active drug abuse with the
following serologies: hepatitis B core IgM negative, hepatitis B DNA 75 IU/mL,
hepatitis A IgG positive, HCV RNA negative.
` Diagnosis

Question 57. Patient with a history of pancreatitis that responded to a course of


tapering medication now presents with elevated ALP and GGT. MRCP reveals
a “chain of lakes” appearance in the biliary tree.
` Diagnosis

Question 58. Cough + Liver cyst with septations.


` Diagnosis

Question 59. Most heavily weighted variable in the MELD scoring system.
` Variable

Question 60. Circle the clinical consequences of hereditary hemochromatosis


that can improve with successful phlebotomy:
Arthropathy
Hypogonadism
Decompensated cirrhosis
Cutaneous hyperpigmentation
Diabetes
228 Chapter 4

Question 61. Liver mass with calcifications and a central scar in the absence
of cirrhosis.
` Diagnosis

Question 62. Undetectable HCV RNA at completion of therapy.


` Diagnosis

Question 63. Portal vein granuloma + Portal hypertension + Absence of other


stigmata of chronic liver disease + Eosinophilia.
` Diagnosis

Question 64. Liver disease + Parkinsonian tremor + Fanconi syndrome.


` Diagnosis

Question 65. Sunflower cataracts.


` Diagnosis

Question 66. Acute hepatitis + Hyperthermia + Bruxism + Young adult.


` Diagnosis

Question 67. Anaphylaxis after a liver cyst filled with “sand” ruptures.
` Diagnosis

Question 68. Unexplained elevated aminotransferases + Short statured +


Osteopenia + Infertility + Bloating.
` Diagnosis

Question 69. Proximal muscle weakness + 10-fold elevations of AST and ALT +
Normal total bilirubin, INR, and ALP.
` Diagnosis

Question 70. Patient with precirrhotic PBC becomes tired and constipated.
` Diagnosis
“Crunch-Time” Self-Test: Time to Get Your Game On 229

Question 71. Patient with PSC develops easy bruisability and elevated INR after
starting antibiotic treatment for cholangitis.
` Diagnosis

Question 72. Patient with PBC develops progressive night blindness.


` Diagnosis

Question 73. Patient with PBC develops problems with balance and gait in the
absence of hepatic encephalopathy.
` Diagnosis

Question 74. Pregnant woman with fatigue, polyuria, palmar erythema, spider
angiomas, and small esophageal varices.
` Most common diagnosis

Question 75. Pregnant woman with cirrhosis presenting with a left upper quad-
rant discomfort and bruit.
` Diagnosis

Question 76. A patient receives a new medication for her seizure disorder. A
month later she is found to have elevated liver tests. A subsequent liver biopsy
reveals microvesicular steatosis.
` Diagnosis

Question 77. Hyperpigmentation + Arthropathy + High ferritin + Abnormal


echocardiogram.
` Diagnosis

Questions 78 to 80. Patient comes from Puerto Rico with diarrhea, malabsorp-
tion, and flat villi on endoscopy. He develops elevated AST and ALT after
beginning therapy for his condition. (Name the condition, medication, and
type of hepatotoxicity with the medication.)
` 78. Diagnosis

` 79. Medication

` 80. Hepatotoxicity
230 Chapter 4

Question 81. Most favorable IL28B polymorphism genotype.


` Genotype

Question 82. Obese patient with diabetes has a 5-cm irregular, hypoechoic
lesion in the right hepatic lobe. Follow-up CT reveals a hypodense, sharply
demarcated mass. The contour and architecture are otherwise not distorted,
with vascular structures passing through the mass normally.
` Diagnosis

Question 83. Patient with essential thrombocytosis has isolated, elevated ALP.
CT reveals multiple hypodense nodules. Biopsy reveals regenerative nodules
clustered around portal triads without fibrosis between the nodules.
` Diagnosis

Question 84. Asian patient with Crohn’s disease starts infliximab and develops
acute liver failure. What underlying infection may have done this?
` Diagnosis

Question 85. HBeAg-positive patient with high HBV viral load is pregnant at
week 34. What can be done to minimize vertical transmission to the new-
born?
` Treatment

Questions 86 to 87. Patient eats an Amanita mushroom and develops acute liver
failure. Name 2 specific therapies for this particular form of liver failure.
` 86. Therapy #1

` 87. Therapy #2

Question 88. HBV genotype that is most prevalent among African-Americans.


` Genotype

Question 89. HBV genotype with best response to interferon therapy.


` Genotype

Questions 90 to 91. Two most prevalent HBV genotypes among Asian patients.
` 90. Genotype

` 91. Genotype
“Crunch-Time” Self-Test: Time to Get Your Game On 231

Question 92. HBV genotype that is most prevalent among patients from Eastern
Europe.
` Genotype

Question 93. Traveler to Indian subcontinent develops acute hepatitis despite


appropriate pre-travel vaccinations.
` Diagnosis

Question 94. Pregnant woman develops severe nausea, vomiting, and fatigue at
8 weeks’ gestation. Labs include elevated AST, ALT, and mildly elevated bili-
rubin. There are no ill contacts. Liver biopsy is normal.
` Diagnosis

Question 95. Most common cause of jaundice in pregnancy.


` Diagnosis

Question 96. Second leading cause of jaundice in pregnancy.


` Diagnosis

Question 97. Fever and cutaneous skin eruptions after starting therapy for
Wilson disease.
` Diagnosis

Question 98. People of Chilean descent are especially at risk for developing this
liver complication of pregnancy.
` Diagnosis

Question 99. Pregnant woman in third trimester develops pruritus, subclinical


steatorrhea, and vitamin K deficiency.
` Diagnosis

Question 100. Liver disease of pregnancy resulting from impairment of intra-


mitochondrial fatty acid oxidation from a mutation causing long-chain 3-
hydroxyacyl-coenyzme A dehydrogenase (LCHAD) deficiency.
` Diagnosis
232 Chapter 4

Question 101. Late third trimester pregnancy is complicated by elevated AST,


ALT, total bilirubin, creatinine, and INR, along with mental status changes
and hypoglycemia.
` Diagnosis

Question 102. Woman with HELLP syndrome develops severe, acute abdominal
pain and hypotension while awaiting emergency C-section.
` Diagnosis

Question 103. Massive liver cysts + Intracranial aneurysm.


` Diagnosis

Question 104. Most common malignancy to develop de novo after liver trans-
plantation.
` Diagnosis

Question 105. Patient with autosomal dominant polycystic kidney disease devel-
ops lower abdominal pain and fever and has rebound tenderness on exam.
` Diagnosis

Question 106. Cirrhosis + Loud P2 + Tricuspid regurgitation + Increased


edema.
` Diagnosis

Question 107. Liver disease + Hypoxemia + Intrapulmonary vasodilation.


` Diagnosis

Question 108. Liver disease + Hypoxemia + >5% brain uptake of technetium


macroaggregated albumin.
` Diagnosis

Question 109. Patient has a 2-cm left lobe amebic liver abscess.
` Treatment
“Crunch-Time” Self-Test: Time to Get Your Game On 233

Question 110. A 50-year-old patient with cirrhosis with a small, 2-cm HCC. Labs:
albumin = 3.5, INR = 1.0, total bilirubin = 1.9, creatinine = 1.0, platelets = 102,
AFP = 40.
` Best treatment

Question 111. DIC + Giant hemangioma.


` Diagnosis

Question 112. Third-trimester pregnancy is complicated by elevated AST, ALT,


total bilirubin, and LDH, along with microangiopathic hemolytic anemia.
` Diagnosis

Question 113. Patient with cirrhosis with platypnea and orthodeoxia.


` Diagnosis

Question 114. AIDS + Elevated ALP and GGT + Bartonella infection.


` Diagnosis

Question 115. Patient with CREST syndrome has a high GGT level +
Steatorrhea.
` Diagnosis

Question 116. Patient with acute diverticulitis develops right upper quadrant
pain and persistent fever with this nonmalignant liver abnormality on CT.
` Diagnosis

Question 117. Third-trimester pregnancy is complicated by liver trouble. Biopsy


reveals periportal hemorrhage, fibrin deposition in the sinusoids, and focal
necrosis.
` Diagnosis

Question 118. Cause of gram-negative septic shock in Gulf Coast fisherman


with cirrhosis and previous leg laceration.
` Diagnosis
234 Chapter 4

Question 119. Recurrence rate of Wilson disease after liver transplantation.


` Recurrence rate

Question 120. 35-year-old man with PSC and 1-cm gallbladder polyp.
` Recommendation

Question 121. Recurrence rate of alpha-1 antitrypsin deficiency (A1AT) after


liver transplantation.
` Recurrence rate

Question 122. Most common phenotype of A1AT.


` Phenotype

Question 123. Most common A1AT phenotype in patients with A1AT-induced


chronic obstructive pulmonary disease.
` Phenotype

Question 124. Most common A1AT phenotype in patients with A1AT-induced


cirrhosis.
` Phenotype

Question 125. This A1AT phenotype serves an intermediate role with other
cofactors that can lead to cirrhosis.
` Phenotype

Question 126. Most common cause of inherited indirect hyperbilirubinemia that


could potentially lead to an artificially high MELD score:
` Diagnosis

Question 127. Currently the most common indication for liver transplantation
in the United States.
` Diagnosis

Question 128. Based on prognostic models, this will be the most common indi-
cation for liver transplantation in the United States 40 years from now.
` Diagnosis
“Crunch-Time” Self-Test: Time to Get Your Game On 235

Question 129. Hepatitis C is an independent risk factor for diabetes mellitus


after liver transplantation. (Circle correct answer.)
True
False

Question 130. This hepatitis C genotype is associated with NAFLD, which can
improve with therapy.
` Genotype

Question 131. This hepatitis C genotype is highly prevalent in Egypt.


` Genotype

Question 132. This hepatitis C genotype is highly prevalent in South Africa.


` Genotype

Question 133. This hepatitis C genotype is highly prevalent in Southeast Asia.


` Genotype

Questions 134 and 135. These questions concern DAAs and HCV genotype 1
subtype comparisons to one another.

134. Which subtype of HCV genotype 1 is more commonly found in North


America and has increased resistance to NS3/4a protease inhibitors?
` Subtype

135. Which subtype of HCV genotype 1 is more commonly found in Europe


and has a decreased resistance to the NS3/4a protease inhibitors?
` Subtype
A
APPENDIX A
ANSWERS TO “CRUNCH-TIME” SELF-TEST
1. Amiodarone hepatotoxicity
2. Hypervitaminosis A
3. Amebic liver abscess
4. Splenic vein thrombosis
5. Cholestasis from trimethoprim-sulfamethoxazole or amoxicillin-clavulanate
6. Acute hepatitis B during immune reconstitution
7. Interferon-induced skin necrosis at injection site
8. Mixed cryoglobulinemia
9. Ribavirin-induced skin rash
10. 1. Acute viral hepatitis 2. Autoimmune hepatitis
11. Celiac sprue
12. Zinc deficiency
13. Acute liver failure from exertional heat stroke
14. Wilson disease
15. Porphyria cutanea tarda
16. Traumatic arteriovenous fistula
17. Gallstones
18. Primary biliary cirrhosis
19. Wilson disease
20. Wilson disease
Spiegel BMR, Karsan HA.
Acing the Hepatology Questions on the GI Board
237 Exam: The Ultimate Crunch-Time Resource (pp 237-242)
© 2012 SLACK Incorporated
238 Appendix A

21. Nonalcoholic fatty liver disease


22. Spironolactone-induced gynecomastia
23. Pernicious anemia
24. Adrenal insufficiency
25. Erythromycin-induced cholestasis
26. Budd-Chiari syndrome
27. Partial early virologic response or slow responder
28. Amebic liver abscess
29. Systemic mastocytosis with liver infiltration
30. Cruveilhier-Baumgarten murmur from recanalized umbilical vein
31. Amanita mushroom poisoning
32. Alcoholic hepatitis
33. Echinococcosis
34. Autoimmune cholangiopathy
35. Hypothyroidism
36. Budd-Chiari syndrome
37. Focal nodular hyperplasia
38. Hepatic adenoma
39. Salmonella typhi hepatitis
40. Rapid virologic response
41. Complete early virologic response
42. Sustained virologic response
43. Fascioliasis
44. Sinusoidal obstruction syndrome (aka veno-occlusive disease)
45. Budd-Chiari syndrome
46. Celiac sprue (with dermatitis herpetiformis)
47. Hepatocellular carcinoma
48. Hepatic angiosarcoma
49. Clonorchis sinensis
50. Fasciola hepatica
51. Recurrent pyogenic cholangitis
52. Acute herpes hepatitis
53. IL-28B
54. CMV hepatitis
55. Fitz-Hugh–Curtis syndrome
Answers to “Crunch-Time” Self-Test 239

56. Hepatitis D (delta) virus superinfection


57. Autoimmune (IgG4-associated) cholangitis
58. Echinococcal (hydatid) cyst
59. INR
60. Cutaneous hyperpigmentation and diabetes
61. Fibrolamellar hepatocellular carcinoma
62. End of treatment response
63. Schistosomiasis
64. Wilson disease (yes, again)
65. Wilson disease (know it)
66. Ecstasy overdose
67. Echinococcal (hydatid) cyst
68. Celiac sprue
69. Myositis
70. Autoimmune-related hypothyroidism
71. Vitamin K deficiency
72. Vitamin A deficiency
73. Vitamin E deficiency
74. Normal pregnancy
75. Splenic artery aneurysm
76. Valproic acid hepatotoxicity
77. Hemochromatosis
78. Condition: tropical sprue
79. Medication: tetracycline
80. Side effect: microvesicular steatosis
81. CC
82. Focal fatty infiltration (not a tumor)
83. Nodular regenerative hyperplasia
84. Hepatitis B
85. Start oral antiviral in mom; newborn receives HBIG and HBV vaccination
86. Therapy #1: Penicillin G
87. Therapy #2: Milk thistle (silymarin)
88. Genotype A
89. Genotype A
90. Genotype B or C
240 Appendix A

91. Genotype C or B
92. Genotype D
93. Hepatitis E
94. Hyperemesis gravidarum
95. Viral hepatitis
96. Intrahepatic cholestasis of pregnancy
97. D-Penicillamine acute hypersensitivity reaction
98. Intrahepatic cholestasis of pregnancy
99. Intrahepatic cholestasis of pregnancy
100. . Acute fatty liver of pregnancy
101. Acute fatty liver of pregnancy
102. Subcapsular hematoma or rupture
103. Autosomal dominant polycystic kidney disease
104. Squamous cell skin cancer
105. Acute diverticulitis
106. Portopulmonary hypertension
107. Hepatopulmonary syndrome
108. Hepatopulmonary syndrome
109. Surgical drainage procedure
110. Refer for transplant
111. Kasabach-Merritt syndrome
112. HELLP syndrome
113. Hepatopulmonary syndrome
114. Peliosis hepatis
115. May be PBC
116. Pyogenic liver abscess
117. HELLP syndrome
118. Vibrio vulnificus
119. 0% (cured)
120. Cholecystectomy
121. 0% (cured)
122. MM
123. ZZ
124. ZZ
125. MZ
Answers to “Crunch-Time” Self-Test 241

126. Gilbert’s syndrome


127. Hepatitis C
128. NAFLD or NASH
129. True
130. Genotype 3
131. Genotype 4
132. Genotype 5
133. Genotype 6 (Genotypes 7, 8, 9, 10, and 11 also found in Southeast Asia)
134. Genotype 1a
135. Genotype 1b
B
APPENDIX B
“CRUNCH-TIME” SELF-TEST
SCORING GUIDE
135 correct: You cheated.
130–134: You still cheated.
125–129: Impossible to believe.
120–124: Either you cheated, or you’re a monster diagnostician
ready to crush the Boards.
115–119: Assuming you didn’t cheat, that was a crazy good perfor-
mance.
110–114: Outstanding performance—easily more than a standard
deviation above the mean.
105–109: Pretty darn tremendous.
100–104: Highly respectable—well above average for this level of dif-
ficulty.
95–109: Good work—you’re definitely ahead of the curve.
90–94: You’re doing fine—a good effort.
85–89: Don’t despair—these are hard, and you hung in well.
80–84: Not terrible, but you need to start fine-tuning the rough
spots.
75–79: Could be better.
70–74: Look in the mirror. Then say, “I know I can do better. Let’s
kick this up a notch.”
65–69: You’re in the 50% range now—mediocre.
60–64: Not good enough—below average.
55–59: These are tough, but you’re below the curve.
50–54: Inadequate knowledge base. You’re in jeopardy of not pass-
ing the exam.
<50: Wait a while before taking the exam. You’ve got a ways to
go.
Spiegel BMR, Karsan HA.
Acing the Hepatology Questions on the GI Board
243 Exam: The Ultimate Crunch-Time Resource (pp 243-244)
© 2012 SLACK Incorporated
Bibliography

BIBLIOGRAPHY

Adams LA, Lymp JF, St Sauver J, et al. The natural history of nonalcoholic fatty liver disease: a population-based
cohort study. Gastroenterology. 2005;129:113-121.
Aggarwal R, Naik S. Epidemiology of hepatitis E: current status. J Gastroenterol Hepatol. 2009;24:1484-1493.
Anand AC, Nightingale P, Neuberger JM. Early indicators of prognosis in fulminant hepatic failure: an assessment
of the King’s criteria. J Hepatol. 1997;26:62-68.
Andreu V, Mas A, Bruguera M, et al. Ecstasy: a common cause of severe acute hepatotoxicity. J Hepatol.
1998;29:394-397.
Angulo P, Dickson ER, Therneau TM, et al. Comparison of three doses of ursodeoxycholic acid in the treatment of
primary biliary cirrhosis: a randomized trial. J Hepatol. 1999;30:830-835.
Arguedas MR, Abrams GA, Krowka MJ, Fallon MB. Prospective evaluation of outcomes and predictors of mortality
in patients with hepatopulmonary syndrome undergoing liver transplantation. Hepatology. 2003;37:192-197.
Bacon BR. Hemochromatosis: diagnosis and management. Gastroenterology. 2001;120:718-725.
Bacon BR, Gordon SC, Lawitz E, et al. Boceprevir for previously treated chronic HCV genotype 1 infection. N Engl
J Med. 2011;364:1207-1217.
Bacq Y, Zarka O, Brechot JF, et al. Liver function tests in normal pregnancy: a prospective study of 103 pregnant
women and 103 matched controls. Hepatology. 1996;23:1030-1034.
Bader T. The myth of statin-induced hepatotoxicity. Am J Gastroenterol. 2010;105:978-980.
Bass NM, Mullen KD, Sanyal A, et al. Rifaximin treatment in hepatic encephalopathy. N Engl J Med. 2010;362:1071-
1081.
Berger J, Hart J, Millis M, Baker AL. Fulminant hepatic failure from heat stroke requiring liver transplantation. J Clin
Gastroenterol. 2000;30:429-431.
Bonora E, Targher G, Alberiche M, et al. Homeostasis model assessment closely mirrors the glucose clamp tech-
nique in the assessment of insulin sensitivity: studies in subjects with various degrees of glucose tolerance and
insulin sensitivity. Diabetes Care. 2000;23:57-63.
Bornstein JD, Byrd DE, Trotter JF. Relapsing hepatitis A: a case report and review of the literature. J Clin
Gastroenterol. 1999;28:355-356.
Boyer TD, Haskal ZJ. The role of transjugular intrahepatic portosystemic shunt (TIPS) in the management of portal
hypertension: update 2009. Hepatology. 2010;51:306.
Bravi F, Bosetti C, Tavani A, et al. Coffee drinking and hepatocellular carcinoma risk: a meta-analysis. Hepatology.
2007;46:430-435.
Spiegel BMR, Karsan HA.
Acing the Hepatology Questions on the GI Board
245 Exam: The Ultimate Crunch-Time Resource (pp 245-250)
© 2012 SLACK Incorporated
246 Bibliography

Broome U, Glaumann H, Lindstom E, et al. Natural history and outcome in 32 Swedish patients with small duct
primary sclerosing cholangitis (PSC). J Hepatol. 2002;36:586-589.
Broome U, Lofberg R, Veress B, Eriksson LS. Primary sclerosing cholangitis and ulcerative colitis: evidence for
increased neoplastic potential. Hepatology. 1995;22:1404-1408.
Buckles DC, Lindor KD, Larusso NF, Petrovic LM, Gores GJ. In primary sclerosing cholangitis, gallbladder polyps
are frequently malignant. Am J Gastroenterol. 2002;97:1138-1142.
Burak K, Angulo P, Pasha TM, Egan K, Petz J, Lindor KD. Incidence and risk factors for cholangiocarcinoma in
primary sclerosing cholangitis. Am J Gastroenterol. 2004;99:523-526.
Carvalho M, Pontes H, Remiao F, Bastos ML, Carvalho F. Mechanisms underlying the hepatotoxic effects of ecstasy.
Curr Pharm Biotechnol. 2010;11:476-495.
Cassidy WM, Reynolds TB. Serum lactic dehydrogenase in the differential diagnosis of acute hepatocellular injury.
J Clin Gastroenterol. 1994;19:118-121.
Chapman R, Fevery J, Kalloo A, et al. Diagnosis and management of primary sclerosing cholangitis. Hepatology.
2010;51:660-678.
Claessen MM, Vleggaar FP, Tytgat KM, Siersema PD, van Buuren HR. High lifetime risk of cancer in primary scle-
rosing cholangitis. J Hepatol. 2009;50:158-164.
Colina F, Pinedo F, Solis JA, Moreno D, Nevado M. Nodular regenerative hyperplasia of the liver in early histologi-
cal stages of primary biliary cirrhosis. Gastroenterology. 1992;102:1319-1324.
Colombato L. The role of transjugular intrahepatic portosystemic shunt (TIPS) in the management of portal hyper-
tension. J Clin Gastroenterol. 2007;41(suppl 3):S344-S351.
Compston A. Progressive lenticular degeneration: a familial nervous disease associated with cirrhosis of the liver.
by S. A. Kinnier Wilson (From the National Hospital, and the Laboratory of the National Hospital, Queen
Square, London), Brain 1912: 34; 295-509. Brain 2009;132:1997-2001.
Conroy M, Sewell L, Miller OF, Ferringer T. Interferon-beta injection site reaction: review of the histology and
report of a lupus-like pattern. J Am Acad Dermatol. 2008;59:S48-S49.
Corigliano N, Mercantini P, Amodio PM, et al. Hemoperitoneum from a spontaneous rupture of a giant heman-
gioma of the liver: report of a case. Surg Today. 2003;33:459-463.
de Castro F, Bonacini M, Walden JM, Schubert TT. Myxedema ascites. Report of two cases and review of the
literature. J Clin Gastroenterol. 1991;13:411-414.
Dolapci M, Ersoz S, Kama NA. Hepatic artery aneurysm. Ann Vasc Surg. 2003;17:214-216.
Elmberg M, Hultcrantz R, Ekbom A, et al. Cancer risk in patients with hereditary hemochromatosis and in their
first-degree relatives. Gastroenterology. 2003;125:1733-1741.
El-Newihi HM, Alamy ME, Reynolds TB. Salmonella hepatitis: analysis of 27 cases and comparison with acute viral
hepatitis. Hepatology. 1996;24:516-519.
El-Serag HB, Davila JA, Petersen NJ, McGlynn KA. The continuing increase in the incidence of hepatocellular
carcinoma in the United States: an update. Ann Intern Med. 2003;139:817-823.
El-Serag HB, Siegel AB, Davila JA, et al. Treatment and outcomes of treating of hepatocellular carcinoma among
Medicare recipients in the United States: a population-based study. J Hepatol. 2006;44:158-166.
Elta GH, Sepersky RA, Goldberg MJ, Connors CM, Miller KB, Kaplan MM. Increased incidence of hypothyroidism
in primary biliary cirrhosis. Dig Dis Sci. 1983;28:971-975.
Esrailian E, Pantangco ER, Kyulo NL, Hu KQ, Runyon BA. Octreotide/Midodrine therapy significantly improves
renal function and 30-day survival in patients with type 1 hepatorenal syndrome. Dig Dis Sci. 2007;52:742-
748.
European Association for the Study of the Liver. EASL Clinical Practice Guidelines: management of cholestatic liver
diseases. J Hepatol. 2009;51:237-267.
European Association for the Study of the Liver. EASL Clinical Practice Guidelines: management of chronic hepatitis
B. J Hepatol. 2009;50:227-242.
European Association for the Study of the Liver. EASL Clinical Practice Guidelines for HFE hemochromatosis. J
Hepatol. 2010;53:3-22.
Euvrard S, Kanitakis J, Claudy A. Skin cancers after organ transplantation. N Engl J Med. 2003;348:1681-1691.
Fasano A, Berti I, Gerarduzzi T, et al. Prevalence of celiac disease in at-risk and not-at-risk groups in the United
States: a large multicenter study. Arch Intern Med. 2003;163:286-292.
Favennec L, Jave Ortiz J, Gargala G, Lopez Chegne N, Ayoub A, Rossignol JF. Double-blind, randomized, placebo-
controlled study of nitazoxanide in the treatment of fascioliasis in adults and children from northern Peru.
Aliment Pharmacol Ther. 2003;17:265-270.
Fevery J, Verslype C, Lai G, Aerts R, Van Steenbergen W. Incidence, diagnosis, and therapy of cholangiocarcinoma
in patients with primary sclerosing cholangitis. Dig Dis Sci. 2007;52:3123-3135.
Foley WJ, Turcotte JG, Hoskins PA, Brant RL, Ause RG. Intrahepatic arteriovenous fistulas between the hepatic
artery and portal vein. Ann Surg. 1971;174:849-855.
Bibliography 247

Ford AC, Chey WD, Talley NJ, Malhotra A, Spiegel BM, Moayyedi P. Yield of diagnostic tests for celiac disease in
individuals with symptoms suggestive of irritable bowel syndrome: systematic review and meta-analysis. Arch
Intern Med. 2009;169:651-658.
Freedman ND, Curto TM, Lindsay KL, Wright EC, Sinha R, Everhart JE; HALT-C Trial Group. Coffee consump-
tion is associated with response to peginterferon and ribavirin therapy in patients with chronic hepatitis C.
Gastroenterology. 2011;140:1961-1969.
Ge D, Fellay J, Thompson AJ, et al. Genetic variation in IL28B predicts hepatitis C treatment-induced viral clear-
ance. Nature. 2009;461:399-401.
Gleisner AL, Munoz A, Brandao A, et al. Survival benefit of liver transplantation and the effect of underlying liver
disease. Surgery. 2010;147:392-404.
Gordon SC, Reddy KR, Schiff L, Schiff ER. Prolonged intrahepatic cholestasis secondary to acute hepatitis A. Ann
Intern Med. 1984;101:635-637.
Gordon-Weeks AN, Snaith A, Petrinic T. Systematic review of outcome of downstaging hepatocellular cancer
before liver transplantation in patients outside the Milan criteria. Br J Surg. 2011;54:645-647.
Guillevin L, Mahr A, Callard P, et al. Hepatitis B virus-associated polyarteritis nodosa: clinical characteristics, out-
come, and impact of treatment in 115 patients. Medicine (Baltimore). 2005;84:313-322.
Gumaste VV, Dave PB, Weissman D, Messer J. Lipase/amylase ratio. A new index that distinguishes acute episodes
of alcoholic from nonalcoholic acute pancreatitis. Gastroenterology. 1991;101:1361-1366.
Hadad E, Ben-Ari Z, Heled Y, Moran DS, Shani Y, Epstein Y. Liver transplantation in exertional heat stroke: a medi-
cal dilemma. Intensive Care Med. 2004;30:1474-1478.
Hall GW. Kasabach-Merritt syndrome: pathogenesis and management. Br J Haematol. 2001;112:851-862.
Hans D, Durosier C, Kanis JA, Johansson H, Schott-Pethelaz AM, Krieg MA. Assessment of the 10-year probability
of osteoporotic hip fracture combining clinical risk factors and heel bone ultrasound: the EPISEM prospective
cohort of 12,958 elderly women. J Bone Miner Res. 2008;23:1045-1051.
Haque R, Huston CD, Hughes M, Houpt E, Petri WA Jr. Amebiasis. N Engl J Med. 2003;348:1565-1573.
Harrison PM, Keays R, Bray GP, Alexander GJ, Williams R. Improved outcome of paracetamol-induced fulminant
hepatic failure by late administration of acetylcysteine. Lancet. 1990;335:1572-1573.
Harry R, Auzinger G, Wendon J. The clinical importance of adrenal insufficiency in acute hepatic dysfunction.
Hepatology. 2002;36:395-402.
Hashimoto E, Taniai M, Kaneda H, et al. Comparison of hepatocellular carcinoma patients with alcoholic liver
disease and nonalcoholic steatohepatitis. Alcohol Clin Exp Res. 2004;28:164S-168S.
Hashimoto E, Yatsuji S, Tobari M, et al. Hepatocellular carcinoma in patients with nonalcoholic steatohepatitis.
J Gastroenterol. 2009;44(suppl 19):89-95.
Hay JE. Liver disease in pregnancy. Hepatology. 2008;47:1067-1076.
Hazin R, Abu-Rajab Tamimi TI, Abuzetun JY, Zein NN. Recognizing and treating cutaneous signs of liver disease.
Cleve Clin J Med. 2009;76:599-606.
McHutchinson JG, Everson GT, Gordon SC, et al. Telaprevir with peginterferon and ribavirin for chronic HCV
genotype 1 infection. N Engl J Med. 2009;360:1827-1938.
McHutchinson JG, Manns MP, Muir AJ, et al. Telaprevir for previously treated chronic HCV infection. N Engl J
Med. 2010;362:1292-1303.
Hennigar GR, Greene WB, Walker EM, de Saussure C. Hemochromatosis caused by excessive vitamin iron intake.
Am J Pathol. 1979;96:611-624.
Herbert V. Hemochromatosis and vitamin C. Ann Intern Med. 1999;131:475-476.
Hillemanns P, Knitza R, Muller-Hocker J. Rupture of splenic artery aneurysm in a pregnant patient with portal
hypertension. Am J Obstet Gynecol. 1996;174:1665-1666.
Hoefs JC, Morgan TR. Seventy-two weeks of peginterferon and ribavirin for patients with partial early virologic
response? Hepatology. 2007;46:1671-1674.
Holtmann M, Schreiner O, Kohler H, et al. Veno-occlusive disease (VOD) in Crohn’s disease (CD) treated with
azathioprine. Dig Dis Sci. 2003;48:1503-1505.
Holubek WJ, Kalman S, Hoffman RS. Acetaminophen-induced acute liver failure: results of a United States multi-
center, prospective study. Hepatology. 2006;43:880; author reply 882.
Ichai P, Roque Afonso AM, Sebagh M, et al. Herpes simplex virus-associated acute liver failure: a difficult diagnosis
with a poor prognosis. Liver Transpl. 2005;11:1550-1555.
Iwatsuki S, Todo S, Starzl TE. Excisional therapy for benign hepatic lesions. Surg Gynecol Obstet. 1990;171:240-
246.
Jensen K, Gluud C. The Mallory body: morphological, clinical and experimental studies (Part 1 of a literature sur-
vey). Hepatology. 1994;20:1061-1077.
Jones AL, Simpson KJ. Review article: mechanisms and management of hepatotoxicity in ecstasy (MDMA) and
amphetamine intoxications. Aliment Pharmacol Ther. 1999;13:129-133.
248 Bibliography

Kanwal F, Gralnek IM, Martin P, Dulai GS, Farid M, Spiegel BM. Treatment alternatives for chronic hepatitis B virus
infection: a cost-effectiveness analysis. Ann Intern Med. 2005;142:821-831.
Karlsen TH, Schrumpf E, Boberg KM. Gallbladder polyps in primary sclerosing cholangitis: not so benign. Curr Opin
Gastroenterol. 2008;24:395-399.
Karsan HA, Rojter SE, Saab S. Primary prevention of cirrhosis. Public health strategies that can make a difference.
Postgrad Med. 2004;115:25-30.
Keeffe EB, Dieterich DT, Han SH, et al. A treatment algorithm for the management of chronic hepatitis B virus infec-
tion in the United States: 2008 update. Clin Gastroenterol Hepatol. 2008;6:1315-1341; quiz 1286.
Khuroo MS. Hepatitis E virus. Curr Opin Infect Dis. 2008;21:539-543.
Kim WR, Biggins SW, Kremers WK, et al. Hyponatremia and mortality among patients on the liver-transplant wait-
ing list. N Engl J Med. 2008;359:1018-1026.
Knill-Jones RP, Buckle RM, Parsons V, Calne RY, Williams R. Hypercalcemia and increased parathyroid-hormone
activity in a primary hepatoma. Studies before and after hepatic transplantation. N Engl J Med. 1970;282:704-
708.
Kowdley KV, Brandhagen DJ, Gish RG, et al. Survival after liver transplantation in patients with hepatic iron over-
load: the national hemochromatosis transplant registry. Gastroenterology. 2005;129:494-503.
Krowka MJ, Fallon MB, Mulligan DC, Gish RG. Model for end-stage liver disease (MELD) exception for portopul-
monary hypertension. Liver Transpl. 2006;12:S114-S116.
Krowka MJ, Swanson KL, Frantz RP, McGoon MD, Wiesner RH. Portopulmonary hypertension: results from a
10-year screening algorithm. Hepatology. 2006;44:1502-1510.
Larson AM, Polson J, Fontana RJ, et al. Acetaminophen-induced acute liver failure: results of a United States mul-
ticenter, prospective study. Hepatology. 2005;42:1364-1372.
Larsson SC, Wolk A. Coffee consumption and risk of liver cancer: a meta-analysis. Gastroenterology. 2007;132:1740-
1745.
Lee NM, Brady CW. Liver disease in pregnancy. World J Gastroenterol. 2009;15:897-906.
Leese T, Farges O, Bismuth H. Liver cell adenomas. A 12-year surgical experience from a specialist hepato-biliary
unit. Ann Surg. 1988;208:558-564.
Levy C, Zein CO, Gomez J, et al. Prevalence and predictors of esophageal varices in patients with primary biliary
cirrhosis. Clin Gastroenterol Hepatol. 2007;5:803-808.
Li CP, Lee FY, Hwang SJ, et al.. Treatment of mastalgia with tamoxifen in male patients with liver cirrhosis: a random-
ized crossover study. Am J Gastroenterol. 2000;95:1051-1055.
Lindor KD, Gershwin ME, Poupon R, Kaplan M, Bergasa NV, Heathcote EJ. Primary biliary cirrhosis. Hepatology.
2009;50:291-308.
Liu LU, Schiano TD. Long-term care of the liver transplant recipient. Clin Liver Dis. 2007;11:397-416.
Liu S, Chan KW, Wang B, Qiao L. Fibrolamellar hepatocellular carcinoma. Am J Gastroenterol. 2009;104:2617-
2624; quiz 2625.
Llovet JM, Fuster J, Bruix J. Intention-to-treat analysis of surgical treatment for early hepatocellular carcinoma: resec-
tion versus transplantation. Hepatology. 1999;30:1434-1440.
Llovet JM, Ricci S, Mazzaferro V, et al. Sorafenib in advanced hepatocellular carcinoma. N Engl J Med.
2008;359:378-390.
Lok AS, McMahon BJ. Chronic hepatitis B: update 2009. Hepatology. 2009;50:661-662.
Longeville JH, de la Hall P, Dolan P, et al. Treatment of a giant haemangioma of the liver with Kasabach-Merritt
syndrome by orthotopic liver transplant: a case report. HPB Surg. 1997;10:159-162.
Lum G. Significance of low serum alkaline phosphatase activity in a predominantly adult male population. Clin
Chem. 1995;41:515-518.
Malinchoc M, Kamath PS, Gordon FD, Peine CJ, Rank J, ter Borg PC. A model to predict poor survival in patients
undergoing transjugular intrahepatic portosystemic shunts. Hepatology. 2000;31:864-871.
Marcellin P, Forns X, Goeser T, et al. Telaprevir is effective given every 8 or 12 hourse with ribavirin and peginter-
feron alfa-2a or -2b to patients with chronic hepatitis C. Gastroenterol. 2011;140:459-468.
Marchesa P, Lashner BA, Lavery IC, et al. The risk of cancer and dysplasia among ulcerative colitis patients with
primary sclerosing cholangitis. Am J Gastroenterol. 1997;92:1285-1288.
Mas-Coma S. Epidemiology of fascioliasis in human endemic areas. J Helminthol. 2005;79:207-216.
Mazzaferro V, Regalia E, Doci R, et al. Liver transplantation for the treatment of small hepatocellular carcinomas in
patients with cirrhosis. N Engl J Med. 1996;334:693-699.
McDowell Torres D, Stevens R, Gurakar A. Acute liver failure: a management challenge for the practicing gastroen-
terologist. Gastroenterol Hepatol. 2010;6:444-450.
McLaran CJ, Bett JH, Nye JA, Halliday JW. Congestive cardiomyopathy and haemochromatosis—rapid progression
possibly accelerated by excessive ingestion of ascorbic acid. Aust N Z J Med. 1982;12:187-188.
McTigue KM, Harris R, Hemphill B, et al. Screening and interventions for obesity in adults: summary of the evidence
for the U.S. Preventive Services Task Force. Ann Intern Med. 2003;139:933-949.
Bibliography 249

Meguro M, Soejima Y, Taketomi A, et al. Living donor liver transplantation in a patient with giant hepatic heman-
gioma complicated by Kasabach-Merritt syndrome: report of a case. Surg Today. 2008;38:463-468.
Mendes FD, Kim WR, Pedersen R, Therneau T, Lindor KD. Mortality attributable to cholestatic liver disease in the
United States. Hepatology. 2008;47:1241-1247.
Menon KV, Angulo P, Weston S, Dickson ER, Lindor KD. Bone disease in primary biliary cirrhosis: independent
indicators and rate of progression. J Hepatol. 2001;35:316-323.
Moore KP, Wong F, Gines P, et al. The management of ascites in cirrhosis: report on the consensus conference of
the International Ascites Club. Hepatology. 2003;38:258-266.
Moreno-Otero R, Trapero-Marugan M. Extrahepatic conditions associated with primary biliary cirrhosis.
Hepatology. 2010;51:713.
Morgan M, Keeffe EB. Diagnosis and treatment of chronic hepatitis B: 2009 update. Minerva Gastroenterol Dietol.
2009;55:5-22.
Morrison ED, Brandhagen DJ, Phatak PD, et al. Serum ferritin level predicts advanced hepatic fibrosis among U.S.
patients with phenotypic hemochromatosis. Ann Intern Med. 2003;138:627-633.
Morris-Stiff G, Coles G, Moore R, Jurewicz A, Lord R. Abdominal wall hernia in autosomal dominant polycystic
kidney disease. Br J Surg. 1997;84:615-617.
O’Grady JG, Alexander GJ, Hayllar KM, Williams R. Early indicators of prognosis in fulminant hepatic failure.
Gastroenterology. 1989;97:439-445.
O’Shea RS, Dasarathy S, McCullough AJ. Alcoholic liver disease. Am J Gastroenterol. 2010;105:14-32; quiz 33.
Ostapowicz G, Fontana RJ, Schiodt FV, et al. Results of a prospective study of acute liver failure at 17 tertiary care
centers in the United States. Ann Intern Med. 2002;137:947-954.
Pachera S, Nishio H, Yamada H, et al. Superextended hepatectomy for resection of multiple giant hemangiomas:
report of a case. Surg Today. 2009;39:452-455.
Palma DT, Fallon MB. The hepatopulmonary syndrome. J Hepatol. 2006;45:617-625.
Papatheodoridis GV, Manolakopoulos S. EASL clinical practice guidelines on the management of chronic hepatitis
B: the need for liver biopsy. J Hepatol. 2009;51:226-227.
Pardi DS, Loftus EV Jr, Kremers WK, Keach J, Lindor KD. Ursodeoxycholic acid as a chemopreventive agent in
patients with ulcerative colitis and primary sclerosing cholangitis. Gastroenterology. 2003;124:889-893.
Passarella M, Fallon MB, Kawut SM. Portopulmonary hypertension. Clin Liver Dis. 2006;10:653-663, x.
Pastor CM, Schiffer E. Therapy insight: hepatopulmonary syndrome and orthotopic liver transplantation. Nat Clin
Pract Gastroenterol Hepatol. 2007;4:614-621.
Pawlotsky JM. EASL Clinical Practice Guidelines. J Hepatol. 2009;50:243.
Pietrangelo A. Hereditary hemochromatosis: pathogenesis, diagnosis, and treatment. Gastroenterology.
2010;139:393-408, 408 e1-2.
Polson J, Lee WM. AASLD position paper: the management of acute liver failure. Hepatology. 2005;41:1179-1197.
Poordad F, McCone J Jr, Bacon BR, et al. Boceprevir for untreated chronic HCV genotype 1 infection. N Engl J Med.
2011;364:1195-1206.
Riely CA. Liver disease in the pregnant patient. Am J Gastroenterol. 1999;94:1728-1732.
Roberts EA, Schilsky ML. Diagnosis and treatment of Wilson disease: an update. Hepatology. 2008;47:2089-2111.
Roberts JP, Venook A, Kerlan R, Yao F. Hepatocellular carcinoma: ablate and wait versus rapid transplantation. Liver
Transpl. 2010;16:925-929.
Rubin RA, Mitchell DG. Evaluation of the solid hepatic mass. Med Clin North Am. 1996;80:907-928.
Runyon BA. Care of patients with ascites. N Engl J Med. 1994;330:337-342.
Sanchez-Tapias JM, Costa J, Mas A, Bruguera M, Rodes J. Influence of hepatitis B virus genotype on the long-term
outcome of chronic hepatitis B in western patients. Gastroenterology. 2002;123:1848-1856.
Sandhu BS, Sanyal AJ. Pregnancy and liver disease. Gastroenterol Clin North Am. 2003;32:407-436, ix.
Sansonno D, Gesualdo L, Manno C, Schena FP, Dammacco F. Hepatitis C virus-related proteins in kidney tissue
from hepatitis C virus-infected patients with cryoglobulinemic membranoproliferative glomerulonephritis.
Hepatology. 1997;25:1237-1244.
Sanyal AJ. AGA technical review on nonalcoholic fatty liver disease. Gastroenterology. 2002;123:1705-1725.
Scheff RT, Zuckerman G, Harter H, Delmez J, Koehler R. Diverticular disease in patients with chronic renal failure
due to polycystic kidney disease. Ann Intern Med. 1980;92:202-204.
Schiodt FV, Rochling FA, Casey DL, Lee WM. Acetaminophen toxicity in an urban county hospital. N Engl J Med.
1997;337:1112-1117.
Schmidt LE, Dalhoff K. Alpha-fetoprotein is a predictor of outcome in acetaminophen-induced liver injury.
Hepatology. 2005;41:26-31.
Sharma MP, Dasarathy S, Verma N, Saksena S, Shukla DK. Prognostic markers in amebic liver abscess: a prospec-
tive study. Am J Gastroenterol. 1996;91:2584-2588.
Shetty K, Rybicki L, Brzezinski A, Carey WD, Lashner BA. The risk for cancer or dysplasia in ulcerative colitis
patients with primary sclerosing cholangitis. Am J Gastroenterol. 1999;94:1643-1649.
250 Bibliography

Soetikno RM, Lin OS, Heidenreich PA, Young HS, Blackstone MO. Increased risk of colorectal neoplasia in patients
with primary sclerosing cholangitis and ulcerative colitis: a meta-analysis. Gastrointest Endosc. 2002;56:48-
54.
Sorokin A, Brown JL, Thompson PD. Primary biliary cirrhosis, hyperlipidemia, and atherosclerotic risk: a systematic
review. Atherosclerosis. 2007;194:293-299.
Spiegel BM, DeRosa VP, Gralnek IM, Wang V, Dulai GS. Testing for celiac sprue in irritable bowel syndrome with
predominant diarrhea: a cost-effectiveness analysis. Gastroenterology. 2004;126:1721-1732.
Spiegel BM, Targownik L, Dulai GS, Karsan HA, Gralnek IM. Endoscopic screening for esophageal varices in cir-
rhosis: Is it ever cost effective? Hepatology. 2003;37:366-377.
Springer JE, Cole DE, Rubin LA, et al. Vitamin D-receptor genotypes as independent genetic predictors of decreased
bone mineral density in primary biliary cirrhosis. Gastroenterology. 2000;118:145-151.
Stratopoulos C, Papakonstantinou A, Terzis I, et al. Changes in liver histology accompanying massive weight loss
after gastroplasty for morbid obesity. Obes Surg. 2005;15:1154-1160.
Sturniolo GC, Molokhia MM, Shields R, Turnberg LA. Zinc absorption in Crohn’s disease. Gut. 1980;21:387-391.
Swanson KL, Wiesner RH, Krowka MJ. Natural history of hepatopulmonary syndrome: impact of liver transplanta-
tion. Hepatology. 2005;41:1122-1129.
Swanson KL, Wiesner RH, Nyberg SL, Rosen CB, Krowka MJ. Survival in portopulmonary hypertension: Mayo
Clinic experience categorized by treatment subgroups. Am J Transplant. 2008;8:2445-2453.
Sylvestre PB, Batts KP, Burgart LJ, Poterucha JJ, Wiesner RH. Recurrence of primary biliary cirrhosis after liver trans-
plantation: histologic estimate of incidence and natural history. Liver Transpl. 2003;9:1086-1093.
Targownik LE, Spiegel BM, Dulai GS, Karsan HA, Gralnek IM. The cost-effectiveness of hepatic venous pressure
gradient monitoring in the prevention of recurrent variceal hemorrhage. Am J Gastroenterol. 2004;99:1306-
1315.
Tavill AS. Diagnosis and management of hemochromatosis. Hepatology. 2001;33:1321-1328.
Tenner S, Dubner H, Steinberg W. Predicting gallstone pancreatitis with laboratory parameters: a meta-analysis. Am
J Gastroenterol. 1994;89:1863-1866.
ter Borg MJ, Leemans WF, de Man RA, Janssen HL. Exacerbation of chronic hepatitis B infection after delivery.
J Viral Hepat. 2008;15:37-41.
Terriff BA, Gibney RG, Scudamore CH. Fatality from fine-needle aspiration biopsy of a hepatic hemangioma.
Am J Roentgenol. 1990;154:203-204.
Thompson AJ, Muir AJ, Sulkowski MS, et al. Interleukin-28B polymorphism improves viral kinetics and is the stron-
gest pretreatment predictor of sustained virologic response in genotype 1 hepatitis C virus. Gastroenterology.
2010;139:120-129 e18.
Thorsen S, Ronne-Rasmussen J, Petersen E, Isager H, Seefeldt T, Mathiesen L. Extra-intestinal amebiasis: clinical
presentation in a non-endemic setting. Scand J Infect Dis. 1993;25:747-750.
Tillmann HL, Hadem J, Leifeld L, et al. Safety and efficacy of lamivudine in patients with severe acute or fulminant
hepatitis B, a multicenter experience. J Viral Hepat. 2006;13:256-263.
Toso C, Mentha G, Kneteman NM, Majno P. The place of downstaging for hepatocellular carcinoma. J Hepatol.
2010;52:930-936.
Vasan RS, Pencina MJ, Cobain M, Freiberg MS, D’Agostino RB. Estimated risks for developing obesity in the
Framingham Heart Study. Ann Intern Med. 2005;143:473-480.
Veluru C, Atluri D, Chadalavada R, Burns E, Mullen KD. Skin rash during chronic hepatitis C therapy. Gastroenterol
Hepatol. 2010;6:323-325.
Vera A, Gunson BK, Ussatoff V, Nightingale P, et al. Colorectal cancer in patients with inflammatory bowel disease
after liver transplantation for primary sclerosing cholangitis. Transplantation. 2003;75:1983-1988.
Watt KD, Pedersen RA, Kremers WK, Heimbach JK, Charlton MR. Evolution of causes and risk factors for mortality
post-liver transplant: results of the NIDDK long-term follow-up study. Am J Transplant. 2010;10:1420-1427.
Weibrecht K, Dayno M, Darling C, Bird SB. Liver aminotransferases are elevated with rhabdomyolysis in the
absence of significant liver injury. J Med Toxicol. 2010;6:294-300.
Wolf G. A history of vitamin A and retinoids. FASEB J. 1996;10:1102-1107.
Yang HI, Lu SN, Liaw YF, et al. Hepatitis B e antigen and the risk of hepatocellular carcinoma. N Engl J Med.
2002;347:168-174.
Yen TC, Hwang SJ, Wang CC, Lee SD, Yeh SH. Hypercalcemia and parathyroid hormone-related protein in hepa-
tocellular carcinoma. Liver. 1993;13:311-315.
Yeoman AD, Al-Chalabi T, Karani JB, et al. Evaluation of risk factors in the development of hepatocellular carcinoma
in autoimmune hepatitis: implications for follow-up and screening. Hepatology. 2008;48:863-870.
Yu L, Ioannou GN. Survival of liver transplant recipients with hemochromatosis in the United States.
Gastroenterology. 2007;133:489-495.
Zein CO, Jorgensen RA, Clarke B, et al. Alendronate improves bone mineral density in primary biliary cirrhosis: a
randomized placebo-controlled trial. Hepatology. 2005;42:762-771.
Index

abdominal pain persistent hypotension in, 117–119


in acute fatty liver of pregnancy, 202–204 acyclovir, for herpes simplex virus hepatitis, 183–184
in amebic abscess, 105–108 adenomas, hepatic, 159–160
in arteriovenous fistula, 127, 129 adrenal insufficiency, in acute liver failure, 117–119
in Budd-Chiari syndrome, 25–28 AFP (alpha-fetoprotein), elevated
in extrahepatic hepatic artery aneurysm, 127–129 in heat stroke, 114–116
in gallstone pancreatitis, 93–95, 209–210 in hepatocellular carcinoma, 73, 185–187
in HELLP syndrome, 202, 204–206 AIDS. See HIV infection
in herpes simplex virus hepatitis, 183–184 albendazole, for hydatid cyst, 213
in Kasabach-Merritt syndrome, 13–15 alcohol use
after liver transplantation, 148–150 acute hepatitis in, 76–80, 127, 128
in mastocytosis, 169–170 cirrhosis in, 108–110
in mushroom toxicity, 133–134 hepatopulmonary syndrome with, 29–31
in pelvic inflammatory disease, 22–24 pancreatitis in, 16–19, 93–95
in spontaneous bacterial peritonitis, 175, 178–179 zinc deficiency in, 151, 153–154
in typhoid fever, 111–113 aldactone, for ascites, 110
abscess alkalosis, contraction, 174, 176–177
amebic, 105–108 ALP (alkaline phosphatase)
pyogenic, 107–108 deficiency of
acetaminophen, toxicity of, 81–84, 117–119 conditions causing, 154
N-acetylcysteine, for acetaminophen toxicity, 83 in hypothyroidism, 151, 152
acquired immunodeficiency syndrome. See HIV infection in pernicious anemia, 151, 153
acute fatty liver of pregnancy, 202–204 in Wilson disease, 151, 153
acute liver failure in zinc deficiency, 151, 153–154
in acetaminophen toxicity, 81–84, 117–119 elevated
in adrenal insufficiency, 117–119 in Budd-Chiari syndrome, 26
definition of, 83 in hydatid cyst, 211–213
in heat stroke, 114–116 in mastocytosis, 169–170
in hepatitis B virus infection, 40, 89–90 in normal pregnancy, 166–168
in herpes simplex virus hepatitis, 183–184 in overlap syndrome of autoimmune hepatitis and
in MDMA toxicity, 20–21 primary biliary cirrhosis, 139–140
in mushroom poisoning, 133–134 in primary biliary cirrhosis, 99, 101, 189, 190

251
252 Index

in primary sclerosing cholangitis, 189, 190 in hemosiderosis, 189, 196


alpha-1 antitrypsin deficiency, liver histology in, 189, 194 in herpes simplex virus hepatitis, 183–184
ALT (alanine aminotransferase), elevated in polymyositis, 91–92
in acute viral hepatitis, 9–12 asterixis, in hepatic encephalopathy, 43–44
coffee consumption and, 131–132 ataxia, in vitamin E deficiency, 135, 136
in gallstone pancreatitis, 93–95 atherosclerosis
in hemosiderosis, 189, 196 extrahepatic hepatic artery aneurysm in, 127–129
in herpes simplex virus hepatitis, 183–184 in morbid obesity, 161–165
in polymyositis, 91–92 atorvastatin, 62
Amanita mushrooms, toxicity of, 133–134 autoimmune cholangiopathy, 139–140
amebiasis, 125 autoimmune hepatitis
amebic abscess, 105–108 drug-induced, 58
amiloride, for ascites, 110 histology of, 189, 192
aminotransferases, elevated autosomal-dominant polycystic kidney disease
in acute viral hepatitis, 9–12 liver cysts with, 144–147
causes of, 85–86 liver transplantation for, 148–150
in celiac sprue, 85–86 azathioprine, toxicity of, 54, 59–60
amiodarone, toxicity of, 54, 57
amoxicillin-clavulanate, toxicity of, 58 balance disorders, in vitamin E deficiency, 135, 136
anabolic steroids, peliosis hepatis due to, 55, 60–61 ballooning hepatocytes, in nonalcoholic steatohepatitis,
“anchovy paste” 189, 190
in amebic abscess, 106 bariatric surgery, for morbid obesity, 161–165
in hydatid cyst, 213 beta blockers, for esophageal varices, 51–52
anemia beta-carotene, toxicity of, 55, 61
in celiac sprue, 85–86 biliary cirrhosis, primary. See primary biliary cirrhosis
hemolytic, in HELLP syndrome, 202, 204–206 biliary colic
pernicious, 151, 153 in fascioliasis, 123–126
aneurysm in gallstone pancreatitis, 209–210
extrahepatic hepatic artery, 127–129 bilirubin
intracranial, with polycystic kidney disease, 146 in acute alcoholic hepatitis, 78
splenic artery, in hepatitis C infections, 32–34 in amebic abscess, 105–106
angiomas, spider biopsy
in hepatopulmonary syndrome, 29–31 for alpha-1 antitrypsin deficiency, 189, 194
in normal pregnancy, 167–168 for autoimmune hepatitis, 189, 192
anorexia for cavernous hemangioma, 14
in hepatitis E virus infections, 141–143 for congestive hepatopathy, 189, 193
in polycystic kidney disease, 144–147 for fatty liver of pregnancy, 202–204
antibiotics for hemochromatosis, 67–68
bleeding due to, 135, 137 for hemosiderosis, 189, 196
toxicity of, 54, 57–58 for hepatitis B virus infections, 189, 195
antimitochondrial antibodies, in primary biliary cirrhosis, for hyperemesis gravidarum, 203
101 for morbid obesity, 161–165
antinuclear antibodies, in overlap syndrome of autoim- for nonalcoholic fatty liver disease, 103–104
mune hepatitis and primary biliary cirrhosis, 139–140 for nonalcoholic steatohepatitis, 189, 190
anti-smooth muscle antibody, in overlap syndrome of auto- for overlap syndrome of autoimmune hepatitis and
immune hepatitis and primary biliary cirrhosis, 139–140 primary biliary cirrhosis, 139–140
arteriovenous fistula, hepatic bruit in, 127, 129 for polymyositis, 92
arthropathy, in hemochromatosis, 64–67 for primary biliary cirrhosis, 101, 189, 190
ascites for primary sclerosing cholangitis, 121, 189, 191
in alcoholic hepatitis, 76–80 for Wilson disease, 49, 102
in Budd-Chiari syndrome, 25–28 bleeding
in hepatitis C cirrhosis, 35–37 from arteriovenous fistula, 127, 129
in hypothyroidism, 151, 152 from esophageal varices, 50–53
in portal hypertension, 127, 129 from fundic varices, 16–19
in portopulmonary hypertension, 200–201 in HELLP syndrome, 202, 204–206
refractory, 109, 174, 176–177 from hepatic adenoma, 160
spironolactone for, gynecomastia due to, 108–110 in Kasabach-Merritt syndrome, 13–15
spontaneous bacterial peritonitis with, 175, 178–179 from splenic artery aneurysm, 32–34
treatment of, neomycin toxicity in, 174, 176 variceal, in pregnancy, 32–34
aspiration in vitamin K deficiency, 135, 137
of amebic abscess, 106 bloating
of hydatid cyst, 213 in Budd-Chiari syndrome, 25–28
AST (aspartate aminotransferase), elevated in Kasabach-Merritt syndrome, 13–15
in acute viral hepatitis, 9–12 boceprevir, for hepatitis C virus infections, 198
coffee consumption and, 131–132 body mass index, classification of, 163
Index 253

brain aneurysm, with polycystic kidney disease, 146 in hepatitis B virus infections, 127, 128
Brazil, amebic abscess acquired in, 105–108 in hepatitis C virus infections, 35–37
“bronze diabetes,” in hemochromatosis, 64–67 hepatocellular carcinoma in, 186–188
bruits, hepatic hepatopulmonary syndrome with, 29–31
in acute alcoholic hepatitis, 127, 128 kidney disorders with
in extrahepatic hepatic artery aneurysm, 127–129 hepatorenal syndrome, 175, 179–180
in hepatocellular carcinoma, 127, 128 mixed cryoglobulinemia and, 174, 178
in liver laceration, 127, 129 neomycin toxicity, 174, 176
in portosystemic shunt, 127, 129 nonsteroidal anti-inflammatory drugs toxicity, 174, 178
bruxism, in MDMA toxicity, 20–21 refractory ascites, 174, 176–177
Budd-Chiari syndrome, 25–28 spontaneous bacterial peritonitis with, 175, 178–179
in hydatid cyst, 212 portal hypertension in, 127, 129
nutmeg liver in, 189, 193 portopulmonary hypertension in, 200–201
clinical threshold values, 215–219
caffeine consumption, 130–132 clonorchiasis, 125–126
calcification, in hepatocellular carcinoma, 72–75 clubbing, in hepatopulmonary syndrome, 29–31
caput medusae, in portal hypertension, 129 coagulopathy, in Kasabach-Merritt syndrome, 13–15
carcinomas coffee consumption, 130–132
cholangiocarcinoma, 121 cognitive function, evaluation of, in hepatic encephalo-
hepatocellular. See hepatocellular carcinoma pathy, 42–46
cardiovascular disease, after liver transplantation, 149–150 colectomy, for ulcerative colitis, primary sclerosing cholan-
cataract, in Wilson disease, 48 gitis after, 207–208
cavernous hemangiomas, in Kasabach-Merritt syndrome, college students, MDMA toxicity in, 20–21
13–15 colonic perforation, after liver transplantation, 149–150
celiac sprue, 85–86, 135, 137–138 colonoscopy, for primary sclerosing cholangitis, 121
cellulitis, interferon-induced, 69–70 colorectal cancer, ulcerative colitis and, 208
central scar, in hepatocellular carcinoma, 72–75 computed tomography
ceruloplasmin deficiency, in Wilson disease, 48–49, 99, for amebic abscess, 106
102–103 for extrahepatic hepatic artery aneurysm, 127–129
Chagas disease, 126 for hepatocellular carcinoma, 185–188
chaparral, toxicity of, 56 for hydatid cyst, 212
chemoembolization, for hepatocellular carcinoma, 187 for recurrent pyogenic cholangitis, 181–182
“chicken-wire” fibrosis, in nonalcoholic fatty liver disease, congestion, hepatic, 25–28, 189, 193
99, 103–104 constipation, in hypothyroidism, 151, 152
Chile, intrahepatic cholestasis of pregnancy in, 202, 203 contraction alkalosis, 174, 176–177
chills, in typhoid fever, 111–113 copper, foods containing, 173
Chinese medicine, hepatotoxicity of, 54, 56 copper accumulation, in Wilson disease, 47–49, 99,
Chlamydial infections, pelvic, 22–24 102–103, 171–173
cholangiocarcinoma, in primary sclerosing cholangitis, cough
121 in amebic abscess, 105–108
cholangiography, for recurrent pyogenic cholangitis, 181– in hydatid cyst, 211–213
182 creatinine kinase, elevated, in polymyositis, 92
cholangiopathy, autoimmune, 139–140 CREST syndrome, in primary biliary cirrhosis, 97
cholangitis Crohn’s disease, zinc deficiency in, 151, 153–154
in hydatid cyst, 212 Cruveilhier-Baumgarten murmur, 127, 129
primary, 120–122, 189, 190 cryoglobulinemia, in hepatitis C virus infections, 70,
primary sclerosing, after colectomy, 207–208 178–179, 275
recurrent pyogenic, 181–182 cyanosis, in hepatopulmonary syndrome, 29–31
cholecystectomy, in gallstone pancreatitis, 209–210 cysts
cholelithiasis, in pregnancy, 210 hydatid, 211–213
cholestasis in polycystic kidney disease, 144–147
antibiotic-induced, 57–58 cytomegalovirus infections, 142
intrahepatic, of pregnancy, 202, 203
in primary sclerosing cholangitis, 189, 191 Darier’s sign, in mastocytosis, 169–170
prolonged, after hepatitis A virus infection, 87–88 “daughter cysts,” in hydatid cyst, 212
in vitamin D deficiency, 137 “death cap” (Amanita mushroom), 133–134
in vitamin E deficiency, 135, 136 delirium, in mushroom toxicity, 133–134
cirrhosis. See also primary biliary cirrhosis dermatographism, in mastocytosis, 169–170
alcoholic, 76–80, 108–110 dermatologic manifestations. See also skin disorders
antibiotics for, bleeding in, 135, 137 as favorite board topic, 3
coffee consumption and, 131–132 diabetes mellitus
esophageal varices in, 50–53 after liver transplantation, 149–150
as favorite board topic, 1–2 in morbid obesity, 161–165
vs. hemochromatosis, 65 nonalcoholic fatty liver disease with, 99, 103–104
hepatic encephalopathy in, 42–46 diarrhea, in mastocytosis, 169–170
254 Index

diastase-resistant globules, in alpha-1 antitrypsin defi- esophageal varices


ciency, 189, 194 in cirrhosis, 50–53
diclofenac, toxicity of, 54, 58–59 in normal pregnancy, 166–168
diet in polycystic kidney disease, 144–147
copper avoidance in, 49, 173 esophageal vein, in portal circulation, 17–18
vitamin D sources in, 137 ethnic variations. See genetic factors
dirt, eating, fascioliasis due to, 123–126 exertional heat stroke, acute liver failure during, 114–116
disseminated intravascular coagulopathy, in Kasabach-
Merritt syndrome, 13–15 fascioliasis, 123–126
diuretics, for ascites, 109–110 fatigue
refractory, 174, 176–177 in acute alcoholic hepatitis, 76–80
renal toxicity and, 174, 176 in amebic abscess, 105–108
diverticulitis, after liver transplantation, 149–150 in HELLP syndrome, 202, 204–206
drainage in hepatitis E virus infections, 141–143
of amebic abscess, 106 in hepatocellular carcinoma, 185–188
of hydatid cyst, 213 in hypothyroidism, 151, 152
drug side effects in normal pregnancy, 166–168
gynecomastia, 108–110 in overlap syndrome of autoimmune hepatitis and
liver injury primary biliary cirrhosis, 139–140
acetaminophen, 81–84, 117–119 in polymyositis, 91–92
amiodarone, 54, 57 in portopulmonary hypertension, 200–201
anabolic steroids, 55, 60–61 fatty liver
antibiotics, 54, 58–59 in alcohol use, 76-80
azathioprine, 54, 59–60 in pregnancy, 202–205
diclofenac, 54, 58–59 in Wilson disease, 47–49
as favorite board topic, 2 fatty liver disease, nonalcoholic, 99, 103–104, 161–165
hepatocanalicular, 56–58 ferritin, elevated, in hemochromatosis, 67–68
kava kava, 54, 56 fever
MDMA, 20–21 in hepatitis E virus infections, 141–143
6-mercaptopurine, 54, 59–60 in herpes simplex virus hepatitis, 183–184
oral contraceptives, 160 in pelvic inflammatory disease, 22–24
peliosis hepatis, 55, 60–61 penicillamine-induced, 171–173
statins, 55, 61–63 in typhoid fever, 111–113
vitamin A, 55, 61 fibrolamellar hepatocellular carcinoma, 72–75
penicillamine, 171–173 fibrosis
renal toxicity, 3, 174, 176, 178 in nonalcoholic steatohepatitis, 189, 190
skin reactions, 68–71 in primary sclerosing cholangitis, 189, 191
“ductopenia,” in primary sclerosing cholangitis, 189, 191 fistulas, arteriovenous, 127, 129
dyspepsia, in hepatic adenoma, 159–160 Fitz-Hugh-Curtis syndrome, 22–24
dyspnea “flask-shaped” ulcerations, in amebic abscess, 106–107
in hepatopulmonary syndrome, 29–31 flat parasites, 123–126
in portopulmonary hypertension, 200–201 “florid duct lesion,” in primary biliary cirrhosis, 101, 189,
190
early response, to hepatitis C therapy, 156–158 flukes, liver, 123–126
early satiety, in polycystic kidney disease, 144–147 flushing, in mastocytosis, 169–170
echinococcosis, 125–126, 211–213 fluvastatin, 62
ecstasy (MDMA) toxicity, 20–21 focal nodular hyperplasia, vs. hepatocellular carcinoma,
edema 73–75
in HELLP syndrome, 202, 204–206 fundic varices, 16–19
in hepatopulmonary syndrome, 29–31 furosemide, for ascites, 108–110, 174, 176–177
in hydatid cyst, 212
in mixed cryoglobulinemia, 174, 178 gait disturbance, in vitamin E deficiency, 135, 136
in portopulmonary hypertension, 200–201 gallbladder polyps, in primary sclerosing cholangitis, 121
“eggshell” appearance, of hydatid cyst, 212 gallstone pancreatitis, 93–95, 209–210
encephalopathy, hepatic. See hepatic encephalopathy gastric bypass, for morbid obesity, 164
entecavir, for hepatitis B virus vertical transmission preven- gastric varices, 16–19
tion, 39, 40 gastroparesis, erythromycin-induced, 57–58
Entamoeba histolytica, in amebic abscess, 105–108 genetic factors
ERCP (endoscopic retrograde cholangiopancreatography) in body mass index, 163
for gallstone pancreatitis, 209–210 in fatty liver of pregnancy, 202–204
for recurrent pyogenic cholangitis, 181–182 in hepatitis B virus, 40
erythema, palmar, in normal pregnancy, 167–168 in hepatitis C virus, 156–158, 197–199
erythromycin in intrahepatic cholestasis of pregnancy, 202, 203
for peliosis hepatis, 61 germander, toxicity of, 56
toxicity of, 57–58 giant hemangiomas, in Kasabach-Merritt syndrome, 13–15
Index 255

globules, diastase-resistant, in alpha-1 antitrypsin defi- hepatocellular carcinoma in, 127, 128
ciency, 189, 194 histology of, 189, 195
gluten enteropathy (celiac sprue), 85–86, 135, 137–138 in pregnancy, vertical transmission of, 38–41
granulomatous destruction, in primary biliary cirrhosis, 99, hepatitis C virus infections
101, 189, 190 cirrhosis in
“ground-glass” appearance, in hepatitis B virus infections, esophageal varices in, 50–53
189, 195 portopulmonary hypertension in, 200–201
gynecomastia, spironolactone-induced, 108–110 transjugular intrahepatic portosystemic shunt consid-
eration for, 35–37
hair changes as favorite board topic, 3
in hypothyroidism, 151, 152 interferon for, rash in, 68–71
in pernicious anemia, 151, 153 skin disorders in, 68–71
headache, in HELLP syndrome, 202, 204–206 splenic artery aneurysm in, 32–34
heart failure, nutmeg liver in, 189, 193 therapy for, response to, 155–158, 197–199
heartburn, in normal pregnancy, 166–168 hepatitis E virus infections, 141–143
heat stroke, acute liver failure during, 114–116 hepatocanalicular injury, from antibiotics, 57–58
HELLP syndrome, 202, 204–206 hepatocellular carcinoma
hemangiomas, cavernous, 13–15 coffee consumption and, 131–132
hematemesis, in fundic varices, 16–19 epidemiology of, 128
hemidiaphragm elevation, in amebic abscess, 106 fibrolamellar, 72–75
hemochromatosis in hemochromatosis, 66
vs. hemosiderosis, 196 hepatic bruit in, 127, 128
hereditary, 64–67 paraneoplastic syndrome in, 185–188
hemolysis, in HELLP syndrome, 202, 204–206 hepatocytes
hemosiderosis ballooning, in nonalcoholic steatohepatitis, 189, 190
vs. hemochromatosis, 196 diastase-resistant, in alpha-1 antitrypsin deficiency, 189,
histology of, 189, 196 194
heparin, for Budd-Chiari syndrome, 28 “ground-glass,” in hepatitis B virus infections, 189, 195
hepatic adenomas, 159–160 hepatolenticular degeneration. See Wilson disease
hepatic artery hepatomegaly
elevated diameter of, in acute alcoholic hepatitis, 128 azathioprine-induced, 54, 60
extrahepatic, aneurysm of, 127–129 in Budd-Chiari syndrome, 25–28
fistula of, 127, 129 in mastocytosis, 169–170
hepatic bruits. See bruits, hepatic in MDMA toxicity, 20–21
hepatic encephalopathy, 42–46 in polycystic kidney disease, 146
in acetaminophen toxicity, 81–84 hepatopathy, congestive, 25–28, 189, 193
in renal insufficiency, 174, 176–177 hepatopulmonary syndrome, 29–31
stages of, 44 hepatotoxicity. See also drug side effects, liver injury
treatment of, 45–46 of Amanita mushrooms, 133–134
hepatic fetor, in hepatitis C cirrhosis, 35–37 herpes simplex virus hepatitis, 142–143
hepatic vein, in portal circulation, 17–18 HIV infection
hepatic venous pressure gradient (HVPG), in cirrhosis, herpes simplex virus hepatitis in, 183–184
51–52 peliosis hepatis in, 61
hepatitis testing for, in hepatitis C infections, 33–34
acute HPVG (hepatic venous pressure gradient), in cirrhosis, 51–52
in MDMA toxicity, 20–21 human immunodeficiency virus infection. See HIV infection
vs. polymyositis, 91–92 hydatid cysts, 211–213
in Wilson disease, 99, 102–103 hyperbilirubinemia, in amebic abscess, 106–107
alcoholic, 76–80, 127, 128 hypercalcemia, in hepatocellular carcinoma, 186–188
autoimmune hyperemesis gravidarum, 202, 203, 205
drug-induced, 54, 58–59 hyperglycemia
histology of, 189, 192 in hemochromatosis, 64–67
overlap with primary biliary cirrhosis, 139–140 in morbid obesity, 161–165
cytomegalovirus, 142 hyperkeratosis, in vitamin A deficiency, 136
herpes simplex virus, 142, 183–184 hyperlipidemia
interface, in autoimmune hepatitis, 189, 192 after liver transplantation, 149–150
in NASH. See nonalcoholic steatohepatitis (NASH) in morbid obesity, 161–165
relapsing, 88 in nonalcoholic fatty liver disease, 99, 103–104
salmonella, 111–113 in primary biliary cirrhosis, 97–98
viral. See specific viruses hypersensitivity, to penicillamine, 171–173
hepatitis A virus infections, prolonged symptoms of, 87–88 hypertension
hepatitis B virus, genotypes in, 40 portal. See portal hypertension
hepatitis B virus infections portopulmonary, 200–201
acute liver failure in, 89–90 systemic
as favorite board topic, 3 in HELLP syndrome, 202, 204–206
256 Index

after liver transplantation, 149–150 Kasabach-Merritt syndrome, cavernous hemangioma in,


in morbid obesity, 161–165 13–15
hyperthermia kava kava toxicity, 54, 56
acute liver failure during, 114–116 Kayser-Fleischer rings, in Wilson disease, 48–49, 99,
in MDMA toxicity, 20–21 102–103
hypervitaminosis A, 55, 61 kidney
hypoalbuminemia, in amebic abscess, 106–107 dysfunction of, in hepatorenal syndrome, 175, 179–180
hyporeflexia, in vitamin E deficiency, 135, 136 failure of, after liver transplantation, 149–150
hyposplenism, in celiac sprue, 85–86 hypoperfusion of, in diuretic overdosage, 174, 176–177
hypotension polycystic disease of, 144–150
in acute liver failure, 117–119 stones in, 185–188
in mushroom toxicity, 133–134 toxicity to, 174, 178
hypothyroidism, in primary biliary cirrhosis, 97–98 King’s College Hospital criteria, for liver transplantation,
hypoxemia, in hepatopulmonary syndrome, 29–31 82
Kupffer cells, hemosiderin in, 189, 196
immune reconstitution, in hepatitis B virus infections,
89–90 lactitol, for hepatic encephalopathy, 45
immunization, hepatitis B virus, 39–41 lactulose, for hepatic encephalopathy, 45
immunoglobulin 4, elevation of, in primary sclerosing laparoscopic adjustable gastric banding, for morbid obe-
cholangitis, 121 sity, 164
India, hepatitis E virus infections in, 142–143 LDH (lactate dehydrogenase), elevated, in acute viral
infertility, in celiac sprue, 85–86 hepatitis, 9–12
inflammation, portal, in nonalcoholic steatohepatitis, 189, leishmaniasis, 126
190 leukocytic vasculitis, in hepatitis C virus infections, 70
inflammatory bowel disease leukocytosis, in amebic abscess, 106–107
drugs for, toxicity of, 54, 59–60 leukopenia, in herpes simplex virus hepatitis, 183–184
primary sclerosing cholangitis with, 121 LFTs (liver function tests), 11–12
vitamin D deficiency in, 135, 137–138 liver
injection site reactions, to interferon, 69 acute failure of. See acute liver failure
insulin resistance cysts of, in polycystic kidney disease, 144–147
in morbid obesity, 161–165 rupture of, in HELLP syndrome, 202, 204–206
in nonalcoholic fatty liver disease, 99, 103–104 “liver rot,” in fascioliasis, 124–125
interface hepatitis, in autoimmune hepatitis, 189, 192 liver transplantation
interferon for acetaminophen toxicity, 81–84, 118
for hepatitis B virus vertical transmission prevention, 39–41 for Budd-Chiari syndrome, 28
for hepatitis C virus infections, response to, 155–158, criteria for, 82
197–199 for hemochromatosis, 66
reactions to, 68–71 for hepatitis C cirrhosis, 35–37
interleukin-28B gene variations, in hepatitis C virus treat- for hepatitis E virus infections, 142
ment response, 197–199 for hepatocellular carcinoma, 186–187
intracranial aneurysm, with polycystic kidney disease, 146 for hepatopulmonary syndrome, 30–31
intracranial pressure, monitoring of, in acute liver failure, for liver failure in heat stroke, 114–116
117–119 long-term complications of, 148–150
intrahepatic cholestasis of pregnancy, 202, 203, 205 Milan criteria for, 186–187
intrapulmonary shunting, in hepatopulmonary syndrome, for mushroom poisoning, 134
29–31 for polycystic kidney disease, 146, 148–150
iron deposition for portopulmonary hypertension, 201
in hemochromatosis, 64–67, 196 for primary biliary cirrhosis, 97–98
in hemosiderosis, 196 primary sclerosing cholangitis after, 207–208
iron-deficiency anemia, in celiac sprue, 85–86 for Wilson disease, 49
irritable bowel syndrome, in celiac sprue, 85–86 lobular inflammation
in autoimmune hepatitis, 189, 192
jaundice in nonalcoholic steatohepatitis, 189, 190
in acute alcoholic hepatitis, 76–80, 127, 128 long-chain 3-hydroxyacyl-coenzyme A dehydrogenase
in acute fatty liver of pregnancy, 202–204 deficiency, fatty liver of pregnancy in, 202–204
in heat stroke, 114–116 lovastatin, toxicity monitoring of, 62
in hepatitis A virus infections, 87–88 lymphadenopathy, penicillamine-induced, 171–173
in hepatitis E virus infections, 141–143 lymphoplasmacytic infiltrate, in primary biliary cirrhosis,
in herpes simplex virus hepatitis, 183–184 189, 190
in intrahepatic cholestasis of pregnancy, 202, 203
in Kasabach-Merritt syndrome, 13–15 ma huang, toxicity of, 56
in MDMA toxicity, 20–21 macrocytosis, in pernicious anemia, 151, 153
in mushroom toxicity, 133–134 magnetic resonance cholangiopancreatography, for pri-
in primary biliary cirrhosis, 99, 101 mary sclerosing cholangitis, 120–121
in recurrent pyogenic cholangitis, 181–182
Index 257

magnetic resonance imaging in HELLP syndrome, 202, 204–206


for hepatic adenoma, 159–160 interferon-induced, 69–70
for liver cysts, 144 neomycin
malabsorption for hepatic encephalopathy, 45
in celiac sprue, 85–86 renal toxicity of, 174, 176
in vitamin D deficiency, 137–138 nephrotic syndrome, in mixed cryoglobulinemia, 174, 178
malaise neurologic disorders
in acute fatty liver of pregnancy, 202–204 in hepatic encephalopathy, 42–46
in hepatitis C cirrhosis, 35–37 in vitamin E deficiency, 135, 136
in herpes simplex virus hepatitis, 183–184 in Wilson disease, 48–49, 99, 102–103
Mallory bodies neutropenia, penicillamine-induced, 171–173
in amiodarone toxicity, 100, 101 night blindness, in vitamin A deficiency, 135, 136
description of, 100 nitazoxanide, for liver parasites, 125
in nonalcoholic fatty liver disease, 99, 103–104 nitrofurantoin, toxicity of, 58
in nonalcoholic steatohepatitis, 189, 190 nodular regenerative hyperplasia, in primary biliary cir-
in primary biliary cirrhosis, 99, 101 rhosis, 97–98
in Wilson disease, 99, 102–103 nomogram, for acetaminophen toxicity, 83
marathon, acute liver failure during, 114–116 nonalcoholic fatty liver disease, 99, 103–104, 161–165
mastalgia, spironolactone-induced, 108–110 nonalcoholic steatohepatitis (NASH), 99, 103–104
mastocytosis, 169–170 histology of, 189, 190
MDMA (methylenedioxymethamphetamine) toxicity, 20–21 vs. Wilson disease, 48
“Medusa’s head,” in portal hypertension, 129 nonsteroidal anti-inflammatory drugs, toxicity of, 54, 58–
MELD (Model for End-Stage Liver Disease) score 59, 174, 178
in acute alcoholic hepatitis, 78 nonviral hepatitis, as favorite board topic, 3–4
in hepatitis C cirrhosis, transjugular intrahepatic porto- null response, to hepatitis C therapy, 156–158
systemic shunt consideration and, 35–37 number connection test (“trail test”), for hepatic encepha-
in hepatocellular carcinoma, 186–187 lopathy, 42–46
in polycystic kidney disease, 146 nutmeg liver, 189, 193
6-mercaptopurine, toxicity of, 54, 59–60 nutritional deficiencies, as favorite board topic, 2–3
metabolic liver disease, as favorite board topic, 2
metabolic syndrome obesity
in morbid obesity, 161–165 bariatric surgery for, 161–165
nonalcoholic fatty liver disease with, 99, 103–104 after liver transplantation, 149–150
metacercariae, in fascioliasis, 124 nonalcoholic fatty liver disease with, 99, 103–104
methyldopa, toxicity of, 58 nonalcoholic steatohepatitis in, 189, 190
methylenedioxymethamphetamine toxicity, 20–21 oliguria, in hepatorenal syndrome, 175, 179–180
Mexico “onion-skin” pattern, in primary sclerosing cholangitis,
hydatid cysts in, 211–213 121, 189, 191
recurrent cholangitis in, 181–182 oral contraceptives, hepatic adenomas and, 159–160
micrographia, in Wilson disease, 48 Oriental cholangitis, 181–182
microvesicular steatosis, in acute fatty liver of pregnancy, osteoporosis
204 in celiac sprue, 85–86
Milan criteria, for liver transplantation, 186–187 in primary biliary cirrhosis, 97–98
milk thistle extract, for mushroom poisoning, 134 in vitamin D deficiency, 135, 137–138
minocycline, toxicity of, 58 “owl’s eye” appearance, in cytomegalovirus infections,
mistletoe, toxicity of, 56 142–143
mixed cryoglobulinemia, in hepatitis C virus infections, 70
muscle injury, abnormal liver results in, 91–92 pain
muscle wasting, in portopulmonary hypertension, 200–201 abdominal. See abdominal pain
mushrooms, toxicity of, 133–134 biliary colic as, 123–126, 209–210
myxedema, 151, 152 in gynecomastia, 108–110
shoulder, in amebic abscess, 105–108
NASH. See nonalcoholic steatohepatitis (NASH) palmar erythema, in normal pregnancy, 167–168
National Institutes of Health, body mass index classifica- pancreatitis
tion of, 163 alcoholic
nausea and vomiting fundic varices in, 16–19
in acute fatty liver of pregnancy, 202–204 vs. gallstone pancreatitis, 93–95
in gallstone pancreatitis, 93–95, 209–210 gallstone, 93–95, 209–210
in HELLP syndrome, 202, 204–206 in hydatid cyst, 212
in herpes simplex virus hepatitis, 183–184 paracentesis, for ascites, 174, 176–177
in MDMA toxicity, 20–21 paraneoplastic syndrome, in hepatocellular carcinoma,
in mushroom toxicity, 133–134 185–188
in normal pregnancy, 166–168 parathyroid hormone-related protein, in hepatocellular
in pregnancy, 202, 203 carcinoma, 186–188
necrosis pegylated interferon-alpha, for hepatitis C virus infections,
258 Index

response to, 197–199 splenic artery aneurysm in, 32–34


peliosis hepatis, steroid-induced, 55, 60–61 primary biliary cirrhosis, 96–99, 101
pelvic exam, for pelvic inflammatory disease, 22–24 histology of, 189, 190
penicillamine, for Wilson disease, 49, 102, 171–173 overlap with autoimmune hepatitis, 139–140
penicillin G, for mushroom poisoning, 134 primary sclerosing cholangitis, 120–122
pentoxifylline, for acute alcoholic hepatitis, 78–79 after colectomy, 207–208
perihepatitis, in pelvic inflammatory disease, 22–24 histology of, 189, 191
peripheral neuropathy, in vitamin E deficiency, 135, 136 proctocolectomy, primary sclerosing cholangitis after, 121
“periportal stage,” in primary biliary cirrhosis, 101 propylthiouracil, toxicity of, 58
peritonitis, spontaneous bacterial, cirrhosis with, 175, proteinuria, in HELLP syndrome, 202, 204–206
178–179 prothrombin time, in acute alcoholic hepatitis, 78
pernicious anemia, 151, 153 pruritus
phallotoxins, in mushrooms, 134 in intrahepatic cholestasis of pregnancy, 202, 203
phenytoin, toxicity of, 58 in mastocytosis, 169–170
phlebotomy, for hemochromatosis, 65–67 in primary biliary cirrhosis, 99, 101
“phospholipid-laden lysosomal lamellar bodies,” in amio- pyogenic abscess, vs. amebic abscess, 107–108
darone toxicity, 57 pyogenic cholangitis, recurrent, 181–182
pica, fascioliasis due to, 123–126
plasma cell infiltrate, in autoimmune hepatitis, 189, 192 radiofrequency ablation, for hepatocellular carcinoma, 187
pleuritic pain, in pelvic inflammatory disease, 22–24 rapid response, to hepatitis C therapy, 156–158
poisoning. See also hepatotoxicity rash, interferon-induced, 68–71
mushroom, 134–135 Rave (methylenedioxymethamphetamine) toxicity, 20–21
polyarteritis nodosa, hepatitis B virus infection and, 89– recurrent pyogenic cholangitis, 181–182
90 refractory ascites, 109, 174, 176–177
polycystic kidney disease relapsing hepatitis, in hepatitis A virus infections, 88
autosomal-dominant, 144–147, 148–150 retinoids
liver cysts with, 144–147 deficiency of, 135, 136
liver transplantation for, 148–150 toxicity of, 55, 61
polycythemia vera, in Budd-Chiari syndrome, 25–28 rhabdomyolysis, abnormal liver results in, 91–92
polymorphisms. See genetic factors rheumatoid factor, in mixed cryoglobulinemia, 174, 178
polypectomy, bleeding after, in vitamin K deficiency, 135, ribavirin
137 for hepatitis C virus infections, response to, 155–158,
polyps, gallbladder, in primary sclerosing cholangitis, 121 197–199
polyuria, in normal pregnancy, 166–168 skin disorders due to, 69
porphyria cutanea tarda, 68–71 rifaximin, for hepatic encephalopathy, 45
portal circulation, anatomy of, 17–18 right hemidiaphragm elevation, in amebic abscess, 106
portal hypertension “rose spots,” in typhoid fever, 112–113
hepatic bruits in, 127, 129 Roux-en-Y gastric bypass, for morbid obesity, 164
in hydatid cyst, 212
in polycystic kidney disease, 144–147 Salmonella paratyphi infections, 111–113
in primary biliary cirrhosis, 97–98 Salmonella typhi infections, 111–113
portal inflammation, in nonalcoholic steatohepatitis, 189, sand, hydatid, 212
190 scar, in hepatocellular carcinoma, 72–75
“portal stage,” of primary biliary cirrhosis, 101 schistosomiasis, 125–126
portal vein sclerosing cholangitis, primary, 120–122, 189, 190, 207–208
fistula of, 127, 129 sheep
thrombosis of fascioliasis in, 123–126
in alcoholic hepatitis, 76–80 hydatid cyst related to, 211–213
in cirrhosis, 52 shock liver, vs. acute viral hepatitis, 11–13
portopulmonary hypertension, 200–201 shoulder pain, in amebic abscess, 105–108
portosystemic shunt, hepatic bruits in, 127, 129 shunt procedures
pravastatin, 62 for Budd-Chiari syndrome, 28
prednisolone, for acute alcoholic hepatitis, 78–79 for gastric varices, 18
pregnancy sicca syndrome, in primary biliary cirrhosis, 97–98
acute fatty liver of, 202–205 silymarin, extract of, or mushroom poisoning, 134
Budd-Chiari syndrome in, 26 simvastatin, toxicity monitoring of, 62
cirrhosis in, 32–34 sinusoidal obstruction syndrome, 54, 60
as favorite board topic, 1 skin disorders
gallstone pancreatitis in, 209–210 cancer, after liver transplantation, 149–150
hepatitis B virus infections in, 38–41 in celiac sprue, 85–86
hepatitis C virus infections in, 32–34 in hepatitis A virus infections, 87–88
hepatitis E virus infections in, 141–143 interferon-induced, 68–71
hyperemesis gravidarum in, 202, 203, 205 in normal pregnancy, 166–168
intrahepatic cholestasis of, 202, 203, 205 in vitamin A deficiency, 136
normal physiologic conditions in, 166–168 sleep disorders, in hepatic encephalopathy, 43–44
Index 259

slow responders, to hepatitis C therapy, 156–158 tocopherol, deficiency of, 135, 136
small-duct primary sclerosing cholangitis, 120–122 “tortuous tracks,” in fascioliasis, 124–125
sodium restriction, for ascites, 109–110 “trail test,” for hepatic encephalopathy, 42–46
spider angiomas transaminemia, severe, 9–12
in hepatopulmonary syndrome, 29–31 transarterial chemoembolization, for hepatocellular carci-
in normal pregnancy, 167–168 noma, 187
“spider web” appearance, in Budd-Chiari syndrome, 27 transferrin saturation, in hemosiderosis, 189, 196
spironolactone transjugular intrahepatic portosystemic shunt, consultation
for ascites, 174, 176–177 on, 35–37
gynecomastia due to, 108–110 transplantation, liver. See liver transplantation
splenectomy, for fundic varices, 18 trauma
splenic artery aneurysm, in hepatitis C infections, 32–34 extrahepatic hepatic artery aneurysm in, 127–129
splenic vein, thrombosis of, in pancreatitis, 16719 hepatic bruit in, 127, 129
splenomegaly traveler’s disorders
in esophageal varices, 50–53 Amanita mushroom poisoning, 133–134
in hepatitis C cirrhosis, 35–37 amebic abscess, 105–108
splenorenal shunt, for gastric varices, 18 fascioliasis, 123–126
spontaneous bacterial peritonitis, with cirrhosis, 175, 178– hepatitis A virus infections, 87–88
179 hepatitis E virus infections, 141–143
statins, toxicity monitoring of, 55, 61–63 hydatid cyst, 211–213
steatohepatitis, nonalcoholic. See nonalcoholic steato- recurrent pyogenic cholangitis, 181–182
hepatitis (NASH) typhoid fever, 111–113
steatorrhea, in intrahepatic cholestasis of pregnancy, 202, trematodes, 123–126
203 tricuspid regurgitation, in portopulmonary hypertension,
steatosis 201
in acute fatty liver of pregnancy, 204 trientine, for Wilson disease, 49, 102, 172–173
in alcoholic hepatitis, 76–80 trimethoprim-sulfamethoxazole, toxicity of, 57–58
in NASH. See nonalcoholic steatohepatitis (NASH) trypanosomiasis, 126
stepwise fever, in typhoid fever, 112–113 tumor(s). See also hepatocellular carcinoma
Stevens Johnson syndrome, interferon-induced, 69–70 cavernous hemangiomas, 13–15
stomach, varices in, 16–19 hepatic adenoma, 159–160
stones, in recurrent pyogenic cholangitis, 181–182 typhoid fever, 111–113
Sugiura procedure, for gastric varices, 18
sunflower cataract, in Wilson disease, 48 ulcerations, “flask-shaped,” in amebic abscess, 106–107
surgery, for hydatid cyst, 213 ulcerative colitis, primary sclerosing cholangitis and, 121,
sustained virologic response, in therapy, 156 207–208
sweating, in pelvic inflammatory disease, 22–24 ultrasonography
for hepatic adenoma, 159–160
tachycardia for hydatid cyst, 212
in Budd-Chiari syndrome, 25–28 upper quadrant pain
in mushroom toxicity, 133–134 in amebic abscess, 105–108
tachypnea, in Budd-Chiari syndrome, 25–28 in Budd-Chiari syndrome, 25–28
tamoxifen, for gynecomastia, 110 in pelvic inflammatory disease, 22–24
teas, herbal, toxicity of, 56 ursodeoxycholic acid
telangiectasias, in hepatopulmonary syndrome, 29–31 for overlap syndrome of autoimmune hepatitis and pri-
telaprevir, for hepatitis C virus infections, 198 mary biliary cirrhosis, 139–140
telbivudine, for hepatitis B virus vertical transmission pre- for primary biliary cirrhosis, 97–98
vention, 39 urticaria, in mastocytosis, 169–170
temperature-pulse dissociation, in typhoid fever, 112–113
tenofovir, for hepatitis B virus vertical transmission preven- varices
tion, 39–40 esophageal
tetracycline, toxicity of, 58 in cirrhosis, 33–34, 50–53
thiopurine methyltransferase, genetic variations in, 59–60 in normal pregnancy, 166–168
threshold values, 215–219 in polycystic kidney disease, 144–147
thrombocytopenia fundic, 16–19
in azathioprine toxicity, 54, 60 vascular disease, as favorite board topic, 2
in esophageal varices, 51 veno-occlusive disease, 54, 60
in HELLP syndrome, 202, 204–206 viral hepatitis. See specific viruses
penicillamine-induced, 171–173 vitamin A
thrombolysis, for Budd-Chiari syndrome, 28 deficiency of, 135, 136
thrombosis toxicity of, 55, 61, 136
portal vein vitamin D
in alcoholic hepatitis, 76–80 deficiency of, 135, 137–138
in cirrhosis, 52 for primary biliary cirrhosis, 97
splenic vein, 16719 vitamin E deficiency, 135, 136
260 Index

vitamin K deficiency, 135, 137, 202, 203 World Health Organization, body mass index classifica-
vomiting. See nausea and vomiting tion of, 163

waist circumference, in obesity, 163 xerophthalmia, in vitamin A deficiency, 136


warfarin, for Budd-Chiari syndrome, 28
weight loss, for morbid obesity, 161–165 zinc
Wilson disease, 99, 102–103 deficiency of, depressed ALP in, 151, 153–154
ALP levels in, 151, 153 for Wilson disease, 49, 172–173
fatty liver in, 47–49
penicillamine for, 171–173
WAIT… There’s More!
SLACK Incorporated’s Health Care Books and Journals offers a wide selection of products in the field of
Gastroenterology and Hepatology. We are dedicated to providing important works that educate, inform and
improve the knowledge of our customers. Don’t miss out on our other informative titles that will enhance
your collection.

Acing the GI Board Exam: With its focus on pearl after pearl, emphasis on images,
The Ultimate Crunch-Time Resource and attention to high-yield “tough stuff” vignettes you
Brennan Spiegel MD, MSHS don’t know the answers to (yet), Acing the GI Board
256 pp., Soft Cover, 2009, Exam is truly the ultimate crunch-time resource for
ISBN 13 978-1-55642-868-5, Order# 78685, $87.95 acing the GI and Hepatology examination, taking re-
Acing the GI Board Exam by Dr. Brennan Spiegel aims certifying examinations, looking good on clerkship
to fill the unmet need in board review by presenting rounds, or for just challenging yourself with interesting
time-tested and high-yield information in a rational, and entertaining vignettes.
useful, and contextually appropriate format.

Please visit www.slackbooks.com


to order this title! 24 Hours a Day…7 Days a Week!

Attention Industry Partners!


Whether you are interested in buying multiple copies of a book, chapter reprints, or
looking for something new and different—we are able to accommodate your needs.

Multiple Copies
SLACK Incorporated At attractive discounts starting for purchases as low as 25 copies for a single title,
SLACK Incorporated will be able to meet all of your needs.
Health Care Books and Journals
Chapter Reprints
6900 Grove Road SLACK Incorporated is able to offer the chapters you want in a format that will lead
Thorofare, NJ 08086 to success. Bound with an attractive cover, use the chapters that are a fit specifically
for your company. Available for quantities of 100 or more.
1-800-257-8290 Customize
SLACK Incorporated is able to create a specialized custom version of any of our
or 1-856-848-1000 products specifically for your company.
Fax: 1-856-848-6091
Please contact the Marketing Communications Director of Health Care
E-mail: orders@slackinc.com
Books and Journals for further details on multiple copy purchases, chapter
Visit: www.slackbooks.com reprints or custom printing at 1-800-257-8290 or 1-856-848-1000.
*Please note all conditions are subject to change.

You might also like

pFad - Phonifier reborn

Pfad - The Proxy pFad of © 2024 Garber Painting. All rights reserved.

Note: This service is not intended for secure transactions such as banking, social media, email, or purchasing. Use at your own risk. We assume no liability whatsoever for broken pages.


Alternative Proxies:

Alternative Proxy

pFad Proxy

pFad v3 Proxy

pFad v4 Proxy